Human Anatomy, 7E Marieb Test Bank

January 15, 2017 | Author: Kitties | Category: N/A
Share Embed Donate


Short Description

This is the Test Bank off of Pearson's Official Website for the textbook Human Anatomy, 7/E by Elaine N. Marieb) ...

Description

 

Human Anatomy, 7e (Marieb/Mitchell/Smith) Chapter 1 The Human Body: An Orientation 1.1 Multiple Choice Questions Use the diagram below to answer the following questions.

Figure 1.1 1) Which letter indicates the mental region? A) A B) B C) C D) D E) E Answer: A Diff: 2 Page Ref: 7 1 Copyright © 2014 Pearson Education, Inc.

 

2) Which letter indicates the umbilical region? A) A B) B C) C D) D E) E Answer: B Diff: 2 Page Ref: 7 3) Which letter indicates the hallux? A) A B) B C) C D) D E) E Answer: E Diff: 2 Page Ref: 7 4) Which letter indicates the inguinal region? A) A B) B C) C D) D E) E Answer: C Diff: 2 Page Ref: 7 5) Which letter indicates the femoral region? A) A B) B C) C D) D E) E Answer: D Diff: 2 Page Ref: 7

2 Copyright © 2014 Pearson Education, Inc.

 

Figure 1.2 Use the diagram above to answer the following questions. 6) Which letter indicates the lumbar region? A) A B) B C) C D) D E) E Answer: D Diff: 2 Page Ref: 7

3 Copyright © 2014 Pearson Education, Inc.

 

7) Which letter indicates the acromial region? A) A B) B C) C D) D E) E Answer: B Diff: 3 Page Ref: 7 8) Which letter indicates the scapular region? A) A B) B C) C D) D E) E Answer: C Diff: 2 Page Ref: 7 9) Which letter indicates the popliteal region? A) A B) B C) C D) D E) E Answer: E Diff: 2 Page Ref: 7 10) Which letter indicates the occipital region? A) A B) B C) C D) D E) E Answer: A Diff: 3 Page Ref: 7 11) The chest is ________ to the abdomen. A) superior B) lateral C) anterior D) proximal E) deep Answer: A Diff: 2 Page Ref: 7-8

4 Copyright © 2014 Pearson Education, Inc.

 

12) The sternal region is ________ to the scapular region. A) superior B) lateral C) anterior D) proximal E) deep Answer: C Diff: 2 Page Ref: 7-8 13) The knee is ________ to the foot. A) superior B) lateral C) anterior D) proximal E) deep Answer: D Diff: 3 Page Ref: 7-8 14) The brain is ________ to the skull. A) superior B) lateral C) anterior D) proximal E) deep Answer: E Diff: 2 Page Ref: 7-8 15) The thumb is ________ to the index finger. A) superior B) lateral C) anterior D) proximal E) deep Answer: B Diff: 3 Page Ref: 7-8 16) Muscles are ________ to the skin. A) superior B) lateral C) anterior D) proximal E) deep Answer: E Diff: 1 Page Ref: 7-8

5 Copyright © 2014 Pearson Education, Inc.

 

17) The axillary region is ________ to the sternum. A) superior B) lateral C) anterior D) proximal E) deep Answer: B Diff: 2 Page Ref: 7-8 18) The lip is ________ to the chin. A) superior B) lateral C) anterior D) proximal E) deep Answer: A Diff: 2 Page Ref: 7-8 19) The eye is ________ to the occipital region. A) superior B) lateral C) anterior D) proximal E) deep Answer: C Diff: 2 Page Ref: 7-8 20) The acromial region is ________ to the scapular region. A) superior B) lateral C) anterior D) proximal E) deep Answer: A Diff: 2 Page Ref: 7-8 21) The gluteal region is ________ to the popliteal region. A) superior B) lateral C) anterior D) proximal E) deep Answer: A Diff: 2 Page Ref: 7-8

6 Copyright © 2014 Pearson Education, Inc.

 

22) The femoral region is ________ to the plantar region. A) superior B) lateral C) anterior D) proximal E) deep Answer: D Diff: 2 Page Ref: 7-8 23) The heart is ________ to the sternum. A) superior B) lateral C) anterior D) proximal E) deep Answer: E Diff: 2 Page Ref: 7-8 24) The pubic area is ________ to the gluteal region. A) superior B) lateral C) anterior D) proximal E) deep Answer: C Diff: 2 Page Ref: 7-8 25) The umbilical region is ________ to the lumbar region. A) superior B) lateral C) anterior D) proximal E) deep Answer: C Diff: 2 Page Ref: 7-8 26) The smallest living unit is A) a cell. B) an organ. C) a human being. D) a molecule. Answer: A Diff: 1 Page Ref: 3

7 Copyright © 2014 Pearson Education, Inc.

 

27) Which branch of anatomy studies the structural changes that occur as one ages? A) developmental anatomy B) pathological anatomy C) regional anatomy D) surface anatomy Answer: A Diff: 2 Page Ref: 2 28) Which organ system includes the pancreas, thymus, testes, and pituitary gland? A) integumentary B) endocrine C) reproductive D) lymphatic Answer: B Diff: 2 Page Ref: 3 &5 29) A coronal section through the human body can A) pass through both the nose and the occipital region. B) pass through both ears. C) provide mirror right and left images. D) lie in a horizontal plane. Answer: B Diff: 2 Page Ref: 7, 9 30) During the process of ________, noncellular artifacts can be introduced into histology samples. A) time B) observation C) staining D) photography Answer: C Diff: 2 Page Ref: 14 31) The cervical region is the A) thigh. B) calf. C) neck. D) head. Answer: C Diff: 1 Page Ref: 7

8 Copyright © 2014 Pearson Education, Inc.

 

32) The "CT" in "CT scanning" stands for A) cut transversely. B) Charles Thorgaard, the inventor's name. C) correlated thickness. D) computed tomography. Answer: D Diff: 1 Page Ref: 16 33) What is the function of serous membranes? A) They act like wrapping paper to hold visceral organs together. B) They contain gland cells that secrete mucus. C) They halt the spread of infection. D) They reduce friction so that viscera move freely. Answer: D Diff: 2 Page Ref: 12 34) The dorsal hollow nerve cord A) develops into the brain and spinal cord. B) is a primitive supporting rod. C) contains the notochord. D) is the same as the human backbone. Answer: A Diff: 2 Page Ref: 10 35) The main purpose of fixation is A) to preserve the tissue. B) to mend breaks in tissue sections. C) to make an organ easier to section. D) to stick tissue sections to a glass slide. Answer: A Diff: 2 Page Ref: 14 36) A histologist examines a specimen that has an epithelium overlying some smooth muscle. This specimen is part of A) a molecule. B) a cell. C) a tissue. D) an organ. Answer: D Diff: 2 Page Ref: 5

9 Copyright © 2014 Pearson Education, Inc.

 

37) An example of a tissue in the body is A) the stomach. B) a muscle cell. C) epithelium. D) a macromolecule. Answer: C Diff: 2 Page Ref: 4 38) An example of an organ is A) a fat cell. B) the intestine. C) epithelium. D) the cardiovascular system (but not the circulatory system). Answer: B Diff: 1 Page Ref: 5 39) Which organ system consists of vessels that do not carry blood, but pick up fluids (and some cells) that are leaked from the blood? A) urinary B) endocrine C) integumentary D) lymphatic Answer: D Diff: 3 Page Ref: 4-5 40) Large molecules such as proteins are called A) cells. B) macromolecules. C) multi-atom units. D) cellular organelles. Answer: B Diff: 1 Page Ref: 4 41) Which organ system covers the external surface of the body, but not the internal surface of the mouth? A) lymphatic B) digestive C) integumentary D) cutaneous Answer: C Diff: 2 Page Ref: 4-5

10 Copyright © 2014 Pearson Education, Inc.

 

42) Which organ system includes the spinal cord? A) skeletal B) muscular C) nervous D) integumentary Answer: C Diff: 1 Page Ref: 4-5 43) The height of an average person's trunk, from neck to perineum, is about A) 5 meters. B) 10 centimeters. C) 1000 µm. D) 1 meter. Answer: D Diff: 3 Page Ref: 3 44) Which statement concerning the anatomical position is FALSE? A) The palms face anteriorly. B) The toes point anteriorly, but the fingers point inferiorly. C) The knees, elbow, and neck are straight (not bent). D) The person is lying down, as straight as possible. Answer: D Diff: 2 Page Ref: 6 45) Bilateral symmetry can apply to objects as well as to animal bodies. Which of the following capital letters of the alphabet is not bilaterally symmetrical? A) A B) M C) L D) O Answer: C Diff: 2 Page Ref: 10 46) Which structure is not covered by visceral serosa? A) lungs B) ribs C) stomach D) uterus Answer: B Diff: 2 Page Ref: 12

11 Copyright © 2014 Pearson Education, Inc.

 

47) The femoral region is the A) buttocks. B) hip. C) thigh. D) toes. Answer: C Diff: 1 Page Ref: 7 48) The inguinal region lies A) anterior to the elbow joint. B) on the anterior neck. C) where the thigh joins the trunk. D) on the external genitals. Answer: C Diff: 2 Page Ref: 7 49) The perineal region is the A) side of the leg. B) region between the external genitals and the anus. C) point of the shoulder. D) superior part of the gluteal region. Answer: B Diff: 2 Page Ref: 7 50) Which structure is not present in the mediastinum? A) esophagus B) heart C) lung D) trachea Answer: C Diff: 2 Page Ref: 11 51) A frontal plane is the same as a ________ plane. A) midsagittal B) transverse C) coronal D) sagittal Answer: C Diff: 2 Page Ref: 7, 9 52) Another name for the midsagittal plane is A) parasagittal. B) oblique. C) coronal. D) median. Answer: D Diff: 2 Page Ref: 7 12 Copyright © 2014 Pearson Education, Inc.

 

53) What point or structure in the body is located farthest laterally? (Hint: Questions always refer to the body in the anatomical position.) A) the coxal region B) ear C) little toe D) tip of thumb Answer: D Diff: 3 Page Ref: 7-8 54) Although transmission electron microscopy is usually used for high-magnification viewing, it is certainly possible to use it at low magnification as well. That is, one can produce similar micrographs of tissues taken by light microscopy and electron microscopy at the same magnification. Even at the same magnification, however, you can easily tell the two kinds of micrographs apart. How? A) The image in the electron micrograph is still sharper. B) Tissue viewed by electron microscopy is colored, whereas light micrographs are always black and white (and shades of gray). C) Tissue for light microscopy cannot be fixed (no fixation). D) Tissue for electron microscopy cannot be sectioned. Answer: A Diff: 2 Page Ref: 14 55) What is the main advantage of MRI as a medical imaging technique? A) It is safe. B) The patient feels less pain during the procedure than with any other imaging technique. C) It is very inexpensive. D) It shows soft tissues very clearly. Answer: D Diff: 2 Page Ref: 18 56) The extremities are the same as A) the ears. B) the fingers and toes. C) the limbs. D) all structures in the head. Answer: C Diff: 2 Page Ref: 7 57) Which of the following pairs of organs/structures is located ipsilateral? A) cecum : sigmoid colon B) descending colon : spleen C) mouth : navel D) right lung : left lung Answer: B Diff: 3 Page Ref: 8 13 Copyright © 2014 Pearson Education, Inc.

 

58) A physician viewing an injury to the back would look at the patient's ________ side. A) lateral B) posterior C) cranial D) ventral Answer: B Diff: 2 Page Ref: 8 59) The ________ body cavity contains the brain. A) dorsal B) ventral C) serous D) lateral Answer: A Diff: 2 Page Ref: 11 60) The roots of anatomical terminology lie mainly in A) German and French. B) Latin and Greek. C) Esperanto. D) Russian and Old English. Answer: B Diff: 2 Page Ref: 6 61) The ________ cavity contains the heart and lungs. A) abdominopelvic B) dorsal C) thoracic D) lateral Answer: C Diff: 2 Page Ref: 11 62) Hormones are regulatory proteins that are secreted by the ________ system. A) urinary B) endocrine C) integumentary D) lymphatic Answer: B Diff: 2 Page Ref: 4-5

14 Copyright © 2014 Pearson Education, Inc.

 

63) Which organ system keeps blood constantly supplied with oxygen, removes carbon dioxide, and contains many air tubes? A) urinary B) endocrine C) circulatory D) respiratory Answer: D Diff: 2 Page Ref: 4-5 64) The elimination of nitrogenous wastes from body fluids is regulated by the ________ system. A) urinary B) endocrine C) integumentary D) lymphatic Answer: A Diff: 1 Page Ref: 4-5 65) Which structures are evidence of the vertebrate characteristic of segmentation? A) branches of the blood vessels B) multiple joints of fingers C) subdivisions of the gastrointestinal tract D) vertebral column Answer: D Diff: 2 Page Ref: 11 66) Which statement about visceral serosa is false? A) It clings to the surface of organs. B) It is continuous with the membrane that covers the outer body wall. C) It is deep to the parietal serosa. D) It lines the internal surface of hollow organs. Answer: D Diff: 2 Page Ref: 12 67) How many centimeters are there in a meter? A) 10 B) 100 C) 1,000 D) 1,000,000 Answer: B Diff: 3 Page Ref: 6

15 Copyright © 2014 Pearson Education, Inc.

 

68) Which organ is not found in the ventral body cavity? A) heart B) liver C) spinal cord D) urinary bladder Answer: C Diff: 2 Page Ref: 11 69) The ankle lies ________ to the thigh. A) distal B) proximal C) lateral D) inferior Answer: A Diff: 2 Page Ref: 8 70) The axillary artery is found in the region of the A) posterior surface of the knee. B) vertebral column. C) armpit. D) long axis of any limb. Answer: C Diff: 2 Page Ref: 7 71) As an anatomical region, lumbar refers to A) the infero-medial aspect of the back. B) part of the lower limb. C) the wrist. D) the breast. Answer: A Diff: 2 Page Ref: 7 72) The buccal region is the A) cheeks. B) waist. C) calf of the leg. D) underside of the foot. Answer: A Diff: 2 Page Ref: 7 73) The popliteal region is A) the side of the leg. B) in the cervical region. C) the posterior surface of the knee. D) the inferior part of the gluteal region. Answer: C Diff: 2 Page Ref: 7 16 Copyright © 2014 Pearson Education, Inc.

 

74) The coxal region is A) the same as the inguinal region. B) the skin over the "tailbone." C) the hip. D) the posterior surface of the wrist. Answer: C Diff: 2 Page Ref: 7 75) Which structure is located entirely within the right upper quadrant? A) appendix B) gallbladder C) spleen D) stomach Answer: B Diff: 2 Page Ref: 13 1.2 True/False Questions 1) Serous cavities include the pleural cavity. Answer: TRUE Diff: 1 Page Ref: 12 2) Serous cavities contain air. Answer: FALSE Diff: 2 Page Ref: 12 3) The peritoneal cavity is a serous cavity. Answer: TRUE Diff: 2 Page Ref: 12 4) EM has much greater resolution than LM. Answer: TRUE Diff: 2 Page Ref: 14 5) The dorsal body cavity is subdivided into a cranial cavity and a vertebral cavity. Answer: TRUE Diff: 2 Page Ref: 11 6) Pathological anatomy deals with structural changes caused by disease. Answer: TRUE Diff: 2 Page Ref: 2 7) A CT scan produces an image of a transverse section of the body. Answer: TRUE Diff: 2 Page Ref: 16-17 17 Copyright © 2014 Pearson Education, Inc.

 

8) MRI techniques can show only images that are hard and deflect the X-rays. Answer: FALSE Diff: 2 Page Ref: 18 9) Ultrasound techniques are used to image a fetus because they are less damaging than other techniques. Answer: TRUE Diff: 2 Page Ref: 18 10) Angiography imaging is used primarily in the study of blood supply to the heart wall and brain. Answer: TRUE Diff: 2 Page Ref: 17 11) Most adults are between 1.5 and 2 meters tall. Answer: TRUE Diff: 3 Page Ref: 6 12) All vertebrate embryos have a dorsal hollow nerve cord. Answer: TRUE Diff: 2 Page Ref: 10 13) A transverse plane could cut the head off the body! Answer: TRUE Diff: 2 Page Ref: 7, 9 14) In anatomical position, the palms of the hands face medially toward the thighs. Answer: FALSE Diff: 1 Page Ref: 6 15) The mediastinum contains the trachea and lungs. Answer: FALSE Diff: 2 Page Ref: 11 1.3 Short Answer Questions 1) Describe the terms proximal and distal. Answer: Proximal means closer to the point of attachment to the main part of the body; distal is farther from the point of attachment. Diff: 2 Page Ref: 8 2) Describe the location of the femoral region in relation to the crural region. Answer: The femoral region is proximal to the crural region. Diff: 2 Page Ref: 7-8

18 Copyright © 2014 Pearson Education, Inc.

 

3) Describe the location of the brachial region in relation to the pollex. Answer: The brachial region is proximal to the pollex. Diff: 2 Page Ref: 7-8 4) In humans, what term is synonymous with posterior? Answer: dorsal Diff: 2 Page Ref: 8 5) Clinicians refer to ________ anatomy when locating blood vessels to draw blood, feeling pulses, and avoiding nerves while giving injections. Answer: surface Diff: 1 Page Ref: 2 6) In humans, the region between the anus and the external genitals is the ________ region. Answer: perineal Diff: 2 Page Ref: 7 7) The olecranal region is posterior to what region? Answer: antecubital Diff: 2 Page Ref: 7 8) The head, neck, and trunk comprise the ________ region. Answer: axial Diff: 2 Page Ref: 7 9) What is the anatomical term for the thumb? Answer: pollex Diff: 2 Page Ref: 7 10) One could say that the forearm is ________ to the brachial region. Answer: distal Diff: 2 Page Ref: 8 11) Cutting the body along the median plane produces a ________ section Answer: midsagittal Diff: 1 Page Ref: 7, 9 12) One could describe the scalp as being ________ to the skull. Answer: superficial Diff: 2 Page Ref: 8 13) A ________ plane separates the body into equal left and right halves. Answer: midsagittal (or median) Diff: 2 Page Ref: 7, 9

19 Copyright © 2014 Pearson Education, Inc.

 

14) The measurement typically used for structures within a cell is the ________. Answer: micrometer Diff: 3 Page Ref: 6 15) The ________ system is involved in immunity. Answer: lymphatic Diff: 3 Page Ref: 4-5 1.4 Essay Questions 1) Identify each of the systems that are found in the arm. Answer: The arm contains elements of the skeletal, nervous, cardiovascular, muscular, lymphatic, and integumentary systems. Diff: 3 Page Ref: 4-5 2) Which of the organ systems found in the arm are not found in the leg? Answer: None Diff: 3 Page Ref: 4-5 3) List the six unique features found in all vertebrates at some stage of their life. Answer: Tube-within-a-tube body plan, bilateral symmetry, dorsal hollow nerve cord, notochord and vertebrae, segmentation, and pharyngeal pouches. Diff: 3 Page Ref: 10-11 4) In adult humans, what remnants of segmentation remain? Answer: The ribs and the vertebrae, with their segmental spinal nerves, are remnants of segmentation. Diff: 3 Page Ref: 11 5) Select from the following techniques the best method for assessing brain function in a stroke patient: X-ray imaging, sonography, MRI. Explain why the other choices are not the best choice. Answer: MRI is the best method of assessing brain function. X-rays do not image soft tissues well or with high resolution. They also produce only two-dimensional images of a structure. Sonography cannot be used to study the brain because sound waves cannot pass through the body skull. Diff: 3 Page Ref: 15-19

20 Copyright © 2014 Pearson Education, Inc.

 

Human Anatomy, 7e (Marieb/Mitchell/Smith) Chapter 2 The Living Units 2.1 Multiple Choice Questions

Figure 2.1 Use the diagram above to answer the following questions. 1) Which letter indicates the rough endoplasmic reticulum? A) A B) B C) C D) D E) E Answer: D Diff: 2 Page Ref: 23 2) Which letter indicates the nucleolus? A) A B) B C) C D) D E) E Answer: A Diff: 2 Page Ref: 23 1 Copyright © 2014 Pearson Education, Inc.

 

3) Which letter indicates the microvilli? A) A B) B C) C D) D E) E Answer: C Diff: 2 Page Ref: 23 4) Which letter indicates the mitochondrion? A) A B) B C) C D) D E) E Answer: B Diff: 2 Page Ref: 23 5) Which letter indicates the Golgi apparatus? A) A B) B C) C D) D E) E Answer: E Diff: 2 Page Ref: 23

2 Copyright © 2014 Pearson Education, Inc.

 

Figure 2.2 Use the diagram above to answer the following questions. 6) Which letter indicates the DNA molecule? A) A B) B C) C D) D E) E Answer: A Diff: 2 Page Ref: 36 7) Which letter indicates the chromatid? A) A B) B C) C D) D E) E Answer: D Diff: 2 Page Ref: 36 3 Copyright © 2014 Pearson Education, Inc.

 

8) Which letter indicates a nucleosome? A) A B) B C) C D) D E) E Answer: C Diff: 2 Page Ref: 36 9) Which letter indicates histones? A) A B) B C) C D) D E) E Answer: B Diff: 2 Page Ref: 36 10) Which letter indicates the metaphase chromosome? A) A B) B C) C D) D E) E Answer: E Diff: 2 Page Ref: 36 11) This organelle is involved in production of cellular energy. A) Golgi apparatus B) lysosome C) rough endoplasmic reticulum D) mitochondria E) peroxisome Answer: D Diff: 1 Page Ref: 32 12) This organelle is characterized by folded membranes called cristae. A) Golgi apparatus B) lysosome C) rough endoplasmic reticulum D) mitochondria E) peroxisome Answer: D Diff: 2 Page Ref: 32

4 Copyright © 2014 Pearson Education, Inc.

 

13) When a phagocytic white blood cell ingests a foreign bacterial cell, the vesicle fuses with this organelle. A) Golgi apparatus B) lysosome C) rough endoplasmic reticulum D) mitochondria E) peroxisome Answer: B Diff: 3 Page Ref: 31 14) This membranous organelle is the site of protein synthesis for proteins that are secreted by the cell. A) Golgi apparatus B) lysosome C) rough endoplasmic reticulum D) mitochondria E) peroxisome Answer: C Diff: 2 Page Ref: 29-30 15) This organelle detoxifies a number of toxic substances. A) Golgi apparatus B) lysosome C) rough endoplasmic reticulum D) mitochondria E) peroxisome Answer: E Diff: 2 Page Ref: 32 16) Cisternae of this organelle are continuous with the nuclear envelope. A) Golgi apparatus B) lysosome C) rough endoplasmic reticulum D) mitochondria E) peroxisome Answer: C Diff: 3 Page Ref: 29-30 17) This organelle has both a cis and a trans face. A) Golgi apparatus B) lysosome C) rough endoplasmic reticulum D) mitochondria E) peroxisome Answer: A Diff: 3 Page Ref: 30 5 Copyright © 2014 Pearson Education, Inc.

 

18) This membranous organelle contains oxidase and catalase enzymes. A) Golgi apparatus B) lysosome C) rough endoplasmic reticulum D) mitochondria E) peroxisome Answer: E Diff: 3 Page Ref: 32 19) These organelles are often called the "demolition crew" of the cell. A) Golgi apparatus B) lysosome C) rough endoplasmic reticulum D) mitochondria E) peroxisome Answer: B Diff: 2 Page Ref: 31 20) This organelle primarily modifies products from the rough ER, and it resembles a stack of hollow saucers, one cupped inside the next. A) Golgi apparatus B) lysosome C) rough endoplasmic reticulum D) mitochondria E) peroxisome Answer: A Diff: 2 Page Ref: 30 21) This organelle is primarily a sac of powerful digestive enzymes called acid hydrolases. A) Golgi apparatus B) lysosome C) rough endoplasmic reticulum D) mitochondria E) peroxisome Answer: B Diff: 2 Page Ref: 31 22) This organelle is defective in the inherited disorder Tay-Sachs disease. A) Golgi apparatus B) lysosome C) rough endoplasmic reticulum D) mitochondria E) peroxisome Answer: B Diff: 2 Page Ref: 31

6 Copyright © 2014 Pearson Education, Inc.

 

23) This organelle is numerous in liver and kidney cells. A) Golgi apparatus B) lysosome C) rough endoplasmic reticulum D) mitochondria E) peroxisome Answer: E Diff: 2 Page Ref: 32 24) This organelle produces ATP molecules. A) Golgi apparatus B) lysosome C) rough endoplasmic reticulum D) mitochondria E) peroxisome Answer: D Diff: 2 Page Ref: 32 25) This organelle contains a single DNA molecule and is capable of self-replication. A) Golgi apparatus B) lysosome C) rough endoplasmic reticulum D) mitochondria E) peroxisome Answer: D Diff: 2 Page Ref: 32 26) Mitosis refers only to nuclear division. Separation of the entire cell following mitosis is A) meiosis. B) karyokinesis. C) cytokinesis. D) telophase. Answer: C Diff: 2 Page Ref: 40 27) Phospholipids of the plasma membrane are arranged A) around a central layer of cholesterol. B) in a single layer with polar heads facing outwards. C) as a bilayer with their nonpolar tails sandwiched between the polar heads. D) as a bilayer with their polar heads sandwiched between the nonpolar tails. Answer: C Diff: 2 Page Ref: 26

7 Copyright © 2014 Pearson Education, Inc.

 

28) Which of the following cytoskeleton elements are the largest in diameter? A) microtubules B) microfilaments C) intermediate filaments D) centrioles Answer: A Diff: 2 Page Ref: 32-33 29) Which of the following statements about integral proteins in the plasma membrane is false? A) Most extend all the way through the membrane. B) Some attach to the glycocalyx. C) They determine which molecules are transported through the membrane. D) They are more abundant by volume than the membrane phospholipids. Answer: D Diff: 2 Page Ref: 26 30) Which type of endocytosis ingests the most specific type of molecule? A) fluid-phase endocytosis B) phagocytosis C) pinocytosis D) receptor-mediated endocytosis Answer: D Diff: 2 Page Ref: 27 31) Hormones are secreted by A) phagocytosis. B) pinocytosis. C) exocytosis. D) osmosis. Answer: C Diff: 2 Page Ref: 28 32) Of the following, the only organelle that has a double membrane structure is the A) centriole. B) Golgi apparatus. C) endoplasmic reticulum. D) mitochondrion. Answer: D Diff: 2 Page Ref: 32 33) Functions of the Golgi apparatus include all of the following except A) synthesis of lysosomes. B) DNA replication. C) plasma membrane formation. D) production of secretory granules. Answer: B Diff: 3 Page Ref: 30 8 Copyright © 2014 Pearson Education, Inc.

 

34) Which of the following statements about the rough endoplasmic reticulum is false? A) It consists of stacked envelopes called cisternae. B) It makes the digestive enzymes contained in the lysosomes. C) It stores lipids as inclusions. D) It makes the integral proteins of the cell membrane. Answer: C Diff: 2 Page Ref: 29-30 35) Which of the following is not a cytoskeleton element? A) microtubule B) microfilament C) intermediate filament D) centriole Answer: D Diff: 1 Page Ref: 32-33 36) Which type of proteins are required for exocytosis? A) caveolin B) coatomer proteins C) clathrin D) SNARE proteins Answer: D Diff: 3 Page Ref: 28 37) In chromatin, the DNA molecule wraps around proteins called A) nucleotides. B) codons. C) integral protein. D) histones. Answer: D Diff: 1 Page Ref: 36 38) In the cell life cycle, DNA is replicated during A) interphase G1. B) interphase S. C) prophase I. D) prophase II. Answer: B Diff: 2 Page Ref: 37

9 Copyright © 2014 Pearson Education, Inc.

 

39) The longest arrays of microtubules that assemble from each centrosome during prophase form filaments called A) mitotic spindle fibers. B) kinetochores. C) asters. D) the nuclear envelope. Answer: A Diff: 3 Page Ref: 38-39 40) During anaphase, motor proteins attached to mitotic spindle fibers serve to A) pull the chromosomes to opposite poles of the cell. B) pull together the replicated chromosomal strands. C) re-form the nuclear envelope. D) form the aster. Answer: A Diff: 2 Page Ref: 38-39 41) The ________ face of the Golgi apparatus is ________ to receive spherical vesicles from the rough endoplasmic reticulum. A) cis; convex B) trans; concave C) cis; flattened D) trans; convex Answer: A Diff: 3 Page Ref: 30 42) Which membranous organelle stores calcium and is a primary site of lipid metabolism? A) Golgi apparatus B) smooth endoplasmic reticulum C) mitochondrion D) peroxisome Answer: B Diff: 3 Page Ref: 30 43) Which organelle is important in neutralizing free radicals? A) Golgi apparatus B) lysosome C) mitochondrion D) peroxisome Answer: D Diff: 2 Page Ref: 32

10 Copyright © 2014 Pearson Education, Inc.

 

44) Which of the following statements accurately describes the function of the nuclear envelope? A) separation of nucleoplasm and cytoplasm B) regulation of passage of substances into and out of the cell membrane C) transcription of DNA D) protein synthesis Answer: A Diff: 3 Page Ref: 34 45) Peroxisomes function to A) synthesize and degrade hydrogen peroxide. B) store cellular free radicals. C) produce pigments. D) regulate membrane permeability. Answer: A Diff: 2 Page Ref: 32 46) Dyneins and kinesins are motor proteins that ________. A) enable a cell to send out and retract extensions called pseudopods. B) move organelles along microtubules through the cytoplasm. C) are molecular components of telomeres D) resist pulling forces that are placed on cells. Answer: B Diff: 3 Page Ref: 33 47) Cell division is analogous to A) two buildings duplicating their parts and fusing. B) a building duplicating its blueprint and then forming a new building by splitting in two. C) a building forming another building by random accumulation of materials. D) a building forming another building through a loss of some of its parts. Answer: B Diff: 2 Page Ref: 36 48) The plasma membrane is important for all the following reasons except A) it determines what substances enter and exit the cell. B) it separates the ECF from the ICF. C) it acts as a site for cell-to-cell interaction and recognition. D) it is an important site for DNA transcription. Answer: D Diff: 3 Page Ref: 24-27 49) The plasma membrane is composed of all of the following except A) glycoproteins. B) tubulin protein. C) cholesterol. D) phospholipids. Answer: B Diff: 2 Page Ref: 26 11 Copyright © 2014 Pearson Education, Inc.

 

50) Materials that are to be exocytosed by cells are enclosed in vesicles synthesized by the A) nucleosome. B) ribosome. C) Golgi apparatus. D) mitochondrion. Answer: C Diff: 2 Page Ref: 30 51) Which of the following does not pass through nuclear pores? A) chromatin B) messenger RNA C) proteins D) ribosomal RNA Answer: A Diff: 2 Page Ref: 34 52) Which of the following is associated with protein synthesis? A) mitochondria B) ribosomes C) chloroplasts D) smooth endoplasmic reticulum Answer: B Diff: 2 Page Ref: 29 53) Ribosomes may be either free within the cytoplasm or bound to a membrane system known as the A) Golgi apparatus. B) microtubule organizing center. C) cytoskeleton. D) rough endoplasmic reticulum. Answer: D Diff: 2 Page Ref: 29 54) Which is not part of interphase? A) G1 B) G2 C) M D) S Answer: C Diff: 1 Page Ref: 37

12 Copyright © 2014 Pearson Education, Inc.

 

55) Embedded in the plasma membrane of cells, cholesterol molecules act to A) stabilize the membrane. B) make the membrane more resistant to freezing. C) destabilize the membrane, leading to heart attacks. D) participate in pinocytosis. Answer: A Diff: 2 Page Ref: 26 56) The endocytotic process in which small vesicles of fluid are brought into the cell is called A) phagocytosis. B) pinocytosis. C) exocytosis. D) xenocytosis. Answer: B Diff: 2 Page Ref: 27 57) The double membrane structure is unique to the A) lysosome. B) peroxisome. C) mitochondrion. D) nucleolus. Answer: C Diff: 2 Page Ref: 32 58) Peroxisomes A) are the toxic waste removal system of the cell. B) are involved in the production of ATP. C) contain some of the code necessary for their own duplication. D) synthesize proteins for use outside the cell. Answer: A Diff: 2 Page Ref: 32 59) The stiffest elements of the cytoskeleton, analogous to the bones of the human body, are A) microtubules. B) microfilaments. C) intermediate filaments. D) the cytosol. Answer: A Diff: 2 Page Ref: 32-33 60) The mitotic spindle forms from the A) nucleus. B) Golgi apparatus. C) centrosome matrix. D) nucleolus. Answer: C Diff: 2 Page Ref: 33 13 Copyright © 2014 Pearson Education, Inc.

 

61) The nuclear envelope is continuous with the rough ER, but it differs from the rough ER in that it A) is not associated with ribosomes. B) has unique pores. C) consists of two membranes separated by a space. D) consists of tubes, like the smooth ER. Answer: B Diff: 3 Page Ref: 34 62) Membrane-bound organelles have the same type of membrane as the plasma membrane except A) for the absence of a glycocalyx. B) for the absence of cholesterol. C) the nonpolar tails face outward. D) they are all covered with ribosomes. Answer: A Diff: 3 Page Ref: 29 63) In the process of phagocytosis, the organelles whose enzymes break down ingested foreign cells are the A) nucleoli. B) smooth endoplasmic reticulum. C) peroxisomes. D) lysosomes. Answer: D Diff: 2 Page Ref: 27 64) During mitosis, the kinetochore microtubules of the mitotic spindle A) attach to chromatids and align them at the metaphase plate. B) push on the chromatids. C) anchor the centriole to the cell membrane. D) push the two poles of the cell apart. Answer: A Diff: 2 Page Ref: 38-39 65) The theory proposing that aging results from the effects of free radicals is primarily a theory of A) wear and tear. B) genetically programmed aging. C) progressive disorder of immunity. D) cross-linking of glucose. Answer: A Diff: 2 Page Ref: 41

14 Copyright © 2014 Pearson Education, Inc.

 

66) The cytoskeletal elements that are analogous to the muscles of the body which generate pseudopodia and contractile forces in conjunction with myosin are A) microtubules. B) microfilaments. C) intermediate filaments. D) integral proteins. Answer: B Diff: 2 Page Ref: 32-33 67) Transcription of DNA requires the presence of A) centrosomes. B) extended chromatin. C) histones. D) nucleosomes. Answer: B Diff: 2 Page Ref: 36 68) The process of cellular aging may involve all of the following except A) accumulated damage by free radicals. B) decreased production of lysosomes. C) excessive metabolic rate. D) progressive shortening of telomeres. Answer: B Diff: 3 Page Ref: 41 69) During what phase of mitosis does the mitotic spindle break down and disappear? A) metaphase B) anaphase C) telophase D) late prophase Answer: C Diff: 2 Page Ref: 38-39 70) The cytoskeletal elements that form a ring to "squeeze" the two daughter cells apart during cytokinesis are A) microtubules. B) microfilaments. C) intermediate filaments. D) the microtrabecular lattice. Answer: B Diff: 2 Page Ref: 40

15 Copyright © 2014 Pearson Education, Inc.

 

71) During what phase of the cell cycle is the DNA duplicated? A) metaphase B) anaphase C) interphase D) prophase Answer: C Diff: 2 Page Ref: 37 72) The plasma membrane is A) a single-layered membrane that surrounds the nucleus of the cell. B) a single-layered membrane enclosing the plasma. C) the membrane surrounding the cell. D) a membrane composed of tiny shelves or cristae. Answer: C Diff: 2 Page Ref: 24-27 73) The cell that gathers information and controls body functions is a A) macrophage. B) fat cell. C) sperm cell. D) neuron. Answer: D Diff: 2 Page Ref: 40-41 74) The temporary structures in the cytoplasm include all of the following except A) pigments. B) glycosomes. C) lipid droplets. D) the Golgi apparatus. Answer: D Diff: 2 Page Ref: 33 75) Which of the following is an inclusion, not an organelle? A) lysosome B) microtubule C) mitochondrion D) glycosome Answer: D Diff: 2 Page Ref: 33 2.2 True/False Questions 1) The smooth ER contains its own molecules of DNA. Answer: FALSE Diff: 3 Page Ref: 30

16 Copyright © 2014 Pearson Education, Inc.

 

2) Hypercholesterolemia is an inherited disease in which the body's cells lack the protein receptors that bind to cholesterol-delivering LDLs. Answer: TRUE Diff: 3 Page Ref: 28 3) Ribosomes consist of two subunits, each surrounded by a membrane. Answer: FALSE Diff: 2 Page Ref: 29 4) Peroxisomes are important in detoxification of a number of toxic substances, for instance, hydrogen peroxide. Answer: TRUE Diff: 2 Page Ref: 32 5) The nucleolus serves as the cell's ribosome-producing machine. Answer: TRUE Diff: 2 Page Ref: 34-35 6) Microtubules are composed of actin. Answer: FALSE Diff: 2 Page Ref: 33 7) Chromatin is composed of DNA wound around proteins known as actin. Answer: FALSE Diff: 2 Page Ref: 36 8) An example of a type of cell with high rates of mitosis is a cell of the skin. Answer: TRUE Diff: 2 Page Ref: 37 9) During the S phase, cells are characterized by rapid growth. Answer: FALSE Diff: 2 Page Ref: 37 10) During the G1 phase, DNA is replicated in the cytoplasm. Answer: FALSE Diff: 2 Page Ref: 37 11) Telomeres are structures that limit the maximum number of times cells can divide. Answer: TRUE Diff: 2 Page Ref: 41 12) Extended chromatin is tightly wound around histones. Answer: FALSE Diff: 2 Page Ref: 36

17 Copyright © 2014 Pearson Education, Inc.

 

13) A mitotic spindle develops during early telophase of mitosis. Answer: FALSE Diff: 2 Page Ref: 38-39 14) During anaphase, the chromosomes are moved to the center of the cell. Answer: FALSE Diff: 2 Page Ref: 38-39 15) Cytokinesis is the physical division of the cytoplasm between the two newly formed cells that result from mitosis. Answer: TRUE Diff: 2 Page Ref: 40 2.3 Short Answer Questions 1) This phase is the physical division of the cytoplasm between the two newly formed cells that result from mitosis. Answer: cytokinesis Diff: 2 Page Ref: 40 2) What is the transport mechanism by which substances move from the cytoplasm to the outside of the cell? Answer: exocytosis Diff: 3 Page Ref: 28 3) Cell aging may be related to production of what charged molecules produced by the mitochondria? Answer: radicals (free radicals) Diff: 2 Page Ref: 41 4) Identify the two different types of membrane-associated molecules that comprise the glycocalyx. Answer: glycolipids and glycoproteins Diff: 2 Page Ref: 26 5) What would extended chromatin wrapped around a group of eight histones be called? Answer: a nucleosome Diff: 2 Page Ref: 36 6) This is the phase in which a cell grows and carries on all its usual metabolic activities. Answer: G1 phase of interphase Diff: 1 Page Ref: 37 7) These are the smallest living units in the body. Answer: cells Diff: 2 Page Ref: 22 18 Copyright © 2014 Pearson Education, Inc.

 

8) This is the outermost continuous boundary of a human cell. Answer: plasma membrane (plasmalemma) Diff: 1 Page Ref: 24-27 9) This is the name for the currently held theory describing the plasma membrane structure. Answer: fluid mosaic model Diff: 2 Page Ref: 26 10) The phospholipid molecules of the plasma membrane are primarily composed of ________. Answer: a non-polar tail comprised of 2 fatty acid chains attached to a polar head Diff: 2 Page Ref: 26 11) This network of rods running throughout the cytosol acts as a cell's "bones," "muscles," and "ligaments." Answer: cytoskeleton Diff: 2 Page Ref: 32-33 12) This is the mechanism by which large particles and macromolecules enter a cell. Answer: endocytosis Diff: 2 Page Ref: 27 13) This is the diffusion of water molecules across a membrane. Answer: osmosis Diff: 3 Page Ref: 27 14) This is the type of protein involved in transport mechanisms across the plasma membrane. Answer: integral proteins (transmembrane proteins) Diff: 3 Page Ref: 26 15) This is an inherited disease that leads to an accumulation of undigested glycolipids especially in the lysosomes of neurons. Answer: Tay-Sachs disease Diff: 2 Page Ref: 31 2.4 Essay Questions 1) Differentiate phagocytosis from receptor-mediated endocytosis. Answer: In phagocytosis, the cell extends pseudopods and engulfs the foreign protein/foreign cell, which is often degraded after the phagocytic vesicle fuses with a lysosome. In receptormediated endocytosis, specific membrane receptors bind specific extra-cellular molecules. Once bound, the membrane deforms inward, creating a vesicle with the receptors and molecules inside. The vesicle contents are released into the cytoplasm or fuse with a lysosome, with the receptors recycled back to the membrane. Diff: 3 Page Ref: 27-28

19 Copyright © 2014 Pearson Education, Inc.

 

2) Describe how cellular differentiation results in structural variation among cells in the human body. Answer: Cellular differentiation is the result of highly regulated gene activation/inactivation in the developing embryo. The products of gene activation are proteins. As the embryo develops, certain cells will begin to produce proteins that neighboring cells do not produce. As development progresses, these unique protein "signatures" lead to differences in cellular function. For example, in muscle cells actin and myosin proteins predominate which results in their unique contractile properties. Diff: 3 Page Ref: 40-41 3) Describe the two checkpoints that occur during interphase. Answer: The G1 checkpoint ensures that the cell has reached a maximum size and has replicated the necessary organelles and enzymes to synthesize DNA. The G2 checkpoint, checks to see whether replication errors or DNA damage has occurred during DNA synthesis. Diff: 3 Page Ref: 37 4) Describe the mitochondria. Answer: These are long, thin organelles, that have their own DNA molecule which allows for self-replication. They produce ATP molecules, which are the equivalent of cellular energy. They are bound by two membranes. The inner one is highly folded into cristae, where many of the critical molecules involved in ATP production are imbedded. Diff: 2 Page Ref: 32 5) Describe the three major types of cytoskeletal elements. Answer: Microtubules are the largest in diameter and are formed by the protein tubulin. They are stiff, but bendable. Microtubules are important in the trafficking of organelles within the cytoplasm. Microfilaments are the smallest in diameter. They are strands of the protein actin, are contractile proteins, which are typically very labile. Intermediate filaments are of intermediate diameter. They are very stabile and permanent, functioning to resist shearing forces within and between adjacent cells. Diff: 2 Page Ref: 32-33

20 Copyright © 2014 Pearson Education, Inc.

 

Human Anatomy, 7e (Marieb/Mitchell/Smith) Chapter 3 Basic Embryology 3.1 Multiple Choice Questions

Figure 3.1 Use the diagram above to answer the following questions. 1) Which letter indicates the blastocyst? A) A B) B C) C D) D E) E Answer: D Diff: 2 Page Ref: 49 2) Which letter indicates the uterus? A) A B) B C) C D) D E) E Answer: E Diff: 2 Page Ref: 49

1 Copyright © 2014 Pearson Education, Inc.

 

3) Which letter indicates the oocyte? A) A B) B C) C D) D E) E Answer: A Diff: 2 Page Ref: 49 4) Which letter indicates the zygote? A) A B) B C) C D) D E) E Answer: B Diff: 2 Page Ref: 49 5) Which letter indicates the morula? A) A B) B C) C D) D E) E Answer: C Diff: 2 Page Ref: 49 6) Quickening occurs (mother feels fetus moving). A) 8 weeks (end of embryonic period) B) 9-12 weeks (month 3) C) 13-16 weeks (month 4) D) 17-20 weeks (month 5) E) 21-30 weeks (months 6 and 7) Answer: D Diff: 2 Page Ref: 59 7) Limbs are complete. A) 8 weeks (end of embryonic period) B) 9-12 weeks (month 3) C) 13-16 weeks (month 4) D) 17-20 weeks (month 5) E) 21-30 weeks (months 6 and 7) Answer: A Diff: 2 Page Ref: 58

2 Copyright © 2014 Pearson Education, Inc.

 

8) Sex can be determined from the genitals. A) 8 weeks (end of embryonic period) B) 9-12 weeks (month 3) C) 13-16 weeks (month 4) D) 17-20 weeks (month 5) E) 21-30 weeks (months 6 and 7) Answer: B Diff: 2 Page Ref: 58 9) Eyes open and lungs develop. A) 8 weeks (end of embryonic period) B) 9-12 weeks (month 3) C) 13-16 weeks (month 4) D) 17-20 weeks (month 5) E) 21-30 weeks (months 6 and 7) Answer: E Diff: 2 Page Ref: 59 10) All major organs are present in rudimentary form. A) 8 weeks (end of embryonic period) B) 9-12 weeks (month 3) C) 13-16 weeks (month 4) D) 17-20 weeks (month 5) E) 21-30 weeks (months 6 and 7) Answer: A Diff: 2 Page Ref: 58 11) This develops into the brain and spinal cord. A) endoderm B) ectoderm C) mesoderm—somites D) mesoderm—lateral plate E) intermediate mesoderm Answer: B Diff: 2 Page Ref: 57 12) This develops into the lining of the gastrointestinal tract. A) endoderm B) ectoderm C) mesoderm—somites D) mesoderm—lateral plate E) intermediate mesoderm Answer: A Diff: 2 Page Ref: 57

3 Copyright © 2014 Pearson Education, Inc.

 

13) This develops into the epidermis of the skin. A) endoderm B) ectoderm C) mesoderm—somites D) mesoderm—lateral plate E) intermediate mesoderm Answer: B Diff: 2 Page Ref: 57 14) The epiblast develops a primitive streak during days 14-15, the initial cells that ingress through the primitive streak become this germ layer. A) endoderm B) ectoderm C) mesoderm—somites D) mesoderm—lateral plate E) intermediate mesoderm Answer: A Diff: 3 Page Ref: 50 15) Mucus-producing glands of the respiratory tract arise from this layer. A) endoderm B) ectoderm C) mesoderm—somites D) mesoderm—lateral plate E) intermediate mesoderm Answer: A Diff: 3 Page Ref: 57 16) What embryonic epiblast cells that remain on the surface of the trilaminar disc are ultimately called. A) endoderm B) ectoderm C) mesoderm—somites D) mesoderm—lateral plate E) intermediate mesoderm Answer: B Diff: 3 Page Ref: 50-51 17) The neural tube develops from this layer. A) endoderm B) ectoderm C) mesoderm—somites D) mesoderm—lateral plate E) intermediate mesoderm Answer: B Diff: 2 Page Ref: 52 4 Copyright © 2014 Pearson Education, Inc.

 

18) The mesoderm located deep to the neural tube forms the kidneys and the gonads is the ________. A) endoderm B) ectoderm C) mesoderm—somites D) mesoderm—lateral plate E) intermediate mesoderm Answer: E Diff: 3 Page Ref: 57 19) The ribs and vertebrae form from this primary germ layer. A) endoderm B) ectoderm C) mesoderm—somites D) mesoderm—lateral plate E) intermediate mesoderm Answer: C Diff: 3 Page Ref: 57 20) The ectoderm develops into the epithelium of the skin, whereas this layer gives rise to the epithelial lining of the gut tube. A) endoderm B) ectoderm C) mesoderm—somites D) mesoderm—lateral plate E) intermediate mesoderm Answer: A Diff: 3 Page Ref: 57 21) This layer will give rise to the bones, ligaments and joints of the embryonic limbs. A) endoderm B) ectoderm C) mesoderm—somites D) mesoderm—lateral plate E) intermediate mesoderm Answer: D Diff: 2 Page Ref: 57 22) Neural crest cells are derived from this germ layer. A) endoderm B) ectoderm C) mesoderm—somites D) mesoderm—lateral plate E) intermediate mesoderm Answer: B Diff: 2 Page Ref: 57 5 Copyright © 2014 Pearson Education, Inc.

 

23) The dermis of the dorsal body region is derived from somites. However, the dermis of the ventral body region is derived from A) endoderm. B) ectoderm. C) mesoderm—somites. D) mesoderm—lateral plate. E) intermediate mesoderm. Answer: D Diff: 2 Page Ref: 57 24) Blood vessels and the heart arise from this primary germ layer. A) endoderm B) ectoderm C) mesoderm—somites D) mesoderm—lateral plate E) intermediate mesoderm Answer: D Diff: 2 Page Ref: 57 25) The splanchnic mesoderm forms next to this layer. A) endoderm B) ectoderm C) mesoderm—somites D) mesoderm—lateral plate E) intermediate mesoderm Answer: A Diff: 2 Page Ref: 57 26) Conception occurs in the A) lateral third of the uterine tube. B) ovary. C) peritoneal cavity. D) uterine cavity. Answer: A Diff: 2 Page Ref: 48 27) The middlemost embryonic germ layer to develop is A) ectoderm. B) endoderm. C) mesoderm. D) protoderm. Answer: C Diff: 2 Page Ref: 50-51

6 Copyright © 2014 Pearson Education, Inc.

 

28) The embryonic notochord will eventually be replaced by the developing ________, leaving only remnants that persist as the nucleus pulposus of intervertebral discs. A) spinal cord B) pharynx C) lateral muscle D) vertebral column Answer: D Diff: 2 Page Ref: 50, 55 29) Which of the following is the proper sequence for neurulation? A) neural tube, neural plate, neural groove B) neural groove, neural plate, neural tube C) neural plate, neural groove, neural tube D) neural groove, brain, neural tube Answer: C Diff: 2 Page Ref: 52-53 30) Limb buds emerge in the human embryo about A) day 9. B) day 15. C) day 28. D) day 60. Answer: C Diff: 2 Page Ref: 56 31) Based on the proportions of the adult body, the 3-month fetus still shows disproportionately large A) hands. B) stomach. C) head. D) heart. Answer: C Diff: 2 Page Ref: 58 32) Division of cells in a zygote is called A) blastulation. B) cleavage. C) gastrulation. D) induction. Answer: B Diff: 2 Page Ref: 48

7 Copyright © 2014 Pearson Education, Inc.

 

33) The gonads arise from what embryonic structure? A) notochord B) endoderm C) splanchnic mesoderm D) intermediate mesoderm Answer: D Diff: 2 Page Ref: 57 34) Which of the following is endodermal in origin? A) liver B) spleen C) blood vessels D) skin Answer: A Diff: 2 Page Ref: 55 35) Somites differentiate into A) the epidermis. B) myotomes. C) the gastrointestinal lining. D) muscles. Answer: B Diff: 3 Page Ref: 55 36) The sclerotome will develop into the A) peritoneum. B) visceral serosa. C) vertebrae. D) heart. Answer: C Diff: 2 Page Ref: 55 37) Splanchnic mesoderm forms the A) heart. B) notochord. C) brain. D) skin. Answer: A Diff: 2 Page Ref: 57 38) Approximately ________ percent of all children have some birth defect. A) 0.01 B) 1 C) 3 D) 20 Answer: C Diff: 2 Page Ref: 60 8 Copyright © 2014 Pearson Education, Inc.

 

39) A transverse section through the 24-day embryo, dorsal to ventral, would first contact the A) neural tube. B) notochord. C) somites. D) myotomes. Answer: A Diff: 2 Page Ref: 55 40) The embryonic layer that invades the lining of the uterus and forms the placenta is the A) trophoblast. B) embryoblast. C) inner cell mass. D) endometrium. Answer: A Diff: 2 Page Ref: 48 41) The expression "breaking water" during birthing refers to the A) amnion. B) allantois. C) chorion. D) yolk sac. Answer: A Diff: 2 Page Ref: 50 42) Which characteristic is not true of a blastocyst? A) It contains a fluid-filled cavity. B) It develops 24-36 hours after conception. C) It has an inner cell mass and an outer trophoblast. D) It implants into the uterus. Answer: B Diff: 3 Page Ref: 48 43) The solid mass of cells that results from cleavage of the zygote is the A) trophoblast. B) blastocyst. C) morula. D) gastrula. Answer: C Diff: 2 Page Ref: 48

9 Copyright © 2014 Pearson Education, Inc.

 

44) Which of the following does not distinguish mesoderm from either endoderm or ectoderm? A) It is a mesenchyme tissue. B) It is derived from the epiblast. C) It is the last of the three germ layers to develop. D) Its cells can migrate widely within the embryo. Answer: B Diff: 3 Page Ref: 50 45) The term bilaminar disc refers to the embryonic ________ and ________. A) epidermis; dermis B) ectoderm; mesoderm C) epiblast; hypoblast D) inner cell mass; trophoblast Answer: C Diff: 2 Page Ref: 50 46) Mesenchyme would refer to ________ but not ________. A) mesoderm; ectoderm B) ectoderm; mesoderm C) endoderm; ectoderm D) ectoderm; endoderm Answer: A Diff: 2 Page Ref: 51 47) Male and female fetuses can first be distinguished by their genitalia at A) 1 week. B) 3 weeks. C) 3 months. D) 7 months. Answer: C Diff: 2 Page Ref: 58 48) Thalidomide, once used to alleviate morning sickness in pregnant women, resulted in A) severe limb abnormalities. B) heart defects. C) mental retardation. D) multiple births. Answer: A Diff: 2 Page Ref: 60 49) Chemical, physical, or biological agents that can induce birth defects are called A) mutagens. B) teratogens. C) free radicals. D) carcinogens. Answer: B Diff: 2 Page Ref: 60 10 Copyright © 2014 Pearson Education, Inc.

 

50) The most common cause of mental retardation in the United States is A) anencephaly. B) spina bifida. C) fetal alcohol syndrome. D) thalidomide. Answer: C Diff: 2 Page Ref: 60 51) Implantation of the blastocyst on the uterine lining takes about A) 1 week. B) 2 weeks. C) 3 weeks. D) 4 weeks. Answer: A Diff: 2 Page Ref: 49 52) Fertilization is to zygote as A) egg is to sperm. B) oocyte is to ovary. C) cleavage is to morula. D) birth is to parturition. Answer: C Diff: 3 Page Ref: 48 53) In human embryos, the yolk sac is important because it A) stores yolk. B) gives rise to earliest blood cells and blood vessels. C) stores nitrogenous wastes. D) allows for gas exchange through the placenta. Answer: B Diff: 2 Page Ref: 50 54) The body's axis of development is determined by the A) blastomeres. B) epiblast. C) notochord. D) primitive streak. Answer: C Diff: 2 Page Ref: 51

11 Copyright © 2014 Pearson Education, Inc.

 

55) Most organ systems are fully formed and ready to function in the fetus by the sixth month. Which of the following systems is the exception to this generalization because it takes longer to finish development? A) the circulatory system B) the respiratory system C) the urinary system D) the integumentary system Answer: B Diff: 3 Page Ref: 59 56) The urinary bladder is derived from which embryonic layer? A) ectoderm B) mesoderm C) endoderm D) neural crest Answer: C Diff: 2 Page Ref: 57 57) A teratogen is A) a deformed baby. B) a specific sedative used in the 1950s in Europe. C) a chemical agent, infectious agent or environmental factor that causes birth defects. D) a stage of development in the mid-fetal period. Answer: C Diff: 2 Page Ref: 60 58) The primary germ layer that ultimately produces the hair, fingernails, and toenails is A) ectoderm. B) mesoderm. C) endoderm. D) neural crest. Answer: A Diff: 2 Page Ref: 55, 57 59) The primary germ layer that ultimately forms the serous membranes of the peritoneum is A) ectoderm. B) mesoderm. C) endoderm. D) neural crest. Answer: B Diff: 2 Page Ref: 56

12 Copyright © 2014 Pearson Education, Inc.

 

60) The neural tube of the embryo develops into the A) spinal cord. B) neural crest. C) sensory neurons. D) dorsal skin. Answer: A Diff: 1 Page Ref: 52 61) Somatic mesoderm gives rise to all of the following except A) some bones. B) some serous membranes. C) dermis of the skin of the belly. D) epidermis of the upper limb. Answer: D Diff: 3 Page Ref: 54 62) Production of surfactant begins during week 22-26 of the gestational period. Premature infants born after week 26 will A) not survive. B) will not need medical assistance for breathing. C) will need medical assistance for breathing. D) will be at increased risk for visual disorders and mental impairment. Answer: B Diff: 2 Page Ref: 59 63) Another name for birth defect is A) birth illness. B) teratogenesis. C) congenital abnormality. D) prenatal pathology. Answer: C Diff: 2 Page Ref: 60 64) Which of the following adult structures derives from neural crest? A) all the nerve cells in the brain B) all the nerve cells in the spinal cord C) all the pigment-producing cells in the body D) all bones Answer: C Diff: 2 Page Ref: 55

13 Copyright © 2014 Pearson Education, Inc.

 

65) As a result of folding, the embryo acquires a tadpole shape by A) day 9. B) day 12. C) day 15. D) day 24. Answer: D Diff: 3 Page Ref: 54 66) The amniotic sac is derived from the epiblast, while the placenta develops from the A) bilaminar disc. B) hypoblast. C) neural crest. D) trophoblast. Answer: D Diff: 2 Page Ref: 48 67) Identical (monozygotic) twins are A) formed from splitting of the inner cell mass. B) formed from splitting of the zygote. C) formed from the release of two eggs. D) formed from two sperm fertilizing one egg. Answer: A Diff: 2 Page Ref: 49 68) If a mutation occurs in the hypoblast cells of a bilaminar embryo, one might expect birth defects to be present in the A) blood cells or vessels. B) brain and spinal cord. C) hair and skin. D) musculoskeletal system. Answer: A Diff: 3 Page Ref: 50 69) Which of the following is not a property of mesenchyme? A) It is present before birth. B) Its cells migrate. C) Its cells are star-shaped. D) Its cells attach closely to one another and form sheets. Answer: D Diff: 2 Page Ref: 51

14 Copyright © 2014 Pearson Education, Inc.

 

70) The ability of one cell to influence the development of its neighboring cells is called A) blastulation. B) cleavage. C) gastrulation. D) induction. Answer: D Diff: 2 Page Ref: 53 71) Which of the following appears latest in human development? A) limb buds B) somites C) heart D) separate fingers Answer: D Diff: 2 Page Ref: 58-59 72) Conjoined twins occur as a result of A) complete separation of cells in the early morula. B) failure of the blastocyst to implant in the uterine lining. C) incomplete division of the inner cell mass. D) fertilization of the ovum by two different sperm Answer: C Diff: 2 Page Ref: 49 73) The cavity in the mesoderm that ultimately develops into the peritoneal, pericardial, and pleural cavities is called: A) an antrum. B) a blastocoel. C) a coelom. D) a dermatome. Answer: C Diff: 2 Page Ref: 54 74) Which of the following is not a germ layer? A) ectoderm B) mesoderm C) epidermis D) endoderm Answer: C Diff: 2 Page Ref: 50-51

15 Copyright © 2014 Pearson Education, Inc.

 

75) The correct sequence for embryonic development is A) zygote, blastocyst, morula. B) zygote, morula, blastocyst. C) blastocyst, morula, zygote. D) zygote, fetus, embryo. Answer: B Diff: 2 Page Ref: 48-49 3.2 True/False Questions 1) Spina bifida results when the neural tube fails to close in the head. Answer: FALSE Diff: 2 Page Ref: 53 2) The fetal period is the longer and later of the prenatal growth periods. Answer: TRUE Diff: 2 Page Ref: 47 3) The inner cell mass will form the embryo, and the trophoblast will form the placenta. Answer: TRUE Diff: 2 Page Ref: 48 4) The outer membrane of the amniotic sac is called the amnion. Answer: TRUE Diff: 2 Page Ref: 50 5) The epiblast gives rise to the three primary germ layers. Answer: TRUE Diff: 2 Page Ref: 50 6) The developing embryo, about 72 hours after fertilization, is called the zygote. Answer: FALSE Diff: 2 Page Ref: 48 7) During the blastocyst stage, there are three cell layers evident. Answer: FALSE Diff: 2 Page Ref: 48 8) It is easy to remember that in humans, 3 weeks after fertilization, there are three germ layers. Answer: TRUE Diff: 1 Page Ref: 50-51 9) The three primary germ layers are the endoderm, myotome, and exoderm. Answer: FALSE Diff: 1 Page Ref: 50-51

16 Copyright © 2014 Pearson Education, Inc.

 

10) The yolk sac, formed from the hypoblast, has little yolk in the human embryo. Answer: TRUE Diff: 2 Page Ref: 50 11) Premature birth is one that occurs 38 weeks after conception. Answer: FALSE Diff: 3 Page Ref: 60 12) Endoderm forms many glands, such as those associated with the gastrointestinal tract. Answer: TRUE Diff: 2 Page Ref: 57 13) The embryonic notochord gives rise to part of the intervertebral discs in the adult human. Answer: TRUE Diff: 2 Page Ref: 55 14) Identical twins arise from the splitting of a three-layered embryo. Answer: FALSE Diff: 3 Page Ref: 49 15) The ability of one group of cells to influence development of neighboring cells is called gastrulation. Answer: FALSE Diff: 2 Page Ref: 53 3.3 Short Answer Questions 1) The embryonic ________ ultimately form the arms and legs of the adult. Answer: limb buds Diff: 2 Page Ref: 56 2) By the end of week 3, the mesoderm has divided into the somites, intermediate mesoderm, and ________. Answer: lateral plate Diff: 2 Page Ref: 54 3) Splanchnic mesoderm gives rise to components of the cardiovascular system, including ________. Answer: the heart and blood vessels Diff: 2 Page Ref: 57 4) Somites differentiate into dermatomes, myotomes, and ________. Answer: sclerotomes Diff: 2 Page Ref: 55

17 Copyright © 2014 Pearson Education, Inc.

 

5) The sex of the fetus can be determined externally by approximately month ________. Answer: 3 Diff: 2 Page Ref: 58 6) The hand develops with ________ between the fingers. Answer: webs (webbing) Diff: 2 Page Ref: 58 7) This is the most common preventable cause of mental retardation in the United States. Answer: fetal alcohol syndrome Diff: 2 Page Ref: 60 8) Defects in the development of the neural tube may be linked to inadequate dietary intake of ________. Answer: folic acid Diff: 2 Page Ref: 53-54 9) ________ is a clinical procedure that provides a genetic profile for a fetus. Answer: Amniocentesis Diff: 2 Page Ref: 61 10) A ________ abortion is one in which the fetus dies and is naturally aborted. Answer: spontaneous Diff: 2 Page Ref: 61 11) The wall and lining of the gut develop from the mesoderm and ________, respectively. Answer: endoderm Diff: 2 Page Ref: 57 12) Name three primary germ layers in the embryo. Answer: endoderm, ectoderm, and mesoderm Diff: 2 Page Ref: 51-52 13) The ________ is the embryonic stage that implants in the uterus. Answer: blastocyst Diff: 2 Page Ref: 48 14) A primitive characteristic that an adult human retains only in part, but is important in determining bilateral symmetry is the ________. Answer: notochord Diff: 3 Page Ref: 51 15) The intermediate mesoderm forms the ________. Answer: kidneys and gonads Diff: 2 Page Ref: 57

18 Copyright © 2014 Pearson Education, Inc.

 

3.4 Essay Questions 1) Explain how conjoined twins may occur. Answer: Conjoined twins result with the incomplete division of the inner cell mass during the twinning process. The twins may be joined at any/multiple body regions and often share organs. Diff: 2 Page Ref: 49 2) Briefly describe the two cellular layers that comprise the bilaminar embryonic disc and explain their importance in embryonic development. Answer: Cells within the inner cell mass of the blastocyst begin to differentiate into two separate layers approximately nine days after fertilization occurs. These two layers will give rise to all of the cell types of the body. The epiblast will give rise to the three primary germ layers; ectoderm, mesoderm and endoderm. The hypoblast gives rise to the yolk sac, which will develop into part of the digestive tube and tissue around the yolk sac gives rise to the earliest blood cells and blood vessels. Diff: 2 Page Ref: 50 3) Describe the types of tissues that make up the endoderm, ectoderm, and mesoderm, and explain how these tissues relate to their developmental processes. Answer: Both endoderm and ectoderm are epithelial tissues. These cells are joined together in sheets that form external or internal linings of the body. The mesoderm consists of mesenchyme tissue whose cells don't stick together. Instead, the cells and groups of cells are free to migrate within the embryo to eventually give rise to muscles, bones, and viscera. Diff: 3 Page Ref: 50-51 4) Define the term teratogen, and explain why exposure to teratogens during the first 8 weeks of pregnancy is particularly dangerous. Answer: A teratogen is any chemical, biological, or physical factor that disrupts normal embryo development. Exposure to teratogenic agents is most dangerous during the first 8 weeks of embryo development because it is during this time that most of the major organ systems initially develop. Disruption to these early cell lines may either be inconsistent with life and cause spontaneous abortion or may cause significant malformations that will persist throughout the fetal period. Diff: 2 Page Ref: 60 5) Briefly describe the formation of the nervous system. Answer: The nervous system develops by an infolding of ectodermal cells from the neural plate. These cells continue to fold toward each other, forming initially the neural groove, which then becomes the completed neural tube. The cranial region of the neural tube becomes the brain, the caudal portion becomes the spinal cord. Just lateral to the developing neural groove, on either side, a ridge begins to form called the neural crest, again formed by ectodermal cells. These lateral ridges fuse; forming a superficial layer of ectoderm, surrounding the hollow, deeper neural tube Migratory neural crest cell derivatives will later become sensory nerve cells, pigment-producing cells, and bone-forming cells of the face. Diff: 2 Page Ref: 52-53, 57

19 Copyright © 2014 Pearson Education, Inc.

 

Human Anatomy, 7e (Marieb/Mitchell/Smith) Chapter 4 Tissues 4.1 Multiple Choice Questions

Figure 4.1 Use the diagram above to answer the following questions. 1) Which letter indicates microvilli? A) A B) B C) C D) D E) E Answer: E Diff: 2 Page Ref: 65 2) Which letter indicates the connective tissue layer? A) A B) B C) C D) D E) E Answer: C Diff: 2 Page Ref: 65 1 Copyright © 2014 Pearson Education, Inc.

 

3) Which letter indicates the epithelium? A) A B) B C) C D) D E) E Answer: B Diff: 2 Page Ref: 65 4) Which letter indicates the basement membrane? A) A B) B C) C D) D E) E Answer: D Diff: 2 Page Ref: 65 5) Which letter on the diagram indicates motile structural elements of the epithelial membrane? A) A B) B C) C D) D E) E Answer: A Diff: 2 Page Ref: 65

2 Copyright © 2014 Pearson Education, Inc.

 

Figure 4.2 Use the diagram above to answer the following questions. 6) Which letter indicates a cellular junction that prevents molecules from passing between epithelial cells? A) A B) B C) C D) D E) E Answer: A Diff: 2 Page Ref: 74-76 7) Which letter indicates the acellular layer that acts as a selective filter and a scaffold for regenerating epithelial cells? A) A B) B C) C D) D E) E Answer: C Diff: 2 Page Ref: 74-76 8) Which letter indicates tunnel-like junctions in the lateral membranes of adjacent epithelial cells? A) A B) B C) C D) D E) E Answer: D Diff: 1 Page Ref: 74-76

3 Copyright © 2014 Pearson Education, Inc.

 

9) Which letter indicates junctions commonly associated with tissues under mechanical stress? A) A B) B C) C D) D E) E Answer: B Diff: 2 Page Ref: 74-76 10) Which letter indicates the cytoplasmic protein fiber that connects desmosomes? A) A B) B C) C D) D E) E Answer: E Diff: 3 Page Ref: 74-76 11) Which category of tissue is characterized by cells that have adhesion proteins and specialized cell-to-cell junctions? A) muscle B) nervous C) connective D) epithelial Answer: D Diff: 3 Page Ref: 74 12) Which major category of tissues has the most diverse and most numerous types of tissue? A) muscle B) nervous C) connective D) epithelial Answer: C Diff: 2 Page Ref: 77 13) Histologists consider blood to be an example of what primary tissue category? A) muscle B) nervous C) connective D) epithelial Answer: C Diff: 2 Page Ref: 77

4 Copyright © 2014 Pearson Education, Inc.

 

14) Cells lining the digestive tube are characteristic of this tissue category. A) muscle B) nervous C) connective D) epithelial Answer: D Diff: 1 Page Ref: 67 15) Pseudostratified columnar is one of the tissues of this category. A) muscle B) nervous C) connective D) epithelial Answer: D Diff: 2 Page Ref: 67 16) Which major tissue category includes tissues that function in secretion, absorption, and filtration? A) muscle B) nervous C) connective D) epithelial Answer: D Diff: 2 Page Ref: 65-66 17) Bone and cartilage are examples of tissues in this category. A) muscle B) nervous C) connective D) epithelial Answer: C Diff: 1 Page Ref: 77 18) Which tissue category contains tissues comprised of cells containing myofilaments? A) muscle B) nervous C) connective D) epithelial Answer: A Diff: 2 Page Ref: 93

5 Copyright © 2014 Pearson Education, Inc.

 

19) The primary contractile tissue of the heart characterizes tissues in this category. A) muscle B) nervous C) connective D) epithelial Answer: A Diff: 1 Page Ref: 93 20) This tissue type, in addition to certain muscle tissues, is capable of generating electrical impulses. A) muscle B) nervous C) connective D) epithelial Answer: B Diff: 2 Page Ref: 95 21) Ligaments are examples of tissues in this category. A) muscle B) nervous C) connective D) epithelial Answer: C Diff: 2 Page Ref: 86 22) The brain and spinal cord are composed primarily of this tissue type. A) muscle B) nervous C) connective D) epithelial Answer: B Diff: 2 Page Ref: 95 23) This type of tissue includes smooth and cardiac. A) muscle B) nervous C) connective D) epithelial Answer: A Diff: 2 Page Ref: 93

6 Copyright © 2014 Pearson Education, Inc.

 

24) Tissues in this category are characterized by an extracellular matrix that holds some amount of fluid. A) muscle B) nervous C) connective D) epithelial Answer: C Diff: 3 Page Ref: 78 25) Adipocytes are found in tissues belonging to this category. A) muscle B) nervous C) connective D) epithelial Answer: C Diff: 2 Page Ref: 81 26) Which of the following is not associated with connective tissue? A) areolar B) collagen C) goblet cells D) chondrocytes Answer: C Diff: 2 Page Ref: 73 27) The major function of stratified squamous epithelium is A) diffusion. B) absorption. C) filtration. D) protection from abrasion. Answer: D Diff: 3 Page Ref: 65, 72 28) Simple squamous epithelia would not provide A) protection from abrasion. B) diffusion. C) filtration. D) secretion. Answer: A Diff: 1 Page Ref: 67

7 Copyright © 2014 Pearson Education, Inc.

 

29) What do goblet cells secrete? A) enzymes B) histamine C) mucin D) antibodies Answer: C Diff: 2 Page Ref: 73 30) A structure that secretes hormones into the blood is A) a plasma cell. B) a goblet cell. C) an endocrine gland. D) an exocrine gland. Answer: C Diff: 2 Page Ref: 72 31) Of the types of lining and covering membranes, the only one that is drier than the others is A) cutaneous. B) serous. C) mucous. D) parietal. Answer: A Diff: 2 Page Ref: 89 32) The difference between a basal lamina and a basement membrane is that A) the basal lamina is thicker. B) they lie on opposite surfaces of the epithelium. C) a basal lamina plus a layer of reticular fibers equals a basement membrane. D) only the basal lamina can be seen by light microscopy. Answer: C Diff: 3 Page Ref: 76 33) Which of the following is not a characteristic of epithelia? A) highly cellular with little extracellular matrix B) innervated C) specialized cell-cell junctions D) vascular Answer: D Diff: 2 Page Ref: 66 34) Which of the following provides the least resistance to diffusion? A) simple columnar epithelium B) simple cuboidal epithelium C) simple squamous epithelium D) stratified squamous epithelium Answer: C Diff: 2 Page Ref: 67 8 Copyright © 2014 Pearson Education, Inc.

 

35) The major function of pseudostratified columnar epithelium is A) secretion. B) filtration. C) protection. D) absorption. Answer: A Diff: 2 Page Ref: 72 36) A type of connective tissue that is not a connective tissue proper is A) dense irregular connective tissue. B) areolar tissue. C) adipose tissue. D) cartilage. Answer: D Diff: 3 Page Ref: 79 37) A stratified epithelium that thins and flattens as the tissue stretches is a A) pseudostratified epithelium. B) stratified cuboidal epithelium. C) stratified squamous epithelium. D) transitional epithelium. Answer: D Diff: 3 Page Ref: 71-72 38) Only one type of fiber in connective tissue can be stretched and then recoil—return to its original length and shape. This fiber is the Answer: elastic fiber. Diff: 2 Page Ref: 80 39) A highly branched gland whose walls do not balloon into spherical sacs is called A) compound alveolar. B) compound tubular. C) compound tubuloalveolar. D) simple branched alveolar. Answer: B Diff: 2 Page Ref: 73-74 40) Which one of the following basic types of tissues functions to control the body? A) epithelial B) connective C) muscle D) nervous Answer: D Diff: 2 Page Ref: 93-95

9 Copyright © 2014 Pearson Education, Inc.

 

41) Which one of the following basic types of tissues has an extracellular matrix? A) epithelial B) connective C) muscle D) nervous Answer: B Diff: 2 Page Ref: 78 42) Which one of the following basic types of tissues functions as a covering tissue for the body? A) epithelial B) connective C) muscle D) nervous Answer: A Diff: 1 Page Ref: 65 43) An epithelium in which the basal layer of cells is cuboidal and the apical layer has cells that are thin, flat and irregular in shaper is a ________ epithelium. A) stratified squamous B) simple columnar C) stratified cuboidal D) simple cuboidal Answer: A Diff: 2 Page Ref: 72 44) The thinnest type of epithelium is A) simple squamous. B) pseudostratified columnar. C) stratified squamous. D) stratified columnar. Answer: A Diff: 2 Page Ref: 67 45) Microvilli can perform all of the following functions except A) enhance the rate of absorption. B) enhance the rate of secretion. C) moving and propelling fluids. D) anchoring mucus. Answer: C Diff: 3 Page Ref: 76

10 Copyright © 2014 Pearson Education, Inc.

 

46) The type of cell junction that permits small sugar molecules and ions to pass from the cytoplasm of one cell to that of another is a A) tight junction. B) gap junction. C) desmosome. D) zonula occludens. Answer: B Diff: 2 Page Ref: 76 47) Ligaments consist primarily of which kind of connective tissue? A) loose areolar B) dense irregular C) cartilage D) dense regular Answer: D Diff: 2 Page Ref: 86 48) In connective tissue proper, ground substance is manufactured and secreted by which type of cell? A) fibroblast B) endothelial cell C) chondroblasts D) osteocytes Answer: A Diff: 2 Page Ref: 78-79 49) Use logic to deduce which of the following tissues is on the anterior surface of a person's eyeball. A) epithelial B) connective C) muscle D) nervous Answer: A Diff: 3 Page Ref: 66 50) The distinguishing characteristic between tight junctions and adherens junctions is that A) adherens junctions attach to the actin myofilaments of the cytoskeleton, whereas tight junctions do not. B) adherens junctions involve integral membrane proteins, whereas tight junctions do not. C) tight junctions are present near apical lateral borders, whereas adherens junctions are located near basal lateral borders. D) tight junctions involve fusion of proteins on adjacent cells, whereas adherens junctions create tunnels between adjacent cells. Answer: A Diff: 3 Page Ref: 74-75

11 Copyright © 2014 Pearson Education, Inc.

 

51) Connective tissue that must resist forces pulling it from many different directions without stretching would most likely be A) areolar connective tissue. B) dense irregular connective tissue. C) dense regular connective tissue. D) reticular connective tissues. Answer: B Diff: 3 Page Ref: 84 52) Which of the following is not a characteristic of the inflammatory response? A) attacks and eliminates specific foreign molecules B) involves release of chemicals that cause local vasodilation C) occurs within connective tissue D) results in edema and delivery of chemicals needed for repair Answer: A Diff: 2 Page Ref: 95-97 53) Glycosaminoglycans and proteoglycans are components of A) blood. B) inflammation. C) ground substance. D) reticular fibers. Answer: C Diff: 3 Page Ref: 80-81 54) It might be argued that all the defense mechanisms in areolar connective tissue act to prevent infection from reaching which other tissue type? A) bone B) epithelium C) blood D) muscle Answer: C Diff: 2 Page Ref: 81 55) Despite their name, many types of cell junction are not very strong at joining and holding cells together. The cell junctions that best keep cells from being pulled apart are A) tight junctions. B) gap junctions. C) desmosomes. D) zonula occludens. Answer: C Diff: 2 Page Ref: 75

12 Copyright © 2014 Pearson Education, Inc.

 

56) Fat is a form of ________ tissue. A) epithelial B) connective C) muscle D) nervous Answer: B Diff: 1 Page Ref: 79 57) During repair of a scrape or cut through the skin, a blood clot is replaced with collagen-rich granulation tissue in a process called A) inflammation. B) organization. C) repair. D) regeneration. Answer: B Diff: 2 Page Ref: 97 58) All lining and covering membranes consist of A) an epithelium only. B) a serosa. C) an epithelium, a connective tissue, and smooth muscle. D) epithelium and connective tissue. Answer: D Diff: 2 Page Ref: 89 59) Inflamed tissue is often warm to the touch because of A) accumulation of bacterial toxins. B) increased blood flow to the area. C) infiltration by neutrophils. D) leakage of clotting proteins. Answer: B Diff: 1 Page Ref: 95-97 60) The best definition of fibrosis is A) the production of collagen fibers. B) the production of elastic fibers. C) the formation of dense regular connective tissue. D) the formation of scar tissue. Answer: D Diff: 2 Page Ref: 97

13 Copyright © 2014 Pearson Education, Inc.

 

61) Cancer can arise from each of the following except A) induced apoptosis. B) exposure to a carcinogen. C) inactivation of a tumor-suppressor gene. D) turning on of oncogenes. Answer: A Diff: 3 Page Ref: 99 62) In connective tissue, extracellular matrix is not exactly the same as ground substance, because the matrix also contains A) fibers. B) cells. C) an epithelium. D) blood. Answer: A Diff: 2 Page Ref: 80 63) When cells from a cancerous tumor travel to other parts of the body, this process is called A) carcinogenesis. B) malignancy. C) metastasis. D) transformation. Answer: C Diff: 2 Page Ref: 99 64) A muscle fiber is A) the same as a collagen fiber that is viewed by electron microscopy. B) part of a striation of a muscle cell. C) a muscle cell. D) an intercalated disc. Answer: C Diff: 3 Page Ref: 93 65) Of the following, the tissue with the greatest capacity for regeneration is A) skeletal muscle. B) neurons. C) epithelium. D) smooth muscle. Answer: C Diff: 2 Page Ref: 66

14 Copyright © 2014 Pearson Education, Inc.

 

66) Cartilage is a form of ________ tissue. A) epithelial B) connective C) muscle D) nervous Answer: B Diff: 2 Page Ref: 79 67) A common example of a unicellular exocrine gland is A) a fibroblast. B) any hormone-secreting cell. C) an endothelial cell. D) a goblet cell. Answer: D Diff: 2 Page Ref: 73 68) Tissue types which are known as mucous and serous membranes are comprised of sheets of epithelial cells and A) adipose tissue. B) areolar connective tissue. C) reticular connective tissue D) elastic connective tissue. Answer: B Diff: 2 Page Ref: 89 69) When unstretched, transitional epithelium looks most like ________ epithelium. A) simple squamous B) pseudostratified columnar C) stratified squamous D) stratified cuboidal Answer: D Diff: 2 Page Ref: 71-72 70) A pseudostratified epithelium is always A) simple. B) ciliated. C) stratified. D) cuboidal. Answer: A Diff: 1 Page Ref: 67

15 Copyright © 2014 Pearson Education, Inc.

 

71) The embryonic tissue that gives rise to the adult connective tissues, consisting of star-shaped cells separated by large amounts of extracellular matrix, is A) ectoderm. B) epithelium. C) mesenchyme. D) hypoblast. Answer: C Diff: 2 Page Ref: 78, 82 72) The type of cell junction that closes off the space between epithelial cells, thereby limiting the passage of small molecules, is the A) tight junction. B) adherens junction. C) desmosome. D) gap junction. Answer: A Diff: 2 Page Ref: 74 73) Cell junctions occur in which plasma membrane of epithelial cells? A) apical B) lateral C) nuclear D) basal lamina Answer: B Diff: 1 Page Ref: 74 74) In connective tissues, the primary extracellular molecules responsible for resisting tension are A) intermediate filaments in fibroblasts. B) collagen fibers. C) proteoglycans D) elastin fibers. Answer: B Diff: 2 Page Ref: 80 75) A connective tissue that has a liquid matrix is A) areolar connective tissue. B) tissue fluid. C) blood. D) epithelium. Answer: C Diff: 2 Page Ref: 89

16 Copyright © 2014 Pearson Education, Inc.

 

4.2 True/False Questions 1) Multilayered epithelia are named for the cell shape found in the basal layer. Answer: FALSE Diff: 2 Page Ref: 66 2) The mesothelium comprises the superficial layer of the serous membranes of the peritoneal, pleural, and pericardial cavities. Answer: TRUE Diff: 3 Page Ref: 89 3) In bony tissues, chondrocytes are found in the lacunae. Answer: FALSE Diff: 2 Page Ref: 89 4) Dense regular connective tissue is characterized by giving strength in all directions. Answer: FALSE Diff: 2 Page Ref: 86 5) Areolar connective tissue forms the lamina propria of mucous membranes. Answer: TRUE Diff: 2 Page Ref: 89 6) Superficial fascia refers to the fatty hypodermis below the skin, whereas deep fascia are the sheets of tissue that wrap around muscles, large blood vessels, etc. Answer: TRUE Diff: 3 Page Ref: 81, 86 7) Much of the body's adipose tissue is found in the hypodermis. Answer: TRUE Diff: 3 Page Ref: 81 8) Microvilli are common on almost all epithelial cells, but they are more obvious and abundant on absorption cells in the small intestine and the kidney. Answer: TRUE Diff: 2 Page Ref: 76 9) Unlike other connective tissues, bone is not considered to be a living tissue. Answer: FALSE Diff: 2 Page Ref: 89 10) In blood, the matrix is the liquid blood plasma. Answer: TRUE Diff: 2 Page Ref: 89

17 Copyright © 2014 Pearson Education, Inc.

 

11) Fibrosis involves the proliferation of a fibrous connective tissue called scar tissue. Answer: TRUE Diff: 2 Page Ref: 97 12) The cutaneous membrane is the skin. Answer: TRUE Diff: 1 Page Ref: 89 13) Cardiac muscle cells are connected to each other by intercalated discs. Answer: TRUE Diff: 2 Page Ref: 93 14) Skeletal muscle is characterized by obvious striations (seen microscopically) and multinucleate cells. Answer: TRUE Diff: 2 Page Ref: 93 15) Dense regular connective tissue gains its strength from the multitude of actin fibers that make up the bulk of the matrix. Answer: FALSE Diff: 3 Page Ref: 86 4.3 Short Answer Questions 1) The smallest diameter blood vessels, capillaries are comprised of ________. Answer: simple squamous epithelium Diff: 2 Page Ref: 67 2) This is the sheet of tissue between the epithelium and the connective tissue below it. Answer: basal lamina Diff: 2 Page Ref: 76 3) An ________ gland releases its products through ducts onto body surfaces or into body cavities. Answer: exocrine Diff: 2 Page Ref: 72 4) Damage to the basal lamina due to untreated diabetes may lead to kidney failure and pathological degeneration of what other organ? Answer: retina of the eye Diff: 3 Page Ref: 76 5) Linker proteins in the extracellular space join cytoplasmic plaques on adjacent epithelial cells in this type of cell junction. Answer: desmosome Diff: 2 Page Ref: 75 18 Copyright © 2014 Pearson Education, Inc.

 

6) The disease scurvy results from a deficiency of vitamin C, leading to a disruption in the synthesis of ________ fibers. Answer: collagen Diff: 2 Page Ref: 80 7) Elastic fibers are comprised of the protein ________. Answer: elastin Diff: 1 Page Ref: 80 8) The outermost layers of intervertebral discs and the pubic symphysis are comprised of this type of cartilage. Answer: fibrocartilage Diff: 2 Page Ref: 87 9) This type of fat is important in babies to aid in thermoregulation. Answer: brown fat Diff: 2 Page Ref: 84 10) Identify two different locations in the human body where elastic cartilage is found. Answer: Pinna of the ear, epiglottis of the larynx Diff: 1 Page Ref: 87 11) This term refers to the accumulation of fluid at a site of inflammation. Answer: edema Diff: 2 Page Ref: 96 12) Deep ________ is a dense regular connective tissue that wraps around muscle groups, large vessels, and nerves. Answer: fascia Diff: 2 Page Ref: 86 13) Nervous tissue and ________ tissues are generally thought to lack the capability to regenerate. Answer: cardiac muscle Diff: 2 Page Ref: 97 14) Stem cells may be characterized as being ________ cells; in other words, they can produce new tissue cells as needed. Answer: undifferentiated Diff: 3 Page Ref: 98 15) After a wound has healed, the scar that remains is nearly entirely composed of ________ fibers. Answer: collagen Diff: 2 Page Ref: 97

19 Copyright © 2014 Pearson Education, Inc.

 

4.4 Essay Questions 1) Describe the medical potential seen for the use of stem cells. Answer: Stem cells may be used to "make" replacement tissues; may help to alleviate degenerative diseases; and could form new tissues, such as bone marrow cells, cardiac cells, and other tissues that have been damaged or have died. Further, they may be used to treat degenerative neurological tissues, because nerve tissue cannot effectively repair itself. Diff: 2 Page Ref: 98 2) Describe the classification of epithelial tissues. Answer: Epithelia may be simple (single layered) or stratified (in more than one layer). The cells that form it may be flat and platelike (squamous), roughly square (cuboidal), or tall and slender (columnar). Diff: 2 Page Ref: 66-67 3) List at least four characteristics that distinguish epithelial tissues from other types of tissues. Answer: Epithelia are composed almost entirely of cells, have specialized contacts, have a free apical surface and a lower basal surface, are supported by underlying connective tissue, are avascular but innervated, and have a high capacity for regeneration. Diff: 2 Page Ref: 66 4) Briefly outline the steps of tissue repair in a skin wound. Include in your response the terms organization, granulation, and regeneration. Answer: 1) An inflammatory response increases blood flow to the site of injury, which promotes the formation of a blood clot. 2) During the "organization" phase of tissue repair, blood vessels invade the clot and allow fibroblasts to arrive. These cells deposit collagen to tie the sides of the wound together with granulation tissue. 3) Macrophages dispose of bacteria, dead cells, and debris. 4) Regeneration involves the replacement of epithelium over the surface of the scar tissue. Diff: 3 Page Ref: 97 5) Describe the basal lamina, its location, composition, and function. Answer: The basal lamina lies between the epithelium and the connective tissue below it. It is a thin, noncellular sheet of proteins secreted by the epithelial cells. It functions as a selective filter. When the epithelium has been damaged or scraped off, the basal lamina aids in regeneration of the epithelium by serving as a scaffold for migrating epithelial cells. Diff: 2 Page Ref: 76

20 Copyright © 2014 Pearson Education, Inc.

 

Human Anatomy, 7e (Marieb/Mitchell/Smith) Chapter 5 The Integumentary System 5.1 Multiple Choice Questions

Figure 5.1 Use the diagram above to answer the following questions. 1) Which letter indicates projections from the dermis that reduce blister formation? A) A B) B C) C D) D E) E Answer: E Diff: 3 Page Ref: 104, 108

1 Copyright © 2014 Pearson Education, Inc.

 

2) Which letter indicates a specialized sensory receptor that responds to deep pressure? A) A B) B C) C D) D E) E Answer: D Diff: 3 Page Ref: 104 3) Which letter indicates the integument layer that corresponds to leather hides from an animal? A) A B) B C) C D) D E) E Answer: B Diff: 2 Page Ref: 104, 108 4) Which letter indicates the integument layer that has no vascularization? A) A B) B C) C D) D E) E Answer: A Diff: 3 Page Ref: 104, 107 5) Which letter indicates the integument layer composed of keratinized stratified squamous cells? A) A B) B C) C D) D E) E Answer: A Diff: 2 Page Ref: 104-105

2 Copyright © 2014 Pearson Education, Inc.

 

Figure 5.2 Use the diagram above to answer the following questions. 6) Which letter indicates cells that produce melanin in the hair root? A) A B) B C) C D) D E) E Answer: D Diff: 2 Page Ref: 112-114 7) Which letter indicates the portion of a hair that projects above the skin surface? A) A B) B C) C D) D E) E Answer: A Diff: 2 Page Ref: 112-114

3 Copyright © 2014 Pearson Education, Inc.

 

8) Which letter indicates the innermost portion of hair that is comprised of large epithelial cells and air spaces? A) A B) B C) C D) D E) E Answer: C Diff: 2 Page Ref: 112-114 9) Which letter indicates the portion of the hair root that contains a single capillary cluster? A) A B) B C) C D) D E) E Answer: E Diff: 2 Page Ref: 112-114 10) Which letter indicates a layer of cells that are derived from the epidermis? A) A B) B C) C D) D E) E Answer: B Diff: 2 Page Ref: 112-114 11) Which primary layer of the skin contains melanocytes? A) dermis B) hypodermis C) epidermis Answer: C Diff: 2 Page Ref: 105-106 12) Which layer of the integument contains the stratum basale? A) dermis B) hypodermis C) epidermis Answer: C Diff: 2 Page Ref: 105-106

4 Copyright © 2014 Pearson Education, Inc.

 

13) Which tissue layer deep to the integument acts as a shock and thermal insulator? A) dermis B) hypodermis C) epidermis Answer: B Diff: 1 Page Ref: 110 14) Which layer of the integument are toenails and fingernails derived from? A) dermis B) hypodermis C) epidermis Answer: C Diff: 3 Page Ref: 111-112 15) Which primary layer of the skin provides a pink undertone to Caucasian skin? A) dermis B) hypodermis C) epidermis Answer: A Diff: 2 Page Ref: 111 16) Which layer of the skin invaginates downward to form sebum-producing glands? A) dermis B) hypodermis C) epidermis Answer: C Diff: 1 Page Ref: 111, 115 17) When doctors make incisions on the body based on the lines of cleavage of the skin, these cleavage lines are based on the arrangement of collagen fibers in which layer of the skin? A) dermis B) hypodermis C) epidermis Answer: A Diff: 3 Page Ref: 109 18) Diffusion of chemicals from a transdermal patch must reach what layer of the skin to be effective? A) dermis B) hypodermis C) epidermis Answer: A Diff: 2 Page Ref: 109

5 Copyright © 2014 Pearson Education, Inc.

 

19) Which primary layer of the skin contains lamellated granules that aid in waterproofing? A) dermis B) hypodermis C) epidermis Answer: C Diff: 3 Page Ref: 107 20) Identify the layer of the skin that contains most of the sensory receptors. A) dermis B) hypodermis C) epidermis Answer: A Diff: 1 Page Ref: 105 21) Following their production in red bone marrow, dendritic cells circulate to what primary layer of the skin? A) dermis B) hypodermis C) epidermis Answer: C Diff: 3 Page Ref: 107 22) Visible stretch marks on the skin surface are actually the result of microscopic tears in collagen fibers in which layer? A) dermis B) hypodermis C) epidermis Answer: A Diff: 2 Page Ref: 108-109 23) When a blister forms due to friction, liquid pools between the epidermis and what other layer of the integument? A) dermis B) hypodermis C) epidermis Answer: A Diff: 1 Page Ref: 108 24) If you were to have liposuction, you would have tissue removed from which layer? A) dermis B) hypodermis C) epidermis Answer: B Diff: 2 Page Ref: 110

6 Copyright © 2014 Pearson Education, Inc.

 

25) Which layer of the integument is found deep to the dermis? A) dermis B) hypodermis C) epidermis Answer: B Diff: 1 Page Ref: 110 26) Is the human integumentary system best adapted for hot or cold climates? A) cold, because we have many skin features that keep us from overheating B) cold, because we have a thick hypodermis C) warm, because sweating cools us in hot climates but we would die in cold climates without clothing D) warm, because sebaceous glands aid us in hot climates but evaporative cooling would kill us in cold climates Answer: C Diff: 3 Page Ref: 115-116 27) The cells in the skin that store the most melanin granules are A) dendritic cells. B) tactile epithelial cells. C) basal keratinocytes. D) melanocytes. Answer: C Diff: 3 Page Ref: 105-106 28) Light-skinned people have less melanin in the superficial layers of their epidermis because A) they are exposed to less UV radiation. B) they have fewer melanocytes. C) keratinocytes in their deeper layers rapidly digest melanin. D) their melanocytes produce a pinkish color melanin instead of brown. Answer: C Diff: 2 Page Ref: 105-106 29) The type of sweat gland that begins to function at puberty and is involved in sexual signaling is A) apocrine. B) eccrine. C) sebaceous. D) holocrine. Answer: A Diff: 1 Page Ref: 116

7 Copyright © 2014 Pearson Education, Inc.

 

30) The proximal nail fold is also called the nail's A) matrix. B) bed. C) lunule. D) eponychium. Answer: D Diff: 1 Page Ref: 112 31) The lunule, which represents the thickest part of the nail matrix is found at the A) body of the nail. B) eponychium. C) proximal part of the nail bed. D) dermis around the nail. Answer: C Diff: 3 Page Ref: 112 32) Based on the rule of nines, approximately what percent of the total body surface covers the anterior and posterior torso? A) 37% B) 50% C) 58% D) 66% Answer: A Diff: 2 Page Ref: 117 33) Which layer of the epidermis contains keratohyalin granules and lamellated granules? A) stratum basale B) stratum corneum C) stratum granulosum D) stratum spinosum Answer: C Diff: 3 Page Ref: 107 34) The reticular layer of the dermis consists of ________ connective tissue. A) reticular B) stratified squamous C) dense regular D) dense irregular Answer: D Diff: 2 Page Ref: 108

8 Copyright © 2014 Pearson Education, Inc.

 

35) In the thick skin of the palm, the layer of the epidermis directly deep to the stratum corneum is the stratum ________. A) granulosum. B) basale. C) lucidum. D) spinosum. Answer: C Diff: 1 Page Ref: 107 36) The coarse hairs in the skin of the perineum and axillary regions are all ________ hairs. A) pubic B) apocrine C) terminal D) vellus Answer: C Diff: 2 Page Ref: 114 37) Lamellated granules A) burst from sebaceous glands, releasing oils onto the skin. B) contain glycolipids that are secreted from cells in the stratum granulosum. C) help form keratin in fingernails. D) smooth the cuticle of a hair shaft to prevent split ends. Answer: B Diff: 2 Page Ref: 107 38) Which is the most abundant and widespread type of sweat gland? A) eccrine B) apocrine C) holocrine D) endocrine Answer: A Diff: 1 Page Ref: 116 39) The type of burn in which the epidermis and part of the dermis are damaged is ________ degree. A) first B) second C) third D) fourth Answer: B Diff: 1 Page Ref: 116-117

9 Copyright © 2014 Pearson Education, Inc.

 

40) The thickness of skin is determined by the presence of the stratum lucidum and the thickness of the A) papillary layer of the dermis. B) reticular layer of the dermis. C) stratum basale. D) stratum corneum. Answer: D Diff: 2 Page Ref: 107 41) The subpapillary plexus A) nourishes the dermal papillae and epidermis. B) nourishes the hypodermis and reticular layer of the dermis. C) provides the sensation of fine touch. D) provides the sensation of deep pressure. Answer: A Diff: 2 Page Ref: 109 42) The color of red or blonde hair is due to A) carotene. B) oxygenated hemoglobin. C) porphyrins. D) a different type of melanin. Answer: D Diff: 1 Page Ref: 112 43) The most external layer of a hair shaft is the A) medulla. B) cuticle. C) cortex. D) epidermal root sheath. Answer: B Diff: 2 Page Ref: 112 44) Since both hair and skin contain keratin, the hair shaft is most comparable to what part of the epidermis? A) stratum corneum B) melanocytes C) basal lamina D) stratum basale Answer: A Diff: 3 Page Ref: 107 & 112

10 Copyright © 2014 Pearson Education, Inc.

 

45) Pimples begin A) as localized infections in the hypodermis. B) as tiny cuts and nicks in the skin. C) when bacteria enter sweat pores. D) when sebaceous glands become blocked by sebum. Answer: D Diff: 2 Page Ref: 115 46) The black part of a blackhead is A) oxidized sebum. B) dirt. C) a growing hair stub. D) excreted carbon. Answer: A Diff: 2 Page Ref: 115 47) The "cheesy" layer on the skin of a newborn is A) lanugo. B) baby sweat. C) seborrhea. D) vernix caseosa. Answer: D Diff: 2 Page Ref: 119 48) An appendage or layer of the skin that does not develop from ectoderm is the A) stratum spinosum. B) hair follicle. C) stratum corneum. D) dermis. Answer: D Diff: 1 Page Ref: 118 49) The type of skin cancer that is most common and has the lowest metastasis rate is A) basal cell carcinoma. B) squamous cell carcinoma. C) malignant melanoma. D) leukopenia. Answer: A Diff: 2 Page Ref: 118 50) Any condition involving absence or loss of hair is called A) psoriasis. B) alopecia. C) impetigo. D) vitiligo. Answer: B Diff: 2 Page Ref: 119 11 Copyright © 2014 Pearson Education, Inc.

 

51) Lanugo hair is replaced in a fetus by A) nominal hair. B) no hair. C) vellus hair. D) dermal hair. Answer: C Diff: 3 Page Ref: 119 52) Since apocrine glands are almost inactive until puberty, they may function primarily in A) cooling the body. B) sexual signalling, under the influence of androgens. C) keeping hair from falling out. D) strengthening melanin in the skin. Answer: B Diff: 3 Page Ref: 116 53) Soft keratin is present in the A) epidermis. B) hair cuticles. C) dermis. D) body of the nails. Answer: A Diff: 3 Page Ref: 111-112 54) The white half-moon visible under the proximal part of a fingernail is the A) matrix. B) bed. C) lunule. D) cuticle. Answer: C Diff: 2 Page Ref: 112 55) Cleavage lines A) are scars resulting from overstretching of the dermis. B) attach the dermis to underlying bone and muscle. C) form the pattern of epidermal ridges known as fingerprints. D) run parallel to the orientation of collage bundles in the reticular layer. Answer: D Diff: 1 Page Ref: 109

12 Copyright © 2014 Pearson Education, Inc.

 

56) Tattoos are permanent because A) Langerhans cells do not recognize the pigments as foreign. B) the melanocytes are altered to produce different colors. C) the pigments are injected in the dermis. D) the pigments are injected in the stratum basale, which continuously replaces the superficial layers that are sloughed off. Answer: C Diff: 2 Page Ref: 110 57) In the ABCD rule of skin cancer, the letter B stands for A) big. B) blisters. C) border. D) baked skin. Answer: C Diff: 1 Page Ref: 118 58) Which connective tissue layer is discussed in the chapter on the integumentary system but is truly not a part of it? A) dermal papilla B) hair C) nail D) hypodermis Answer: D Diff: 1 Page Ref: 110 59) Which of the following is a sensory receptor? A) keratinocyte B) melanocyte C) dendritic cell D) Pacinian corpuscle Answer: D Diff: 2 Page Ref: 103-105 60) Which of the following is not a current hypothesis proposed for the evolution of skin color? A) preventing damage to DNA B) preventing damage to elastin C) protecting the level of folic acid D) protecting the level of vitamin D Answer: B Diff: 3 Page Ref: 111

13 Copyright © 2014 Pearson Education, Inc.

 

61) A(n) ________ responds to the movement of a single hair. A) arrector pili muscle B) hair matrix C) hair papilla D) root hair plexus Answer: D Diff: 2 Page Ref: 112, 114 62) A sebaceous gland is classified as a ________ gland. A) compound tubular B) simple tubular C) simple alveolar D) compound alveolar Answer: C Diff: 2 Page Ref: 115 63) The epidermis is a keratinized ________ epithelium. A) compound columnar B) stratified squamous C) simple squamous D) simple areolar Answer: B Diff: 2 Page Ref: 105 64) The only cell type found superficial to the stratum spinosum in the epidermis is the A) keratinocyte. B) melanocyte. C) dendritic cell. D) tactile epithelial cell. Answer: A Diff: 3 Page Ref: 105 65) In the ABCD rule for recognizing melanoma, the letter D stands for A) depth. B) diameter. C) dark. D) density. Answer: B Diff: 2 Page Ref: 118 66) The appearance of wrinkles in skin with age is largely due to changes in the A) collagen and elastic fibers in the dermis. B) keratin of the epidermis. C) carotene in the epidermis. D) blood vessels in the dermis. Answer: A Diff: 3 Page Ref: 118-119 14 Copyright © 2014 Pearson Education, Inc.

 

67) Excessive exposure to ultraviolet radiation (UVR) can lead to skin cancer, but minimal UVR exposure can lead to A) porphyria. B) vitiligo. C) vitamin D deficiency. D) ulcerative colitis. Answer: C Diff: 3 Page Ref: 104-5 & 111 68) Damage to the hair ________ would prevent hair growth. A) cortex B) cuticle C) matrix D) medulla Answer: C Diff: 2 Page Ref: 112-114 69) "Goose bumps" on the skin are due to action of the A) arrector pili muscles. B) sweat glands. C) sebaceous glands. D) flexion creases. Answer: A Diff: 1 Page Ref: 114 70) Which statement is not part of the explanation for male pattern baldness? A) Cells of the hair matrix cease mitosis. B) Hair growth cycles shorten. C) Hair shafts may not reach the surface. D) Terminal hairs are replaced by vellus hairs. Answer: A Diff: 2 Page Ref: 114 71) Holocrine secretion A) differentiates the products of ceruminous and mammary glands from other sweat glands. B) involves the bursting of the glandular epithelial cell. C) produces a lipid-rich product that can be metabolized by bacteria. D) results from the exocytosis of protein-rich vesicles. Answer: B Diff: 3 Page Ref: 115

15 Copyright © 2014 Pearson Education, Inc.

 

72) The type of skin cancer that arises from keratinocytes of the stratum spinosum is A) basal cell carcinoma. B) malignant melanoma. C) squamous cell carcinoma. D) psoriasis. Answer: C Diff: 2 Page Ref: 118 73) The integumentary organs that are derived from mesoderm and function in thermoregulation are A) nails. B) hair. C) sebaceous glands. D) arrector pili muscles. Answer: D Diff: 3 Page Ref: 114 74) A partial-thickness burn involves A) the epidermis. B) the epidermis and papillary layer of the dermis. C) the epidermis and the entire dermis. D) the epidermis, dermis, and any portion of the hypodermis. Answer: B Diff: 3 Page Ref: 116-117 5.2 True/False Questions 1) As a person balds, terminal hair is replaced by vellus. Answer: TRUE Diff: 3 Page Ref: 114 2) Skin consists of two distinct tissue layers: the epidermis and dermis. Answer: TRUE Diff: 1 Page Ref: 104 3) Keratinocytes begin to die in the stratum granulosum. Answer: FALSE Diff: 3 Page Ref: 107 4) Chemotherapy causes hair loss because these drugs specifically target rapidly dividing cells in the hypodermis. Answer: FALSE Diff: 2 Page Ref: 114 5) Nutrients enter a hair follicle through the dermal papilla. Answer: TRUE Diff: 2 Page Ref: 114 16 Copyright © 2014 Pearson Education, Inc.

 

6) The letter A in the ABCD rule for melanoma detection stands for asymmetry. Answer: TRUE Diff: 1 Page Ref: 118 7) Skin cancers are the most common types of cancer in the United States, and the most important risk factor is overexposure to ultraviolet radiation. Answer: TRUE Diff: 2 Page Ref: 118 8) An immediate concern with a burn patient is loss of fluids, so fluids must be administered immediately. Answer: TRUE Diff: 2 Page Ref: 116 9) First-degree burns are the most severe type of burn. Answer: FALSE Diff: 1 Page Ref: 116-117 10) If you are truly "smelly" after a workout that causes you to work up a sweat, the smell is due to secretions from your eccrine glands. Answer: FALSE Diff: 2 Page Ref: 116 11) Sebaceous glands secrete oil onto the surface of the epidermis via the hair follicle Answer: TRUE Diff: 2 Page Ref: 115 12) The contraction of skeletal muscle cells of the arrector pili muscles, attached to hair follicles, causes "goose bumps." Answer: FALSE Diff: 2 Page Ref: 114 13) Consumer goods made of leather are produced from the subcutaneous adipose layer of animals. Answer: FALSE Diff: 2 Page Ref: 108 14) By the time keratinocytes reach the stratum corneum, they are dead and filled with collagen. Answer: FALSE Diff: 2 Page Ref: 107-108 15) Ceruminous glands of the ear canal are modified sebaceous glands. Answer: FALSE Diff: 3 Page Ref: 116

17 Copyright © 2014 Pearson Education, Inc.

 

5.3 Short Answer Questions 1) In thin skin, the epidermis is comprised of how many layers? Answer: four Diff: 2 Page Ref: 105 2) In the skin, the rich capillary beds and the ________ glands aid in thermoregulation. Answer: sweat or sudoriferous Diff: 2 Page Ref: 115-116 3) Most ________ glands open into hair follicles. Answer: sebaceous Diff: 1 Page Ref: 115 4) The stratum ________ lies between the stratum granulosum and the stratum corneum of thick skin. Answer: lucidum Diff: 2 Page Ref: 107 5) This stratum of the epidermis contains the greatest number of cellular layers in both thick and thin skin. Answer: stratum corneum Diff: 2 Page Ref: 107-108 6) The epidermal stem cells that give rise to hair matrix cells are located in a bulge in the ________, near the insertion of the arrector pili muscle. Answer: external root sheath Diff: 3 Page Ref: 114 7) Deodorants mask the odor produced by the bacteria that live off the secretions of ________ glands. Answer: apocrine Diff: 3 Page Ref: 116 8) The vitamin produced by the skin is vitamin ________. Answer: D Diff: 1 Page Ref: 105 &111 9) Keratinocytes produce keratin, as well as various enzymes and ________. Answer: antimicrobial compounds Diff: 3 Page Ref: 105 10) The dendritic cells reside in the ________. Answer: stratum spinosum Diff: 1 Page Ref: 107

18 Copyright © 2014 Pearson Education, Inc.

 

11) Melanin functions to "shade" keratinocyte DNA molecules from harmful ________. Answer: UV radiation Diff: 2 Page Ref: 105 12) Mitosis primarily occurs in a layer of the epidermis, the stratum ________. Answer: basale Diff: 2 Page Ref: 105 13) Found in the stratum spinosum, dendritic cells are cells of the ________ system. Answer: immune Diff: 2 Page Ref: 107 14) This layer is found only in thick skin, and appears as thin translucent band when viewed with a light microscope is the ________. Answer: stratum lucidum Diff: 2 Page Ref: 105 15) The process that causes premature damage to skin with prolonged exposure to ultraviolet radiation is called ________. Answer: photoaging Diff: 1 Page Ref: 119 5.4 Essay Questions 1) List the layers of the epidermis found in thick skin from the deepest layer to the most superficial layer, and provide a few descriptive terms that characterize each layer. Answer: Stratum basale: single layer of cells, rapid mitosis. Stratum spinosum: keratinocytes have spiny extensions seen in prepared slides. Stratum granulosum: cells have keratohyalin and lamellated granules. Stratum lucidum: cells are thin, dead keratinocytes. Stratum corneum: outer layer of dead, thin cells filled with keratin, surrounded by lipids. Diff: 2 Page Ref: 105-108 2) Name the location and function of these cells: keratinocytes, tactile epithelial cells, melanocytes, and dendritic cells. Answer: Keratinocytes are found in all layers of the epidermis; they accumulate keratin and produce antimicrobial compounds and enzymes that protect the epidermis. Tactile epithelial cells are found in the stratum basale; they serve as touch receptors. Melanocytes are found in the stratum basale; they produce the pigment melanin, which protects the skin from UV radiation of the sun. Dendritic cells are found in the stratum spinosum; they function in immunity via receptor-mediated endocytosis. Diff: 2 Page Ref: 105-107

19 Copyright © 2014 Pearson Education, Inc.

 

3) Describe the effects of postpubescent aging on sebaceous glands, melanocytes, and the structure of the dermis. Answer: As an individual ages, sebaceous glands decrease their activity. The resulting decline in sebum causes itchy, dry skin. Exposure to UV light causes accumulations of melanin to produce freckles and liver spots. It may also cause various forms of skin cancer. These damaging rays also alter collagen and elastin fibers, causing the skin to become wrinkled, loose, and inelastic. Diff: 2 Page Ref: 118-119 4) Describe the vascularization of the skin. Answer: The dermis contains two vascular plexi. The dermal plexus is located between the hypodermis and the dermis, nourishing the hypodermis and the deeper portions of the dermis. The subpapillary plexus is located just deep to the dermal papillae and supplies the upper dermal layer. It also provides nourishment to cells of the deeper layers of the epidermis by diffusion. Diff: 2 Page Ref: 109 5) Describe the location, characteristics, and function of the hypodermis. Answer: The hypodermis, or superficial fascia, is just deep to the dermal layer of the skin. It is composed primarily of adipose tissue, covered by a more superficial areolar connective tissue. Its functions include thermal and shock insulation, nutrient storage, and anchors the skin to underlying tissues. Diff: 2 Page Ref: 110

20 Copyright © 2014 Pearson Education, Inc.

 

Human Anatomy, 7e (Marieb/Mitchell/Smith) Chapter 6 Bones and Skeletal Tissues 6.1 Multiple Choice Questions

1 Copyright © 2014 Pearson Education, Inc.

Figure 6.1 Use the diagram above to answer the following questions. 1) Which letter indicates the region of the long bone comprised of a network of trabecular plates? A) A B) B C) C D) D E) E Answer: A Diff: 2 Page Ref: 129-131

1 Copyright © 2014 Pearson Education, Inc.

 

2) Which letter indicates the region of a long bone that contains the medullary cavity? A) A B) B C) C D) D E) E Answer: E Diff: 2 Page Ref: 129-131 3) Which letter indicates what is known, in a growing juvenile, as the epiphyseal plate? A) A B) B C) C D) D E) E Answer: C Diff: 2 Page Ref: 129-131 4) Which letter indicates the region known as the epiphysis? A) A B) B C) C D) D E) E Answer: B Diff: 1 Page Ref: 128-129 5) Which letter indicates the region of a long bone that in an adult is filled with yellow bone marrow? A) A B) B C) C D) D E) E Answer: D Diff: 2 Page Ref: 129-131

2 Copyright © 2014 Pearson Education, Inc.

 

Figure 6.2 Use the diagram above to answer the following questions. 6) Which letter indicates the space where microscopic blood vessels and nerves pass through the center of the osteon? A) A B) B C) C D) D E) E Answer: A Diff: 2 Page Ref: 133 7) Which letter indicates the collagen fibers of the periosteum known as the perforating, or Sharpey's, fibers? A) A B) B C) C D) D E) E Answer: C Diff: 2 Page Ref: 131

3 Copyright © 2014 Pearson Education, Inc.

 

8) Which letter indicates the circumferential lamellae? A) A B) B C) C D) D E) E Answer: B Diff: 2 Page Ref: 135 9) Which letter indicates bone that is not arranged in osteons and is called spongy or trabecular bone? A) A B) B C) C D) D E) E Answer: E Diff: 2 Page Ref: 129-130 10) Which letter indicates the perforating, or Volkmann's canals which lie at right angles to central canals? A) A B) B C) C D) D E) E Answer: D Diff: 2 Page Ref: 134 11) Which letter indicates lamellae located outside the lamellae belonging to the osteon? A) A B) B C) C D) D E) E Answer: B Diff: 2 Page Ref: 135 12) Which of these diseases is characterized by a pathological loss of bone density? A) osteomalacia and rickets B) osteomyelitis C) osteoporosis D) achondroplasia E) Paget's disease Answer: C Diff: 1 Page Ref: 143 4 Copyright © 2014 Pearson Education, Inc.

 

13) Which of these would be characterized as a skeletal disorder resulting in weakened bones due to inadequate mineralization? A) osteomalacia and rickets B) osteomyelitis C) osteoporosis D) achondroplasia E) Paget's disease Answer: A Diff: 2 Page Ref: 144 14) Which disease of the skeletal system is often associated with decreasing levels of estrogen, but may also result from inadequate weight-bearing exercise and nutritional deficiencies of calcium, vitamin D and protein? A) osteomalacia and rickets B) osteomyelitis C) osteoporosis D) achondroplasia E) Paget's disease Answer: C Diff: 2 Page Ref: 143-144 15) What disease might result from a compound fracture of a bone? A) osteomalacia and rickets B) osteomyelitis C) osteoporosis D) achondroplasia E) Paget's disease Answer: B Diff: 2 Page Ref: 146 16) Which skeletal disorder is more common in older persons and has an abnormally high ratio of immature woven bone to mature compact bone? A) osteomalacia and rickets B) osteomyelitis C) osteoporosis D) achondroplasia E) Paget's disease Answer: E Diff: 2 Page Ref: 141

5 Copyright © 2014 Pearson Education, Inc.

 

17) Which of these results from a genetic mutation that leads to premature endochondral ossification before adult height can be reached? A) osteomalacia and rickets B) osteomyelitis C) osteoporosis D) achondroplasia E) Paget's disease Answer: D Diff: 2 Page Ref: 139 18) Most bone disorders are characterized by bone loss; which of these diseases is characterized by excessive bone deposition? A) osteomalacia and rickets B) osteomyelitis C) osteoporosis D) achondroplasia E) Paget's disease Answer: E Diff: 2 Page Ref: 141 19) What skeletal disease results from a bacterial infection of the bone and bone marrow? A) osteomalacia and rickets B) osteomyelitis C) osteoporosis D) achondroplasia E) Paget's disease Answer: B Diff: 2 Page Ref: 146 20) Which of these is a skeletal disorder resulting from mutations in a gene that leads to the most common form of dwarfism? A) osteomalacia and rickets B) osteomyelitis C) osteoporosis D) achondroplasia E) Paget's disease Answer: D Diff: 2 Page Ref: 139

6 Copyright © 2014 Pearson Education, Inc.

 

21) Which of these diseases can be treated by administering calcitonin and calcium supplements increased weight-bearing exercise and osteoclast-inhibiting drugs? A) osteomalacia and rickets B) osteomyelitis C) osteoporosis D) achondroplasia E) Paget's disease Answer: C Diff: 3 Page Ref: 143-144 22) Which of these skeletal disorders is characterized in the later stages by excessive rates of bone deposition, where medullary cavities may fill with bony matrix? A) osteomalacia and rickets B) osteomyelitis C) osteoporosis D) achondroplasia E) Paget's disease Answer: E Diff: 2 Page Ref: 141 23) Which of these is a congenital condition primarily affecting long bones? A) osteomalacia and rickets B) osteomyelitis C) osteoporosis D) achondroplasia E) Paget's disease Answer: D Diff: 3 Page Ref: 139 24) Which of these diseases is more prevalent in children, and was exceedingly common in the United States and other industrialized nations before vitamin D was added to milk? A) rickets B) osteomyelitis C) osteoporosis D) achondroplasia E) Paget's disease Answer: A Diff: 2 Page Ref: 144

7 Copyright © 2014 Pearson Education, Inc.

 

25) Which of these diseases, if untreated could lead to excessively long epiphyses and bowed legs in children? A) osteomalacia and rickets B) osteomyelitis C) osteoporosis D) achondroplasia E) Paget's disease Answer: A Diff: 3 Page Ref: 144 26) A long bone that is fractured midshaft has damaged the A) meniscus. B) articular cartilage. C) epiphysis. D) diaphysis. Answer: D Diff: 1 Page Ref: 129-130 27) In the bone matrix, osteoblasts become A) osteocytes. B) chondrocytes. C) osteoclasts. D) mesenchyme cells. Answer: A Diff: 2 Page Ref: 128 28) Which of the following is the correct progression in the healing of a skeletal fracture? A) bone remodeling, bony callus, hematoma, fibrocartilage callus B) fibrocartilage callus, bone remodeling, bony callus, hematoma C) hematoma, fibrocartilage callus, bony callus, bone remodeling D) hematoma, bony callus, bone remodeling, fibrocartilage callus Answer: C Diff: 3 Page Ref: 140-141 29) The type of cartilage that forms the long bones of the embryonic skeleton is A) hyaline cartilage. B) fibrocartilage. C) elastic cartilage. D) calcified cartilage. Answer: A Diff: 2 Page Ref: 125-126, 136-137

8 Copyright © 2014 Pearson Education, Inc.

 

30) The kneecap, or patella, is an example of A) a flat bone. B) an irregular bone. C) a sesamoid bone. D) calcified cartilage. Answer: C Diff: 2 Page Ref: 129 31) Blood vessels to the diaphysis move through the A) epiphyseal plate. B) articular cartilage. C) foramen magnum. D) nutrient foramen. Answer: D Diff: 2 Page Ref: 130 32) The continual process of bone resorption is under the control of the A) parathyroid gland. B) pancreas. C) pituitary gland. D) adrenal gland. Answer: A Diff: 2 Page Ref: 137-138 33) In an adult, the perichondrium A) inhibits growth and repair of cartilage. B) produces new chondrocytes. C) acts like a girdle to prevent matrix extrusion when cartilage is compressed. D) binds the cartilage to adjacent bones. Answer: C Diff: 2 Page Ref: 125 34) Bones in the wrists and ankles are A) long bones. B) short bones. C) irregular bones. D) flat bones. Answer: B Diff: 1 Page Ref: 128 35) Which of the following statements about a long bone is false? A) It has a diaphysis. B) It has two distinct ends. C) It is longer than it is wide. D) It must be more than 5 inches in length. Answer: D Diff: 2 Page Ref: 128-129 9 Copyright © 2014 Pearson Education, Inc.

 

36) An incomplete, splintery fracture that occurs in the flexible bones of children is a(n) ________ fracture. A) greenstick B) depressed C) comminuted D) epiphyseal Answer: A Diff: 2 Page Ref: 142 37) In which type of fracture does the bone fragment into three or more pieces? A) greenstick B) epiphyseal C) comminuted D) spiral Answer: C Diff: 2 Page Ref: 142 38) Osteoid is A) the mineral part of the bone matrix. B) secreted by osteocytes. C) the entire extracellular matrix of bone. D) the organic part of bone matrix prior to mineralization. Answer: D Diff: 3 Page Ref: 128 39) A disease that is characterized by excessive and abnormal remodeling of bone tissue and affects 3% of the elderly is A) Paget's disease. B) osteosarcoma. C) osteomyelitis. D) osteomalacia. Answer: A Diff: 2 Page Ref: 141 40) The process of bone growth is regulated by several hormones throughout the lifetime of an individual. The hormone most influential in skeletal growth prior to adolescence is secreted from the A) parathyroid gland. B) pancreas. C) pituitary gland. D) adrenal gland. Answer: C Diff: 2 Page Ref: 139

10 Copyright © 2014 Pearson Education, Inc.

 

41) The only membrane bone that occurs inferior to the skull is the A) femur. B) sternum. C) rib. D) clavicle. Answer: D Diff: 3 Page Ref: 136 42) Within a long bone of the skeleton, the circumferential lamellae are located in the A) trabeculae. B) osteons. C) external and internal regions of compact bone. D) marrow cavity. Answer: C Diff: 3 Page Ref: 135 43) Muscles are not likely to tear from their bones because A) a circumferential lamella sandwiches the ends of the tendon onto the surface of an adjacent lamella. B) elastin fibers in the tendon can allow for stretching and recoil of the muscle. C) perforating fibers of collagen attach the tendons and periosteum to the cortical surfaces of bone. D) the tendons are partially ossified at their attachment points to the periosteum. Answer: C Diff: 3 Page Ref: 131 44) The primary center of ossification A) occurs in the 4-week embryo. B) is in the diaphysis. C) is in the epiphysis. D) is in membrane bones but not in endochondral bones. Answer: B Diff: 2 Page Ref: 137 45) Osteoblasts probably originate directly from A) mesenchyme cells. B) osteoblasts. C) osteocytes. D) blood stem cells. Answer: A Diff: 3 Page Ref: 136& 140

11 Copyright © 2014 Pearson Education, Inc.

 

46) A hormone that increases the bone-degrading activity of osteoclasts is A) an androgen (male sex hormone). B) an estrogen (female sex hormone). C) thyroid hormone. D) parathyroid hormone. Answer: D Diff: 3 Page Ref: 141 47) Cartilage repairs slowly after adolescence because A) the perichondrium thickens, preventing diffusion of gasses and nutrients. B) the cartilage matrix becomes mineralized. C) chondrocytes no longer are able to divide. D) collagen synthesis ceases. Answer: C Diff: 3 Page Ref: 127 48) Which term applies to a number of disorders in adults in which the bones are inadequately mineralized? A) rickets B) osteomalacia C) Paget's disease D) osteosarcoma Answer: B Diff: 3 Page Ref: 144 49) Rickets most often results from a deficiency of vitamin A) A. B) B. C) C. D) D. Answer: D Diff: 2 Page Ref: 144 50) An osteon is composed of A) a cell body and a long, threadlike extension. B) cartilage. C) layers of bone lamellae surrounding a central canal. D) interstitial lamellae. Answer: C Diff: 1 Page Ref: 132-135

12 Copyright © 2014 Pearson Education, Inc.

 

51) What type of tissue is the embryonic precursor for long bones in the fetal skeleton? A) elastic connective tissue B) dense connective tissue proper C) fibrocartilage D) hyaline cartilage Answer: D Diff: 2 Page Ref: 125-126 52) The type of cartilage that can withstand the strongest compression and tension forces is A) hyaline cartilage. B) elastic cartilage. C) fibrocartilage. D) calcified cartilage. Answer: C Diff: 2 Page Ref: 126-127 53) A long bone is able to withstand extreme torsion or twisting stresses because A) the bone is spongelike with many hollow spaces, such as the central and epiphyseal marrow cavities. B) within each osteon the collagen fibers and mineral crystals in adjacent lamellae are aligned in opposite directions. C) the internal and external surfaces are both covered in irregular connective tissue fibers. D) the trabeculae of spongy bone are aligned along stress trajectories. Answer: B Diff: 2 Page Ref: 133 54) The cartilage of the epiphyseal plates is organized into zones based upon the unique processes occurring in each of these regions. Identify the correct sequence of these processes, from the epiphyseal end toward the diaphysis. A) calcification — hypertrophy — growth — resting — ossification B) ossification — calcification — hypertrophy — growth — resting C) resting — hypertrophy — growth — calcification — ossification D) hypertrophy — growth — resting — calcification — ossification Answer: B Diff: 3 Page Ref: 138-139 55) The type of cartilage that comprises the epiglottis is A) hyaline cartilage. B) fibrocartilage. C) elastic cartilage. D) calcified cartilage. Answer: C Diff: 2 Page Ref: 126

13 Copyright © 2014 Pearson Education, Inc.

 

56) The type of cartilage that forms the costal cartilages at the ends of the ribs is A) hyaline cartilage. B) fibrocartilage. C) elastic cartilage. D) calcified cartilage. Answer: A Diff: 2 Page Ref: 125-126 57) If a bone is thin, flattened, and somewhat curved, that bone is a(n) ________ bone. A) long B) short C) irregular D) flat Answer: D Diff: 3 Page Ref: 128-129 58) If a bone located in a limb is longer than it is wide, that bone is a(n) ________ bone. A) long B) short C) irregular D) flat Answer: A Diff: 1 Page Ref: 128-129 59) In the repair of a simple fracture, the step that follows formation of a hematoma is A) formation of a bony callus. B) inflammation. C) formation of fibrocartilaginous callus. D) bone remodeling. Answer: C Diff: 2 Page Ref: 141-143 60) Which type of fracture tends to occur where cartilage cells are dying and the matrix is calcifying? A) greenstick B) epiphyseal C) spiral D) comminuted Answer: B Diff: 2 Page Ref: 142

14 Copyright © 2014 Pearson Education, Inc.

 

61) Which statement about growth of long bones is false? A) The bone cannot grow longer once the epiphyseal plates are ossified. B) The diaphysis becomes progressively thinner as the bone lengthens. C) The epiphyses are pushed further from the center of the diaphysis. D) The epiphyseal plates remain a constant thickness during growth. Answer: B Diff: 2 Page Ref: 136-138 62) Which hormone is primarily responsible for bone matrix resorption rather than bone matrix deposition during puberty? A) growth hormone B) parathyroid hormone C) sex hormones D) thyroid hormone Answer: B Diff: 1 Page Ref: 140-141 63) In growing cartilage, the chondroblasts associated with the perichondrium are responsible for A) interstitial growth. B) appositional growth. C) endochondral growth. D) calcification. Answer: B Diff: 3 Page Ref: 127 64) Which bone cell type secretes hydrochloric acid? A) osteoblast B) osteoclast C) osteocyte D) osteoprogenitor Answer: B Diff: 2 Page Ref: 139-140 65) Which factor will decrease the rate of bone resorption? A) inadequate vitamin D intake B) long zero-gravity exposure C) mechanical stress D) prolonged bed rest Answer: C Diff: 3 Page Ref: 140-141

15 Copyright © 2014 Pearson Education, Inc.

 

66) Which of the following statements about woven bone is false? A) It is the same as spongy bone of adults. B) It contains no trabeculae. C) It is characteristic of the development of embryonic flat bones. D) As fetal development progresses it is replaced by comapact bone at the periphery. Answer: B Diff: 3 Page Ref: 136 67) In an open reduction to repair a broken bone, A) the ends are close enough to allow them to heal together on their own. B) the ends are joined by pins or wires. C) the ends are repositioned by physician's hands. D) the ends must have chips placed between them to bridge the gap. Answer: B Diff: 2 Page Ref: 141 68) A large, multinucleate cell with a ruffled border is an A) osteoclast. B) adipocyte. C) osteocyte. D) osteoblast. Answer: A Diff: 3 Page Ref: 140 69) The benefits of weight-bearing exercise include stronger muscles A) but weaker bones. B) and stronger bones. C) but bones of the same strength. D) and stronger compact bone, but weaker spongy bone. Answer: B Diff: 3 Page Ref: 141 70) Which of the following is not present in the central canals of osteons? A) blood vessels B) nerves C) endosteum D) osteocytes Answer: D Diff: 2 Page Ref: 131, 133 71) The universal loss of skeletal mass that begins after age 40 A) is slower in women than in men. B) is absolutely uniform throughout the skeleton. C) reflects an imbalance in the bone-remodeling process. D) is due to increased blood flow to bones as individuals age. Answer: C Diff: 2 Page Ref: 145 16 Copyright © 2014 Pearson Education, Inc.

 

72) The cell responsible for secreting the matrix of bone is the A) chondroblast. B) osteoblast. C) osteoclast. D) chondrocyte. Answer: B Diff: 2 Page Ref: 140 73) The cell responsible for secreting the matrix of cartilage is the A) osteocyte. B) osteoblast. C) osteoclast. D) chondroblast. Answer: D Diff: 1 Page Ref: 126 74) Which of the following is not a function of the bony skeleton? A) support and protection B) storage of parathyroid hormone C) transmission of muscular forces by acting as levers D) production of blood cells Answer: B Diff: 1 Page Ref: 127 75) Repair of a simple fracture begins with A) calcification of dense connective tissue. B) granulation tissue formation. C) hyaline cartilage deposition. D) inflammation and hematoma formation. Answer: D Diff: 2 Page Ref: 141 6.2 True/False Questions 1) Microscopic inspection of hyaline cartilage would show prominent parallel bundles of collagen fibers. Answer: FALSE Diff: 2 Page Ref: 125-126 2) In appositional growth, chondrocytes within cartilage divide and secrete new matrix. Answer: FALSE Diff: 3 Page Ref: 127

17 Copyright © 2014 Pearson Education, Inc.

 

3) As a person ages, some calcium phosphate crystals are deposited within cartilage, leading to a condition called calcified cartilage. Answer: TRUE Diff: 2 Page Ref: 127 4) Cartilage is strong in resisting twisting and bending but weak in resisting tension and compression. Answer: FALSE Diff: 3 Page Ref: 125-127 5) A fracture in which the bone breaks cleanly but does not penetrate the skin is a compound fracture. Answer: FALSE Diff: 2 Page Ref: 141 6) The bones of your hand and fingers would all be categorized as long bones. Answer: TRUE Diff: 2 Page Ref: 128-129 7) A long bone may be characterized by two diaphyses and one epiphysis. Answer: FALSE Diff: 2 Page Ref: 128-129 8) Examples of flat bones include the ribs, sternum, the bones of the head surrounding the brain case, and the scapula. Answer: TRUE Diff: 3 Page Ref: 129 9) A sesamoid bone is a specialized short bone that develops within a tendon, such as the patella. Answer: TRUE Diff: 3 Page Ref: 129 10) Osteocytes, residing within lacunae, are connected via canaliculi. Answer: TRUE Diff: 3 Page Ref: 134 11) Circumferential lamellae occur around the entire outer and inner surfaces of compact bone, and they are considered remnants of old osteons that have been disrupted by bone remodeling. Answer: FALSE Diff: 3 Page Ref: 135 12) As cartilage ages, calcium salts are deposited and mineralize the cartilage, turning it into bone. Answer: FALSE Diff: 3 Page Ref: 127

18 Copyright © 2014 Pearson Education, Inc.

 

13) Epiphyseal plates typically close at the time an infant learns to walk. Answer: FALSE Diff: 2 Page Ref: 138 14) Pathologic fractures occur in a diseased bone and involve slight or no physical trauma. Answer: TRUE Diff: 2 Page Ref: 146 15) Membrane bones form directly from mesenchyme without being modeled in cartilage. Answer: TRUE Diff: 1 Page Ref: 136 6.3 Short Answer Questions 1) In ________ growth, cartilage-forming cells from the perichondrium produce new cartilage tissue by actively secreting matrix. Answer: appositional Diff: 2 Page Ref: 127 2) The two primary minerals stored in bone are ________ and ________. Answer: calcium and phosphate Diff: 2 Page Ref: 127 3) ________ canals lie perpendicular to the central, or Haversian, canals. Answer: Perforating or Volkmann's Diff: 2 Page Ref: 133-134 4) The joint surface of each epiphysis is covered with a thin layer of hyaline cartilage called the ________. Answer: articular cartilage Diff: 2 Page Ref: 126 5) You might expect the epiphyseal plate to close between ages ________ years. Answer: 15-23 Diff: 2 Page Ref: 139 6) A blow to the head results in a portion of the broken bone pressed inward; this type of fracture is known as ________. Answer: depressed Diff: 2 Page Ref: 142 7) Treatment of osteoporosis with vitamin D, calcium, and weight-bearing exercise is helpful and not controversial. Treatments that include ________ is controversial because it may lead to increased risk of heart attack, stroke and breast cancer. Answer: estrogen replacement therapy Diff: 2 Page Ref: 144 19 Copyright © 2014 Pearson Education, Inc.

 

8) This disease is characterized by excessive rates of bone deposition and bone resorption. Answer: Paget's disease Diff: 2 Page Ref: 141 9) ________ is a condition in children that was more common early in the nineteenth century, before vitamin D was added to milk in most developed countries. Answer: Rickets Diff: 2 Page Ref: 144 10) ________ is characterized by an increased prevalence in teens and young adults with pain in long bones of the extremities. If untreated, metastasis to the lungs is common. Answer: Osteosarcoma Diff: 2 Page Ref: 145 11) A(n) ________ is common in older persons and is a projection on a bone due to abnormal bone overgrowth. Answer: bone spur Diff: 2 Page Ref: 146 12) A bone graft often uses a piece of the ________ that is placed where bone has been damaged or removed. Answer: ilium Diff: 2 Page Ref: 146 13) ________ is the term for pain in a bone, typical after bone grafting or bone removal. Answer: Ostealgia Diff: 2 Page Ref: 146 14) Bundles of collagen that attach the periosteum to bone are called ________. Answer: perforating collagen fiber bundles, Sharpey's fibers Diff: 2 Page Ref: 131 15) The ________ delivers osteoblasts and osteoclasts to the primary center of ossification. Answer: periosteal bud Diff: 3 Page Ref: 137 6.4 Essay Questions 1) Discuss the changes that occur during osteoporosis and their causes. Answer: In osteoporosis there is loss of bone mass and loss of trabeculae, and the compact bone becomes thinner and less dense. Fractures then become common. It occurs more often in the elderly and post-menopausal women because of the decline in estrogen production. Additional contributing factors include insufficient weight-bearing exercise and a diet insufficient in calcium, vitamin D and protein. Diff: 2 Page Ref: 143-144

20 Copyright © 2014 Pearson Education, Inc.

 

2) Explain the healing of a simple fracture. Answer: First, hematoma formation stops the flow of blood at the point of the break. Fibrocartilage callus formation is facilitated by the cells of the periosteum and endosteum, leading to a soft callus. Bony callus formation follows, by endochondral ossification, and the trabeculae span the break in the bone. Bone remodeling occurs over a number of months and includes removal of excess bony material and the appropriate addition of compact bone. Diff: 2 Page Ref: 141-143 3) Describe the three types of cartilage and their defining characteristics. Answer: Hyaline cartilage is the most abundant. Unstained, it appears "glassy" under the microscope; collagen fibers are not visible using light microscopy but are present in the matrix, providing strength and flexibility. Elastic cartilage contains many more dark-staining elastic fibers in the matrix (which are visible with light microscopy), They allow this cartilage to readily deform and recoil. Fibrocartilage contains thicker collagen fibers, oriented between rows of chondrocytes, and is resistant to both compression and tension forces. Diff: 2 Page Ref: 125-127 4) Explain how the epiphyseal plate remains a constant width until puberty is completed. Answer: Chrondrocytes proximal to the medullary cavity of the long bone are relatively inactive and eventually are deprived of nutrients by calcification of the surrounding matrix. These dead chrondrocytes are replaced by osteoclasts and osteoblasts. Meanwhile, the chrondrocytes closer to the epiphyses rapidly undergo mitosis and are actively secreting additional cartilage matrix. These cells proliferate at about the same rate as others die. In this way, the plate maintains its thickness until the rate of proliferation slows as puberty is completed. Diff: 2 Page Ref: 138-139

21 Copyright © 2014 Pearson Education, Inc.

 

Human Anatomy, 7e (Marieb/Mitchell/Smith) Chapter 7 Bones, Part 1: The Axial Skeleton 7.1 Multiple Choice Questions

Figure 7.1 Use the diagram above to answer the following questions. 1) Which letter indicates a bony projection from the ethmoid bone? A) A B) B C) C D) D E) E Answer: C Diff: 1 Page Ref: 155 2) Which letter indicates the bone that forms the inferior part of the nasal septum? A) A B) B C) C D) D E) E Answer: D Diff: 1 Page Ref: 155 1 Copyright © 2014 Pearson Education, Inc.

 

3) Which letter indicates the facial bone that has as inferior orbital fissure? A) A B) B C) C D) D E) E Answer: B Diff: 2 Page Ref: 155 4) Which letter indicates the bone of the skull that has supraorbital foramen (notch)? A) A B) B C) C D) D E) E Answer: A Diff: 2 Page Ref: 155 5) Which letter indicates the facial bone that has mental foramina? A) A B) B C) C D) D E) E Answer: E Diff: 1 Page Ref: 155

2 Copyright © 2014 Pearson Education, Inc.

 

Figure 7.2 Use the diagram above to answer the following questions. 6) Which letter indicates the mastoid process of the temporal bone? A) A B) B C) C D) D E) E Answer: B Diff: 1 Page Ref: 153 7) Which letter indicates the zygomatic bone? A) A B) B C) C D) D E) E Answer: E Diff: 1 Page Ref: 153 8) Which letter indicates the coronal suture? A) A B) B C) C D) D E) E Answer: A Diff: 1 Page Ref: 153 3 Copyright © 2014 Pearson Education, Inc.

 

9) Which letter indicates the coronoid process of the mandible? A) A B) B C) C D) D E) E Answer: D Diff: 1 Page Ref: 153 10) Which letter indicates the styloid process of the temporal bone? A) A B) B C) C D) D E) E Answer: C Diff: 1 Page Ref: 153

Figure 7.3 Use the diagram above to answer the following questions. 11) Which letter indicates the palatine bone that forms part of the hard palate? A) A B) B C) C D) D E) E Answer: A Diff: 1 Page Ref: 157 4 Copyright © 2014 Pearson Education, Inc.

 

12) Which letter indicates the jugular foramen? A) A B) B C) C D) D E) E Answer: E Diff: 2 Page Ref: 157 13) Which letter indicates the foramen magnum of the occipital bone? A) A B) B C) C D) D E) E Answer: C Diff: 2 Page Ref: 157 14) Which letter indicates the mandibular fossa of the temporal bone? A) A B) B C) C D) D E) E Answer: B Diff: 2 Page Ref: 157 15) Which letter indicates the occipital condyle of the occipital bone? A) A B) B C) C D) D E) E Answer: D Diff: 2 Page Ref: 157 16) On which of these bones would the mental foramen be found? A) mandible B) sphenoid bone C) occipital bone D) temporal bone E) ethmoid bone Answer: A Diff: 2 Page Ref: 162-163

5 Copyright © 2014 Pearson Education, Inc.

 

17) Which of these bones has a foramen that encircles the superior aspect of the spinal cord? A) mandible B) sphenoid bone C) occipital bone D) temporal bone E) ethmoid bone Answer: C Diff: 2 Page Ref: 156 18) Which of these bones has a squamous and petrous region? A) mandible B) sphenoid bone C) occipital bone D) temporal bone E) ethmoid bone Answer: D Diff: 2 Page Ref: 156-158 19) Which of these bones together with the maxilla, functions in chewing? A) mandible B) sphenoid bone C) occipital bone D) temporal bone E) ethmoid bone Answer: A Diff: 1 Page Ref: 162-163 20) Which of these bones forms part of the interior of the nasal cavity and the orbit for the eye? A) mandible B) sphenoid bone C) occipital bone D) temporal bone E) ethmoid bone Answer: E Diff: 2 Page Ref: 162 21) Which of these bones has a ramus and angle? A) mandible B) sphenoid bone C) occipital bone D) temporal bone E) ethmoid bone Answer: A Diff: 2 Page Ref: 162-163

6 Copyright © 2014 Pearson Education, Inc.

 

22) Which of these bones has pterygoid processes and the optic canal? A) mandible B) sphenoid bone C) occipital bone D) temporal bone E) ethmoid bone Answer: B Diff: 2 Page Ref: 158-162 23) Which of these bones has a mastoid process and a styloid process? A) mandible B) sphenoid bone C) occipital bone D) temporal bone E) ethmoid bone Answer: D Diff: 2 Page Ref: 156-158 24) Which of these bones has both the cribriform plate and the crista galli? A) mandible B) sphenoid bone C) occipital bone D) temporal bone E) ethmoid bone Answer: E Diff: 2 Page Ref: 162 25) Which of these bones has "wings" and a body? A) mandible B) sphenoid bone C) occipital bone D) temporal bone E) ethmoid bone Answer: B Diff: 1 Page Ref: 158-162 26) Which of these bones/bony collections listed below is not part of the axial skeleton? A) the skull B) the sternum C) the pelvis D) the sacrum Answer: C Diff: 1 Page Ref: 151

7 Copyright © 2014 Pearson Education, Inc.

 

27) The ribs that are attached posteriorly to the thoracic vertebrae but are not attached anteriorly are known as A) the true ribs. B) the false ribs. C) the floating ribs. D) vertebrosternal ribs. Answer: C Diff: 2 Page Ref: 179 28) Which bony landmark below is most similar to a meatus? A) facet B) foramen C) groove D) suture Answer: B Diff: 3 Page Ref: 152 29) Which is not a region of the temporal bone? A) hypophyseal B) petrous C) squamous D) tympanic Answer: A Diff: 2 Page Ref: 156-158 30) Which of the bones listed below does not contain air sinuses? A) the frontal bone B) the ethmoid C) the maxilla D) the mandible Answer: D Diff: 2 Page Ref: 168 31) The anterior cranial fossa is formed by the A) frontal bone. B) frontal and temporal bones. C) frontal, ethmoid, and sphenoid bones. D) frontal, sphenoid, and temporal bones. Answer: C Diff: 2 Page Ref: 152

8 Copyright © 2014 Pearson Education, Inc.

 

32) Which of the bones listed below is not part of the orbit? A) the frontal B) the zygomatic C) the palatine D) the temporal Answer: D Diff: 2 Page Ref: 169 33) This bone has nuchal lines, the hypoglossal canal, and condyles. A) mandible B) occipital C) sphenoid D) temporal Answer: B Diff: 1 Page Ref: 156 34) Which of the bones listed below does not have a zygomatic process? A) frontal B) maxillary C) temporal D) zygomatic Answer: D Diff: 3 Page Ref: 164 35) Which of the bones listed below is not part of the calvaria? A) mandible B) occipital C) parietal D) temporal Answer: A Diff: 1 Page Ref: 153 36) The jugular foramen is located A) anterior to the foramen lacerum. B) between the temporal and occipital bones. C) posterior to the occipital condyles. D) within the greater wing of the sphenoid. Answer: B Diff: 2 Page Ref: 157

9 Copyright © 2014 Pearson Education, Inc.

 

37) Quasimodo, the main character in Victor Hugo's The Hunchback of Notre Dame, suffered from A) rickets. B) scoliosis. C) lordosis. D) kyphosis. Answer: D Diff: 2 Page Ref: 181 38) The sella turcica of the sphenoid bone A) anchors the brain into the anterior cranial fossa. B) attaches neck muscles the skull. C) forms the superior portion of the nasal septum. D) holds the pituitary gland. Answer: D Diff: 2 Page Ref: 160 39) A condyle is a bone marking that is A) a sharp, slender, pointed projection. B) a long, narrow ridge of bone. C) a large opening through a bone. D) a large projection for bone to bone articulation. Answer: D Diff: 2 Page Ref: 163 40) Which of the facial bones listed below is unpaired? A) palatine B) zygomatic C) lacrimal D) vomer Answer: D Diff: 1 Page Ref: 162-163 41) Which of the bones listed below is not part of the nasal cavity? A) temporal bone B) ethmoid bone C) inferior nasal concha D) palatine bone Answer: A Diff: 2 Page Ref: 165

10 Copyright © 2014 Pearson Education, Inc.

 

42) The external and internal acoustic meatuses are in the ________ and ________ regions of the temporal bone, respectively. A) squamous; mastoid B) tympanic; petrous C) mastoid; petrous D) squamous; sphenoid Answer: B Diff: 2 Page Ref: 156 43) Which of the bones listed below does not contribute to the cranial floor? A) occipital B) sphenoid C) frontal D) palatine Answer: D Diff: 2 Page Ref: 165 44) The ridge-like bone or bones that separate(s) the posterior from the middle cranial fossa is (are) the A) occipital bone. B) sphenoid bone. C) frontal bone. D) temporal bones. Answer: D Diff: 2 Page Ref: 164 45) Which bones are divided by the sagittal suture? A) occipital and parietal B) frontal and parietal C) temporal and parietal D) right and left parietal bones Answer: D Diff: 1 Page Ref: 153 46) In referring to the alveolar regions of the mandible and maxillae, the alveoli are A) glands (salivary glands). B) tooth sockets. C) lips. D) the palate. Answer: B Diff: 1 Page Ref: 163

11 Copyright © 2014 Pearson Education, Inc.

 

47) What does the costal tubercle of a rib articulate with? A) vertebral body of thoracic vertebrae B) transverse process of thoracic vertebrae C) vertebral lamina of thoracic vertebrae D) costal cartilage of thoracic cage Answer: B Diff: 2 Page Ref: 180 48) For each vertebrae, the laminae A) are major components of the vertebral arches. B) are where the ribs attach. C) lie ventral to the vertebral foramen. D) occur in two pairs, so there are four of them. Answer: A Diff: 2 Page Ref: 172 49) Which bone articulates with the dens? A) atlas B) axis C) occipital D) temporal Answer: A Diff: 2 Page Ref: 173 50) When in life does the lumbar curvature of the vertebral column appear? A) in the 2-month embryo B) at the time of birth C) in the toddler (about 1 year old) D) in old age Answer: C Diff: 2 Page Ref: 171 51) The largest fontanelle, forming the largest soft spot on a baby's head, is the A) anterior. B) posterior. C) mastoid. D) sphenoidal. Answer: A Diff: 1 Page Ref: 181

12 Copyright © 2014 Pearson Education, Inc.

 

52) The bone of the skull that has pterygoid processes, greater wings, and a hypophyseal fossa is the A) ethmoid. B) palatine. C) sphenoid. D) temporal. Answer: C Diff: 2 Page Ref: 158-162 53) The function of the anterior longitudinal ligament on the vertebral bodies is A) to prevent hyperextension of the back. B) to allow for extensive lateral movement and rotation of adjacent vertebrae. C) to protect the spinal nerve cord. D) to hold the vertebral laminae together. Answer: A Diff: 3 Page Ref: 171 54) Which type of movement is not possible between the lumbar vertebrae? A) flexion B) extension C) lateral flexion D) circumduction Answer: D Diff: 3 Page Ref: 177 55) The difference between cranium and skull, if any, is that A) the skull contains the hyoid bone but the cranium does not; otherwise they are the same. B) the skull contains facial bones and cranial bones. C) the cranium does not include the floor of the skull. D) the cranium contains the orbits, but the skull does not. Answer: B Diff: 2 Page Ref: 152 56) Which part of the temporal bone projects medially and contains organs of the inner ear? A) styloid B) petrous C) squamous D) zygomatic Answer: B Diff: 2 Page Ref: 158

13 Copyright © 2014 Pearson Education, Inc.

 

57) Which bone listed below has cribriform foramina? A) concha B) ethmoid C) lacrimal D) nasal Answer: B Diff: 1 Page Ref: 162 58) The largest paranasal sinus is the A) frontal. B) maxillary. C) ethmoid. D) sphenoid. Answer: B Diff: 2 Page Ref: 168 59) Which of the bones listed below is not a facial bone? A) lacrimal B) zygomatic C) parietal D) mandible Answer: C Diff: 1 Page Ref: 165 60) The sella turcica is a bony landmark of which bone listed below? A) parietal B) frontal C) sphenoid D) ethmoid Answer: C Diff: 2 Page Ref: 158 61) Swayback, which sometimes occurs in obese individuals, is also called A) lordosis. B) scoliosis. C) kyphosis. D) hunchback. Answer: A Diff: 2 Page Ref: 181 62) The vertebra that lacks a body and helps allow one to nod the head is the A) axis. B) atlas. C) vertebra prominens. D) coccyx. Answer: B Diff: 3 Page Ref: 174 14 Copyright © 2014 Pearson Education, Inc.

 

63) An example of a false rib that is also a floating rib is A) rib 11. B) rib 8. C) rib 1. D) rib 7. Answer: A Diff: 2 Page Ref: 179 64) Which of the bones of the axial skeleton listed below is paired? A) fifth thoracic vertebra B) rib 3 C) the sacrum D) the sphenoid Answer: B Diff: 1 Page Ref: 179 65) Which of the vertebral characteristics described below, would not distinguish a lumbar vertebra from a thoracic vertebra? A) absence of costal facets B) posteromedial facing superior facets C) presence of an inferior vertebral notch D) triangular-shaped vertebral foramen Answer: C Diff: 3 Page Ref: 175-177 66) The primary bone underlying the cheek is the A) maxillary. B) temporal. C) frontal. D) zygomatic. Answer: D Diff: 1 Page Ref: 165 67) Whereas the inferior concha is a projection from the maxilla, the superior and middle nasal conchae are projections of the ________ bone. A) sphenoid B) vomer C) ethmoid D) palatine Answer: C Diff: 3 Page Ref: 162

15 Copyright © 2014 Pearson Education, Inc.

 

68) Which cranial bones meet at the lambdoid suture? A) frontal and parietal B) parietal and occipital C) frontal and nasal D) parietal and temporal Answer: B Diff: 2 Page Ref: 154 69) The thoracic vertebrae articulate with ribs at the A) body and transverse processes. B) spinous processes. C) inferior and superior articular facets. D) head and articular tubercle. Answer: A Diff: 2 Page Ref: 179-180 70) The manubrium articulates with all of the bones listed below except A) rib 1. B) rib 2. C) the xiphoid. D) the clavicle. Answer: C Diff: 2 Page Ref: 179 71) The sternal angle is at the same level as rib number A) 1. B) 2. C) 3. D) 5. Answer: B Diff: 2 Page Ref: 179 72) The most anterior ridge of the sacrum is the A) median sacral crest. B) sacral promontory. C) sacral hiatus. D) third dorsal sacral foramen. Answer: B Diff: 2 Page Ref: 177

16 Copyright © 2014 Pearson Education, Inc.

 

73) The collagen-rich ________ limits expansion of the nucleus pulposus when the spine is compressed. A) annulus fibrosus B) costal cartilage C) lateral masses D) ligamentum flavum Answer: A Diff: 2 Page Ref: 171-172 74) All of the bony landmarks listed below are found within the orbit except the A) superior orbital fissure. B) inferior orbital fissure. C) infraorbital foramen. D) optic canal. Answer: C Diff: 3 Page Ref: 169 75) The lesser and greater wings of the sphenoid are in the ________ and ________ cranial fossae, respectively. A) anterior; posterior B) anterior; middle C) middle; posterior D) posterior; middle Answer: B Diff: 3 Page Ref: 159 7.2 True/False Questions 1) The foramen ovale is bounded posteriorly by the foramen spinosum and anteriorly by the foramen rotundum. Answer: TRUE Diff: 2 Page Ref: 159 2) The axial skeleton includes the skull, vertebral column, and thoracic cage. Answer: TRUE Diff: 2 Page Ref: 151 3) The cranial vault forms the superior, inferior, and lateral aspects of the skull. Answer: FALSE Diff: 2 Page Ref: 153 4) The first 10 pairs of ribs are true ribs. Answer: FALSE Diff: 1 Page Ref: 179

17 Copyright © 2014 Pearson Education, Inc.

 

5) Scoliosis is an accentuated lumbar curvature of the spine. Answer: FALSE Diff: 2 Page Ref: 180-181 6) The term process in anatomical terminology means any prominence on a bone. Answer: TRUE Diff: 1 Page Ref: 133 7) Kyphosis is an exaggerated thoracic curvature of the spine. Answer: TRUE Diff: 3 Page Ref: 181 8) The head of a rib articulates with the transverse process of the vertebra. Answer: FALSE Diff: 3 Page Ref: 180 9) The vomer is an unpaired bone located superior to the palate. Answer: TRUE Diff: 2 Page Ref: 167 10) The superior orbital fissure is part of the sphenoid bone. Answer: TRUE Diff: 2 Page Ref: 164 11) The superior and inferior nasal conchae are part of the ethmoid bone. Answer: FALSE Diff: 2 Page Ref: 164 12) The teeth fit into the sockets of both the mandible and the maxilla. Answer: TRUE Diff: 2 Page Ref: 167 13) The bony framework of the face consists of 14 facial bones. Answer: TRUE Diff: 2 Page Ref: 162 14) The cervical and thoracic curves are primary curves. Answer: FALSE Diff: 2 Page Ref: 171 15) Osteoporosis of the spine often leads to kyphosis, often called the "dowager's hump" in elderly individuals. Answer: TRUE Diff: 3 Page Ref: 180-181

18 Copyright © 2014 Pearson Education, Inc.

 

7.3 Short Answer Questions 1) Fatal whiplash involves what bony structure? Answer: the dens Diff: 2 Page Ref: 174 2) The transverse costal facet articulates with this bony landmark on a rib. Answer: tubercle Diff: 2 Page Ref: 180 3) If a vertebra has a spinous process that is rather slender and points inferiorly, it is most likely what type of vertebra? Answer: thoracic Diff: 2 Page Ref: 175 4) Regarding the thoracic vertebrae, do the superior articular facets face anteriorly or posteriorly? Answer: posteriorly Diff: 3 Page Ref: 175 5) Surgery to remove part of the cranium is referred to as ________. Answer: a craniotomy Diff: 2 Page Ref: 182 6) What bony structure articulates with the inferior apex of the sacrum? Answer: the coccyx Diff: 2 Page Ref: 177-178 7) What are the three regions of the sternum, listed from most superior to most inferior? Answer: manubrium, body, xiphoid process Diff: 3 Page Ref: 179 8) The costal margin is formed by what structures? Be specific! Answer: the costal cartilages of ribs 7 through 10 Diff: 2 Page Ref: 179 9) This is the largest fontanelle of the skull. Answer: anterior fontanelle Diff: 2 Page Ref: 181 10) This bone of the axial skeleton does not articulate with any other bone. Answer: hyoid Diff: 1 Page Ref: 169-170 11) A nasal septum that is markedly off center is a(n) ________. Answer: deviated septum Diff: 2 Page Ref: 167 19 Copyright © 2014 Pearson Education, Inc.

 

12) What is the most common congenital abnormality of the skull? Answer: cleft palate Diff: 2 Page Ref: 180 13) What is the term for a smooth, flat articular surface on vertebrae? Answer: a facet Diff: 2 Page Ref: 173 14) The occipital, sphenoid, and ethmoid bones and parts of the temporal bone form from what type of embryonic tissue? Answer: cartilage Diff: 3 Page Ref: 181 15) Name the four longest sutures of the skull. Answer: coronal, sagittal, squamous, and lambdoid Diff: 3 Page Ref: 153-154 7.4 Essay Questions 1) List the bones that articulate at each of the four major sutures of the skull. Answer: Coronal: frontal and parietal bones. Squamous: parietal and temporal bones. Sagittal: between the parietal bones. Lambdoid suture: parietal and occipital bones. Diff: 2 Page Ref: 153-154 2) List the 14 bones of the face. Identify which are paired (with the letter P) and which are unpaired (using UP). Answer: Mandible (UP), maxillae (P), zygomatics (P), nasals (P), lacrimals (P), palatines (P), inferior nasal conchae (P), vomer (UP) Diff: 1 Page Ref: 165 3) Describe how the longitudinal ligaments, ligamentum flavum, and intervertebral discs control the flexibility of the vertebral column. Answer: The intervertebral discs are located between adjacent vertebrae, giving the vertebral column the ability to stretch and compress. The annulus fibrosus places limits on these motions. The anterior and posterior longitudinal ligaments prevent hyperextension and hyperflexion, respectively. The ligamentum flavum assists in recoil after anterior flexion of the vertebral column. Diff: 3 Page Ref: 171-172 4) What bony structures form the nasal cavity? (Hint: there are seven separate bones, and the number of parts of these bones is greater than the number of bones; name the parts as best you can.) Answer: The cribriform plates and the superior and middle conchae of the ethmoid, the palatine and frontal processes of the maxillae, the horizontal plates and the perpendicular processes of the palatine bones, the nasal bones, and the inferior nasal conchae. Diff: 3 Page Ref: 167-168 20 Copyright © 2014 Pearson Education, Inc.

 

5) Name the four bones that contain the paranasal sinuses. What lines these sinuses? What is the primary function of the sinuses? Why are they called paranasal sinuses? Answer: The ethmoid, sphenoid, frontal, and maxillary bones contain the sinuses. Mucous membranes line these cavities. They function in lightening the weight of the skull. They are called paranasal sinuses because they are in close proximity to the nasal cavity. Diff: 2 Page Ref: 168-169

21 Copyright © 2014 Pearson Education, Inc.

 

Human Anatomy, 7e (Marieb/Mitchell/Smith) Chapter 8 Bones, Part 2: The Appendicular Skeleton 8.1 Multiple Choice Questions

Figure 8.1 Use the diagram above to answer the following questions. 1) Which letter indicates the bone that has tuberosities that are the strongest parts of the hip bones? A) A B) B C) C D) D E) E Answer: E Diff: 2 Page Ref: 195 2) Which letter identifies the anterior superior iliac spine? A) A B) B C) C D) D E) E Answer: B Diff: 2 Page Ref: 195

1 Copyright © 2014 Pearson Education, Inc.

 

3) Which letter indicates a pad of a fibrocartilage known as the pubic symphysis? A) A B) B C) C D) D E) E Answer: D Diff: 2 Page Ref: 195 4) Which letter indicates the iliac crest? A) A B) B C) C D) D E) E Answer: A Diff: 2 Page Ref: 195 5) Which letter indicates a deep hemispherical socket where all three pelvic bones intersect? A) A B) B C) C D) D E) E Answer: C Diff: 1 Page Ref: 195

2 Copyright © 2014 Pearson Education, Inc.

 

Figure 8.2 Use the diagram above to answer the following questions. 6) Which letter indicates the groove that articulates with the patella? A) A B) B C) C D) D E) E Answer: E Diff: 2 Page Ref: 199 7) Which letter indicates the gluteal tuberosity of the femur? A) A B) B C) C D) D E) E Answer: D Diff: 3 Page Ref: 199

3 Copyright © 2014 Pearson Education, Inc.

 

8) Which letter indicates the weakest region of the femur, which may be fractured in a fall? A) A B) B C) C D) D E) E Answer: A Diff: 2 Page Ref: 199 9) Which letter indicates a depression that is the site where a femoral ligament attaches to the acetabulum? A) A B) B C) C D) D E) E Answer: B Diff: 2 Page Ref: 199 10) Which letter indicates the greater trochanter of the femur? A) A B) B C) C D) D E) E Answer: C Diff: 1 Page Ref: 199

4 Copyright © 2014 Pearson Education, Inc.

 

Figure 8.3 Use the diagram above to answer the following questions. 11) Which letter indicates the greater tubercle of the humerus? A) A B) B C) C D) D E) E Answer: E Diff: 2 Page Ref: 189 12) Which letter indicates the fossa where the ulna articulates with the humerus? A) A B) B C) C D) D E) E Answer: C Diff: 2 Page Ref: 189

5 Copyright © 2014 Pearson Education, Inc.

 

13) Which letter indicates the bony landmark of the humerus where the deltoid muscle attaches? A) A B) B C) C D) D E) E Answer: D Diff: 2 Page Ref: 189 14) Which letter indicates the bony landmark of the humerus that articulates with the head of the radius? A) A B) B C) C D) D E) E Answer: B Diff: 2 Page Ref: 189 15) Which letter indicates the anterior-most boundary of the intertubercular groove? A) A B) B C) C D) D E) E Answer: A Diff: 2 Page Ref: 189 16) The coronoid fossa is found on which of the bones listed below? A) radius B) tibia C) ulna D) humerus Answer: D Diff: 2 Page Ref: 190 17) The deltoid tuberosity is found on which of the bones listed below? A) radius B) tibia C) ulna D) humerus Answer: D Diff: 1 Page Ref: 189

6 Copyright © 2014 Pearson Education, Inc.

 

18) The intercondylar eminence is found on which of the bones listed below? A) radius B) tibia C) ulna D) humerus Answer: B Diff: 2 Page Ref: 200 19) The trochlear notch is found on which of the bones listed below? A) radius B) tibia C) ulna D) humerus Answer: C Diff: 3 Page Ref: 190 20) The olecranon process is found on which of the bones listed below? A) radius B) tibia C) ulna D) humerus Answer: C Diff: 2 Page Ref: 190 21) The ulnar notch is found on which of the bones listed below? A) radius B) tibia C) ulna D) humerus Answer: A Diff: 3 Page Ref: 190 22) The medial malleolus is found on which of the bones listed below? A) radius B) tibia C) ulna D) humerus Answer: B Diff: 2 Page Ref: 200

7 Copyright © 2014 Pearson Education, Inc.

 

23) The styloid process on lateral side of the antebrachium is found on which of the bones listed below? A) radius B) tibia C) ulna D) humerus Answer: A Diff: 3 Page Ref: 190 24) The bony landmark which is the attachment point for the patellar ligament is found on which of the bones listed below? A) radius B) tibia C) ulna D) humerus Answer: B Diff: 2 Page Ref: 202 25) The coronoid process is found on which of the bones listed below? A) radius B) tibia C) ulna D) humerus Answer: C Diff: 2 Page Ref: 190 26) Which bone is the keystone of the medial arch of the foot? A) calcaneus B) cuboid C) navicular D) talus Answer: D Diff: 3 Page Ref: 204 27) The tallest arch of the foot is the ________ arch. A) medial B) lateral C) transverse D) intermediate Answer: A Diff: 3 Page Ref: 203-204

8 Copyright © 2014 Pearson Education, Inc.

 

28) All of these bony landmarks contribute to the pelvic inlet (brim) except the A) sacral promontory. B) ischial tuberosities. C) arcuate lines on the ilia. D) pubic crests. Answer: B Diff: 3 Page Ref: 196 29) Which of the following statements about the patella is false? A) It is roughly triangular in shape. B) It is a sesamoid bone. C) It acts to protect the knee joint anteriorly. D) It articulates with the femur, tibia, and fibula. Answer: D Diff: 3 Page Ref: 199 30) The acromion of the scapula A) articulates with the humerus. B) attaches the biceps muscle of the arm. C) is an extension of the scapular spine. D) lies anterior to the coracoid process. Answer: C Diff: 2 Page Ref: 187-188 31) The bone of the forearm that directly and functionally articulates with the carpals is the A) ulna. B) radius. C) humerus. D) lunate. Answer: B Diff: 3 Page Ref: 192 32) The rotator cuff muscles attach to the humerus at the A) deltoid tuberosity. B) epicondyles. C) supracondylar ridges. D) tubercles. Answer: D Diff: 2 Page Ref: 189 33) The pubic bone is characterized by all of the following except that it A) forms part of the obturator foramen. B) forms part of the acetabulum. C) forms part of the greater sciatic notch. D) fuses with the ischium. Answer: C Diff: 2 Page Ref: 196-197 9 Copyright © 2014 Pearson Education, Inc.

 

34) Which of the following statements about the male pelvis is false? A) The bones are heavier and rougher than in the female. B) The male pelvis is narrow and deep. C) The male pubic arch is wider than that of the female. D) The cavity of the true pelvis is smaller in the female. Answer: C Diff: 2 Page Ref: 198 35) How is it possible to tell whether an isolated clavicle is the right or left one? A) The medial end is cone-shaped, the lateral end is flat, and the inferior surface is ridged. B) The lateral end is flat, the medial end is cone-shaped, and the superior surface is curved. C) The medial end is S-shaped, the lateral end is straight, and the superior surface is ridged. D) The coronoid process is on the lateral end, the ridge is on the superior surface, and the socket is in the medial end. Answer: A Diff: 3 Page Ref: 186-187 36) The area between the crest of the ilium and the arcuate line is called the A) false pelvis. B) pelvic inlet. C) pelvic outlet. D) true pelvis. Answer: A Diff: 2 Page Ref: 196 37) In anatomical position, A) the head of the radius is medial to the ulna. B) the radius is lateral to the ulna. C) the styloid process of the radius is medial to the ulna. D) the ulna is lateral to the radius. Answer: B Diff: 2 Page Ref: 190 38) The bone of the palm that articulates with the shortest digit is A) metacarpal I. B) metacarpal II. C) metacarpal III. D) metacarpal V. Answer: D Diff: 1 Page Ref: 192

10 Copyright © 2014 Pearson Education, Inc.

 

39) Which of the following statements concerning the fibula is false? A) It helps stabilize the ankle. B) It helps stabilize the knee. C) It is the thinnest of the two leg bones. D) It forms the lateral malleolus. Answer: B Diff: 3 Page Ref: 201-202 40) Which of the appendicular foramen listed below is "closed up" and has very few vessels or nerves passing through it? A) obturator foramen B) olfactory foramen C) foramen lacerum D) foramen magnum Answer: A Diff: 2 Page Ref: 197 41) Which of the bones listed below is the "heel" bone? A) hamate B) talus C) calcaneus D) cuboid Answer: C Diff: 1 Page Ref: 202-203 42) Which of the bones listed below is not found a carpal bone? A) cuboid B) triquetral C) scaphoid D) hamate Answer: A Diff: 2 Page Ref: 192 43) Which of the bones listed below is not part of the appendicular skeleton? A) patella B) femur C) sacrum D) navicular Answer: C Diff: 2 Page Ref: 194

11 Copyright © 2014 Pearson Education, Inc.

 

44) Which of these bone combinations represents a pair of pivoting bones? A) tibia and fibula B) ulna and radius C) pisiform and triquetrum D) clavicle and scapula Answer: B Diff: 2 Page Ref: 191 45) Which border of the scapula is proximal to the vertebral column? A) superior B) inferior C) lateral D) medial Answer: D Diff: 2 Page Ref: 187-188 46) Which of the bones listed below does not articulate with the clavicle? A) scapula B) rib 1 C) manubrium D) 7th vertebra Answer: D Diff: 3 Page Ref: 186-187 47) The anatomical neck of the humerus lies A) proximal to the tubercles. B) distal to the tubercles. C) beside the deltoid tuberosity. D) near the radial groove. Answer: A Diff: 2 Page Ref: 189 48) Which of the bony landmarks listed below is not part of the distal end of the humerus? A) capitulum B) trochlea C) intertubercular sulcus D) olecranon fossa Answer: C Diff: 2 Page Ref: 189 49) The coronoid process is part of the A) ulna. B) humerus. C) radius. D) carpal bones. Answer: A Diff: 2 Page Ref: 191 12 Copyright © 2014 Pearson Education, Inc.

 

50) Which tarsal bone lies directly anterior to the talus? A) lateral cuneiform B) navicular C) cuboid D) calcaneus Answer: B Diff: 2 Page Ref: 202 51) What is the cause of carpal tunnel syndrome? A) compression of the carpal bones B) fibrosis of the interosseous membrane C) compression of the median nerve D) damage to the radioulnar joint Answer: C Diff: 2 Page Ref: 194 52) The ischium joins the pubis in the acetabulum and at the A) superior pubic ramus. B) inferior pubic ramus. C) sacroiliac joint. D) pubic symphysis. Answer: B Diff: 2 Page Ref: 195 53) Which of the following bones primarily bears the weight of the body? A) talus B) navicular C) cuboid D) cuneiforms Answer: A Diff: 2 Page Ref: 203 54) The boundaries of the pelvic outlet include all those listed below except the A) ischial tuberosity. B) sacrum. C) coccyx. D) gluteal tuberosity. Answer: D Diff: 3 Page Ref: 197

13 Copyright © 2014 Pearson Education, Inc.

 

55) As it runs from the hip to the knee, the femur projects ________ as well as inferiorly. A) medially B) anteriorly C) laterally D) posteriorly Answer: A Diff: 3 Page Ref: 197 56) The pelvic inlet (brim) is widest A) from anterior to posterior. B) from side to side. C) obliquely. D) superior to inferior. Answer: B Diff: 2 Page Ref: 197 57) The auricular surface of the ilium A) attaches gluteal muscles. B) forms the lateral borders of the false pelvis. C) forms the sacroiliac joint. D) lines the interior of the acetabulum. Answer: C Diff: 2 Page Ref: 196 58) When an individual "breaks a hip," the most common location of the fracture is the A) acetabulum of the os coxa. B) diaphysis of the femur. C) iliac crest of the os coxa. D) neck of the femur. Answer: D Diff: 1 Page Ref: 197 59) The fibula A) articulates with the femur. B) crosses the tibia when the leg is rotated. C) forms the lateral border of the knee joint. D) stabilizes the ankle joint. Answer: D Diff: 2 Page Ref: 200, 202 60) At the shoulder joint, the head of the humerus articulates with the scapula at the A) glenoid cavity. B) coracoid process. C) subscapular fossa. D) acromion process. Answer: A Diff: 2 Page Ref: 187 14 Copyright © 2014 Pearson Education, Inc.

 

61) All of these bones are in the proximal row of four carpal bones except the A) scaphoid. B) triquetral. C) pisiform. D) trapezoid. Answer: D Diff: 2 Page Ref: 192 62) The supraspinous fossa is in the ________ region of the scapula. A) anterosuperior B) anteroinferior C) posterosuperior D) posteroinferior Answer: C Diff: 3 Page Ref: 187-188 63) The bony landmark at the lateral angle of the scapula is the A) suprascapular notch. B) glenoid cavity. C) spine. D) acromion. Answer: B Diff: 2 Page Ref: 187-188 64) The ischium has all of these features listed below except the A) ischial tuberosity. B) ischial spine. C) body. D) superior ramus. Answer: D Diff: 2 Page Ref: 196 65) Which of these bones of the appendicular skeleton are unpaired? A) humerus B) clavicle C) os coxae D) None; all appendicular bones are paired. Answer: D Diff: 1 Page Ref: 185-186 66) All of the bones of the digits are collectively known as A) phalanges. B) tarsals. C) carpals. D) metacarpals. Answer: A Diff: 1 Page Ref: 192 15 Copyright © 2014 Pearson Education, Inc.

 

67) The fibula has all of the following features except the A) head. B) inferior tibiofibular joint (facet). C) medial malleolus. D) lateral malleolus. Answer: C Diff: 2 Page Ref: 200 68) Which of the statements below regarding the metatarsals is false? A) They are numbered 1-5 from lateral to medial, just like the metacarpals. B) They articulate proximally to the cuneiforms and cuboid. C) They contribute to the three arches of the feet. D) They support some of the body's weight. Answer: A Diff: 2 Page Ref: 203 69) All of these statements regarding the acetabulum are true except A) The acetabulum is where the three pelvic bones intersect. B) The acetabulum participates in the hip joint. C) The acetabulum is cup-shaped. D) The acetabulum articulates with the sacrum. Answer: D Diff: 2 Page Ref: 194-195 70) The clavicle functions to A) transmit compression forces from the upper limb to the axial skeleton. B) protect the lungs. C) allow the arm to extend further for reaching. D) provide balance for the scapula on the other side of the body. Answer: A Diff: 2 Page Ref: 187 71) By anatomical definition the leg extends from the A) back (including the pelvis) to the tips of the toes. B) the top of the femur to the ankle. C) acetabulum to the metatarsals. D) knee to the ankle. Answer: D Diff: 2 Page Ref: 199, 201 72) The bone in the upper limb that corresponds to the femur in the lower limb is the A) humerus. B) radius. C) ulna. D) ischium. Answer: A Diff: 1 Page Ref: 187, 189 16 Copyright © 2014 Pearson Education, Inc.

 

73) The bone that has a trochlear notch, an olecranon process, and a coronoid process is the A) tibia. B) radius. C) ulna. D) femur. Answer: C Diff: 2 Page Ref: 190-191 74) The bone that has a trochlea, a capitulum, and a greater tubercle is the A) tibia. B) femur. C) humerus. D) fibula. Answer: C Diff: 2 Page Ref: 187, 190 75) These three tarsal bones lie just distal to the navicular are the A) cuneiforms. B) cuboids. C) trapezoids. D) naviculars. Answer: A Diff: 2 Page Ref: 202 8.2 True/False Questions 1) The coracoid process of the scapula articulates with the clavicle. Answer: FALSE Diff: 2 Page Ref: 186 2) The metacarpus is the palm of the hand. Answer: TRUE Diff: 1 Page Ref: 192 3) The thumb has no middle phalanx. Answer: TRUE Diff: 1 Page Ref: 192 4) The posterior inferior iliac spine is superior to the ischial spine. Answer: TRUE Diff: 3 Page Ref: 195 5) The subscapular fossa lies inferior to the spine and faces posteriorly. Answer: FALSE Diff: 2 Page Ref: 188

17 Copyright © 2014 Pearson Education, Inc.

 

6) The proximal end of the fibula is the lateral malleolus. Answer: FALSE Diff: 1 Page Ref: 200, 202 7) The tibia articulates distally with the trochlea of the talus. Answer: TRUE Diff: 3 Page Ref: 203 8) As one ages, the upper-lower (UL) body ratio changes from about 1.7:1 at birth to 1:1 at puberty. Answer: TRUE Diff: 3 Page Ref: 205 9) The greater and lesser tubercles are sites of muscle attachment on the proximal lateral aspect of the femur. Answer: FALSE Diff: 2 Page Ref: 189 10) The calcaneal tuberosity is the region of the foot that makes contact with the ground while walking. Answer: TRUE Diff: 2 Page Ref: 202 11) The distal end of the ulna is the olecranon process. Answer: FALSE Diff: 2 Page Ref: 190 12) The apex of the patella points distally. Answer: TRUE Diff: 2 Page Ref: 199 13) The pubic symphysis is a pad of hyaline cartilage between the two pubic bones. Answer: FALSE Diff: 3 Page Ref: 197 14) The ischium forms the posteroinferior region of the pelvic girdle. Answer: TRUE Diff: 2 Page Ref: 196 15) In the proximal row of carpals, there are the scaphoid, lunate, triquetral, and pisiform, from lateral to medial. Answer: TRUE Diff: 3 Page Ref: 192

18 Copyright © 2014 Pearson Education, Inc.

 

8.3 Short Answer Questions 1) The pectoral girdle consists of the clavicle and the ________. Answer: scapula Diff: 1 Page Ref: 186 2) When examining the clavicle, one would find the ________ tubercle at the acromial end. Answer: conoid Diff: 2 Page Ref: 186-187 3) In anatomical position, the ________ is the most lateral bone in the leg. Answer: fibula Diff: 2 Page Ref: 200 4) In the wrist, lateral to the lunate is the scaphoid, and medial to the lunate is the ________. Answer: triquetrum Diff: 2 Page Ref: 192 5) When the forearm is flexed at the elbow, the coronoid process of the ulna fits into the ________ fossa of the humerus. Answer: coronoid Diff: 2 Page Ref: 190 6) The large processes on the proximal end of the humerus are called tubercles, but the similar landmarks on the femur are called ________. Answer: trochanters Diff: 2 Page Ref: 199 7) The ________ is the tarsal bone lateral to the navicular. Answer: cuboid Diff: 2 Page Ref: 202 8) At the distal humerus, the trochlea articulates with the ________. Answer: ulna Diff: 2 Page Ref: 190 9) Most fingers and toes have a proximal, a middle, and a ________ phalanx. Answer: distal Diff: 1 Page Ref: 192 10) When you sit cross-legged for a while on a hard surface, your "rear end" hurts because you have been distributing weight directly over your bony ________ tuberosities. Answer: ischial Diff: 2 Page Ref: 196

19 Copyright © 2014 Pearson Education, Inc.

 

11) The ________ spine lies between the greater and lesser sciatic notches. Answer: ischial Diff: 2 Page Ref: 195 12) The ________ ligaments run from each lateral surface of the sacrum to the ischial tuberosities to hold the pelvis together. Answer: sacrotuberous Diff: 3 Page Ref: 196 13) The head of the femur has a ________, from which a ligament extends that attaches to the inner surface of the acetabulum. Answer: fovea capitis Diff: 2 Page Ref: 197 14) An interosseous membrane is found between the tibia and the ________ in the lower extremity. Answer: fibula Diff: 2 Page Ref: 200 15) The pelvic girdle consists of the paired ________. Answer: coxal bones Diff: 2 Page Ref: 194 8.4 Essay Questions 1) What are the anatomical differences between male and female athletes that result in a greater incidence of knee problems for women? Answer: The femur courses medially along its length toward the knee; because of the wider pelvis in women, this angle is even more pronounced. This results in a greater angle between the femur and the tibia, which is nearly vertical in position and leads to greater instability at the joint. Diff: 2 Page Ref: 197 2) Describe some of the anatomical characteristics of the female pelvis that are adaptations for childbearing. Answer: The female pelvis is typically wider, shallower, and lighter to provide more room in the true pelvis, through which the infant must pass during childbirth. The pubic arch is broader as well, reflecting the wider pelvis. The pelvis itself is tilted forward and has a larger enclosed cavity. Diff: 2 Page Ref: 198

20 Copyright © 2014 Pearson Education, Inc.

 

3) Identify the heads and all the notches of the radius and ulna, and explain how these bones articulate with each other and with the humerus. Answer: The radius has an ulnar notch at its distal end that articulates with the head of the ulna. The proximal end of the ulna has a radial notch that articulates with the head of the radius. The head of the radius articulates with the capitulum of the humerus. The trochlear notch of the ulna articulates with the trochlea of the humerus. Diff: 3 Page Ref: 190-191 4) Describe carpal tunnel syndrome. Answer: Carpal tunnel syndrome results from entrapment and/or compression of nerves passing through the carpal tunnel, on the anterior surface of the wrist. The median nerve is the primary nerve involved. When it is compressed, numbness, tingling, and weakness of the fingers ensues. Carpal tunnel syndrome is among a group of conditions called repetitive stress injuries. Diff: 2 Page Ref: 194 5) Describe two anatomical characteristics of the glenohumeral joint that contribute to the high degree of mobility at the shoulder. Answer: The scapula and upper limb are attached to the axial skeleton only by the clavicle; the scapula itself does not articulate with the axial skeleton. This allows the scapula to move freely over the thorax. Additionally, the humerus articulates with the essentially flat glenoid cavity, allowing for free rotation of the arm. Diff: 1 Page Ref: 186

21 Copyright © 2014 Pearson Education, Inc.

 

Human Anatomy, 7e (Marieb/Mitchell/Smith) Chapter 9 Joints 9.1 Multiple Choice Questions

Figure 9.1 Use the diagram above to answer the following questions. 1) Which letter indicates a ligament that connects bone to bone and is external to the joint capsule? A) A B) B C) C D) D E) E Answer: A Diff: 2 Page Ref: 212

1 Copyright © 2014 Pearson Education, Inc.

 

2) Which letter indicates an articulating surface that is comprised of hyaline cartilage? A) A B) B C) C D) D E) E Answer: C Diff: 1 Page Ref: 212 3) Which letter indicates the fibrous layer of the articular capsule of this synovial joint? A) A B) B C) C D) D E) E Answer: D Diff: 2 Page Ref: 212 4) Which letter indicates the joint/articular cavity that contains a small amount of synovial fluid? A) A B) B C) C D) D E) E Answer: B Diff: 2 Page Ref: 212 5) Which letter indicates the layer of the articular capsule that is the most highly vascularized? A) A B) B C) C D) D E) E Answer: E Diff: 3 Page Ref: 212

2 Copyright © 2014 Pearson Education, Inc.

 

Figure 9.2 Use the diagram above to answer the following questions. 6) Which letter indicates an example of an interphalangeal joint? A) A B) B C) C D) D E) E Answer: E Diff: 2 Page Ref: 222 7) Which letter indicates a cartilaginous, amphiarthrotic, symphysis type of joint? A) A B) B C) C D) D E) E Answer: B Diff: 2 Page Ref: 222

3 Copyright © 2014 Pearson Education, Inc.

 

8) Which letter indicates the proximal articulation between the tibia and fibula and is a diarthrotic plane joint? A) A B) B C) C D) D E) E Answer: D Diff: 2 Page Ref: 222 9) Which letter indicates an articulation, that in addition to the pubic symphysis, becomes slightly amphiarthrotic during pregnancy? A) A B) B C) C D) D E) E Answer: A Diff: 2 Page Ref: 222 10) Which letter on the diagram indicates a modified hinge joint? A) A B) B C) C D) D E) E Answer: C Diff: 1 Page Ref: 222

4 Copyright © 2014 Pearson Education, Inc.

 

Figure 9.3 Use the diagram above to answer the following questions. 11) Which letter indicates a synovial, diarthrotic, hinge type of joint? A) A B) B C) C D) D E) E Answer: B Diff: 2 Page Ref: 221 12) Which letter indicates the joint that is made more stable by the glenoid labrum? A) A B) B C) C D) D E) E Answer: A Diff: 2 Page Ref: 221

5 Copyright © 2014 Pearson Education, Inc.

 

13) Which letter indicates the knuckle or metacarpophalangeal joint,which is a diarthrotic joint? A) A B) B C) C D) D E) E Answer: E Diff: 1 Page Ref: 221 14) Which letter indicates the distal articulation between the radius and ulna? A) A B) B C) C D) D E) E Answer: C Diff: 1 Page Ref: 221 15) Which letter indicates a synovial, diarthrotic, saddle type of joint? A) A B) B C) C D) D E) E Answer: D Diff: 2 Page Ref: 221 16) Which of these joints would be best described as having a number of bursae, tendon sheaths, an acromion process, and a coracoid process? A) elbow joint B) shoulder joint C) sternoclavicular joint D) hip joint E) ankle joint Answer: B Diff: 3 Page Ref: 224-226 17) Trauma at which of these joints is more likely to result in a bone fracture than a dislocation? A) elbow joint B) shoulder joint C) sternoclavicular joint D) hip joint E) ankle joint Answer: C Diff: 2 Page Ref: 224

6 Copyright © 2014 Pearson Education, Inc.

 

18) Which of these joints is stabilized by an iliofemoral ligament, a pubofemoral ligament, and an ischiofemoral ligament? A) elbow joint B) shoulder joint C) sternoclavicular joint D) hip joint E) ankle joint Answer: D Diff: 1 Page Ref: 228 19) Which of these joints is a hinge joint, primarily involved in articulation with the ulna rather than the radius? A) elbow joint B) shoulder joint C) sternoclavicular joint D) hip joint E) ankle joint Answer: A Diff: 1 Page Ref: 226 20) Which of these joints utilizes the acetabulum? A) elbow joint B) shoulder joint C) sternoclavicular joint D) hip joint E) ankle joint Answer: D Diff: 1 Page Ref: 228 21) Which of these joints incorporates two bones, the first costal cartilage, and an articular disc? A) elbow joint B) shoulder joint C) sternoclavicular joint D) hip joint E) ankle joint Answer: C Diff: 1 Page Ref: 224 22) Which of these joints allows for dorsiflexion and plantar flexion? A) elbow joint B) shoulder joint C) sternoclavicular joint D) hip joint E) ankle joint Answer: E Diff: 1 Page Ref: 232 7 Copyright © 2014 Pearson Education, Inc.

 

23) Which of these joints is one of the most freely moving joints of the body, but requires the stability provided by the rotator cuff muscles to keep it in place? A) elbow joint B) shoulder joint C) sternoclavicular joint D) hip joint E) ankle joint Answer: B Diff: 1 Page Ref: 224 24) Which of these joints is stabilized by glenohumeral ligaments? A) elbow joint B) shoulder joint C) sternoclavicular joint D) hip joint E) ankle joint Answer: B Diff: 1 Page Ref: 224 25) Which of these joints is stabilized with an annular ligament? A) elbow joint B) shoulder joint C) sternoclavicular joint D) hip joint E) ankle joint Answer: A Diff: 2 Page Ref: 226 26) Which structure(s) directly secures the humerus to the glenoid cavity? A) the rotator cuff B) the glenohumeral ligaments C) the coracohumeral ligament D) the tendon of the biceps brachii muscle Answer: D Diff: 2 Page Ref: 224 27) Which of these is not characteristic of a synchondrosis? A) absence of a joint cavity B) bone ends attached by collagen C) composed of hyaline cartilage D) not highly movable Answer: B Diff: 2 Page Ref: 210

8 Copyright © 2014 Pearson Education, Inc.

 

28) What structures are most important in keeping the knee from moving medially to laterally? A) the collateral ligaments B) the menisci C) the patellar ligament D) the oblique popliteal ligaments Answer: A Diff: 2 Page Ref: 229-231 29) A joint capsule (articular capsule) has two layers. The function of the capsule's internal layer is to A) strengthen the joint against tension. B) produce synovial fluid. C) contain all of the nerves and blood vessels of the joint. D) act as a meniscus. Answer: B Diff: 2 Page Ref: 211-212 30) When the fibrous tissue of sutures ossifies, the resulting joint is called a A) symphysis. B) synchondrosis. C) syndesmosis. D) synostosis. Answer: D Diff: 3 Page Ref: 209 31) The condition in which excessive amounts of synovial fluid effuse into a joint cavity is called A) chondromalacia. B) ankylosing spondylitis. C) arthroplasty. D) synovitis. Answer: D Diff: 2 Page Ref: 237 32) An example of a synarthrotic fibrous joint is the A) sagittal suture. B) interosseous membrane between the radius and the ulna. C) pubic symphysis. D) intervertebral discs. Answer: A Diff: 2 Page Ref: 209

9 Copyright © 2014 Pearson Education, Inc.

 

33) Which of the factors listed below contributes least to hip joint stability? A) muscle tone B) ligaments in the articular capsule C) the deep socket D) ligaments attached to the head of the femur Answer: D Diff: 3 Page Ref: 228 34) An example of a pivot joint is the A) atlantoaxial joint B) distal tibiofibular joint. C) sagittal suture. D) suture. Answer: A Diff: 3 Page Ref: 220 35) An example of a saddle-shaped synovial joint is the A) radioulnar joint B) temporomandibular joint. C) carpometacarpal of digit 1. D) glenohumeral joint. Answer: C Diff: 2 Page Ref: 221 36) Articular cartilages are found both in symphyses and in A) sutures. B) synovial joints. C) synchondroses. D) syndesmoses. Answer: B Diff: 2 Page Ref: 211 37) Synovial joint cavities are the only important exceptions to the rule that cavities in the body are always lined with epithelia. The synovial membrane consists of A) loose connective tissue. B) muscle tissue. C) nervous tissue. D) dense irregular connective tissue. Answer: A Diff: 3 Page Ref: 211

10 Copyright © 2014 Pearson Education, Inc.

 

38) The main function of synovial fluid is A) cooling. B) nourishing bone. C) removing metabolic wastes. D) lubrication. Answer: D Diff: 1 Page Ref: 211 39) In abduction of the fingers, the fifth finger moves medially, and the second finger moves A) anteriorly. B) laterally. C) medially. D) by flexing toward the palm. Answer: B Diff: 2 Page Ref: 217 40) By hyperextending a thigh at the hip joint, you could A) hit your chin with your knee. B) perform the same movement as circumduction. C) squeeze both thighs together. D) bring your knee and leg to a position posterior to the thorax. Answer: D Diff: 2 Page Ref: 214 41) The largest ball-and-socket joint in the body is the A) hip. B) shoulder. C) knee. D) interphalangeal joint. Answer: A Diff: 1 Page Ref: 228 42) A condition that commonly leads to neck pain, ear problems, and pain when opening the mouth is A) osteoarthritis. B) temporomandibular disorder. C) sutural disorders. D) gout. Answer: B Diff: 1 Page Ref: 223

11 Copyright © 2014 Pearson Education, Inc.

 

43) Besides helping to "lock" the knee, the posterior cruciate ligament A) prevents posterior sliding of the tibia when the leg is flexed at the knee. B) holds the patella to the tibia. C) prevents all medial rotation on the femoral condyles. D) breaks more frequently than the anterior cruciate ligament. Answer: A Diff: 2 Page Ref: 230-231 44) As an essential stage in the locking mechanism of the knee, A) both cruciate and collateral ligaments tighten. B) the popliteal muscle contracts. C) the fibula is pulled out of the way. D) the femur rotates laterally on the tibial condyles. Answer: A Diff: 3 Page Ref: 230-231 45) Student's elbow is an example of A) tendonitis. B) bursitis. C) osteoarthritis. D) gout. Answer: B Diff: 2 Page Ref: 235 46) A joint between a tooth and its socket is A) a suture. B) a gomphosis. C) an ankylosis. D) a synostosis. Answer: B Diff: 1 Page Ref: 210 47) A chronic disorder of joints in which the articular cartilages degenerate and bony spurs form is A) rheumatoid arthritis. B) osteoarthritis. C) gout. D) Lyme disease. Answer: B Diff: 2 Page Ref: 235

12 Copyright © 2014 Pearson Education, Inc.

 

48) Which of the response pairs listed below does not correctly pair the joint category with its functional degree of mobility? A) suture: synarthrosis B) symphysis: amphiarthrosis C) synchondrosis: amphiarthrosis D) synovial: diarthrosis Answer: C Diff: 2 Page Ref: 209-210 49) Capsular ligaments A) are covered by synovial membrane. B) are located outside the joint capsule. C) are thickened parts of the joint capsule itself. D) separate the synovial cavity into compartments. Answer: C Diff: 2 Page Ref: 211-212 50) Cartilaginous joints A) allow free movement. B) allow movement in only one plane as a hinge. C) are seen in sutural lines. D) include symphyses. Answer: D Diff: 2 Page Ref: 210 51) What movement occurs when one moves the foot from the anatomical position to point the toes laterally, with the foot flat on the floor? A) eversion B) lateral rotation C) inversion D) plantar flexion Answer: B Diff: 2 Page Ref: 215 52) Articular discs are found in all of the following joints except the A) sternoclavicular. B) temporomandibular. C) tibiofemoral. D) vertebrocostal. Answer: D Diff: 3 Page Ref: 213

13 Copyright © 2014 Pearson Education, Inc.

 

53) Of the joints listed below, the only joint with a relatively shallow or flat articular surface is the A) ankle joint. B) hip joint. C) shoulder joint. D) elbow joint. Answer: C Diff: 3 Page Ref: 223-224 54) In pronation, A) the palm faces anteriorly. B) the radius and ulna are crossed. C) the forearm is laterally rotated. D) the ulna rotates, but the radius does not. Answer: B Diff: 2 Page Ref: 217 55) What type of excessive motion do anterior ligaments resist? A) abduction B) adduction C) extension D) flexion Answer: C Diff: 3 Page Ref: 220 56) An example of a diarthrotic synchondrosis A) is the epiphyseal plate. B) is the intertarsal joint. C) is the knee. D) does not exist. Answer: D Diff: 3 Page Ref: 210 57) An example of an amphiarthrotic cartilaginous joint is the A) epiphyseal plate. B) pubic symphysis. C) knee joint. D) sternocostal joint. Answer: B Diff: 2 Page Ref: 210

14 Copyright © 2014 Pearson Education, Inc.

 

58) The main movements occurring at the ankle joint are A) plantar flexion and dorsiflexion. B) inversion and eversion. C) abduction and adduction. D) supination and pronation. Answer: A Diff: 3 Page Ref: 232 59) Which of the following movements is not possible at the condyloid metacarpophalangeal joints of fingers 2-5? A) flexion B) circumduction C) adduction D) rotation Answer: D Diff: 3 Page Ref: 221 60) An example of a multiaxial joint is A) the proximal radioulnar joint. B) an intervertebral joint, between the articular processes. C) the hip. D) the pubic symphysis. Answer: C Diff: 2 Page Ref: 217, 219 61) The only category of articulations with a joint cavity is a A) fibrous joint. B) cartilaginous joint. C) synovial joint. D) synostosis. Answer: C Diff: 1 Page Ref: 210-211 62) Which of these statements about menisci is false? A) They contain fibrocartilage. B) They are found in all synovial joints. C) They can divide the joint cavity. D) They can allow two movements at a single joint. Answer: B Diff: 3 Page Ref: 213

15 Copyright © 2014 Pearson Education, Inc.

 

63) Synovial fluid is A) identical to blood plasma. B) an extract from the bone marrow. C) a filtrate of the blood, with added glycoproteins. D) fluid from edema. Answer: C Diff: 2 Page Ref: 211 64) Tendon sheaths A) surround all tendons. B) contain articular cartilages. C) differ from bursae in that they are not lubricating in function. D) wrap the tendons that are crowded in the carpal tunnel. Answer: D Diff: 2 Page Ref: 213 65) By plantar flexing your feet at the ankle joints, you will A) stand on your toes. B) stand back on your heels. C) stand on the medial margins of your feet. D) turn your big toes laterally. Answer: A Diff: 2 Page Ref: 216-217 66) Gliding movements occur between all these joints except A) adjacent carpals. B) adjacent phalanges. C) articular processes of vertebrae. D) temporomandibular joints. Answer: B Diff: 2 Page Ref: 214, 222-224 67) Based on the definition of luxation, a subluxation must be A) a mild case of Lyme disease. B) rheumatoid arthritis on one side of the body only. C) reduction of a joint. D) a partially dislocated joint. Answer: D Diff: 3 Page Ref: 235 68) All of these stabilizing structures provide structural support to the hip joint except the A) iliofemoral ligament. B) ischiofemoral ligament. C) pubofemoral ligament. D) ligament of the head of the femur. Answer: D Diff: 3 Page Ref: 228-229 16 Copyright © 2014 Pearson Education, Inc.

 

69) A bursa differs from the synovial cavity of a joint in that it A) contains no synovial fluid. B) is not involved in lubrication. C) has no synovial membrane. D) need not be attached to any bone. Answer: D Diff: 2 Page Ref: 213 70) Pulling your shoulders back, or squaring them, involves which motion of the scapula? A) abduction B) pronation C) retraction D) opposition Answer: C Diff: 2 Page Ref: 216-217 71) A pre-adolescent has more joints than an adult because of these immovable cartilaginous joints. A) sutures B) epiphyseal plates C) synovial joints D) gomphoses Answer: B Diff: 3 Page Ref: 209 72) All of the following can be performed at the wrist except A) flexion and extension of the hand. B) circumduction of the hand. C) rotation of the hand. D) abduction and adduction of the hand. Answer: C Diff: 3 Page Ref: 221 73) Biaxial joints cannot A) abduct. B) adduct. C) flex. D) rotate. Answer: D Diff: 1 Page Ref: 218-219

17 Copyright © 2014 Pearson Education, Inc.

 

74) The lateral movement of the arms away from the body is called A) abduction. B) adduction. C) flexion. D) extension. Answer: A Diff: 1 Page Ref: 215 75) Expansion of the rib cage during inhalation is possible because the costovertebral joints are A) amphiarthrotic. B) biaxial. C) synchondroses. D) synovial. Answer: D Diff: 2 Page Ref: 220 9.2 True/False Questions 1) Synarthroses are freely movable joints. Answer: FALSE Diff: 2 Page Ref: 209 2) One type of cartilage, fibrocartilage, characterizes all cartilaginous joints. Answer: FALSE Diff: 2 Page Ref: 210 3) The term synovial joint contains the root word ov, referring to ovum, or egg, because the synovial fluid is viscous like the white of the egg. Answer: TRUE Diff: 3 Page Ref: 211 4) The interosseous membrane is a type of syndesmosis. Answer: TRUE Diff: 3 Page Ref: 210 5) Synovial fluid is slippery because of glycoproteins that are secreted by fibroblasts located within the articular cartilage. Answer: FALSE Diff: 2 Page Ref: 211 6) Typical synovial joints are supplied by blood vessels and nerve fibers. Answer: TRUE Diff: 2 Page Ref: 212

18 Copyright © 2014 Pearson Education, Inc.

 

7) Hyaline cartilage forms many menisci, such as are found in the temporomandibular joint and the knee joint. Answer: FALSE Diff: 2 Page Ref: 213 8) Arthroplasty is the process of replacing a diseased joint with an artificial joint. Answer: TRUE Diff: 3 Page Ref: 237 9) The sternoclavicular joint (SC) is a modified hinge joint. Answer: FALSE Diff: 2 Page Ref: 224 10) Intracapsular ligaments, such as the cruciate ligaments, are covered with a synovial membrane. Answer: TRUE Diff: 3 Page Ref: 211-212 11) Amphiarthroses are more movable than diarthroses. Answer: FALSE Diff: 2 Page Ref: 209 12) One could characterize a bursa as a sac of synovial fluid. Answer: TRUE Diff: 1 Page Ref: 213 13) A tendon sheath is an elongated membranous sac filled with synovial fluid that wraps around a tendon. Answer: TRUE Diff: 2 Page Ref: 213 14) A person who states that they are "double-jointed" is capable of joint hyperextension and has more phalangeal joints than the average person. Answer: FALSE Diff: 3 Page Ref: 214 15) The atlantoaxial joint is a saddle joint. Answer: FALSE Diff: 3 Page Ref: 220 9.3 Short Answer Questions 1) The glenohumeral joint is an example of a(n) ________ joint. Answer: ball-and-socket Diff: 2 Page Ref: 221

19 Copyright © 2014 Pearson Education, Inc.

 

2) This kind of joint is composed of very short connective tissue fibers that are continuous with the periosteum of the flat bone. Answer: suture Diff: 2 Page Ref: 209 3) This joint is structurally reinforced by talofibular and tibiofibular ligaments. Answer: ankle joint Diff: 2 Page Ref: 232-233 4) An important factor in joint stabilization is ________, which is exerted by muscles even when there is no movement at that joint. Answer: muscle tone Diff: 3 Page Ref: 220 5) A(n) ________ is a specialized bursa that wraps around a tendon to reduce friction. Answer: tendon sheath Diff: 3 Page Ref: 213 6) Perhaps the simplest synovial joint, a(n) ________ joint occurs when two flat surfaces of bones move over each other. Answer: gliding Diff: 3 Page Ref: 213 7) This is the action by which the thumb touches the tips of the other fingers. Answer: Opposition Diff: 2 Page Ref: 217 8) ________ causes the forearm to rotate laterally. Answer: Supination Diff: 2 Page Ref: 217 9) The sternoclavicular joint is a(n) ________ joint. Answer: saddle Diff: 2 Page Ref: 224 10) Excessive dorsiflexion of the ankle can overstretch tibiofibular ligaments, resulting in a special type of injury known as a(n) ________ ankle sprain. Answer: syndesmosis Diff: 3 Page Ref: 234 11) The most freely moving large joint of the body is the ________ joint. Answer: shoulder (glenohumeral) Diff: 2 Page Ref: 224 12) Baseball players often damage the ________ cuff at the shoulder region. Answer: rotator Diff: 1 Page Ref: 224 20 Copyright © 2014 Pearson Education, Inc.

 

13) To ________ the foot, move the plantar surface laterally. Answer: evert Diff: 2 Page Ref: 216-217 14) This is a type of arthritis in which crystals of urate accumulate in the synovial membranes. Answer: gout (gouty arthritis) Diff: 2 Page Ref: 236 15) Synovial joints form from ________ tissue that fills the spaces between the cartilaginous bone models in the late embryo. Answer: mesenchymal Diff: 3 Page Ref: 237 9.4 Essay Questions 1) Describe the joint-associated problems that can occur as normal changes to the articular cartilage as an individual ages. Answer: Osteoarthritis to some degree, will typically affect most individuals as they grow older. Damage done to joints when an individual is young can cause problems as a person ages. Metalloproteinase enzymes released during joint damage may break down the matrix of the articular cartilage. As time goes by, recurring microdamage diminishes the thickness of the articular cartilage and ultimately results in chronic inflammation and diminished mobility at the joint(s). Regular exercise and non-steroidal anti-inflammatory drugs (NSAIDs) can minimize inflammation, alleviating some of the symptoms of osteoarthritis. Diff: 2 Page Ref: 235 2) Describe the three types of fibrous joints, including examples of each. Answer: Sutures are located between many of the bones of the skull, with fibers of the joint continuous with the periosteum of the adjacent bones. Syndesmoses are connected by long fibrous ligaments, such as those found at the distal tibiofibular articulation. Considerable motion is possible at these types of joints, even though there is no synovial capsule. Gomphoses are found between the roots of the teeth and the tooth sockets of the mandible and maxilla. The teeth are held in place with numerous, short periodontal ligaments. Diff: 2 Page Ref: 209-210 3) Compare the structure and function of the two major joint surfaces of the wrist. Answer: The radiocarpal joint is a condyloid joint between the radius and the scaphoid and lunate. This joint allows for movements of flexion, extension, abduction, adduction, and circumduction. The intercarpal joints between adjacent carpals are planar joints, which allow for gliding movements. Diff: 3 Page Ref: 221, 227

21 Copyright © 2014 Pearson Education, Inc.

 

4) Characterize articular cartilages. Answer: Articular cartilages are found at diarthrotic synovial joints. They cover the ends (epiphyses) of articulating bones and minimize friction and enhance resistance to compressive forces within the joint capsule. Articular cartilage is hyaline cartilage. Diff: 2 Page Ref: 211 5) Name the major components of the synovial joint of the knee. Answer: Articular cartilages, tendon of the quadriceps muscle, patellar ligament, medial and lateral patellar retinacula, medial and lateral menisci, oblique popliteal ligament, arcuate popliteal ligament, anterior and posterior cruciate ligaments, tibial and fibular collateral ligaments, femur, tibia, fibula, patella. Diff: 3 Page Ref: 229-232

22 Copyright © 2014 Pearson Education, Inc.

 

Human Anatomy, 7e (Marieb/Mitchell/Smith) Chapter 10 Skeletal Muscle Tissue 10.1 Multiple Choice Questions

Figure 10.1 Use the diagram above to answer the following questions. 1) This structure is composed entirely of dense regular connective tissue and connects bone to muscle. A) A B) B C) C D) D E) E Answer: A Diff: 2 Page Ref: 244-245 2) Identify the letter that indicates the endomysium. A) A B) B C) C D) D E) E Answer: D Diff: 2 Page Ref: 244-245 1 Copyright © 2014 Pearson Education, Inc.  

 

3) This indicates an individual fascicle. A) A B) B C) C D) D E) E Answer: C Diff: 2 Page Ref: 244-245 4) Identify the letter that indicates the epimysium. A) A B) B C) C D) D E) E Answer: B Diff: 2 Page Ref: 244-245 5) Identify the letter that indicates the muscle fiber. A) A B) B C) C D) D E) E Answer: E Diff: 2 Page Ref: 244-245

2 Copyright © 2014 Pearson Education, Inc.  

 

Figure 10.2 Use the diagram above to answer the following questions. 6) This structure is composed of thick (myosin) filaments. A) A B) B C) C D) D E) E Answer: E Diff: 2 Page Ref: 246-248 7) This structure is the basic unit of contraction. A) A B) B C) C D) D E) E Answer: B Diff: 2 Page Ref: 246-248

3 Copyright © 2014 Pearson Education, Inc.  

 

8) Identify the letter that indicates the titin filament. A) A B) B C) C D) D E) E Answer: D Diff: 2 Page Ref: 246-248 9) This structure is composed of thin (actin) filaments. A) A B) B C) C D) D E) E Answer: C Diff: 2 Page Ref: 246-248 10) Identify the letter that indicates the I band. A) A B) B C) C D) D E) E Answer: A Diff: 2 Page Ref: 246-248 11) This type of muscle makes up the walls of hollow organs, such as the stomach and uterus. A) skeletal muscle B) smooth muscle C) cardiac muscle Answer: B Diff: 2 Page Ref: 242, 244 12) This type of muscle is found in the heart. A) skeletal muscle B) smooth muscle C) cardiac muscle Answer: C Diff: 1 Page Ref: 242 13) This type of muscle contains intercalated discs. A) skeletal muscle B) smooth muscle C) cardiac muscle Answer: C Diff: 2 Page Ref: 255 4 Copyright © 2014 Pearson Education, Inc.  

 

14) This type of muscle includes the fast oxidative fibers. A) skeletal muscle B) smooth muscle C) cardiac muscle Answer: A Diff: 2 Page Ref: 252-253 15) This type of muscle is found in large vessels leading to and from the heart. A) skeletal muscle B) smooth muscle C) cardiac muscle Answer: B Diff: 2 Page Ref: 242, 244 16) A cell of this type of muscle is striated and can be uninucleated or binucleated. A) skeletal muscle B) smooth muscle C) cardiac muscle Answer: C Diff: 2 Page Ref: 254 17) Both single-unit smooth muscle and this type of muscle have gap junctions. A) skeletal muscle B) smooth muscle C) cardiac muscle Answer: C Diff: 1 Page Ref: 255 18) The "cells" of both skeletal muscle and this muscle type are correctly called muscle fibers. A) serous muscle B) smooth muscle C) cardiac muscle Answer: B Diff: 2 Page Ref: 244 19) Both cardiac muscle and this type of muscle are called involuntary. A) skeletal muscle B) smooth muscle C) cardiac muscle Answer: B Diff: 2 Page Ref: 242, 244

5 Copyright © 2014 Pearson Education, Inc.  

 

20) This type of muscle attaches to bone, but may also attach to skin, cartilage, fascia or a raphe. A) skeletal muscle B) smooth muscle C) cardiac muscle Answer: A Diff: 1 Page Ref: 245-246 21) This type of muscle may be affected by rhabdomyolysis. A) skeletal muscle B) smooth muscle C) cardiac muscle Answer: A Diff: 2 Page Ref: 253 22) This type of muscle, along with cardiac muscle, is called visceral muscle. A) skeletal muscle B) smooth muscle C) cardiac muscle Answer: B Diff: 2 Page Ref: 244 23) This type of muscle composes the largest share of muscle weight in the human body. A) skeletal muscle B) smooth muscle C) cardiac muscle Answer: A Diff: 3 Page Ref: 246-248 24) The biceps and the deltoid muscle are of this type. A) skeletal muscle B) smooth muscle C) cardiac muscle Answer: A Diff: 2 Page Ref: 242 25) Contractions of these muscles are under voluntary control. A) skeletal muscle B) smooth muscle C) cardiac muscle Answer: A Diff: 1 Page Ref: 242

6 Copyright © 2014 Pearson Education, Inc.  

 

26) Much of the natural elasticity of skeletal muscle tissue is provided by A) actin. B) myosin. C) tendons. D) connective tissue sheaths. Answer: D Diff: 2 Page Ref: 245 27) Which of these is not in direct contact with thick myofilaments? A) myosin B) actin C) ATPase D) synaptic vesicles Answer: D Diff: 2 Page Ref: 249-251 28) Sarcoplasmic reticulum lies A) between fibers but in myofilaments. B) between myofibrils but in fibers. C) between fascicles but outside fibers. D) between muscles but in myofilaments. Answer: B Diff: 2 Page Ref: 248-249 29) Which statement about muscle attachments is false? A) An insertion can be at either attachment point of a muscle, depending on body position and the movement being performed. B) At least one joint is present between an origin and insertion. C) Insertions are usually distal to the origin. D) Upon contraction, the origin is pulled toward the insertion. Answer: D Diff: 3 Page Ref: 245-246 30) An aponeurosis A) connects a muscle to underlying structures through a flat sheet or web. B) consists of a neuron and all the muscle fibers it innervates. C) is a type of direct attachment of muscle to bone. D) is the junction between the axon terminus of a neuron to an individual muscle fiber. Answer: A Diff: 2 Page Ref: 246

7 Copyright © 2014 Pearson Education, Inc.  

 

31) A common characteristic of the three types of muscle tissue is that A) they all have striations. B) contraction is triggered by the release of calcium. C) they all contain sarcomeres. D) they can all use aerobic and anaerobic methods to utilize energy. Answer: B Diff: 1 Page Ref: 255 32) Which of the following correctly defines a triad of the sarcomere? A) a T tubule and two adjacent terminal cisterns B) actin, myosin, and titin C) an axon terminal, synaptic cleft, and sarcolemma D) actin, troponin, and tropomyosin Answer: A Diff: 2 Page Ref: 248 33) Eccentric contraction A) generates force as the muscle lengthens. B) generates force without changing the length of the muscle. C) pulls the insertion toward the origin. D) shortens the muscle. Answer: A Diff: 2 Page Ref: 249 34) Sarcopenia is A) the loss of muscle mass with age. B) a decrease in the muscle's need for energy. C) a decrease in the amount of actin in the muscle fiber. D) a decrease in the amount of sarcolemma in the muscle fiber. Answer: A Diff: 2 Page Ref: 258 35) The embryonic origin of muscle tissue is from A) endoderm. B) mesoderm. C) ectoderm. D) epidermis. Answer: B Diff: 2 Page Ref: 257 36) As skeletal muscles enlarge in a weight lifter, all of the following occur except A) muscle cells divide mitotically. B) muscle cells grow larger. C) myofilaments become more abundant in the muscle cells. D) myofibrils become more abundant in the muscle cells. Answer: A Diff: 3 Page Ref: 253, 255 8 Copyright © 2014 Pearson Education, Inc.  

 

37) Delayed-onset muscle soreness is caused by A) accumulation of lactic acid. B) depletion of ATP during prolonged activity. C) microscopic tears and resulting inflammation. D) overnight cramps that occur after strenuous exercise. Answer: C Diff: 3 Page Ref: 249 38) Titin A) connects myosin to actin. B) forms the Z disk to which actin attaches. C) limits the degree to which a muscle may contract. D) prevents a muscle from being overstretched. Answer: D Diff: 2 Page Ref: 248 39) Visceral muscle refers to A) skeletal muscle and cardiac muscle. B) cardiac muscle and smooth muscle. C) skeletal muscle and smooth muscle. D) smooth muscle only. Answer: B Diff: 2 Page Ref: 244 40) The sarcoplasmic reticulum (SR) stores A) calcium. B) sodium. C) ATPase. D) myosin. Answer: A Diff: 1 Page Ref: 248-249 41) Of the various types of skeletal muscle fibers, the type with the most mitochondria is A) slow oxidative fibers. B) fast glycolytic fibers. C) fast oxidative fibers. D) slow glycolytic fibers. Answer: A Diff: 2 Page Ref: 252-253

9 Copyright © 2014 Pearson Education, Inc.  

 

42) Of the various types of skeletal muscle fibers, the type that produces the most power is A) slow oxidative fibers. B) fast glycolytic fibers. C) fast oxidative fibers. D) slow glycolytic fibers. Answer: B Diff: 3 Page Ref: 252-253 43) Of the various types of skeletal muscle fibers, the type containing the most glycogen granules is A) slow oxidative fibers. B) fast glycolytic fibers. C) fast oxidative fibers. D) slow glycolytic fibers. Answer: B Diff: 3 Page Ref: 252-253 44) The only muscle cells that branch are ________ muscle cells. A) skeletal B) smooth C) cardiac D) red Answer: C Diff: 1 Page Ref: 254 45) Which type of muscle fiber has no myofibrils? A) skeletal B) smooth C) cardiac D) red Answer: B Diff: 2 Page Ref: 254 46) Which protein strengthens the sarcolemma by connecting the cytoskeleton with the extracellular matrix? A) actin B) dystrophin C) myosin D) titin Answer: B Diff: 3 Page Ref: 256

10 Copyright © 2014 Pearson Education, Inc.  

 

47) Which type of muscle fiber has caveolae but no T tubules? A) skeletal B) cardiac C) smooth D) white Answer: C Diff: 3 Page Ref: 254 48) The area that contains no thin filaments is known as the A) H zone. B) I band. C) intercalated disc. D) A band. Answer: A Diff: 2 Page Ref: 246 49) With intense resistance training, A) fast glycolytic fibers can convert to fast oxidative fibers. B) fast oxidative fibers can convert to fast glycolytic fibers. C) slow oxidative fibers can convert to fast glycolytic fibers. D) slow oxidative fibers can convert to fast oxidative fibers. Answer: A Diff: 3 Page Ref: 253, 255 50) Myoglobin A) binds and stores oxygen for ATP production. B) is found within the T tubules. C) is released by the sarcoplasmic reticulum. D) provides energy for contraction. Answer: A Diff: 1 Page Ref: 253 51) Myofascial pain syndrome is best described as a condition in which A) microscopic tears in muscle fibers result in swelling and inflammation. B) muscle fibers degenerate because of chronic leakage of extracellular calcium. C) muscle fibers contract when the skin superficial to them is stroked. D) pain occurs in at least 11 of 18 standardized points across the body. Answer: C Diff: 2 Page Ref: 256 52) Age-related loss of muscle mass may be influenced by all of these except: A) an increase in the relative proportion of connective tissue to number of muscle fibers. B) decrease in the level of testosterone. C) degeneration of muscle fibers because of accumulation of calcium in the sarcoplasm. D) exhaustion of the supply of muscle satellite cells. Answer: B Diff: 2 Page Ref: 258 11 Copyright © 2014 Pearson Education, Inc.  

 

53) Acetylcholine A) binds to the myosin head, enabling it to form cross-bridges with actin. B) binds to the sarcolemma and initiates an impulse in the muscle fiber. C) breaks down the neurotransmitter that activates muscle fibers. D) is released from the sarcoplasmic reticulum and binds to actin. Answer: B Diff: 2 Page Ref: 251 54) The sleevelike tubular network within skeletal muscle cells is the A) sarcoplasmic reticulum. B) rough endoplasmic reticulum. C) myofibrils. D) T tubules. Answer: A Diff: 1 Page Ref: 248 55) The type of attachment in which the muscle fibers seem to attach directly to a bone is A) a tendon. B) an aponeurosis. C) an insertion. D) a fleshy attachment. Answer: D Diff: 2 Page Ref: 246 56) Which region of the sarcomere does not change in length during contraction? A) A band B) H zone C) I band D) Z disk to Z disk Answer: A Diff: 2 Page Ref: 250 57) In limbs, the insertions of muscles almost always lie ________ to their origins. A) proximal B) distal C) lateral D) posterior Answer: B Diff: 2 Page Ref: 245-246

12 Copyright © 2014 Pearson Education, Inc.  

 

58) When the distance between two adjacent Z disks grows shorter, the muscle fiber is experiencing A) concentric contraction. B) eccentric contraction. C) isometric contraction. D) relaxation. Answer: A Diff: 3 Page Ref: 250 59) The ability of the sarcolemma of muscle cells to conduct an impulse is an example of A) contractility. B) excitability. C) extensibility. D) elasticity. Answer: B Diff: 2 Page Ref: 242 60) The characteristic of muscle tissue that results more from its connective tissue components than from its muscle cells is A) contractility. B) excitability. C) extensibility. D) elasticity. Answer: D Diff: 3 Page Ref: 242, 245 61) A muscle fascicle is A) a bundle of cells. B) a single muscle cell. C) a bundle of myofilaments. D) a tendon. Answer: A Diff: 1 Page Ref: 245 62) An aponeurosis is A) a large muscle. B) a nerve to a muscle. C) clinical pain in a muscle. D) a sheet of dense connective tissue. Answer: D Diff: 1 Page Ref: 246

13 Copyright © 2014 Pearson Education, Inc.  

 

63) What causes a strain or muscle pull? A) a tear in a muscle B) a tear in a tendon C) pain in a muscle due to any muscle disorder D) a sudden involuntary spasm of a muscle Answer: A Diff: 1 Page Ref: 258 64) Of the three classes of muscle cells, the only one in which the nuclei lie peripherally instead of centrally is A) skeletal. B) cardiac. C) smooth. D) visceral. Answer: A Diff: 2 Page Ref: 254 65) In striated muscle cells, which of these structures stores calcium ions that trigger contraction? A) the internal surface of the plasma membrane B) the terminal cisterns C) T tubules D) the myofibrils Answer: B Diff: 2 Page Ref: 248 66) In striated muscle, the I band is where A) only thick filaments occur. B) thin filaments occur. C) H zones occur. D) thick and thin filaments occur. Answer: B Diff: 2 Page Ref: 248 67) Of the various types of skeletal muscle fibers, the fibers that experience fatigue sooner are A) slow oxidative fibers. B) fast glycolytic fibers. C) fast oxidative fibers. D) slow glycolytic fibers. Answer: B Diff: 2 Page Ref: 252-253

14 Copyright © 2014 Pearson Education, Inc.  

 

68) Of the various types of skeletal muscle fibers, the fibers most resistant to fatigue are A) slow oxidative fibers. B) fast glycolytic fibers. C) fast oxidative fibers. D) slow glycolytic fibers. Answer: A Diff: 2 Page Ref: 252-253 69) Of the various types of skeletal muscle fibers, the ones with the thinnest myofibrils are A) slow oxidative fibers. B) fast glycolytic fibers. C) fast oxidative fibers. D) slow glycolytic fibers. Answer: A Diff: 3 Page Ref: 252-253 70) Of the various types of skeletal muscle fibers, the ones supplied by the most capillaries are A) slow oxidative fibers. B) fast glycolytic fibers. C) fast oxidative fibers. D) slow glycolytic fibers. Answer: A Diff: 2 Page Ref: 252-253 71) One of the largest and strongest muscles in the body is the gluteus maximus in the buttocks, which is important in these diverse muscular activities; walking, running, and climbing stairs. It must consist of A) slow oxidative fibers only. B) fast glycolytic fibers only. C) fast oxidative fibers only. D) a mixture of fiber types. Answer: D Diff: 2 Page Ref: 252-253 72) In skeletal muscle fibers, which band or zone contains both thick and thin myofilaments? A) Z B) H C) I D) A Answer: D Diff: 2 Page Ref: 246-247

15 Copyright © 2014 Pearson Education, Inc.  

 

73) In muscular dystrophy, A) muscles decrease in size because of loss of fat and connective tissue. B) muscle fibers degenerate and atrophy. C) most forms of the disease do not appear to be inherited. D) most cases appear in young females. Answer: B Diff: 3 Page Ref: 255 74) Muscle tissue is endowed with all of the following properties except A) excitability. B) contractibility. C) extensibility. D) transmissibility. Answer: D Diff: 2 Page Ref: 242 75) After a muscle fiber has contracted, the calcium A) is destroyed. B) is chemically bound to the myofilaments. C) is secreted by the Golgi apparatus. D) is actively transported into the sarcoplasmic reticulum. Answer: D Diff: 2 Page Ref: 249 10.2 True/False Questions 1) Muscle tissue can be characterized as being excitable and elastic, but not extensible. Answer: FALSE Diff: 2 Page Ref: 242 2) Skeletal muscle is responsible for movements of the appendages, but not the abdomen. Answer: FALSE Diff: 1 Page Ref: 242 3) The origin of a muscle can also be an insertion, depending on the position of the body and the movement being performed. Answer: TRUE Diff: 3 Page Ref: 245-246 4) Of the surrounding connective tissues of the muscle, the endomysium is the most superficial. Answer: FALSE Diff: 2 Page Ref: 244-245 5) Skeletal muscle fibers are multinucleate, because they arose from a number of embryonic cells. Answer: TRUE Diff: 2 Page Ref: 257 16 Copyright © 2014 Pearson Education, Inc.  

 

6) The repeating segments seen in a skeletal muscle fiber are the T tubules. Answer: FALSE Diff: 2 Page Ref: 257 7) The Z disks, sometimes referred to as Z lines, define each sarcomere at both ends. Answer: TRUE Diff: 2 Page Ref: 246 8) Fascicles are bundles of muscle fibers wrapped together within perimysium. Answer: TRUE Diff: 2 Page Ref: 244-245 9) During contraction, the A band of a sarcomere shortens. Answer: FALSE Diff: 3 Page Ref: 250 10) Delayed-onset muscle soreness is caused by inflammation that results from microscopic tears in muscle fibers following exceptionally strenuous muscle activity. Answer: TRUE Diff: 3 Page Ref: 249 11) Muscular dystrophy is a sex-linked, dominant gene that primarily affects females. Answer: FALSE Diff: 1 Page Ref: 255-256 12) Fibromyalgia is a muscular disease, and its cause is known to be genetic. Answer: FALSE Diff: 2 Page Ref: 256 13) Skeletal muscles never undergo mitosis after they are fully formed. Answer: TRUE Diff: 2 Page Ref: 257 14) Contraction of a muscle cell requires the influx of calcium into the sarcoplasm. Answer: TRUE Diff: 2 Page Ref: 249-250 15) Actin and myosin shorten while the muscle is contracting. Answer: FALSE Diff: 2 Page Ref: 250

17 Copyright © 2014 Pearson Education, Inc.  

 

10.3 Short Answer Questions 1) How do muscles enlarge when undergoing athletic training? Answer: by increasing the number and diameter of contractile proteins; myofibrils, and myofilaments Diff: 2 Page Ref: 253, 255 2) Fusion of embryonic ________ cells from the mesoderm creates multinucleated skeletal muscle fibers. Answer: myoblast Diff: 2 Page Ref: 257 3) During contraction, the ________ shortens, and the ________ disappears completely. Answer: I band, H zone Diff: 2 Page Ref: 250 4) Muscles in the appendages typically have their origins ________ to the insertion. Answer: proximal Diff: 2 Page Ref: 245-246 5) Typically, each skeletal muscle is supplied by one ________ and one ________. Answer: artery, nerve Diff: 2 Page Ref: 245 6) Within a fascicle, every muscle fiber is surrounded by the ________. Answer: endomysium Diff: 2 Page Ref: 244-245 7) The ________ filament is composed of myosin. Answer: thick Diff: 2 Page Ref: 246 8) The mechanism for muscle contraction is the ________. Answer: sliding filament mechanism Diff: 2 Page Ref: 249 9) This protein adds to muscle elasticity. Answer: titin Diff: 3 Page Ref: 248 10) ________ is a specialized smooth endoplasmic reticulum found in muscle fibers. Answer: Sarcoplasmic reticulum Diff: 2 Page Ref: 248 11) Muscle cells that produce ATP anaerobically via glycolysis are called ________. Answer: glycolytic fibers Diff: 2 Page Ref: 252-253 18 Copyright © 2014 Pearson Education, Inc.  

 

12) Most muscles that are involved in maintaining posture contain high percentages of ________ fibers. Answer: slow oxidative Diff: 2 Page Ref: 252-253 13) The difference in color between skeletal muscle fibers is caused by the abundance of ________. Answer: myoglobin Diff: 2 Page Ref: 252-253 14) During contraction, there is no change in the length of the ________ region of the sarcomere. Answer: A band Diff: 2 Page Ref: 250 15) Muscle cells that produce ATP aerobically are called ________. Answer: oxidative fibers Diff: 2 Page Ref: 252-253 10.4 Essay Questions 1) Describe the differences in the mixture of fiber types that are present in the calf muscles of a sprinter compared to those of a long-distance runner. Explain the anatomical basis for the differences between these fiber types. Answer: A sprinter might have a higher proportion of fast glycolytic fibers, whereas the longdistance runner might have relatively more fast-oxidative fibers. Both fiber types generate forces quickly, and their large diameters, which are proportional to the number of myofibrils, enable them to produce substantial forces for the race. The glycolytic fibers have fewer mitochondria and less myoglobin, so they are prone to fatigue more quickly than the oxidative fibers and, therefore, are better suited for short sprints. Diff: 3 Page Ref: 252-253 2) Describe muscular dystrophy. Answer: Muscular dystrophy is a recessive, sex-linked disease and affects 1 in 3500 boys. The most familiar form is Duchenne muscular dystrophy, and it has been found that the muscle fibers lack a protein called dystrophin that links the cytoskeleton to the extracellular matrix. Future treatment may involve injection of myoblasts into the unhealthy muscle fibers. Children affected by this disease slowly lose muscular control and often die of respiratory infections or respiratory failure. Diff: 2 Page Ref: 255-256 3) Name four functions of muscle tissue. Answer: Movement, maintenance of posture, joint stabilization, open and close many body passageways and heat generation Diff: 2 Page Ref: 242

19 Copyright © 2014 Pearson Education, Inc.  

 

4) List the arrangement of muscle fibers in a muscle and their connective tissue fibers. Answer: Each muscle fiber is covered by the endomysium, and fibers are bundled into fascicles, which are covered with perimysium. The fascicles compose the muscle itself, which is covered with the epimysium. Diff: 1 Page Ref: 244-245 5) Describe how the components of a sarcomere change during contraction. Answer: The distance between the Z discs shortens, the I bands narrow, and H zones disappear. The length of A bands during contraction does not change. Diff: 2 Page Ref: 250

20 Copyright © 2014 Pearson Education, Inc.  

 

Human Anatomy, 7e (Marieb/Mitchell/Smith) Chapter 11 Muscles of the Body 11.1 Multiple Choice Questions

Figure 11.1 Use the diagram above to answer the following questions. 1) Identify the letter that indicates the Linea alba. A) A B) B C) C D) D E) E Answer: B Diff: 2 Page Ref: 294 2) Identify the letter that indicates the Internal oblique. A) A B) B C) C D) D E) E Answer: D Diff: 2 Page Ref: 294

1 Copyright © 2014 Pearson Education, Inc.

 

3) Identify the letter that indicates the Serratus anterior. A) A B) B C) C D) D E) E Answer: A Diff: 2 Page Ref: 294 4) Identify the letter that indicates the Rectus abdominis. A) A B) B C) C D) D E) E Answer: C Diff: 2 Page Ref: 294 5) Identify the letter that indicates the Aponeurosis of external oblique. A) A B) B C) C D) D E) E Answer: E Diff: 1 Page Ref: 294

2 Copyright © 2014 Pearson Education, Inc.

 

Figure 11.2 Use the diagram above to answer the following questions. 6) Identify the letter that indicates the Masseter. A) A B) B C) C D) D E) E Answer: E Diff: 2 Page Ref: 280 7) Identify the letter that indicates the Orbicularis oris. A) A B) B C) C D) D E) E Answer: D Diff: 2 Page Ref: 280

3 Copyright © 2014 Pearson Education, Inc.

 

8) Identify the letter that indicates the Temporalis. A) A B) B C) C D) D E) E Answer: B Diff: 2 Page Ref: 280 9) Identify the letter that indicates the Epicranial aponeurosis. A) A B) B C) C D) D E) E Answer: A Diff: 2 Page Ref: 280 10) Identify the letter that indicates the Zygomaticus major and minor. A) A B) B C) C D) D E) E Answer: C Diff: 2 Page Ref: 280

4 Copyright © 2014 Pearson Education, Inc.

 

Figure 11.3 Use the diagram above to answer the following questions. 11) Identify the letter that indicates the Sternocleidomastoid. A) A B) B C) C D) D E) E Answer: A Diff: 2 Page Ref: 301 12) Identify the letter that indicates the Pectoralis minor. A) A B) B C) C D) D E) E Answer: D Diff: 2 Page Ref: 301 13) Identify the letter that indicates the Deltoid. A) A B) B C) C D) D E) E Answer: B Diff: 2 Page Ref: 301

5 Copyright © 2014 Pearson Education, Inc.

 

14) Identify the letter that indicates the Pectoralis major. A) A B) B C) C D) D E) E Answer: C Diff: 2 Page Ref: 301 15) Identify the letter that indicates the Serratus anterior. A) A B) B C) C D) D E) E Answer: E Diff: 2 Page Ref: 301 16) The muscle fascicle arrangement for Orbicularis oris is best characterized as A) parallel. B) bipennate. C) convergent. D) unipennate. E) circular. Answer: E Diff: 2 Page Ref: 263 17) This arrangement of muscle fascicles tends to be triangular in shape. The Pectoralis major is an example of a muscle with this type of muscle fascicle arrangement. A) parallel B) bipennate C) convergent D) unipennate E) circular Answer: C Diff: 2 Page Ref: 263 18) The rectus femoris demonstrates this type of muscle fascicle arrangement. A) parallel B) bipennate C) convergent D) unipennate E) circular Answer: B Diff: 2 Page Ref: 263

6 Copyright © 2014 Pearson Education, Inc.

 

19) Muscles with this type of muscle fascicle arrangement are either fusiform or straplike in appearance. A) parallel B) bipennate C) convergent D) unipennate E) circular Answer: A Diff: 2 Page Ref: 263 20) The Extensor digitorum longus demonstrates this type of fascicle arrangement. A) parallel B) bipennate C) convergent D) unipennate E) circular Answer: D Diff: 2 Page Ref: 263 21) A muscle with fascicles that insert into only one side of the tendon is categorized as A) parallel. B) bipennate. C) convergent. D) unipennate. E) circular. Answer: D Diff: 1 Page Ref: 263 22) A muscle with fascicles that insert into the tendon from both sidesis categorized as A) parallel. B) bipennate. C) convergent. D) unipennate. E) circular. Answer: B Diff: 2 Page Ref: 263 23) In addition to the Pectoralis major, another muscle that displays this pattern of fascicle arrangement is Latissimus dorsi. A) parallel B) bipennate C) convergent D) unipennate E) circular Answer: C Diff: 2 Page Ref: 263 7 Copyright © 2014 Pearson Education, Inc.

 

24) The Sartorius muscle, a thin straplike muscle would have muscle fascicles arranged in A) parallel. B) bipennate. C) convergent. D) unipennate. E) circular. Answer: A Diff: 2 Page Ref: 263 25) This arrangement of fascicles tends to provide the greatest range of motion (shortening distance), though not the greatest power. A) parallel B) bipennate C) convergent D) unipennate E) circular Answer: A Diff: 2 Page Ref: 263 26) An example of a muscle with this type of fascicle arrangement is the orbicularis oris. A) parallel B) bipennate C) convergent D) unipennate E) circular Answer: E Diff: 2 Page Ref: 263 27) The long axes of the fascicles run parallel to the long axis of the muscle is a description of this type of fascicle arrangement. A) parallel B) bipennate C) convergent D) unipennate E) circular Answer: A Diff: 2 Page Ref: 263 28) When muscle fascicles are arranged in concentric rings this arrangement is referred to as A) parallel. B) bipennate. C) convergent. D) unipennate. E) circular. Answer: E Diff: 2 Page Ref: 263 8 Copyright © 2014 Pearson Education, Inc.

 

29) The biceps brachii has this fascicle arrangement, which maximizes the range of motion possible. A) parallel B) bipennate C) convergent D) unipennate E) circular Answer: A Diff: 2 Page Ref: 263 30) The arrangement of muscle fascicles in a sphincter is A) parallel. B) bipennate. C) convergent. D) unipennate. E) circular. Answer: E Diff: 2 Page Ref: 263 31) Prominent process posterior to each ear A) anatomical snuffbox B) gluteal fold C) posterior median furrow D) mastoid E) popliteal fossa Answer: D Diff: 2 Page Ref: 333 32) Neck region containing portions of the cervical and brachial plexus and the subclavian artery A) femoral triangle B) posterior triangle C) medial bicipital furrow D) cubital fossa E) supracristal line Answer: B Diff: 2 Page Ref: 335 33) Vertical groove in the center of the back A) suprachristal line B) linea semilunaris C) medial bicipital furrow D) posterior median furrow E) posterior triangle Answer: D Diff: 2 Page Ref: 336-337

9 Copyright © 2014 Pearson Education, Inc.

 

34) Groove running from the anterior superior iliac spine (ASIS) to the pubic tubercle A) femoral triangle B) inguinal ligament C) linea semilunaris D) midclavicular line E) anatomical snuffbox Answer: B Diff: 2 Page Ref: 335 35) Site of intravenous catheter insertion and blood draws A) femoral triangle B) posterior triangle C) medial bicipital furrow D) cubital fossa E) supracristal line Answer: D Diff: 2 Page Ref: 339 36) Marks the lateral margin of the rectus abdominis A) femoral triangle B) inguinal ligament C) linea semilunaris D) midclavicular line E) anatomical snuffbox Answer: C Diff: 2 Page Ref: 336 37) Ischial tuberosities lie just above the medial aspect of this landmark. A) posterior triangle B) ventral gluteal site C) gluteal fold D) posterior median fossa E) cubital fossa Answer: C Diff: 2 Page Ref: 343 38) Depression defined by the inguinal ligament, sartorius, and adductor longus muscles A) femoral triangle B) inguinal ligament C) linea semilunaris D) midclavicular line E) anatomical snuffbox Answer: A Diff: 2 Page Ref: 343

10 Copyright © 2014 Pearson Education, Inc.

 

39) A horizontal line on the back used for locating the fourth lumbar vertebra A) femoral triangle B) posterior triangle C) medial bicipital furrow D) cubital fossa E) supracristal line Answer: E Diff: 3 Page Ref: 336-337 40) A triangular depression on the anterior surface of the elbow A) femoral triangle B) posterior triangle C) medial bicipital furrow D) cubital fossa E) supracristal line Answer: D Diff: 3 Page Ref: 339 41) Marks the medial boundary of the biceps brachii A) femoral triangle B) posterior triangle C) median bicipital furrow D) cubital fossa E) supracristal line Answer: C Diff: 3 Page Ref: 339 42) Palpation of this landmark may help diagnose a fracture of the scaphoid or radius. A) femoral triangle B) inguinal ligament C) linea semilunaris D) midclavicular line E) anatomical snuffbox Answer: E Diff: 2 Page Ref: 340 43) A depression on the posterior of the knee A) femoral triangle B) posterior triangle C) medial bicipital furrow D) popliteal fossa E) supracristal line Answer: D Diff: 2 Page Ref: 343

11 Copyright © 2014 Pearson Education, Inc.

 

44) This vertical line passes 1 cm medial to the nipple. A) femoral triangle B) inguinal ligament C) linea semilunaris D) midclavicular line E) anatomical snuffbox Answer: D Diff: 2 Page Ref: 335 45) Intramuscular injections in adults are frequently given in this region to avoid the sciatic nerve. A) posterior triangle B) ventral gluteal site C) gluteal fold D) posterior median fossa E) cubital fossa Answer: B Diff: 2 Page Ref: 341 46) The prime mover for trunk flexion when doing a sit-up is the A) rectus abdominis. B) erector spinae. C) splenius capitis. D) splenius cervicis. Answer: A Diff: 2 Page Ref: 293-294 47) Shin splints result from damage to which muscle or muscle group? A) soleus B) hamstrings C) tibialis anterior D) adductors of the thigh Answer: C Diff: 2 Page Ref: 344 48) The fascicles of the biceps brachii muscle have this type of arrangement. A) bipennate B) convergent C) fusiform D) multipennate Answer: C Diff: 2 Page Ref: 263

12 Copyright © 2014 Pearson Education, Inc.

 

49) The extensor muscles of the upper limb lie almost exclusively in the ________ region of that limb. A) posterior B) anterior C) medial D) lateral Answer: A Diff: 1 Page Ref: 271 50) The main "swallowing" muscles are the A) esophageal muscles. B) pharyngeal constrictors. C) buccinators. D) masseters. Answer: B Diff: 2 Page Ref: 284 & 286 51) When the diaphragm contracts, it A) opens a hole in its center through which air flows. B) aids in expiration. C) becomes flatter. D) becomes more dome-shaped. Answer: C Diff: 2 Page Ref: 292 52) Which type of fascicle arrangement shortens the least? A) bipennate B) circular C) fusiform D) parallel Answer: A Diff: 2 Page Ref: 263 53) In this arrangement of fascicles, the fascicles insert into only one side of the tendon. A) multipennate B) bipennate C) unipennate D) parallel Answer: B Diff: 2 Page Ref: 263

13 Copyright © 2014 Pearson Education, Inc.

 

54) A prime mover for flexion of the arm at the shoulder is the A) pectoralis major. B) deltoid. C) latissimus dorsi. D) coracobrachialis. Answer: A Diff: 3 Page Ref: 302 55) A prime mover for extension of the forearm at the elbow is the A) brachialis. B) triceps brachii. C) biceps brachii. D) anconeus. Answer: B Diff: 2 Page Ref: 305 56) A prime mover for flexing the forearm at the elbow is the A) triceps brachii. B) brachialis. C) brachioradialis. D) anconeus. Answer: B Diff: 2 Page Ref: 305 57) An important function of the soleus muscle is to A) extend the leg at the knee. B) evert the foot. C) invert the foot. D) plantar flex the foot. Answer: D Diff: 2 Page Ref: 326 58) A muscle that originates on the medial epicondyle of the humerus is the A) flexor carpi radialis. B) short head of the triceps. C) brachioradialis. D) extensor digitorum. Answer: A Diff: 2 Page Ref: 306 59) The muscle that originates on the anterior superior iliac spine is the A) rectus femoris. B) sartorius. C) pectineus. D) psoas major. Answer: B Diff: 2 Page Ref: 316 14 Copyright © 2014 Pearson Education, Inc.

 

60) A muscle that originates on the lateral surfaces of the superior eight ribs is the A) rectus abdominis. B) subclavius. C) serratus anterior. D) subscapularis. Answer: C Diff: 2 Page Ref: 298 61) The prime movers of jaw closure and biting are the A) temporalis and buccinator muscles. B) masseter and temporalis muscles. C) buccinator and masseter muscles. D) temporalis and lateral pterygoid muscles. Answer: B Diff: 2 Page Ref: 282-283 62) A muscle that attaches to the skeleton at the pubic crest and the xiphoid process and costal cartilages of ribs 5-7 is the A) rectus abdominis. B) external oblique. C) transversus abdominis. D) internal intercostal. Answer: A Diff: 2 Page Ref: 294 63) A muscle that originates along most of the shaft of the femur is the A) iliopsoas. B) adductor magnus. C) vastus intermedius. D) semitendinosus. Answer: C Diff: 2 Page Ref: 316 64) Besides the coracobrachialis, what other muscle originates on the coracoid process of the scapula? A) triceps brachii (part of it) B) biceps brachii (part of it) C) subscapularis D) infraspinatus Answer: B Diff: 3 Page Ref: 305

15 Copyright © 2014 Pearson Education, Inc.

 

65) Which muscle inserts on the lesser tubercle of the humerus? A) subscapularis B) supraspinatus C) biceps brachii D) deltoid Answer: A Diff: 2 Page Ref: 304 66) The main function of the flexor retinaculum in the wrist is to A) reduce friction. B) hold blood vessels. C) bind the radius, ulna, and carpals together. D) prevent the flexor tendons from moving anteriorly when all fingers are flexed. Answer: D Diff: 3 Page Ref: 306 67) A synergist muscle can do all of the following except A) add extra force to a movement. B) hold a bone in place to create a stable base for the prime mover. C) prevent inappropriate extra movements that might otherwise be produced. D) reverse a particular movement caused by the prime mover. Answer: D Diff: 2 Page Ref: 268 68) The most superficial muscle of the anterior abdominal wall is the A) external oblique. B) transversus abdominis. C) internal oblique. D) rectus abdominis. Answer: A Diff: 3 Page Ref: 293 69) Which is not a criterion for naming a muscle? A) action B) color C) location D) shape Answer: B Diff: 1 Page Ref: 270

16 Copyright © 2014 Pearson Education, Inc.

 

70) A nursing infant develops a powerful sucking muscle that adults also use for whistling. This muscle is the A) platysma. B) buccinator. C) zygomaticus. D) masseter. Answer: B Diff: 2 Page Ref: 280 71) Visceral muscle differs in its embryonic origin from skeletal muscle because it is derived from A) myotomes. B) myotomes and somitomeres. C) somitomeres. D) splanchnic mesoderm. Answer: D Diff: 3 Page Ref: 266 72) The major muscles used in throwing a punch are the A) latissimus dorsi and deltoid. B) teres major and brachialis. C) brachioradialis and biceps brachii. D) serratus anterior and triceps brachii. Answer: D Diff: 3 Page Ref: 298 & 305 73) The biceps brachii operates in a ________-class lever system. A) first B) second C) third D) fourth Answer: C Diff: 2 Page Ref: 265-266 74) Based on their embryonic origin, which set of muscles is most closely related? A) chewing muscles—tongue muscles B) extrinsic eye muscles—pharyngeal constrictors C) suprahyoid muscles—infrahyoid muscles D) trapezius—erector spinae Answer: B Diff: 3 Page Ref: 266-268

17 Copyright © 2014 Pearson Education, Inc.

 

75) The gastrocnemius operates in a ________-class lever system. A) first B) second C) third D) fourth Answer: B Diff: 2 Page Ref: 264-266 76) The deepest muscle of the anterior abdominal wall is the A) external oblique. B) transversus abdominis. C) internal oblique. D) rectus abdominis. Answer: B Diff: 3 Page Ref: 293 77) The external urethral sphincter A) is part of the pelvic diaphragm. B) is part of the uterus. C) touches the deep transverse perineal muscle. D) is smooth muscle that involuntarily inhibits urination when we are frightened. Answer: C Diff: 3 Page Ref: 296-297 78) The muscle used to depress the tongue is the A) genioglossus. B) hyoglossus. C) styloglossus. D) digastric. Answer: B Diff: 2 Page Ref: 282 79) Which set correctly matches the function of the designated muscle compartment? A) anterior thigh—thigh extension B) lateral leg—foot eversion C) medial thigh—thigh abduction D) posterior leg—dorsiflexion Answer: B Diff: 2 Page Ref: 276-278 80) The agonist for jaw closure is the A) digastric (posterior head). B) digastric (anterior head). C) medial pterygoid. D) masseter. Answer: D Diff: 3 Page Ref: 282 18 Copyright © 2014 Pearson Education, Inc.

 

81) Which set does not correctly match the function and innervation of the designated muscle compartment? A) anterior thigh—thigh flexion—femoral nerve B) lateral leg—foot inversion—deep fibular nerve C) medial thigh—thigh adduction—obturator nerve D) posterior leg—plantar flexion—tibial nerve Answer: B Diff: 3 Page Ref: 276-278 82) A muscle that inserts on the lateral surface of the ramus of the mandible but not on the coronoid process is the A) lateral pterygoid. B) digastric. C) masseter. D) temporalis. Answer: C Diff: 2 Page Ref: 282 83) A muscle inserting on the pisiform bone of the wrist is A) the flexor carpi ulnaris. B) a lumbrical. C) the extensor digitorum. D) the pronator teres. Answer: A Diff: 3 Page Ref: 307 84) A muscle that originates on both the spine of the scapula and the clavicle is the A) pectoralis major. B) trapezius. C) deltoid. D) subclavius. Answer: C Diff: 2 Page Ref: 302 85) An individual with damage to the radial nerve would be unlikely to contract the A) biceps brachii. B) coracobrachialis. C) extensor digitorum. D) pronator teres. Answer: C Diff: 2 Page Ref: 309-310

19 Copyright © 2014 Pearson Education, Inc.

 

86) A muscle that opens the mouth is the A) orbicularis oculi. B) buccinator. C) orbicularis oris. D) digastric. Answer: D Diff: 2 Page Ref: 284 87) The latissimus dorsi originates primarily on the A) linea alba. B) humerus. C) vertebral column. D) clavicle. Answer: C Diff: 2 Page Ref: 302-303 88) A muscle that inserts on the lesser trochanter of the femur is the A) iliopsoas. B) adductor magnus. C) vastus intermedius. D) semitendinosus. Answer: A Diff: 2 Page Ref: 315 89) A prime mover for extending the arm at the shoulder is the A) latissimus dorsi. B) pectoralis major. C) triceps brachii. D) serratus anterior. Answer: A Diff: 2 Page Ref: 272 90) A prime mover of foot inversion is the A) flexor accessorius. B) abductor hallucis. C) tibialis posterior. D) soleus. Answer: C Diff: 2 Page Ref: 275

20 Copyright © 2014 Pearson Education, Inc.

 

91) When the adductor muscles of the thigh adduct the thigh at the hip, what muscle or muscles act as antagonists to this movement? A) pectineus B) triceps surae C) gluteus minimus and medius D) gluteus maximus Answer: C Diff: 3 Page Ref: 274 92) A prime mover for lateral rotation at the shoulder is the A) pectoralis major. B) supraspinatus. C) teres major. D) teres minor. Answer: D Diff: 2 Page Ref: 272 93) This muscle inserts into an aponeurosis on the anterior of the hand. A) flexor digitorum superficialis B) palmar interossei C) palmaris longus D) pronator quadratus Answer: C Diff: 2 Page Ref: 306-307 94) This muscle inserts into the central tendon of the perineum. A) bulbospongiosus B) coccygeus C) levator ani D) superficial transverse perineal Answer: D Diff: 2 Page Ref: 296 95) Paralysis of which muscle(s) would make a person unable to flex the leg at the knee? A) gastrocnemius and hamstrings B) gluteal muscles C) peroneus D) soleus Answer: A Diff: 2 Page Ref: 319, 326

21 Copyright © 2014 Pearson Education, Inc.

 

96) Which of the following statements concerning surface anatomy is false? A) It studies internal organs as they relate to surface landmarks. B) It serves as a basis for a standard physical examination. C) It is best studied in cadavers. D) It is used to take pulses, insert tubes, make surgical incisions, and perform other clinical procedures. Answer: C Diff: 1 Page Ref: 332 97) The spine of which vertebra is most prominent in the posterior midline of the neck? A) C1 B) C2 C) C7 D) T2 Answer: C Diff: 1 Page Ref: 334 98) Which of the following structures forms the roof of the anterior triangle of the neck? A) the inferior margin of the mandible B) the clavicle C) the sternocleidomastoid muscle D) the midline of the neck Answer: A Diff: 1 Page Ref: 335 99) Which vertebra lies at the intersection of the supracristal lines and is used as the site to insert a needle to obtain a sample of cerebrospinal fluid (CSF)? A) C7 B) T12 C) L1 D) L4 Answer: D Diff: 2 Page Ref: 337 100) The best site to listen to lung sounds is A) the triangle of auscultation. B) the anterior triangle. C) the posterior triangle. D) over the xiphoid process. Answer: A Diff: 2 Page Ref: 337

22 Copyright © 2014 Pearson Education, Inc.

 

101) The sternal angle is used to locate A) the apex of the heart. B) rib 2. C) the cardiac sphincter of the stomach. D) the bifurcation of the trachea. Answer: B Diff: 1 Page Ref: 335 102) The linea semilunaris on the abdomen A) is the lateral groove made by each rectus abdominis muscle in lean people. B) is a crescent spot where appendicitis pain is usually located. C) represents the tendinous insertions of the subdivided rectus abdominis muscle. D) is the edge of the external oblique muscle seen in coughing or straining individuals. Answer: A Diff: 2 Page Ref: 336 103) A deltoid injection is given A) above the acromion. B) lateral to the clavicle. C) 5 cm below the greater tubercle of the humerus. D) 2 inches above the most lateral bony landmark on the superior surface of the shoulder. Answer: C Diff: 1 Page Ref: 339 104) How do physicians diagnose a "Colles fracture" of the radius (fracture near the distal epiphysis) using surface anatomy? A) They palpate the styloid process of the radius and find that it has moved proximally from its normal position. B) They extend the forearm, place three fingers in the lateral dimple, and ask the patient to rotate the hand as if turning a doorknob. C) They ask the patient to flex the forearm against resistance and then look for any bulges that formed. D) They ask the patient to extend the thumb of the affected hand and to point it posteriorly, and then they look for the formation of a triangle on the back of the hand. Answer: A Diff: 3 Page Ref: 191, 340 105) The thenar eminence is located at the A) base of the little finger. B) base of the thumb. C) tips of the fingers. D) bulges formed by the gluteal muscles (commonly known as the "cheeks"). Answer: B Diff: 1 Page Ref: 312

23 Copyright © 2014 Pearson Education, Inc.

 

106) To avoid damaging important structures in the lower limbs of infants, intramuscular injections are administered into the A) gluteus maximus. B) gluteus medius. C) gluteus minimus. D) vastus lateralis. Answer: D Diff: 2 Page Ref: 341 107) The lateral border of the cubital fossa is formed by the A) brachioradialis. B) pronator teres. C) tendon of the biceps brachii. D) median nerve. Answer: A Diff: 2 Page Ref: 339 108) In the neck, the hyoid bone can be felt A) between the thyroid cartilage and the tracheal rings. B) at the laryngeal prominence. C) where the anterior surface of the neck changes from a vertical orientation to an almost horizontal orientation. D) at the jugular notch. Answer: C Diff: 1 Page Ref: 334 109) The vertical groove in the center of the back is called the A) supracristal line. B) posterior median furrow. C) linea semilunaris. D) posterior axillary fold. Answer: B Diff: 2 Page Ref: 336-337 110) The midclavicular line is almost exactly the same as a A) vertical line in the median plane. B) line dropped straight inferiorly from the posterior axillary fold. C) line extending straight superiorly from the mid-inguinal point. D) horizontal line between the two clavicles. Answer: C Diff: 2 Page Ref: 335-336

24 Copyright © 2014 Pearson Education, Inc.

 

111) The inguinal ligament can be located by drawing an imaginary line between two palpable bony surface landmarks: the anterior superior iliac spine and the A) pubic tubercle. B) superficial inguinal ring. C) pubic symphysis. D) posterior superior iliac spine. Answer: A Diff: 3 Page Ref: 335 112) The triangular fossa whose base is formed by a horizontal line between the humeral epicondyles is the A) anatomical snuff box. B) anterior triangle. C) posterior triangle. D) cubital fossa. Answer: D Diff: 2 Page Ref: 339 113) If an intramuscular gluteal injection is not situated properly, all of the following structures could be damaged except A) the sciatic nerve. B) gluteal blood vessels. C) the vastus lateralis. D) the gluteal nerves. Answer: C Diff: 2 Page Ref: 341 114) Which of the following structures is not used to define the borders of femoral triangle? A) the anterior superior iliac spine B) the sartorius C) the inguinal ligament D) the adductor longus Answer: A Diff: 3 Page Ref: 343 115) The popliteal fossa is located A) posterior to the knee. B) anterior to the knee. C) in the groin. D) in the groove between the big toe and the second metatarsal. Answer: A Diff: 2 Page Ref: 343

25 Copyright © 2014 Pearson Education, Inc.

 

116) Inability to raise the eyebrow could be associated with damage to the A) mastoid process. B) orbicularis oculi. C) temporalis muscle. D) frontal belly of the epicranius muscle. Answer: D Diff: 2 Page Ref: 333 117) The lacrimal fossa is felt on the ________ surface of the eye socket. A) medial B) lateral marginal C) superior D) inferior Answer: A Diff: 1 Page Ref: 333 118) When you stand on your tiptoes, the lateral and medial bulges seen in your calf are the A) soleus. B) heads of the gastrocnemius. C) hamstrings. D) Achilles tendon. Answer: B Diff: 2 Page Ref: 343 119) The lateral border of the femoral triangle is formed by the A) femur. B) sartorius muscle. C) greater saphenous vein. D) rectus femoris muscle. Answer: B Diff: 2 Page Ref: 342-343 120) The superior lateral border of the popliteal fossa is formed by the A) knee joint. B) condyles of the femur. C) sciatic nerve. D) biceps femoris. Answer: D Diff: 2 Page Ref: 343 121) Which of the following structures does not lie within the anterior triangle of the neck? A) submandibular gland B) infrahyoid muscles C) subclavian artery D) suprahyoid muscles Answer: C Diff: 3 Page Ref: 335 26 Copyright © 2014 Pearson Education, Inc.

 

122) Which of the following structures does not lie in the posterior triangle of the neck? A) accessory nerve B) cervical plexus C) trunks of the brachial plexus D) external carotid artery Answer: D Diff: 3 Page Ref: 335 123) In the ventral gluteal site, an injection is made into the A) gluteal prominence. B) gluteus maximus. C) gluteus medius. D) gluteus minimus. Answer: C Diff: 2 Page Ref: 341 124) The jugular notch is a notch in the A) clavicle. B) manubrium. C) common carotid artery. D) hyoid bone. Answer: B Diff: 1 Page Ref: 335 125) A prime mover for leg extension at the knee is the A) rectus femoris. B) sartorius. C) semimembranosus. D) biceps femoris. Answer: A Diff: 2 Page Ref: 274 126) By palpation, one locates the temporomandibular joint A) directly anterior to the external auditory opening. B) by feeling the pulse of the facial artery. C) directly posterior to the last upper molars (wisdom teeth). D) by feeling the pulse of the superficial temporal artery. Answer: A Diff: 2 Page Ref: 334

27 Copyright © 2014 Pearson Education, Inc.

 

127) The medial border of the triangle of auscultation is formed by the A) medial border of the scapula. B) spinous process of the seventh thoracic vertebra. C) trapezius. D) seventh rib. Answer: C Diff: 2 Page Ref: 337 128) This structure is located between the laryngeal prominence and the jugular notch of the manubrium. A) anterior triangle B) cricoid cartilage C) epiglottis D) infrahyoid muscles Answer: B Diff: 2 Page Ref: 335 129) Which blood vessel lies deep to the sternocleidomastoid muscle? A) axillary artery B) common carotid artery C) subclavian artery D) thoracoacromial trunk Answer: B Diff: 2 Page Ref: 335 130) The phrenic nerve can be located by this surface landmark. A) along the midclavicular line B) anterior triangle C) fifth intercostal space, one hand width lateral to the sternum D) posterior triangle Answer: D Diff: 2 Page Ref: 335 131) The midclavicular line A) runs horizontally across the chest. B) runs inferomedially through the naval. C) runs lateral to the nipple. D) runs medial to the nipple. Answer: D Diff: 2 Page Ref: 335-336

28 Copyright © 2014 Pearson Education, Inc.

 

132) Which of the following statements about the pubic tubercle is false? A) It bears most of the body's weight in the seated position. B) It defines the inferior limit of the inguinal ligament. C) It is located just lateral to the pubic symphysis. D) It serves as an insertion point of the external obliques. Answer: A Diff: 1 Page Ref: 335 133) The line on the back drawn between the iliac crests is called the A) linea alba. B) linea semilunaris. C) posterior median furrow. D) supracristal line. Answer: D Diff: 2 Page Ref: 337 134) Which forearm muscle is absent in about 30% of individuals? A) extensor pollicis B) flexor carpi ulnaris C) palmaris longus D) pronator quadratus Answer: C Diff: 2 Page Ref: 340 135) Pain associated with the anatomical snuffbox may indicate A) dislocation of the elbow joint. B) dislocation of the shoulder joint. C) fracture of the scaphoid bone. D) inflammation of the carpal tunnel. Answer: C Diff: 2 Page Ref: 340 136) A dimple located two to three finger widths lateral from the midline of the back at the level of the hips is a landmark for the A) ischial tuberosities. B) anterior inferior iliac spine. C) sacroiliac joint. D) transverse processes of the L4 vertebra. Answer: C Diff: 2 Page Ref: 340

29 Copyright © 2014 Pearson Education, Inc.

 

137) Which bony landmark of the pelvis cannot be palpated? A) ischial tuberosities B) posterior superior iliac spine C) sacroiliac joint D) sacral promontory Answer: D Diff: 2 Page Ref: 340 138) When the forearm is extended, the olecranon process of the ulna lines up horizontally with the lateral epicondyles of the humerus, then A) the elbow joint is dislocated. B) the elbow joint is functioning normally. C) the patient suffers from tennis elbow. D) the radius is likely fractured. Answer: B Diff: 2 Page Ref: 339 139) With a flexed thigh, which landmark can be palpated along the superior medial aspect of the gluteal fold? A) greater sciatic notch B) greater trochanter C) ischial spine D) ischial tuberosities Answer: D Diff: 2 Page Ref: 343 11.2 True/False Questions 1) The elbow joint is an example of a fulcrum as the biceps brachii flexes the forearm. Answer: TRUE Diff: 2 Page Ref: 265-266 2) In both parallel and unipennate muscles, the muscle fibers extend the length of the muscle, from origin to insertion. Answer: FALSE Diff: 2 Page Ref: 263 3) The prime mover is also known as the antagonist. Answer: FALSE Diff: 1 Page Ref: 268 4) When a prime mover is contracted, the antagonists are stretched, but typically relaxed. Answer: TRUE Diff: 2 Page Ref: 268

30 Copyright © 2014 Pearson Education, Inc.

 

5) Interestingly, some synergists may act by cancelling out some of the actions of a prime mover. Answer: TRUE Diff: 3 Page Ref: 268 6) Third-class lever systems, like most skeletal muscles, enable great strength but sacrifice speed and distance of movement. Answer: FALSE Diff: 3 Page Ref: 266 7) Muscles arise from the mesoderm in the embryo. Answer: TRUE Diff: 2 Page Ref: 266 8) Muscles of the pharyngeal arch muscle group include muscles of facial expression and chewing muscles. Answer: TRUE Diff: 2 Page Ref: 266-267 9) In anatomical position, the pronator teres, the brachioradialis, and the extensor carpi radialis all lie on the anterior surface of the forearm. Answer: FALSE Diff: 2 Page Ref: 274 10) The hamstrings consist of the biceps femoris, the semitendinosus, and the semimembranosus. Answer: TRUE Diff: 2 Page Ref: 319 11) The erector spinae are the prime movers of back extension and consist of three columns of paired muscles. Answer: TRUE Diff: 2 Page Ref: 289 12) The external and internal intercostals are the prime movers of respiration. Answer: FALSE Diff: 2 Page Ref: 291 13) The respiratory diaphragm is the prime mover in inspiration. Answer: TRUE Diff: 1 Page Ref: 291 14) The common origin for many of the extensor muscles of the forearm is the medial epicondyle of the humerus. Answer: FALSE Diff: 2 Page Ref: 309

31 Copyright © 2014 Pearson Education, Inc.

 

15) The pelvic diaphragm consists of the levator ani, coccygeus, and transverse perineal muscles. Answer: FALSE Diff: 3 Page Ref: 296 16) Femoral and inguinal hernias both occur below the inguinal ligament. Answer: FALSE Diff: 1 Page Ref: 293 17) Placing fingers directly anterior to the external acoustic meatus enables palpation of the head of the mandible at the tempomandibular joint. Answer: TRUE Diff: 2 Page Ref: 334 18) The posterior triangle of the neck is bordered by the sternocleidomastoid, the trapezius, and the clavicle. Answer: TRUE Diff: 2 Page Ref: 335 19) The triangle of auscultation in the back is bordered by the latissimus dorsi, teres major, and medial border of the scapula. Answer: FALSE Diff: 2 Page Ref: 337 20) A lever system operating at a mechanical advantage can move a large load a short distance with a small amount of effort. Answer: TRUE Diff: 2 Page Ref: 264 21) The elbow has dislocated if the olecranon is not in a horizontal line with the epicondyles of the humerus. Answer: TRUE Diff: 2 Page Ref: 339 22) The supracristal line is used to determine the location of a lumbar puncture. Answer: TRUE Diff: 2 Page Ref: 336-337 23) The dimple indicating the posterior superior iliac spine is a landmark for the sacroiliac joint. Answer: TRUE Diff: 2 Page Ref: 340 24) The femoral triangle is formed by the inguinal ligament, sartorius, and gracilis muscles. Answer: FALSE Diff: 2 Page Ref: 342-343

32 Copyright © 2014 Pearson Education, Inc.

 

25) The two heads of the gastrocnemius form the inferior borders of the popliteal fossa. Answer: TRUE Diff: 2 Page Ref: 343 26) The head of the radius is felt by pressing into the dimple on the posterior lateral surface of the elbow while extending and rotating the forearm. Answer: TRUE Diff: 3 Page Ref: 340 27) Intramuscular injections in the deltoid should be given about 1 inch superior to the greater tubercle. Answer: FALSE Diff: 3 Page Ref: 338-339 28) The external carotid arteries and the carotid pulse can be palpated just posterior to the sternocleidomastoid. Answer: FALSE Diff: 2 Page Ref: 335 29) In lean, muscular individuals, the linea semilunaris marks the lateral borders of the rectus abdominis. Answer: TRUE Diff: 2 Page Ref: 336 30) You can locate the iliac crests by resting your hands on your hips. Answer: TRUE Diff: 2 Page Ref: 335 11.3 Short Answer Questions 1) The ________ muscle compresses the abdominal contents. Answer: transversus abdominis Diff: 2 Page Ref: 293 2) The ________ muscle helps stabilize and depress the pectoral girdle. Answer: subclavius Diff: 2 Page Ref: 298 3) The ________ muscle is the prime mover of head flexion, as well as rotation. Answer: sternocleidomastoid Diff: 1 Page Ref: 287 4) The temporalis and ________ muscles are the prime movers of jaw closure as they elevate the mandible. Answer: masseter Diff: 1 Page Ref: 282-283 33 Copyright © 2014 Pearson Education, Inc.

 

5) The three extrinsic muscles of the tongue are ________. Answer: genioglossus, hyoglossus, and styloglossus Diff: 3 Page Ref: 282 6) The four suprahyoid muscles are ________. Answer: digastric, stylohyoid, mylohyoid, and geniohyoid Diff: 3 Page Ref: 284 7) The psoas major and ________ muscles are the prime movers in thigh flexion. Answer: iliacus Diff: 2 Page Ref: 315 8) The ________ is the longest muscle in the body, spanning two joints. Answer: sartorius Diff: 2 Page Ref: 316 9) The four muscles of the quadriceps femoris are ________. Answer: rectus femoris, vastus lateralis, vastus medialis, and the vastus intermedius Diff: 3 Page Ref: 316 10) The ________ and gluteus maximus muscles insert on the iliotibial tract. Answer: tensor fasciae latae Diff: 2 Page Ref: 315 11) The elbow joint serves as a(n) ________ as the biceps brachii muscle flexes the forearm. Answer: fulcrum Diff: 3 Page Ref: 265-266 12) Most skeletal muscles are ________-class lever systems. Answer: third Diff: 3 Page Ref: 266 13) The three hamstrings are ________. Answer: biceps femoris, semitendinosus, and semimembranosus Diff: 2 Page Ref: 319 14) The ________ is composed of the iliocostalis, longissmus, and spinalis. Answer: erector spinae Diff: 2 Page Ref: 289 15) The rhomboids are acting as a(n) ________ when they stabilize the scapula, holding it in place as the arm moves. Answer: fixator Diff: 3 Page Ref: 270

34 Copyright © 2014 Pearson Education, Inc.

 

16) Contraction of the ________ muscle wrinkles the forehead. Answer: Frontal belly of epicranius Diff: 2 Page Ref: 279 17) The deltoid muscle is an example of a(n) ________ fascicle arrangement. Answer: multipennate Diff: 2 Page Ref: 263 18) Another name for the pharyngeal arch muscles is ________. Answer: branchiomeric muscles Diff: 2 Page Ref: 266 19) Muscles that oppose or reverse a particular movement act as ________. Answer: antagonists Diff: 2 Page Ref: 268 20) The seven criteria used in naming muscles are ________. Answer: location, shape, size, direction of fascicles, location of attachments, and number of origins Diff: 2 Page Ref: 270 21) Gluteal intramuscular injections must occur superior to the midpoint of a line extending from the posterior superior iliac spine to the ________. Answer: greater trochanter Diff: 3 Page Ref: 341 22) The ________ is a broad sheet of connective tissue that slides freely over the superior skull. Answer: epicranial aponeurosis Diff: 2 Page Ref: 333 23) The anterior border of the posterior triangle of the neck is formed by the ________ muscle. Answer: sternocleidomastoid Diff: 2 Page Ref: 335 24) The ________ muscle is the prime mover of dorsiflexion. Answer: tibialis anterior Diff: 2 Page Ref: 323 11.4 Essay Questions 1) Describe the diaphragm. Answer: The diaphragm is the most important muscle in respiration. It is dome-shaped when in the relaxed condition; in the center is the central tendon. During contraction, it flattens, and inspiration occurs. When it relaxes, expiration occurs. It can also be contracted to produce pressure on the abdominal organs and can aid in evacuating the bladder or large intestine or in giving birth. Diff: 2 Page Ref: 291 35 Copyright © 2014 Pearson Education, Inc.

 

2) Compare and contrast the function of the muscles contained in the anterior and posterior compartments of the upper and lower limbs. Explain the embryological reason for the differences. Answer: The anterior compartments of the upper limbs contain flexor muscles, whereas the anterior compartments of the lower limbs contain extensor muscles. Similarly, the posterior compartments contain extensor muscles in the upper limbs but flexor muscles in the lower limbs. The reason for these differences in muscle location is that during embryo development, the upper and lower limbs rotate in opposite directions from each other. Diff: 3 Page Ref: 267-268 3) Describe the functions of the retinacula found at the wrist and ankle. Answer: These keep the tendons of muscles that attach either to the hand or the foot, and if they were not present, these tendons would "bowstring" out. They prevent this action when the hand or foot is hyperextending or flexing. Diff: 1 Page Ref: 306, 323 4) What muscles are important in mastication (excluding the muscles of the tongue)? Answer: Masseter, temporalis, medial pterygoid, lateral pterygoid, and buccinator Diff: 2 Page Ref: 282 5) Describe the range of movements that can be accomplished by the quadriceps. Describe their common insertion. Answer: Extends leg at the knee, flexes thigh at the hip, stabilizes knee, stabilizes patella. They come together as the quadriceps tendon, named as such because it connects muscle to bone (the patella), then it encloses the patella. Distal to the patella, it is called the patellar ligament, connecting bone to bone (the tibial tuberosity). Diff: 2 Page Ref: 316 6) Describe how one could locate the acromion and acromioclavicular joint. Answer: The acromion is located by following the spine of the scapula to its lateral end. Then the clavicle can be palpated anterior and medially. When the arm is thrust anteriorly, movement can be detected at the acromioclavicular joint. Diff: 2 Page Ref: 338 7) Briefly describe the landmarks used in making a gluteal intramuscular injection. Answer: A line is drawn from the posterior superior iliac spine to the greater trochanter of the femur. The posterior superior iliac spine is located by a dimple that is two to three finger widths lateral to the midline of the back. The greater trochanter is palpated on the lateral hip just anterior to a hollow about a hand breadth inferior to the iliac crest. The injection should occur 2 inches superior to the midpoint of this line. Diff: 3 Page Ref: 341

36 Copyright © 2014 Pearson Education, Inc.

 

8) Describe the boundaries of the triangle of auscultation, and explain how a clinician can take advantage of this anatomical feature. Answer: The triangle of auscultation is found between the trapezius muscle, the latissimus dorsi muscle, and the medial border of the scapula. When the clinician directs the patient to flex the trunk and fold the upper limbs across the chest, the scapula is moved anteriorly to maximize the area of this triangle. Doing so will expose a region of the rib cage that is not covered by superficial muscles and allows the clinician to better listen to pulmonary sounds. Diff: 2 Page Ref: 337

37 Copyright © 2014 Pearson Education, Inc.

 

Human Anatomy, 7e (Marieb/Mitchell/Smith) Chapter 12 Fundamentals of the Nervous System and Nervous Tissue 12.1 Multiple Choice Questions

Figure 12.1 Use the diagram pictured above to answer the following questions. 1) Identify the letter that indicates the region of a neuron with a name that means "little hill." A) A B) B C) C D) D E) E Answer: B Diff: 2 Page Ref: 353-354 2) Identify the letter that indicates the region of a neuron where neurotransmitters are released. A) A B) B C) C D) D E) E Answer: E Diff: 1 Page Ref: 353-354

1 Copyright © 2014 Pearson Education, Inc.

 

3) Identify the letter that indicates a Schwann cell. A) A B) B C) C D) D E) E Answer: C Diff: 2 Page Ref: 353-354 4) Identify the letter that indicates the gaps between Schwann cells that are known as myelin sheath gaps (nodes of Ranvier). A) A B) B C) C D) D E) E Answer: D Diff: 2 Page Ref: 353-354 5) Axodendritic synapses occur between letter "E" on the diagram and this region on a proximal neuron. A) A B) B C) C D) D E) E Answer: A Diff: 1 Page Ref: 353-355

2 Copyright © 2014 Pearson Education, Inc.

 

Figure 12.2 Use the diagrams above to answer the following questions. 6) Identify which diagram represents a microglial cell. A) A B) B C) C D) D E) E Answer: C Diff: 2 Page Ref: 358-359 7) Identify which diagram represents a cell that produces a myelin sheath in the central nervous system. A) A B) B C) C D) D E) E Answer: E Diff: 2 Page Ref: 358-359

3 Copyright © 2014 Pearson Education, Inc.

 

8) Identify which letter represents the most abundant category of glial cells in the CNS. A) A B) B C) C D) D E) E Answer: B Diff: 2 Page Ref: 358-359 9) Identify which letter represents an oligodendrocyte. A) A B) B C) C D) D E) E Answer: E Diff: 1 Page Ref: 358-359 10) Identify which diagram represents cells that produce and circulate cerebrospinal fluid. A) A B) B C) C D) D E) E Answer: D Diff: 1 Page Ref: 358-359 11) This is the site of communication between neurons. A) synapse B) axon terminal C) axon D) cell body E) dendrite Answer: A Diff: 2 Page Ref: 353-355 12) The ________ of a presynaptic neuron associates with the dendrite of a postsynaptic neuron. A) synapse B) axon terminal C) axon D) cell body E) dendrite Answer: B Diff: 2 Page Ref: 353-355

4 Copyright © 2014 Pearson Education, Inc.

 

13) This tends to be the longest cytoplasmic projection froma neuron. A) synapse B) axon terminal C) axon D) cell body E) dendrite Answer: C Diff: 1 Page Ref: 353-355 14) These regions of a neuron are also referred to as terminal boutons. A) synapse B) axon terminal C) axon D) cell body E) dendrite Answer: B Diff: 1 Page Ref: 353-355 15) Chemical signals diffuse between neurons at this location. A) synapse B) axon terminal C) axon D) cell body E) dendrite Answer: A Diff: 1 Page Ref: 353-355 16) Mitochondria are particularly abundant here. A) synapse B) axon terminal C) axon D) cell body E) dendrite Answer: B Diff: 2 Page Ref: 354-355 17) An axosomatic synapse occurs between the axon terminals of one neuron and the ________ of a proximal neuron. A) synapse B) axon terminal C) axon D) cell body E) dendrite Answer: D Diff: 2 Page Ref: 353-355

5 Copyright © 2014 Pearson Education, Inc.

 

18) In an axodendritic synapse, the region of the postsynaptic neuron that binds the released neurotransmitter is the ________. A) synapse B) axon terminal C) axon D) cell body E) dendrite Answer: E Diff: 3 Page Ref: 353-355 19) This region of the neuron contains a single nucleus surrounded by cytoplasm. A) synapse B) axon terminal C) axon D) cell body E) dendrite Answer: D Diff: 2 Page Ref: 353-355 20) These regions of a neuron are characterized by numerous, short cytoplasmic extensions and are often referred to as receiving regions. A) synapse B) axon terminal C) axon D) cell body E) dendrite Answer: E Diff: 2 Page Ref: 353-355 21) These regions of the neuron direct electrical currents toward the cell body. A) synapse B) axon terminal C) axon D) cell body E) dendrite Answer: E Diff: 2 Page Ref: 353-355 22) In this part of a neuron, neurofilaments, actin microfilaments, and microtubules are particularly abundant, providing structural support and a transport network. A) synapse B) axon terminal C) axon D) cell body E) dendrite Answer: C Diff: 2 Page Ref: 353-355 6 Copyright © 2014 Pearson Education, Inc.

 

23) This part of the neuron may have branching collaterals. A) synapse B) axon terminal C) axon D) cell body E) dendrite Answer: C Diff: 2 Page Ref: 353-355 24) This region of a neuron contains chromatophilic substance or Nissl bodies. A) synapse B) axon terminal C) axon D) cell body E) dendrite Answer: D Diff: 2 Page Ref: 353-355 25) This neuronal region transmits electrical impulses away from the cell body. A) synapse B) axon terminal C) axon D) cell body E) dendrite Answer: C Diff: 2 Page Ref: 353-355 26) A somatic motor neuron carries A) information that signals muscle contraction in the organs in the ventral cavity. B) information, such as pain, from the viscera in the ventral cavity to the CNS. C) motor commands to the skeletal musculature. D) information from the skin to the CNS. Answer: C Diff: 2 Page Ref: 352 27) Most synapses transmit communicating signals using A) chemical signaling molecules-neurotransmitters. B) electrical impulses travelling through gap junctions. C) physical contact between adjacent neurons. D) chemical signaling molecules released from neuroglia. Answer: A Diff: 2 Page Ref: 354-355

7 Copyright © 2014 Pearson Education, Inc.

 

28) Which of the following is not considered a special somatic sense? A) smell B) taste C) pain D) equilibrium Answer: C Diff: 2 Page Ref: 352 29) Which of the following is not a characteristic of neurons? A) longevity B) inability to divide C) high metabolic rate D) ability to survive without oxygen Answer: D Diff: 2 Page Ref: 353 30) Neurofibrils A) form synapses with axons of postsynaptic neurons. B) help circulate cerebrospinal fluid. C) prevent the neuron from being pulled apart when subjected to tensile forces. D) receive incoming stimuli and pass the signal toward the cell body. Answer: C Diff: 2 Page Ref: 353 31) Ganglia represent A) groups of axons. B) groups of dendrites. C) groups of neuron cell bodies. D) groups of synapses. Answer: C Diff: 2 Page Ref: 353 32) Which of the following is the correct path an impulse takes across a synapse? A) axon of postsynaptic neuron, dendrite of presynaptic neuron, synaptic cleft B) dendrite of presynaptic neuron, synaptic cleft, axon of postsynaptic neuron C) axon of presynaptic neuron, synaptic cleft, dendrite of postsynaptic neuron D) synaptic cleft, dendrite of postsynaptic neuron, axon of presynaptic neuron Answer: C Diff: 3 Page Ref: 354-355 33) The majority of neurons in the body are A) multipolar. B) bipolar. C) unipolar. D) pseudounipolar. Answer: A Diff: 1 Page Ref: 355 8 Copyright © 2014 Pearson Education, Inc.

 

34) Which cells are the macrophages of the CNS? A) ependymal cells B) microglial cells C) satellite cells D) Schwann cells Answer: B Diff: 2 Page Ref: 358-359 35) Of the following, which is the only structure that is in the PNS, as opposed to the CNS? A) a tract B) white matter C) a ganglion D) gray matter Answer: C Diff: 2 Page Ref: 353 36) Which of the following statements about an axon is false? A) It is also referred to as a nerve fiber. B) It has branches. C) It carries nerve impulses toward the cell body. D) It has a uniform diameter. Answer: C Diff: 2 Page Ref: 354 37) Which of the following statements concerning sensory neurons is incorrect? A) Most are pseudounipolar. B) Most have their cell bodies in ganglia outside the CNS. C) They have peripheral and central processes. D) They contain only dendrites. Answer: D Diff: 2 Page Ref: 357 38) Interneurons are found A) only in the visceral nervous system. B) only in the CNS. C) only in the PNS. D) only in the autonomic nervous system. Answer: B Diff: 2 Page Ref: 357

9 Copyright © 2014 Pearson Education, Inc.

 

39) Most tumors in the brain originate from A) glial cells. B) multipolar neurons. C) bipolar neurons. D) unipolar neurons. Answer: A Diff: 2 Page Ref: 359 40) Which of the following statements about neurotransmitters is false? A) They alter the permeability of the postsynaptic cell membrane. B) They are waves of positive charges that travel down axons. C) They are released from synaptic vesicles. D) They diffuse across the synaptic cleft. Answer: B Diff: 2 Page Ref: 355 41) Which cells provide the myelin sheath for neurons in the CNS? A) Schwann cells B) astrocytes C) microglial cells D) oligodendrocyctes Answer: D Diff: 2 Page Ref: 359 42) Which of the following is the correct arrangement of a reflex arc? A) receptor, sensory neuron, integration center, motor neuron, effector B) integration center, receptor, sensory neuron, motor neuron, effector C) effector, sensory neuron, integration center, motor neuron, receptor D) receptor, motor neuron, integration center, effector, sensory neuron Answer: A Diff: 3 Page Ref: 363-364 43) Which relatively rare type of neuron has two processes extending from opposite sides of the cell body? A) bipolar B) multipolar C) pseudounipolar D) unipolar Answer: A Diff: 2 Page Ref: 355-356

10 Copyright © 2014 Pearson Education, Inc.

 

44) The complexity of the CNS can be attributed to A) the large number of sensory neurons carrying information to the CNS. B) the different types of receptors outside the CNS. C) the large number of interneurons in the CNS. D) the large number of motor neurons leaving the CNS. Answer: C Diff: 3 Page Ref: 357 45) White matter represents A) aggregations of neuron cell bodies in the brain. B) aggregations of neuron cell bodies in the spinal cord. C) myelinated axons traveling together in the CNS. D) dendrites traveling together in the ANS. Answer: C Diff: 2 Page Ref: 361 46) These glial cells surround the cell bodies of sensory neurons within ganglia of the PNS. A) astrocytes B) microglia C) satellite cells D) Schwann cells Answer: C Diff: 2 Page Ref: 359 47) These glial cells arise from embryonic white blood cells. A) ependymal cells B) microglia C) oligodendrocytes D) satellite cells Answer: B Diff: 3 Page Ref: 359 48) Action potentials travel along the A) axon membrane. B) dendrite membrane. C) cell body. D) myelin. Answer: A Diff: 2 Page Ref: 354

11 Copyright © 2014 Pearson Education, Inc.

 

49) The peripheral nerve fibers that measure the degree of stretch in the biceps brachii muscle and its tendons are classified as A) general somatic sensory. B) general somatic motor. C) special visceral sensory. D) general visceral motor. Answer: A Diff: 3 Page Ref: 352 50) All of the following are characteristics of dendrites except that they A) are more extensive branching than axons. B) always conduct action potentials. C) conduct signals toward the cell body. D) typically occur as more than one per cell. Answer: B Diff: 3 Page Ref: 354 51) Myelin on axons functions to A) make the axons live longer. B) store nutrients (fat) for use by the neurons. C) cover nodes of Ranvier. D) speed the rate of impulse conduction and insulate neighboring axons from one another. Answer: D Diff: 2 Page Ref: 359 52) Which cells provide the myelin sheath for neurons in the PNS? A) astrocytes B) microglial cells C) Schwann cells D) oligodendrocytes Answer: C Diff: 2 Page Ref: 359 53) Nonmyelinated axons A) are thicker than myelinated axons. B) are not associated with any Schwann cells. C) conduct impulses more slowly than myelinated axons. D) occur in the PNS, but not in the CNS. Answer: C Diff: 3 Page Ref: 359

12 Copyright © 2014 Pearson Education, Inc.

 

54) Which of the following most immediately encases the smallest component of a nerve? A) endoneurium B) epineurium C) fascicle D) perineurium Answer: A Diff: 2 Page Ref: 361-362 55) Motor neurons arise primarily from the A) alar plate. B) basal plate. C) ependyma. D) neural crest. Answer: B Diff: 2 Page Ref: 369 56) A nerve cell is the same as a A) nerve. B) nerve fiber. C) neurilemmocyte. D) neuron. Answer: D Diff: 1 Page Ref: 353 57) An example of proprioception is A) the contraction of the triceps brachii. B) the contraction of pharyngeal arch muscles used in chewing. C) sensing a feather touch the skin. D) the sensation you feel during a wake-up stretch. Answer: D Diff: 3 Page Ref: 352 58) Sensory neurons arise primarily from the A) neural crest. B) alar plate. C) basal plate. D) neural tube. Answer: A Diff: 2 Page Ref: 369 59) Some small neurons in the CNS have no axon, only dendrites, and are A) multipolar. B) bipolar. C) unipolar. D) pseudounipolar. Answer: A Diff: 2 Page Ref: 355 13 Copyright © 2014 Pearson Education, Inc.

 

60) A man walking barefoot stepped on a piece of glass. His foot jerked upward in which type of reflex? A) somatic, polysynaptic withdrawal reflex B) visceral, monosynaptic stretch reflex C) somatic, monosynaptic withdrawal reflex D) visceral, polysynaptic withdrawal reflex Answer: A Diff: 2 Page Ref: 363-364 61) A node of Ranvier (myelin sheath gap) A) occurs only in ganglia. B) is one segment of the myelin sheath. C) occurs in the PNS but not in the CNS. D) is a bare region of axonal membrane in myelinated axons only. Answer: D Diff: 2 Page Ref: 359 62) The difference between myelinated and unmyelinated axons is that A) Schwann cells are not associated with unmyelinated axons. B) Schwann cells form more widely spaced nodes of Ranvier in unmyelinated axons. C) Schwann cells simultaneously surround multiple axons in myelinated axons. D) Schwann cells wrap around myelinated axons in concentric layers. Answer: D Diff: 3 Page Ref: 359-361 63) An axon collateral from one neuron that circles back and synapses with a previous neuron describes A) a converging circuit. B) parallel processing. C) a reverberating circuit. D) serial processing. Answer: C Diff: 3 Page Ref: 365 64) In the region of the spinal cord, the cell bodies of most interneurons lie in A) sensory ganglia. B) the PNS. C) the dorsal half of the gray matter. D) the ventral half of the white matter. Answer: C Diff: 2 Page Ref: 361

14 Copyright © 2014 Pearson Education, Inc.

 

65) A monosynaptic reflex arc is an example of A) a converging circuit. B) parallel processing. C) a reverberating circuit. D) serial processing. Answer: D Diff: 3 Page Ref: 365 66) Ciliated neuroglial cells that form an epithelium and play an active role in forming and moving cerebrospinal fluid are A) ependymal cells. B) Schwann cells. C) oligodendrocytes. D) astrocytes. Answer: A Diff: 2 Page Ref: 359 67) The chemical substance that is released at axon terminals is called a A) hormone. B) neurotransmitter. C) synaptic vesicle. D) Nissl body. Answer: B Diff: 2 Page Ref: 354-355 68) The covering of a fascicle within a nerve is the A) endoneurium. B) perineurium. C) ectoneurium. D) epineurium. Answer: B Diff: 2 Page Ref: 361-362 69) The entire nerve is surrounded by a tough fibrous sheath called the A) endoneurium. B) perineurium. C) ectoneurium. D) epineurium. Answer: D Diff: 2 Page Ref: 361-362

15 Copyright © 2014 Pearson Education, Inc.

 

70) The peripheral nerve fibers that speed up the movement of the digestive tract are classified as A) general visceral sensory (afferent). B) special visceral sensory. C) general visceral motor (efferent). D) special somatic motor. Answer: C Diff: 2 Page Ref: 352- 353 71) Regeneration of peripheral axons requires A) construction of a collateral that branches from the point of damage. B) deposition of neurofilaments to bridge the gap between the original axon fragments. C) formation of a tube by Schwann cells to guide growth. D) migration of neural stem cells from the hippocampus. Answer: C Diff: 2 Page Ref: 367-368 72) Vesicles containing neurotransmitters are located in A) a synaptic cleft. B) axon terminals. C) the postsynaptic region of dendrites. D) the nodes of Ranvier. Answer: B Diff: 1 Page Ref: 354-355 73) The type of axon that conducts impulses most slowly is A) thick, myelinated. B) thick, unmyelinated. C) thin, myelinated. D) thin, unmyelinated. Answer: D Diff: 2 Page Ref: 359 74) During embryo development, neurons that make "bad" connections A) are inhibited by chemicals released from astrocytes. B) are turned off by inhibitory synapses in reverberating circuits. C) develop collaterals that seek out appropriate target cells. D) die by apoptosis. Answer: D Diff: 2 Page Ref: 369-370 75) Gray matter in the CNS contains all of the following except A) neuron cell bodies. B) neuroglia. C) dendrites. D) fiber tracts. Answer: D Diff: 2 Page Ref: 361 16 Copyright © 2014 Pearson Education, Inc.

 

12.2 True/False Questions 1) All of the neuron's organelles are localized to the cell body. Answer: FALSE Diff: 2 Page Ref: 354-355 2) A neuron is a collection of nerve fibers in the PNS. Answer: FALSE Diff: 1 Page Ref: 353 3) Special somatic senses have receptors that are located mostly in the head, including hearing and balance and vision. Answer: TRUE Diff: 3 Page Ref: 352 4) General visceral sensory impulses include pain, temperature, nausea, and hunger. Answer: TRUE Diff: 2 Page Ref: 352 5) Most neurons in the body are multipolar neurons. Answer: TRUE Diff: 2 Page Ref: 355 6) The somatic motor subdivision of the peripheral nervous system is considered to be an involuntary nervous system. Answer: FALSE Diff: 2 Page Ref: 352 7) The supporting cells of the nervous system that surround and wrap neurons are derived from embryonic neuroepithelial cells from the ectoderm. Answer: TRUE Diff: 3 Page Ref: 369 8) The white matter is the site where neuron cell bodies are clustered. Answer: FALSE Diff: 2 Page Ref: 361 9) Any long axon is called a nerve fiber. Answer: TRUE Diff: 2 Page Ref: 354 10) Chromatophilic bodies are clusters of rough ER and free ribosomes that produce the large amount of proteins needed by a neuron. Answer: TRUE Diff: 2 Page Ref: 353 17 Copyright © 2014 Pearson Education, Inc.

 

11) Gliomas are tumors of glial cells. This type of cancer is difficult to treat, and the one-year survival rate is poor. Answer: TRUE Diff: 3 Page Ref: 359 12) Microglia are ciliated to help circulate cerebrospinal fluid (CSF). Answer: FALSE Diff: 2 Page Ref: 358-359 13) Oligodendrocytes and Schwann cells myelinate neurons within the peripheral nervous system. Answer: FALSE Diff: 2 Page Ref: 358-360 14) Less than 15% of all neurons are interneurons. Answer: FALSE Diff: 2 Page Ref: 357 15) Microglia are derived from blood stem cells rather than from neural tube and crest cells. Answer: TRUE Diff: 3 Page Ref: 359 12.3 Short Answer Questions 1) The ________ matter of the spinal cord forms a butterfly-shaped structure. Answer: gray Diff: 2 Page Ref: 361 2) ________ is a progressive disease that destroys patches of myelin in both the brain and spinal cord. Answer: Multiple sclerosis (MS) Diff: 2 Page Ref: 367-368 3) ________ form the myelin sheaths in the brain and spinal cord. Answer: Oligodendrocytes Diff: 2 Page Ref: 358 4) ________ is the term for the previously held belief that fully differentiated neurons could not divide. Answer: The no-new-neurons doctrine Diff: 2 Page Ref: 370 5) ________ are chemicals that control the growth of newly forming neurons. Answer: Neurotrophins Diff: 2 Page Ref: 369

18 Copyright © 2014 Pearson Education, Inc.

 

6) Cytoplasm, organelles, and neurotransmitters move through the cytoplasm of the axon by a mechanism called ________. Answer: axonal transport Diff: 2 Page Ref: 354 7) White matter is white in color because of ________. Answer: myelinated axons Diff: 2 Page Ref: 361 8) A(n) ________ is a medical specialist in the study of the nervous system and its disorders. Answer: neurologist Diff: 2 Page Ref: 371 9) Nearly all somatic sensory neurons are of this type, based on the classification of the type and number of processes. Answer: pseudounipolar Diff: 2 Page Ref: 355-357 10) Nearly all motor neurons are of this type, based on the classification of the type and number of processes. Answer: multipolar Diff: 2 Page Ref: 357 11) ________ are the neuroglial cells with cilia. Answer: Ependymal cells Diff: 2 Page Ref: 359 12) These neurons reside only in the CNS and connect motor and sensory neurons. Answer: interneurons, or association neurons Diff: 2 Page Ref: 357 13) Neurotransmitters are found packaged inside ________ in the axon terminal. Answer: synaptic vesicles Diff: 2 Page Ref: 355 14) ________ neurons develop from neural crest cells. Answer: Sensory Diff: 3 Page Ref: 370 15) The simplest of all reflexes is the ________ reflex. Answer: monosynaptic Diff: 2 Page Ref: 363-364

19 Copyright © 2014 Pearson Education, Inc.

 

12.4 Essay Questions 1) Name four characteristics that distinguish neuroglial cells from neurons. Answer: Glial cells are smaller, they have a very darkly staining nucleus, they can divide throughout life, and they greatly outnumber neurons. Diff: 2 Page Ref: 358 2) Distinguish between converging, diverging, and reverberating neuronal circuits. Identify the role of axon collaterals and terminal branches in these circuits. Answer: In converging circuits, a single neuron receives stimuli from multiple neurons. The target neuron must integrate the multiple incoming signals. In diverging circuits, a single neuron synapses with multiple target cells through many terminal branches of its axon. In reverberating circuits, a postsynaptic neuron will branch, and its axon collateral will synapse with one of its presynaptic neurons and alter the activity of that neuron. Diff: 3 Page Ref: 364-365 3) Describe the myelin sheath. Answer: Schwann cells in the PNS and oligodendrocytes in the CNS produce the myelin sheath. These cells wrap around the larger axons of the body. They function as an insulating layer that prevents the electrical current from diminishing in strength as it travels the length of the axon This results in extremely efficient transmission of neuronal impulses to the target tissue. Diff: 2 Page Ref: 359-361 4) Describe the levels of layers of myelin and connective tissue surrounding a nerve. Answer: The axon is myelinated by Schwann cells, with each individual axon covered with a loose connective tissue layer called the endoneurium. Groups of these axons are found in bundles, called fascicles, surrounded by the perineurium. These fascicles are held together by an outer fibrous connective tissue layer — the epineurium, which collectively is the nerve. Embedded in the connective tissue layers are blood vessels which provide oxygen and nutrients to all of the cells enclosed by the epineurium. Diff: 2 Page Ref: 361-362 5) Distinguish monosynaptic reflexes from polysynaptic reflexes. Answer: Monosynaptic reflexes involve a single sensory neuron, with the cell body in the dorsal root ganglion and the axon forming a synapse with the motor neuron in the ventral horn of the gray matter of the spinal cord. The axon of the motor neuron (somatic or visceral) exits the gray matter and directs the motor impulse towards skeletal muscle or glandular tissue. Polysynaptic reflexes have more than one synapse and include interneurons, which may be localized to the same level of the spinal cord but most commonly involve multiple synapses, and neural connections may run to other levels of the spinal cord, as well as to the brain. Diff: 3 Page Ref: 363-364

20 Copyright © 2014 Pearson Education, Inc.

 

Human Anatomy, 7e (Marieb/Mitchell/Smith) Chapter 13 The Central Nervous System 13.1 Multiple Choice Questions

Figure 13.1 Use the diagram above to answer the following questions. 1) Which letter indicates the primary visual cortex, which receives information from neurons in the retina of the eye? A) A B) B C) C D) D E) E Answer: E Diff: 2 Page Ref: 392 2) Which letter indicates Broca's area, which controls the motor movements necessary for speaking? A) A B) B C) C D) D E) E Answer: A Diff: 2 Page Ref: 392,394

1 Copyright © 2014 Pearson Education, Inc.

 

3) Which letter indicates Wernicke's area, which is important for understanding spoken words? A) A B) B C) C D) D E) E Answer: D Diff: 2 Page Ref: 392, 395 4) Which letter indicates an area that is characterized by huge neurons known as pyramidal cells? A) A B) B C) C D) D E) E Answer: B Diff: 2 Page Ref: 392, 394 5) Which letter indicates the primary somatosensory cortex? A) A B) B C) C D) D E) E Answer: C Diff: 2 Page Ref: 392, 393

2 Copyright © 2014 Pearson Education, Inc.

 

Figure 13.2 Use the diagram above to answer the following questions. 6) Which letter indicates the corpora quadrigemina, meaning "four bodies?" A) A B) B C) C D) D E) E Answer: D Diff: 2 Page Ref: 386 7) Which letter indicates the medulla, the functional neural connection between the pons and the spinal cord? A) A B) B C) C D) D E) E Answer: E Diff: 2 Page Ref: 386

3 Copyright © 2014 Pearson Education, Inc.

 

8) Which letter indicates the region of the brain that has the greatest surface area due to the numerous surface convolutions? A) A B) B C) C D) D E) E Answer: A Diff: 2 Page Ref: 386 9) Which letter indicates a choroid plexus, which produces cerebrospinal fluid (CSF)in all four ventricles of the brain? A) A B) B C) C D) D E) E Answer: C Diff: 2 Page Ref: 386 10) Which letter indicates the corpus callosum, which is an area of white matter where axons from one cerebral hemisphere cross the midline to the opposite hemisphere? A) A B) B C) C D) D E) E Answer: B Diff: 2 Page Ref: 386 11) The hindbrain of the embryo develops into the pons, the medulla oblongata, part of the spinal cord, and what other structure? A) hypothalamus B) cerebrum C) cerebellum D) thalamus Answer: C Diff: 2 Page Ref: 375 12) The telencephalon develops into what adult brain structure? A) hypothalamus B) cerebrum C) cerebellum D) thalamus Answer: B Diff: 2 Page Ref: 375 4 Copyright © 2014 Pearson Education, Inc.

 

13) The diencephalon develops into the thalamus, the epithalamus, and what other structure? A) hypothalamus B) cerebrum C) cerebellum D) thalamus Answer: A Diff: 2 Page Ref: 375 14) During the human embryonic and fetal periods, the brain grows rapidly, most noticeably the growth of what structure? A) hypothalamus B) cerebrum C) cerebellum D) thalamus Answer: B Diff: 1 Page Ref: 375 15) Which of these regions of the brain regulates body temperature, hunger, and thirst? A) hypothalamus B) cerebrum C) cerebellum D) thalamus Answer: A Diff: 2 Page Ref: 385-387 16) Which of these regions of the brain, besides the cerebrum, has an outer cortical layer of gray matter? A) hypothalamus B) cerebrum C) cerebellum D) thalamus Answer: C Diff: 2 Page Ref: 383 17) Which of these regions has two hemispheres connected by the corpus callosum? A) hypothalamus B) cerebrum C) cerebellum D) thalamus Answer: B Diff: 2 Page Ref: 396

5 Copyright © 2014 Pearson Education, Inc.

 

18) Which of these regions superior-most aspect is found in close proximityl to the fourth ventricle and lies lies just posterior to the pons? A) hypothalamus B) cerebrum C) cerebellum D) thalamus Answer: C Diff: 2 Page Ref: 383-384 19) Which of these regions of the brain has frontal, parietal, temporal, and occipital lobes, as well as the insula? A) hypothalamus B) cerebrum C) cerebellum D) thalamus Answer: B Diff: 1 Page Ref: 388-390 20) The insula is considered to be part of the ________. A) hypothalamus B) cerebrum C) cerebellum D) thalamus Answer: B Diff: 2 Page Ref: 388-390 21) Which of these regions of the brain contains the basal forebrain nuclei in the deep gray matter? A) hypothalamus B) cerebrum C) cerebellum D) thalamus Answer: B Diff: 2 Page Ref: 390, 399 22) Which of these regions is the largest part of the diencephalon and forms the upper and lateral walls of the third ventricle? A) hypothalamus B) cerebrum C) cerebellum D) thalamus Answer: D Diff: 2 Page Ref: 385

6 Copyright © 2014 Pearson Education, Inc.

 

23) Which of these regions functions to direct nearly all sensory input, except olfactory impulses, to the cerebral cortex? A) hypothalamus B) cerebrum C) cerebellum D) thalamus Answer: D Diff: 2 Page Ref: 385 24) Which of these regions of the brain regulates hormonal secretions from the pituitary gland (hypophysis)? A) hypothalamus B) cerebrum C) cerebellum D) thalamus Answer: A Diff: 2 Page Ref: 385-387 25) Which of these regions lies between the optic chiasma and the posterior border of the mammillary bodies? A) hypothalamus B) cerebrum C) cerebellum D) thalamus Answer: A Diff: 3 Page Ref: 386 26) A "motor homunculus" can be visualized as an overlay on the precentral gyrus. The reason why the facial region of this homunculus covers such a large surface area is because A) our ears are large for hearing. B) our eyes are large for seeing. C) we have very expressive faces. D) we have very sensitive lips. Answer: C Diff: 2 Page Ref: 393-394 27) It is easy to confuse the terms sulcus and gyrus (on the cerebral cortex). The difference between these two terms is that A) a sulcus corresponds to a folia on the cerebellum, whereas a gyrus corresponds to a fissure. B) a sulcus is the same as a fissure on the cerebral cortex, whereas a gyrus is a lobe. C) a sulcus is a groove, and a gyrus is a deeper groove. D) a gyrus is a ridge, and a sulcus is a groove. Answer: D Diff: 1 Page Ref: 389

7 Copyright © 2014 Pearson Education, Inc.

 

28) The cell bodies located in the anteriormost region of the spinal cord's gray matter belong to this group. A) somatic motor B) somatic sensory C) visceral motor D) visceral sensory Answer: A Diff: 3 Page Ref: 411-412 29) The axons in the inferior olivary nuclei relay sensory impulses for A) vision. B) proprioception. C) sound. D) smell. Answer: B Diff: 3 Page Ref: 378-379 30) The visual association area of the cerebral cortex is located in the A) frontal lobe. B) insula. C) parietal lobe. D) occipital lobe. Answer: D Diff: 2 Page Ref: 390, 39 31) The fluid-filled ventricles within the cerebrum are similar in function to the ________ of the spinal cord. A) subarachnoid space B) central canal C) pia mater D) dura mater Answer: B Diff: 2 Page Ref: 377 32) The function of the blood-brain barrier is to A) keep neurons from innervating blood vessels. B) prevent all contact between bloodborne molecules and brain tissue. C) help protect the central nervous system. D) provide an impenetrable barrier between blood and brain, because the brain gets all its nourishment from the cerebrospinal fluid. Answer: C Diff: 2 Page Ref: 407

8 Copyright © 2014 Pearson Education, Inc.

 

33) Three "seeing/vision" nuclei that occupy the midbrain of humans are the A) red nucleus, substantia nigra, and cerebral peduncle. B) inferior colliculi, reticular formation, and periaqueductal gray. C) lateral geniculate, red nucleus, and visual association area. D) superior colliculi, motor nucleus for oculomotor nerve, and motor nucleus for trochlear nerve. Answer: D Diff: 3 Page Ref: 382-383, 436 34) Which of the following is associated with Alzheimer's disease? A) accumulation of protein plaques around neurons B) cerebral ischemia C) deterioration of the substantia nigra of the midbrain D) inadequate vitamin B Answer: A Diff: 2 Page Ref: 421 35) Shearing of axons in a diffuse axonal injury causes such widespread disruptions because such an injury involves A) association fibers. B) commissural fibers. C) projection fibers. D) pyramidal tract fibers. Answer: C Diff: 3 Page Ref: 420 36) Which of the following is a descending motor spinal tract? A) dorsal column pathway B) pyramidal pathway C) spinocerebellar pathway D) spinothalamic pathway Answer: B Diff: 2 Page Ref: 418 37) What types of sensory information are conveyed toward the brain in the lateral spinothalamic tracts? A) discriminative touch B) equilibrium C) pain and temperature D) visual senses Answer: C Diff: 2 Page Ref: 414-416

9 Copyright © 2014 Pearson Education, Inc.

 

38) Cerebrospinal fluid is produced by choroid plexuses in all the following locations except the A) central canal. B) fourth ventricle. C) lateral ventricle. D) third ventricle. Answer: A Diff: 1 Page Ref: 405-407 39) Which of these lobes of the cerebrum lies anterior to the central sulcus? A) occipital lobe B) cerebellum C) temporal lobe D) frontal lobe Answer: D Diff: 1 Page Ref: 388- 389 40) Which region of the cerebellum functionally sequences complex voluntary muscle contractions to adjust posture as body position changes? A) the deep cerebellar nuclei B) white matter C) the cortex D) the flocculonodular lobes Answer: D Diff: 2 Page Ref: 383 41) The second largest region of the brain is the A) cerebrum. B) brain stem. C) diencephalon. D) cerebellum. Answer: D Diff: 1 Page Ref: 383 42) The sheet of dura mater that separates the right and left cerebral hemispheres is the A) falx cerebri. B) tentorium cerebri. C) falx cerebelli. D) superior sagittal sinus. Answer: A Diff: 2 Page Ref: 404

10 Copyright © 2014 Pearson Education, Inc.

 

43) At what vertebral level does the spinal cord terminate (inferiorly) in the average adult? A) S5 B) between L1 and L2 C) between L5 and S1 D) C3 Answer: B Diff: 2 Page Ref: 409 44) The cell bodies located in the posteriormost region of the spinal cord's gray matter belong to this group. A) somatic motor B) somatic sensory C) visceral motor D) visceral sensory Answer: B Diff: 2 Page Ref: 411-412 45) All of the following are structures of the limbic system except the A) hippocampus. B) cingulate gyrus. C) amygdaloid nucleus. D) caudate nucleus. Answer: D Diff: 2 Page Ref: 399-401 46) Which of the following is not a hindbrain structure? A) fourth ventricle B) pons C) medulla D) basal nuclei Answer: D Diff: 2 Page Ref: 375-376 47) Which of the following is not a function of nuclei within the reticular formation? A) adjust respiratory and cardiovascular functions B) consolidate and store memories C) coordinate cranial nerve reflexes D) maintain consciousness and alertness Answer: B Diff: 3 Page Ref: 400-401

11 Copyright © 2014 Pearson Education, Inc.

 

48) The subarachnoid space lies between what two layers of meninges? A) arachnoid and pia B) arachnoid and bone of skull C) arachnoid and dura D) dura and epidura Answer: A Diff: 2 Page Ref: 404 49) Cell bodies of the sensory neurons of the spinal nerves are located in A) the dorsal root ganglia external to the spinal cord. B) the posterior gray horn of the spinal cord. C) the thalamus. D) nuclei of the cranial nerves. Answer: A Diff: 3 Page Ref: 4118 50) The tough, dense fibrous connective tissue meningeal layer is the A) dura mater. B) subarachnoid. C) arachnoid. D) pia mater. Answer: A Diff: 1 Page Ref: 403 51) Cutting the corpus callosum would lead to which of the following? A) The right cerebral hemisphere cannot communicate directly with the left hemisphere. B) Visual reflexes are lost. C) Emotional problems develop. D) One-half of the body is paralyzed. Answer: A Diff: 2 Page Ref: 396 52) What is the anatomical distinction between the basal ganglia and the basal forebrain nuclei? A) The basal forebrain nuclei are located in the prefrontal cortex, whereas the basal ganglia are within the midbrain. B) The basal forebrain nuclei are located closer to the hypothalamus, whereas the basal ganglia lie near the internal capsule. C) The basal ganglia are located in the brain stem, whereas the basal forebrain nuclei are located within the cerebrum. D) The basal ganglia are located outside the CNS, whereas the basal forebrain nuclei are located within the cerebrum. Answer: B Diff: 3 Page Ref: 398-399

12 Copyright © 2014 Pearson Education, Inc.

 

53) The abstract representation of the homunculus overlaid on the somatosensory cerebral cortex has the A) head and face proximal to the longitudinal fissure. B) feet and ankles distal to the longitudinal fissure. C) thumbs and fingers proximal to the longitudinal fissure. D) genitals proximal to the longitudinal fissure. Answer: D Diff: 2 Page Ref: 391, 393 54) Which of the following is a motor area for vision? A) frontal eye field B) premotor cortex C) primary visual cortex D) visual association area Answer: A Diff: 2 Page Ref: 394 55) Posterior gray horns of the spinal cord consist of A) cell bodies of interneurons. B) dorsal roots. C) sensory ganglia. D) motor neurons. Answer: A Diff: 2 Page Ref: 411-412 56) The cauda equina A) consists of hair like the tail of a horse. B) describes the radiating patterns of projection fibers. C) is a series of nerve roots in the lumbar and sacral region. D) is confined to the thoracic and cervical segments of the spinal cord. Answer: C Diff: 2 Page Ref: 409 57) The calcarine sulcus contains which functional area? A) primary somatosensory B) auditory association C) frontal eye field D) primary visual Answer: D Diff: 3 Page Ref: 393

13 Copyright © 2014 Pearson Education, Inc.

 

58) A lesion to the inferior olivary nucleus in the medulla would interfere with the function of which other structure(s)? A) spinal motor neurons B) the limbic system C) the reticular formation D) the cerebellum Answer: D Diff: 3 Page Ref: 378-379 59) One of the distinctions between the cerebrum and cerebellum is that A) gray matter is found superficially and deep in the cerebrum but is present only superficially in the cerebellum. B) fibers of the cerebellum enter and leave ipsilaterally, whereas those of the cerebrum enter and leave contralateraly. C) the cerebrum is folded into gyri and lobes, whereas the cerebellum is only folded into folia but lacks lobes. D) the cerebellum is involved only in refining motor responses, whereas the cerebrum is involved in both motor function and cognition. Answer: B Diff: 3 Page Ref: 384 60) Cerebrospinal fluid is located within the A) subarachnoid space. B) orbits. C) cerebral cortex. D) superior sagittal sinus. Answer: A Diff: 2 Page Ref: 404 61) The only one of the meninges that follows the brain surface into a cerebral sulcus is the A) dura mater. B) arachnoid mater. C) pia mater. D) alma mater. Answer: C Diff: 2 Page Ref: 404 62) All of the following can lead to hydrocephalus except A) meningitis that scars the arachnoid mater. B) an overactive choroid plexus. C) blockage of the cerebral aqueduct by a small brain tumor. D) Alzheimer's disease. Answer: D Diff: 1 Page Ref: 407

14 Copyright © 2014 Pearson Education, Inc.

 

63) Which areas of the brain are most likely to process and store motor skill memories? A) the ependymal cells and ventricles B) the amygdala and cingulate gyrus C) the thalamus and hypothalamus D) the premotor cortex, basal ganglia, and cerebellum Answer: D Diff: 3 Page Ref: 383 394 64) Which of the following grooves separates the cerebrum from the cerebellum? A) lateral sulcus B) central sulcus C) longitudinal fissure D) transverse cerebral fissure Answer: D Diff: 1 Page Ref: 398 65) After someone faints, smelling salts of ammonia may be placed under the person's nose. The person breathes the ammonia vapors, which deliver a sharp jolt to the nasal membranes, and the person wakes up. This illustrates A) how smells bring about emotions. B) the function of the hypothalamus in controlling sleep. C) the function of sensory input to the reticular activating system. D) how smells elicit memories of being awake, causing the person to awaken. Answer: C Diff: 2 Page Ref: 400-401 66) Two parts of the brain that are most involved in emotions are the A) medulla and cerebellum. B) cingulate gyrus and hypothalamus. C) superior and inferior colliculi. D) red nucleus and substantia nigra. Answer: B Diff: 2 Page Ref: 399-400 67) A cerebrovascular accident patient who is unable to recognize faces and objects but can still visually perceive spatial arrangement of objects most likely has damage to A) the primary visual cortex. B) the occipital lobe. C) the posterior association area. D) the temporal lobe. Answer: D Diff: 3 Page Ref: 395

15 Copyright © 2014 Pearson Education, Inc.

 

68) The main visceral control center of the brain is the A) cerebral cortex. B) thalamus. C) reticular formation. D) hypothalamus. Answer: D Diff: 2 Page Ref: 385-386 69) Sleep-wake cycles are controlled by the pineal gland and the A) hypothalamus. B) medulla oblongata. C) thalamus. D) visual cortex. Answer: A Diff: 2 Page Ref: 387 70) If the most caudal part of the CNS is the conus medullaris, then the most rostral part is/are the A) prefrontal lobes. B) hypothalamus. C) precentral and postcentral gyri. D) cauda equina. Answer: A Diff: 2 Page Ref: 392 71) Which of the following is not part of the basal ganglia? A) putamen B) Wernicke's area C) globus pallidus D) caudate nucleus Answer: B Diff: 2 Page Ref: 398-399 72) Which of the following is not a midbrain structure? A) third ventricle B) cerebral peduncles C) corpora quadrigemina D) red nucleus Answer: A Diff: 2 Page Ref: 381-382

16 Copyright © 2014 Pearson Education, Inc.

 

73) The brain stem consists of the A) cerebrum, pons, midbrain, and medulla. B) midbrain, medulla, and pons. C) pons, medulla, cerebellum, and midbrain. D) midbrain only. Answer: B Diff: 2 Page Ref: 378 74) Neural centers that control heart rate, respiration, and blood pressure are located in the A) thalamus. B) medulla. C) midbrain. D) cerebrum. Answer: B Diff: 2 Page Ref: 380 75) A shallow groove on the surface of the cerebral cortex is called a A) sulcus. B) fissure. C) gyrus. D) furrow. Answer: A Diff: 1 Page Ref: 389 13.2 True/False Questions 1) The hypothalamus, in addition to the cerebral cortex, is in control of emotional responses. Answer: TRUE Diff: 2 Page Ref: 385-387 2) The pituitary gland (hypophysis) is connected to the hypothalamus and receives regulatory impulses that control the secretion of hormones. Answer: TRUE Diff: 2 Page Ref: 385-387 3) The ventral portion of the cerebral peduncles connect the midbrain to the cerebellum and contain the pyramidal (corticospinal) motor tracts descending to the spinal cord. Answer: FALSE Diff: 3 Page Ref: 382 4) The vasomotor center of the medulla regulates blood pressure. Answer: TRUE Diff: 2 Page Ref: 380

17 Copyright © 2014 Pearson Education, Inc.

 

5) In the olivary nuclei in the medulla, mostly proprioceptive sensory impulses are relayed to the cerebellum. Answer: TRUE Diff: 2 Page Ref: 378-379 6) The medullary respiratory center controls the rhythm and rate of breathing. Answer: TRUE Diff: 2 Page Ref: 380 7) Spinal nerve pairs in the cervical region of the spinal cord are found inferior to the vertebral bodies of the same number. Answer: FALSE Diff: 2 Page Ref: 409 8) The cerebellum has an outer cortex, internal white matter, and areas of gray matter buried deep in the white matter. Answer: TRUE Diff: 2 Page Ref: 383 9) Cerebrospinal fluid passes through choroidal villi to enter the superior sagittal sinus. Answer: FALSE Diff: 2 Page Ref: 404 10) The reticular activating system (RAS) contains structures that regulate and cause emotional responses. Answer: FALSE Diff: 2 Page Ref: 400-401 11) The amygdala is part of the limbic system and has neurons that process the emotions and reactions to fear. Answer: TRUE Diff: 2 Page Ref: 399-400 12) The limbic system has neural ties to the autonomic nervous system, the hypothalamus, and the reticular formation. Answer: TRUE Diff: 2 Page Ref: 399-400 13) The meninges of the brain differ from those of the spinal cord in that the dura mater of the brain splits into two layers and that there is no epidural space. Answer: TRUE Diff: 2 Page Ref: 403-404, 412 14) Damage to the ventral root causes spastic paralysis, whereas damage to the descending tracts causes flaccid paralysis. Answer: FALSE Diff: 3 Page Ref: 412 18 Copyright © 2014 Pearson Education, Inc.

 

15) The white matter of the spinal cord contains ascending and descending pathways known as tracts. Answer: TRUE Diff: 2 Page Ref: 413-414 13.3 Short Answer Questions 1) This is a term for neural pathways crossing from one side of the CNS to the other. Answer: decussation Diff: 2 Page Ref: 413 2) The white matter on both sides of the spinal cord is divided into three white columns, also known as ________. Answer: funiculi (singular: funiculus) Diff: 2 Page Ref: 411 3) This ascending pathway of the spinal cord carries neural impulses generated from sensory receptors sensitive to fine touch, pressure, and some parts of proprioception. Answer: dorsal column pathway Diff: 3 Page Ref: 414 4) What is another term for the descending pyramidal tracts that relay motor impulses from the brain and down the spinal cord? Answer: corticospinal tracts Diff: 2 Page Ref: 418 5) The internal white matter of the cerebellum is called the ________, or "tree of life." Answer: arbor vitae Diff: 1 Page Ref: 383 6) Cerebrovascular accident is a long name for what type of brain injury? Answer: stroke Diff: 2 Page Ref: 419 7) The pineal gland secretes the hormone ________, which signals the body to prepare for sleep. Answer: melatonin Diff: 2 Page Ref: 387 8) This type of spinal cord damage is due to severe damage to the anterior horn or ventral motor roots. Answer: flaccid paralysis Diff: 2 Page Ref: 412 9) The most common form of spina bifida is ________. Answer: spina bifida cystica Diff: 2 Page Ref: 421 19 Copyright © 2014 Pearson Education, Inc.

 

10) ________ is a common CNS-related congenital condition in which the affected person has difficulty throughout life controlling voluntary muscles and which may be due to infection of the placenta. Answer: Cerebral palsy Diff: 2 Page Ref: 422 11) Hyperactivity in the amygdala and dysfunction in the limbic association area and the hippocampus are involved in the extreme response to triggered memory experienced by individuals suffering from ________. Answer: post-traumatic stress disorder (PTSD) Diff: 2 Page Ref: 400 12) The ________ can be thought of as the "gateway" to the cerebral cortex. Answer: thalamus Diff: 2 Page Ref: 385 13) Long-term storage of memories involves the ________ of the limbic system. Answer: hippocampal formation (hippocampus) Diff: 2 Page Ref: 400 14) The massive development of the ________ distinguishes the human brain from many other vertebrate species. Answer: cerebrum Diff: 1 Page Ref: 388 15) There are three types of white matter in the cerebrum. These include the association fibers, projection fibers, and ________. Answer: commisural fibers Diff: 2 Page Ref: 396-398 13.4 Essay Questions 1) Describe the difference between the primary sensory areas of the cerebrum and their corresponding association areas. Answer: The primary sensory regions of the cerebral cortex receive impulses initiated from peripheral sensory receptors. For example, the primary auditory area receive impulses from the cochlea that convey the pitch and loudness of a sound. The auditory association areas allow multiple neuronal circuits to compare and integrate incoming impulses. Simultaneously, previously inactive neural circuits are re-activated resulting in coordination of memory, interpretation and execution of cognitive or motor action plans. In this example, would allow one to give meaning to words spoken by another. Diff: 2 Page Ref: 391-394

20 Copyright © 2014 Pearson Education, Inc.

 

2) Briefly describe the three types of cerebral white matter and their functions. Answer: Commissures connect corresponding gray areas of the right and left hemispheres, association fibers connect neuron to neuron in the same hemisphere, and projection fibers connect the cerebrum to distal/caudal regions of the brain and spinal cord and vice versa. Diff: 2 Page Ref: 396 3) Describe the structure and functions of the thalamus. Answer: The thalamus makes up most of the diencelphalon and encloses much of the third ventricle. The right and left half are connected by the interthalamic adhesion. Overall, the thalamus processes and relays impulses to and from the cerebral cortex. There are about 12 different major nuclei, and their functions vary. Some are relay nuclei for sensory impulses to the rest of the brain; these include nuclei that receive and pass on impulses from the retina of the eye and the cochlea of the inner ear. All sensory imoulses that are relayed to the cerebrum are routed through the thalamus; thus, the thalamus is a "relay center." It can also be thought of as a sensory impulse filter, because it may amplify or diminish the "strength"/frequency of the original impulse. Diff: 2 Page Ref: 385 4) Briefly identify the role of the following structures in controlling voluntary skeletal movement: cerebellum, primary motor cortex, premotor cortex, pyramidal tracts, inferior olivary nucleus, and cerebral and cerebellar peduncles. Answer: The premotor cortex and primary motor cortex of the frontal lobe of the cerebrum sequence and coordinate voluntary contraction of skeletal muscles. Pyramidal neurons of the cortex relay impulses to specific muscle groups. These pyramidal tracts (axons) descend through the brain stem and continue down the spinal cord. However, in the midbrain, these tracts will pass through the cerebral peduncles, which will simultaneously relay descending impulses to the cerebellum. Then the superior cerebellar peduncles will direct impulses in the reverse direction, back toward the pre-motor cortex of the cerebrum. At the same time the cerebellum also receives proprioceptive sensory information via the inferior olivary nucleus of the medulla, and it compares actual body movements to the intended movement generated by the cerebrum. The cerebellum is responsible for constantly refining the actual movement to achieve the intended movement. Diff: 3 Page Ref: 383-385, 394, 418 5) Describe the three meninges of the brain. Answer: The dura mater is the outer membrane and is extremely thick and inelastic. It is composed of two layers around the brain. The outermost layer, the periosteal layer, is continuous with the periosteum of the skull bones. The innermost layer of the dura mater is the meningeal layer. The dura mater has three distinct regions the falx cerebri, the falx cerebelli, and the tentorium cerebelli. The middlemost meninges is the arachnoid mater. Supporting trabeculae from the arachnoid layer raise this membrane above the innermost layer—the pia mater. In the space between these two deeper meninges, the subarachnoid space, circulates cerebrospinal fluid (CSF). The most delicate membrane is the pia mater, which adheres tightly to the cortical surface of the brain, covering every sulcus and gyrus. This inner meninges is also the carrier of microscopic blood vessels. Diff: 2 Page Ref: 403-404 21 Copyright © 2014 Pearson Education, Inc.

 

Human Anatomy, 7e (Marieb/Mitchell/Smith) Chapter 14 The Peripheral Nervous System 14.1 Multiple Choice Questions

Figure 14.1 Use the diagram above to answer the following questions. 1) Which letter indicates a tactile corpuscle (Meissner's corpuscle) that is responsive to light pressure? A) A B) B C) C D) D E) E Answer: B Diff: 2 Page Ref: 429

1 Copyright © 2014 Pearson Education, Inc.

 

2) Which letter indicates a lamellar corpuscle (Pacinian corpuscle) that rapidly adapts to deep pressure stimuli? A) A B) B C) C D) D E) E Answer: D Diff: 2 Page Ref: 429 3) Which letter indicates receptors that respond chiefly to pain and temperature? A) A B) B C) C D) D E) E Answer: C Diff: 2 Page Ref: 429 4) Which letter indicates a bulbous corpuscle (Ruffini ending) that slowly adapts to deep pressure stimuli? A) A B) B C) C D) D E) E Answer: E Diff: 2 Page Ref: 429-430 5) Which letter indicates tactile epithelial cells (Merkel discs), light pressure receptors found in the epidermis? A) A B) B C) C D) D E) E Answer: A Diff: 1 Page Ref: 429

2 Copyright © 2014 Pearson Education, Inc.

 

Figure 14.2 Use the diagram above to answer the following questions. 6) Which letter indicates the cranial nerve with nerve fibers that pass through the hypoglossal canal? A) A B) B C) C D) D E) E Answer: E Diff: 2 Page Ref: 432, 443 7) Which letter indicates the cranial nerve that has three major branches, and is cranial nerve V? A) A B) B C) C D) D E) E Answer: C Diff: 2 Page Ref: 433, 434

3 Copyright © 2014 Pearson Education, Inc.

 

8) Which letter indicates the cranial nerve that transmits olfactory impulses? A) A B) B C) C D) D E) E Answer: A Diff: 2 Page Ref: 432, 433 9) Which letter indicates the structure where cranial nerve fibers from the retina cross to the opposite side of the brain? A) A B) B C) C D) D E) E Answer: B Diff: 2 Page Ref: 433 10) Which letter indicates the cranial nerve that has motor fibers that innervate muscles of facial expression? A) A B) B C) C D) D E) E Answer: D Diff: 2 Page Ref: 433, 434 11) Which cranial nerve, along with cranial nerve pair II, relays specific somatic sensory impulses? A) I B) II C) IV D) V E) VIII Answer: E Diff: 2 Page Ref: 434

4 Copyright © 2014 Pearson Education, Inc.

 

12) Which cranial nerve, along with the abducens and oculomotor nerves, has somatic motor fibers that pass through the superior orbital fissure? A) I B) II C) IV D) VII E) VIII Answer: C Diff: 3 Page Ref: 434, 436 13) Which cranial nerve pair has two sensory branches which both pass through the internal acoustic meatus? A) I B) II C) IV D) V E) VIII Answer: E Diff: 2 Page Ref: 440 14) Of the cranial nerve pairs listed below, which nerve, along with XI and XII, relays only somatic motor impulses? A) I B) II C) IV D) V E) VIII Answer: C Diff: 3 Page Ref: 434 15) Which cranial nerve pair is characterized by somatic sensory fibers crossing at the ventral midline of the brain? A) I B) II C) IV D) V E) VIII Answer: B Diff: 2 Page Ref: 435

5 Copyright © 2014 Pearson Education, Inc.

 

16) Which cranial nerve pair, along with cranial nerve III, has fibers that pass through the superior orbital fissure? A) I B) II C) IV D) VII E) VIII Answer: C Diff: 2 Page Ref: 436-437 17) Which cranial nerve pair, similar to cranial nerve II, runs through a canal that bears the same name? A) IV B) V C) VIII D) XI E) XII Answer: E Diff: 2 Page Ref: 443 18) Damage to which cranial nerve pair can cause difficulties in speech and swallowing? A) IV B) V C) VIII D) XI E) XII Answer: E Diff: 2 Page Ref: 443 19) The spinal root of which cranial nerve pair innervates the sternocleidomastoid and trapezius muscles? A) IV B) V C) VIII D) XI E) XII Answer: D Diff: 2 Page Ref: 443 20) Damage to which cranial nerve pair can cause deafness and problems with equilibrium? A) IV B) V C) VIII D) XI E) XII Answer: C Diff: 2 Page Ref: 440 6 Copyright © 2014 Pearson Education, Inc.

 

21) Which cranial nerve pair has three major branches, the maxillary, mandibular, and ophthalmic divisions? A) IV B) V C) VIII D) XI E) XII Answer: B Diff: 2 Page Ref: 437-438 22) Which of the cranial nerve listed below is the largest in diameter? A) IV B) V C) VIII D) XI E) XII Answer: B Diff: 2 Page Ref: 438 23) Which cranial nerve pair along with the vagus and glossopharyngeal nerves, exits through the jugular foramen? A) IV B) V C) VIII D) XI E) XII Answer: D Diff: 2 Page Ref: 443 24) Sensory fibers of which cranial nerve pair pass through the cribriform plate of the ethmoid bone? A) I B) II C) IV D) V E) VIII Answer: A Diff: 2 Page Ref: 435

7 Copyright © 2014 Pearson Education, Inc.

 

25) Damage to which cranial nerve pair may result in anosmia? A) I B) II C) IV D) V E) VIII Answer: A Diff: 2 Page Ref: 435 26) An example of an encapsulated receptor is A) a hair follicle receptor. B) a tendon organ. C) a tactile epithelial cell (Merkel disc). D) a lamellar corpuscle (Pacinian corpuscle). Answer: D Diff: 2 Page Ref: 430-431 27) A lesion to the common fibular nerve causes A) footdrop. B) paralysis of the peroneal muscles. C) hammer toe. D) the same deficit as a ruptured calcaneal tendon causes. Answer: A Diff: 2 Page Ref: 459 28) A pudendal nerve block may be given A) to treat pain from sciatica. B) in surgery to halt breathing temporarily. C) in surgery to relax the muscles that cover the fibula. D) to a woman giving birth. Answer: D Diff: 2 Page Ref: 459 29) Sensory impulses from the skin would be transmitted into the spinal cord on the A) ventral root of spinal nerves. B) dorsal root of spinal nerves. C) sympathetic nerve pathway. D) pathway of parasympathetic neurons. Answer: B Diff: 1 Page Ref: 444

8 Copyright © 2014 Pearson Education, Inc.

 

30) A receptor that monitors stretch of the patellar tendon can be classified as a A) mechanoreceptor and interoceptor. B) mechanoreceptor and proprioceptor. C) nociceptor and exteroceptor. D) baroreceptor and proprioceptor. Answer: B Diff: 3 Page Ref: 429-430 31) A receptor that monitors stretch of the stomach can be classified as a A) mechanoreceptor and interoceptor. B) mechanoreceptor and proprioceptor. C) nociceptor and exteroceptor. D) baroreceptor and proprioceptor. Answer: A Diff: 2 Page Ref: 429-430 32) Which spinal nerve relays motor impulses to most of the muscles of the posterior forearm? A) ulnar B) median C) axillary D) radial Answer: D Diff: 2 Page Ref: 453 33) The tibial and common fibular nerves branch from the ________ nerve. A) sciatic B) pudendal C) axillary D) femoral Answer: A Diff: 1 Page Ref: 455 34) If one likens a spinal nerve to the trunk of a tree, then the two roots of this tree are the dorsal and ventral roots. What would represent the two large branches of this tree? A) rami communicantes B) dorsal and ventral root ganglia C) ventral and dorsal rami D) the brachial plexus Answer: C Diff: 2 Page Ref: 444

9 Copyright © 2014 Pearson Education, Inc.

 

35) Anesthetic injection to which nerve may block pain during childbirth? A) sciatic B) femoral C) pudendal D) obturator Answer: C Diff: 2 Page Ref: 459 36) Which of the cranial nerves listed below provides innervation to one of the extrinsic eye muscles? A) trigeminal B) trochlear C) optic D) facial Answer: B Diff: 1 Page Ref: 436 37) The major nerve plexus to the upper limbs is the A) brachial plexus. B) sacral plexus. C) cervical plexus. D) lumbar plexus. Answer: A Diff: 1 Page Ref: 447-448 38) Contraction rate of the diaphragm is controlled by which nerve? A) phrenic B) vagus C) trigeminal D) trochlear Answer: A Diff: 1 Page Ref: 447 39) Of the following, the nerve that forms from fibers of both the medial and lateral cords of the brachial plexus is the A) median. B) axillary. C) ulnar. D) musculocutaneous. Answer: A Diff: 1 Page Ref: 448-449

10 Copyright © 2014 Pearson Education, Inc.

 

40) Extrafusal muscle fibers that surround a muscle spindle and resist excessive muscle stretching are innervated by A) primary sensory endings. B) secondary sensory endings. C) α efferent neurons. D) γ efferent neurons. Answer: C Diff: 3 Page Ref: 432 41) What is the only cranial nerve that has fibers that enter and leave the skull through different foramen? (Hint: It enters through the foramen magnum and leaves through the jugular foramen.) A) VI B) XII C) XI D) I Answer: C Diff: 1 Page Ref: 434, 443 42) Which of the cranial nerves listed below is a special somatic sensory nerve? A) facial B) vestibulocochlear C) accessory D) trochlear Answer: B Diff: 2 Page Ref: 434 43) Which cranial nerves have fibers that relay somatic motor impulses to pharyngeal muscles during swallowing? A) I and II B) V and XII C) VI and VII D) IX and X Answer: D Diff: 2 Page Ref: 441-442 44) Which of these receptors would not be found within a synovial joint capsule? A) free nerve endings B) tactile corpuscles (Meissner's corpuscles) C) lamellar corpuscles (Pacinian corpuscles) D) bulbous corpuscles (Ruffini endings) Answer: B Diff: 2 Page Ref: 430

11 Copyright © 2014 Pearson Education, Inc.

 

45) Mixed cranial nerves containing both motor and sensory fibers include all of the following except the A) facial. B) optic. C) trigeminal. D) vagus. Answer: B Diff: 1 Page Ref: 434 46) The cranial nerves that have neural connections with the tongue include all of the following except the A) trigeminal. B) facial. C) glossopharyngeal. D) accessory. Answer: D Diff: 1 Page Ref: 438-443 47) Which of these nerves does not arise primarily from the brachial plexus? A) median B) phrenic C) radial D) ulnar Answer: B Diff: 2 Page Ref: 447 48) The gastrocnemius muscle is innervated by the ________ nerve. A) obturator B) common peroneal C) tibial D) femoral Answer: C Diff: 1 Page Ref: 455 49) Which of the following regions is not innervated by nerves of the sacral plexus (L4 - S4)? A) posterior thigh B) anterior thigh muscles C) foot D) external genitalia Answer: B Diff: 2 Page Ref: 455-457

12 Copyright © 2014 Pearson Education, Inc.

 

50) Which of these cranial nerve pairs do not pass through the superior orbital fissures? A) abducens B) oculomotor C) optic D) trochlear Answer: C Diff: 2 Page Ref: 435 51) Spinal nerves found at the level of L4 to about S4 form the A) lumbar plexus. B) femoral plexus. C) sacral plexus. D) thoracic plexus. Answer: C Diff: 2 Page Ref: 455-459 52) The abducens nerve (CN VI) A) innervates the lateral rectus muscle of the eye. B) relays sensory information from the taste buds on the tongue. C) exits from the medulla. D) if paralyzed, results in Bell's palsy. Answer: A Diff: 1 Page Ref: 438 53) An injury to the ulnar nerve causes A) claw hand. B) inability to oppose the thumb. C) the "hand of benediction." D) wrist drop. Answer: A Diff: 1 Page Ref: 452 54) Spinal nerves are A) present in 31 pairs. B) located exclusively in the vertebral canal. C) present only in the thoracic region where plexuses are absent. D) purely sensory nerves. Answer: A Diff: 1 Page Ref: 444 55) A person who cannot blink or smile could have damage to which cranial nerve? A) I B) III C) V D) VII Answer: D Diff: 2 Page Ref: 440 13 Copyright © 2014 Pearson Education, Inc.

 

56) The trigeminal nerve contains which class(es) of nerve fibers? A) somatic sensory only B) somatic motor and general somatic sensory C) somatic sensory, visceral sensory, and visceral motor D) somatic motor only Answer: B Diff: 3 Page Ref: 434, 437-438 57) The primary nerve to muscles of the anterior thigh is the A) sciatic. B) obturator. C) sural. D) femoral. Answer: D Diff: 1 Page Ref: 455 58) Which of these foramen does not include a branch of the trigeminal nerve? A) mandibular B) ovale C) rotundum D) spinosum Answer: D Diff: 3 Page Ref: 438 59) To avoid double vision, which of the following groups of cranial nerves must be functioning correctly? A) I, IX, and X B) VII, VIII, and XII C) V, XI, and XII D) III, IV, and VI Answer: D Diff: 3 Page Ref: 436-438 60) What specific dermatome lies at the level of the navel? A) C2 B) S4 C) T10 D) T1 Answer: C Diff: 1 Page Ref: 460

14 Copyright © 2014 Pearson Education, Inc.

 

61) Tactile corpuscles (Meissner's corpuscles) A) occur in hairy skin. B) occur in dermal papillae. C) are nociceptors. D) are free unencapsulated dendritic endings. Answer: B Diff: 2 Page Ref: 430 62) Which cranial nerve pair is comprised of ventral rootlets from C1-C5? A) abducens B) accessory C) glossopharyngeal D) hypoglossal Answer: B Diff: 3 Page Ref: 443 63) Which of these nerves arises from the posterior cord of the brachial plexus? A) median B) musculocutaneous C) radial D) ulnar Answer: C Diff: 3 Page Ref: 448, 453 64) Which of these statements about intercostal nerves is true? A) They are part of a nerve plexus. B) They innervate only the intercostal muscles. C) The ventral rami of T1-T12 spinal nerves lie just inferior to each rib. D) They consist of 12 pairs. Answer: C Diff: 2 Page Ref: 444 65) The lateral cord of the brachial plexus forms directly from A) the upper, middle, and lower trunks. B) the posterior divisions of the middle and lower trunks. C) the anterior divisions of the upper and middle trunks. D) roots C7-T1. Answer: C Diff: 3 Page Ref: 448-449 66) Which nerve does not arise from the anterior division of the brachial plexus? A) median B) musculocutaneous C) radial D) ulnar Answer: C Diff: 3 Page Ref: 449 15 Copyright © 2014 Pearson Education, Inc.

 

67) Migraine headaches result when cerebral arteries dilate in response to neurotransmitters released from the A) abducens nerve. B) facial nerve. C) trigeminal nerve. D) vestibulocochlear nerve. Answer: C Diff: 2 Page Ref: 461 68) The cranial nerve pair with large branches that passes through the foramen ovale of the skull is A) I. B) III. C) V. D) VII. Answer: C Diff: 3 Page Ref: 438 69) A lesion/injury to the radial nerve causes A) footdrop. B) claw hand. C) the "hand of benediction." D) wrist-drop. Answer: D Diff: 2 Page Ref: 452 70) The phrenic nerve receives fibers from A) C3-C5. B) T1-T7. C) CN X. D) CN XI. Answer: A Diff: 2 Page Ref: 447 71) The only cranial nerve that travels into the abdomen is the A) vagus. B) hypoglossal. C) glossopharyngeal. D) vestibulocochlear. Answer: A Diff: 1 Page Ref: 442

16 Copyright © 2014 Pearson Education, Inc.

 

72) The muscles that initiate flexion at the elbow joint (biceps brachii and brachialis) are innervated by what nerve from the lateral cord? A) radial B) median C) ulnar D) musculocutaneous Answer: D Diff: 1 Page Ref: 449-450 73) Motor neurons to skeletal muscle belong to the A) somatic division of the nervous system. B) autonomic nervous system. C) afferent division of the autonomic nervous system. D) visceral division of the nervous system. Answer: A Diff: 2 Page Ref: 428 74) Disorders of balance may follow trauma to which nerve? A) abducens B) vestibulocochlear C) trigeminal D) accessory Answer: B Diff: 2 Page Ref: 440 75) Starting at the spinal cord and proceeding distally, the subdivisions of the brachial plexus are A) rami, trunks, divisions, cords. B) rami, divisions, cords, trunks. C) divisions, rami, trunks, cords. D) trunks, divisions, cords, rami. Answer: A Diff: 2 Page Ref: 448 14.2 True/False Questions 1) When you contract your Biceps brachii muscle, you are utilizing the somatic sensory portion of the nervous system. Answer: FALSE Diff: 1 Page Ref: 428 2) Both the parasympathetic and sympathetic divisions are part of the somatic motor nervous division. Answer: FALSE Diff: 2 Page Ref: 428

17 Copyright © 2014 Pearson Education, Inc.

 

3) All of the special senses taste, smell, vision, hearing, and equilibrium are part of the afferent division of the peripheral nervous system. Answer: TRUE Diff: 2 Page Ref: 428 4) Exteroceptors process information from visceral organs. Answer: FALSE Diff: 1 Page Ref: 429 5) All of the following are exteroceptors: tactile corpuscles, lamellar corpuscles, and tactile epithelial cells. Answer: TRUE Diff: 3 Page Ref: 429-430 6) Of the exteroceptors, only tactile corpuscles, lamellar corpuscles, and bulbous corpuscles are encapsulated. Answer: TRUE Diff: 3 Page Ref: 429-430 7) The primary proprioceptors are muscle spindles, tendon organs, and joint kinesthetic receptors. Answer: TRUE Diff: 3 Page Ref: 429-430 8) One of the largest branches of the sacral plexus is the sciatic nerve. Answer: TRUE Diff: 2 Page Ref: 455 9) The accessory nerve (cranial nerve CN XI) contains fibers that assist the hypoglossal nerve (CN XII). Answer: FALSE Diff: 3 Page Ref: 443 10) A dermatome is an area of skin that is innervated by the cutaneous branches from one spinal nerve pair. Answer: TRUE Diff: 2 Page Ref: 460 11) Shingles is a latent viral infection in adults that involves both the nervous system and the cutaneous system. Answer: TRUE Diff: 2 Page Ref: 461 12) The vagus nerve (CN X) is the only cranial nerve pair found inferior to the diaphragm. Answer: TRUE Diff: 2 Page Ref: 442 18 Copyright © 2014 Pearson Education, Inc.

 

13) A nerve plexus is formed by interconnected dorsal and ventral rami of spinal nerves. Answer: FALSE Diff: 2 Page Ref: 446 14) The sequence of brachial plexus components from the spinal cord to the distal targets is as follows: roots, trunks, divisions, cords. Answer: TRUE Diff: 2 Page Ref: 448 15) An interoceptor receives information from receptors in the skin, such as temperature, pressure, and pain. Answer: FALSE Diff: 2 Page Ref: 429 14.3 Short Answer Questions 1) The portion of the nervous system that does NOT include the brain and spinal cord. Answer: peripheral Diff: 1 Page Ref: 427 2) This term refers to clusters of peripheral cell bodies outside the CNS. Answer: ganglia Diff: 2 Page Ref: 428 3) The capsules of tactile and lamellar corpuscles consist of layer(s) of ________ wrapped by connective tissue. Answer: Schwann cells Diff: 3 Page Ref: 431 4) This category of receptors elicit a response of "Ouch!" Answer: nociceptors Diff: 2 Page Ref: 429 5) These sensory receptors are found not only on your tongue and nose, but also internally so your body can respond to changes in blood chemistry. Answer: chemoreceptors Diff: 1 Page Ref: 429 6) Ventral rami from all spinal nerves except numbers ________ contribute to nerve plexuses. Answer: T2-T12 Diff: 1 Page Ref: 446 7) Clinicians use these to identify the level of spinal injuries and to trace the site of adult herpes zoster infections. Answer: dermatomes Diff: 2 Page Ref: 460 19 Copyright © 2014 Pearson Education, Inc.

 

8) Nerves from the ________ cords of the brachial plexus innervate the anterior flexor muscles of the upper limb. Answer: lateral and medial Diff: 3 Page Ref: 448-449 9) Damage to this nerve shows a typical hand position known as the "hand of benediction," or the "ape hand." Answer: median nerve Diff: 3 Page Ref: 452 10) The cervical plexus is found deep in the neck region covered by by what superficial muscle? Answer: sternocleidomastoid Diff: 1 Page Ref: 446 11) A nerve from the ________ cord of the brachial plexus innervates the posterior extensor muscles of the upper limb. Answer: posterior Diff: 3 Page Ref: 453 12) The obturator nerve is part of which nerve plexus? Answer: lumbar Diff: 1 Page Ref: 455 13) This nerve innervates the diaphragm. Answer: phrenic Diff: 2 Page Ref: 447 14) The ________ and optic nerves are the only cranial nerves that do not have nuclei within the brain. Answer: olfactory Diff: 1 Page Ref: 435-436 15) Damage to this nerve may cause difficulties in swallowing, decrease in saliva production and some difficulties in tasting, in particular sour and bitter substances. Answer: glossopharyngeal Diff: 2 Page Ref: 441

20 Copyright © 2014 Pearson Education, Inc.

 

14.4 Essay Questions 1) Distinguish the trunks, roots, and divisions of the brachial plexus, and identify their relationship to the flexor and extensor muscle groups of the upper limb. Answer: The ventral rami of spinal nerves C5-T1 combine and subdivide into three trunks: upper, middle, and lower. Each of these trunks further branches into an anterior and posterior division. The divisions recombine to form cords, from which the peripheral nerves arise. The nerves that innervate the flexor muscles of the upper limb (median, musculocutaneous, ulnar) arise from the anterior divisions, whereas those that innervate the extensor muscles (radial) arise from the posterior division. Diff: 2 Page Ref: 447-453 2) List and describe the four types of joint kinesthetic receptors. Answer: Lamellar corpuscles (Pacinian corpuscles ) are stretch receptors measuring acceleration and rapid movement of the joints. Bulbous corpuscles (Ruffini endings) are slower-adapting stretch receptors than the lamellar corpuscles. They measure the positions of nonmoving joints and the stretch of joints that undergo slow sustained movements. Free nerve endings are pain receptors. Receptor tendon organs are also found, but their function is not known. Diff: 3 Page Ref: 432 3) Describe the action and innervation of the six extrinsic muscles/ extraocular muscles of the eyeball. Answer: (1) The medial rectus muscle, (2) superior rectus, (3) inferior oblique muscle, and (4) inferior rectus are all innervated by cranial nerve pair III—the oculomotor nerves. The muscles with the name rectus move the eye in that same direction. For example, the superior rectus muscle moves the eyeball superiorly. The inferior oblique moves the eye up and out or superolaterally, (5) the superior oblique muscle is innervated by the trochlear muscle and acts to move the eye down and out or inferolaterally, and (6) the lateral rectus abducts/moves the eye laterally and is innervated by the abducens nerve. Diff: 3 Page Ref: 434 4) Name the three divisions of the trigeminal nerve and the foramina through which they run. Answer: (1) The ophthalmic division runs from the forehead and upper eyelid to the pons via superior orbital fissures and the supraorbital foramina; (2) the maxillary division runs from the skin, gums and lips superficial to the maxilla to the pons via the foramen rotundum and the infraorbital foramina; and (3) the mandibular division passes through the foramen ovale, the mandibular foramen, and the mental foramen. Diff: 2 Page Ref: 437-438

21 Copyright © 2014 Pearson Education, Inc.

 

5) Describe the ventral rami, the terminal branches and the pathway of the sciatic nerve. Answer: The sciatic nerve is the largest branch of the sacral plexus, and it is the largest and longest nerve in the body. It is comprised of ventral rami from L4 to S3.It innervates nearly all of the lower limb except the anterior and medial regions of the thigh. This nerve is actually two nerves (tibial and common fibular nerves) in a common sheath. It passes through the greater sciatic notch, then moves deep to the gluteal muscles before descending into the posterior compartment of the thigh. The upper fibers innervate the hamstring muscles. Just superior to the popliteal fossa it branches into the tibial and common fibular nerve. The tibial nerve provides innervation to the posterior calf muscles—plantar flexors and flexors of the toes. The common fibular nerve innervates muscles of the anterolateral leg, providing dorsiflexion and eversion of the foot and extension of the toes. Diff: 2 Page Ref: 456-457

22 Copyright © 2014 Pearson Education, Inc.

 

Human Anatomy, 7e (Marieb/Mitchell/Smith) Chapter 15 The Autonomic Nervous System and Visceral Sensory Neurons 15.1 Multiple Choice Questions

Figure 15.1 Use the diagram above to answer the following questions. 1) Which letter indicates the celiac ganglion and plexus? A) A B) B C) C D) D E) E Answer: C Diff: 2 Page Ref: 474 1 Copyright © 2014 Pearson Education, Inc.

 

2) Which letter indicates the aortic plexus? A) A B) B C) C D) D E) E Answer: D Diff: 2 Page Ref: 474 3) Which letter indicates the left vagus nerve? A) A B) B C) C D) D E) E Answer: A Diff: 1 Page Ref: 474 4) Which letter indicates the cardiac plexus? A) A B) B C) C D) D E) E Answer: B Diff: 2 Page Ref: 474 5) Which letter indicates the pelvic sympathetic trunk? A) A B) B C) C D) D E) E Answer: E Diff: 2 Page Ref: 474 6) The general visceral motor division of the peripheral nervous system describes the A) parasympathetic division. B) sympathetic division. C) autonomic nervous system. Answer: C Diff: 1 Page Ref: 468

2 Copyright © 2014 Pearson Education, Inc.

 

7) Division of the ANS responsible for the fight-or-flight response describes the A) parasympathetic division. B) sympathetic division. C) autonomic nervous system. Answer: B Diff: 1 Page Ref: 470-471 8) Division of the ANS most active during vigorous exercise describes the A) parasympathetic division. B) sympathetic division. C) autonomic nervous system. Answer: B Diff: 2 Page Ref: 470-471 9) Division of the ANS most active when the body is at rest describes the A) parasympathetic division. B) sympathetic division. C) autonomic nervous system. Answer: A Diff: 2 Page Ref: 470-471 10) This division can also be called the craniosacral division. A) parasympathetic division B) sympathetic division C) autonomic nervous system Answer: A Diff: 1 Page Ref: 470-471 11) Cell body of this autonomic neuron lies outside the CNS. A) collateral ganglia B) rami communicantes C) pre-ganglionic neuron D) splanchnic nerves E) post-ganglionic neuron Answer: E Diff: 2 Page Ref: 468 12) Cell body of this autonomic neuron lies within the CNS. A) collateral ganglia B) rami communicantes C) pre-ganglionic neuron D) splanchnic nerves E) post-ganglionic neuron Answer: C Diff: 2 Page Ref: 468

3 Copyright © 2014 Pearson Education, Inc.

 

13) Division that is dominant during exercise, excitement, or emergencies. A) parasympathetic B) enteric nervous system C) inferior hypogastric plexus D) sympathetic E) cholinergic Answer: D Diff: 1 Page Ref: 470 14) The overall integrating center for the ANS. A) hypothalamus B) medulla C) medulla oblongata D) rami communicantes E) sympathetic trunk Answer: A Diff: 1 Page Ref: 483 15) Division that is dominant during rest, digestion, and excretion. A) parasympathetic B) enteric nervous system C) inferior hypogastric plexus D) sympathetic E) cholinergic Answer: A Diff: 1 Page Ref: 470-471 16) A descriptive term that identifies the type of neurotransmitter released by sympathetic postganglionic fibers. A) postganglionic neuron B) adrenergic C) medulla D) cholinergic E) none of the above Answer: B Diff: 2 Page Ref: 471 17) A class of preganglionic fibers present in both sympathetic and parasympathetic divisions. A) postganglionic neuron B) adrenergic C) medulla D) cholinergic E) none of the above Answer: D Diff: 2 Page Ref: 471

4 Copyright © 2014 Pearson Education, Inc.

 

18) Short nerve branches that connect the ventral rami of spinal nerves to the sympathetic ganglia. A) hypothalamus B) medulla C) medulla oblongata D) rami communicantes E) sympathetic trunk Answer: D Diff: 3 Page Ref: 476 19) Sympathetic structures of the abdomen that lie anterior to the vertebral column. A) collateral ganglia B) rami communicantes C) pre-ganglionic neuron D) hypothalamus E) medulla oblongata Answer: A Diff: 3 Page Ref: 476 20) Adrenergic hormones are secreted into the bloodstream in this region of the adrenal gland. A) hypothalamus B) medulla C) medulla oblongata D) rami communicantes E) sympathetic trunk Answer: B Diff: 1 Page Ref: 480 21) Three-neuron reflex arcs that exist entirely within the walls of the digestive tract. A) parasympathetic B) enteric nervous system C) inferior hypogastric plexus D) sympathetic E) cholinergic Answer: B Diff: 2 Page Ref: 482 22) Location in the brain of the cardiorespiratory and vasomotor control centers. A) hypothalamus B) medulla C) medulla oblongata D) rami communicantes E) sympathetic trunk Answer: C Diff: 2 Page Ref: 483

5 Copyright © 2014 Pearson Education, Inc.

 

23) Ganglia of this structure are connected to the ventral rami of spinal nerves through the rami communicantes. A) collateral ganglia B) sympathetic trunk C) pre-vertebral ganglia D) inferior hypogastric plexus E) celiac ganglion Answer: B Diff: 2 Page Ref: 476 24) Network of autonomic axons giving rise to nerves of the pelvic organs. A) aortic plexus B) pulmonary plexus C) inferior hypogastric plexus D) stellate ganglion E) dorsal root ganglion Answer: C Diff: 3 Page Ref: 480 25) Sacral nerves carrying parasympathetic innervation to the pelvic organs. A) pelvic splanchnic nerves B) sacral splanchnic nerves C) thoracic splanchnic nerves D) vagus nerve E) enteric nervous system Answer: A Diff: 3 Page Ref: 473 26) The defecation and urination reflexes are integrated in the A) brain stem. B) cerebral cortex. C) hypothalamus. D) spinal cord. Answer: D Diff: 2 Page Ref: 481 27) Which of these is not innervated by the ANS? A) cardiac muscle B) smooth muscle C) skeletal muscle D) glands Answer: C Diff: 2 Page Ref: 468

6 Copyright © 2014 Pearson Education, Inc.

 

28) Which division of the autonomic nervous system innervates the sweat glands in skin? A) sympathetic B) parasympathetic C) somatic D) integumentary Answer: A Diff: 1 Page Ref: 477 29) Which division of the autonomic nervous system has preganglionic fibers within the facial nerve? A) sympathetic B) parasympathetic C) somatic D) CNS Answer: B Diff: 1 Page Ref: 472-473 30) Another name for the ANS is the A) general visceral motor system. B) general somatic motor system. C) branchial motor system. D) general peripheral nervous system. Answer: A Diff: 1 Page Ref: 468 31) Focusing the eye on a nearby object in the field of vision is a function of which division of the ANS? A) sympathetic B) parasympathetic C) somatic D) cranial Answer: B Diff: 1 Page Ref: 472 32) Which division of the nervous system is characterized by highly branched postganglionic fibers that innervate multiple organs? A) parasympathetic B) somatic motor C) sympathetic D) visceral sensory Answer: C Diff: 1 Page Ref: 479

7 Copyright © 2014 Pearson Education, Inc.

 

33) The gray and white rami communicantes attach to the A) dorsal root ganglia. B) ventral rami. C) dorsal roots. D) spinal nerves. Answer: B Diff: 1 Page Ref: 476 34) All of the following have only sympathetic innervation except the A) blood vessels. B) sweat glands. C) arrector pili. D) salivary glands. Answer: D Diff: 1 Page Ref: 475 35) Which of the following cranial nerves does not supply parasympathetic fibers to the head? A) oculomotor B) facial C) vagus D) glossopharyngeal Answer: C Diff: 2 Page Ref: 473 36) A nurse encountered a patient in a wheelchair 3 months after the patient had sustained a crushing injury to his cervical spinal cord at the level of C6 in a motorcycle accident. Although the patient was paralyzed, his limbs were thrashing, his face was bright red, and he was sweating profusely. What was happening? A) Horner's syndrome B) vagotomy C) mass reflex D) Raynaud's disease Answer: C Diff: 1 Page Ref: 482 37) Which of these statements concerning the glossopharyngeal nerve is incorrect? A) The preganglionic fibers originate in the inferior salivatory nucleus. B) The pre- and postganglionic fibers travel in this nerve. C) The pre- and postganglionic fibers synapse in the otic ganglion. D) Stimulation induces secretion from the parotid gland. Answer: B Diff: 3 Page Ref: 473

8 Copyright © 2014 Pearson Education, Inc.

 

38) Which of these characteristics definitively distinguishes the autonomic nervous system from the somatic nervous system? A) cell bodies outside the CNS B) cell bodies within the CNS C) cholinergic fibers D) myelinated axons Answer: A Diff: 1 Page Ref: 468-470 39) Which of these is not a characteristic of the parasympathetic division? A) branched fibers B) cholinergic fibers C) intramural ganglia D) myelinated fibers Answer: A Diff: 2 Page Ref: 471 40) Only about half a dozen parasympathetic ganglia have specific names. Two of these are the A) celiac and superior mesenteric. B) inferior and superior hypogastric. C) ciliary and pterygopalatine. D) chain and prevertebral. Answer: C Diff: 2 Page Ref: 472-473 41) What stimulates the adrenal medulla to secrete its excitatory neurohormones? A) postganglionic sympathetic neurons B) preganglionic sympathetic neurons C) preganglionic parasympathetic neurons D) postganglionic parasympathetic neurons Answer: B Diff: 2 Page Ref: 480 42) All of the following plexi contain fibers from the vagus nerve except A) the otic. B) the esophageal. C) the celiac. D) the superior mesenteric. Answer: A Diff: 3 Page Ref: 473

9 Copyright © 2014 Pearson Education, Inc.

 

43) The parasympathetic nervous system is characterized by A) peripheral ganglia near the organs, and short postganglionic fibers. B) peripheral ganglia near the organs, and long postganglionic fibers. C) peripheral ganglia near the spinal cord, and short postganglionic fibers. D) peripheral ganglia near the spinal cord, and long postganglionic fibers. Answer: A Diff: 2 Page Ref: 471 44) Which of these statements concerning the sympathetic division is incorrect? A) The preganglionic cell bodies form the lateral gray horn in some areas of the spinal cord. B) There is a sympathetic trunk ganglion for each of the 31 spinal nerves. C) There is approximately one chain ganglion for each spinal nerve. D) It is more complex than the parasympathetic system because it supplies more structures than the parasympathetic system. Answer: B Diff: 2 Page Ref: 476 45) Which of these statements is not a characteristic of collateral ganglia of the sympathetic division? A) They lie in the abdomen and pelvis. B) They are paired and segmented. C) They lie anterior to the vertebral column. D) They are located mostly on the abdominal aorta. Answer: B Diff: 2 Page Ref: 476 46) The sympathetic system causes A) decreased blood glucose and peristalsis, and increased heart rate and blood pressure. B) increased blood glucose and peristalsis, and decreased heart rate and blood pressure. C) increased blood glucose, heart rate, and blood pressure, and decreased peristalsis. D) decreased blood glucose, heart rate, and blood pressure, and increased peristalsis. Answer: C Diff: 2 Page Ref: 470 47) Which of these statements concerning gray rami communicantes is incorrect? A) They carry postganglionic fibers to peripheral structures. B) Their fibers are unmyelinated. C) They contain all the preganglionic fibers traveling to the sympathetic chain. D) They are associated with sympathetic trunk ganglia. Answer: C Diff: 2 Page Ref: 476

10 Copyright © 2014 Pearson Education, Inc.

 

48) Which of these descriptions is not a result of parasympathetic stimulation? A) production of goose bumps B) constriction of the pupils C) increased peristalsis of the digestive viscera D) contraction of the bladder wall in urination Answer: A Diff: 2 Page Ref: 470 49) The site of origin of the preganglionic fibers of the parasympathetic nervous system is the A) thoracolumbar region of the spinal cord. B) higher brain centers. C) chain and prevertebral ganglia. D) brain stem and the sacral region of the cord. Answer: D Diff: 2 Page Ref: 471 50) Cranial parasympathetic outflow is contained in all of the following cranial nerves except the A) accessory. B) facial. C) glossopharyngeal. D) vagus. Answer: A Diff: 2 Page Ref: 472-473 51) The micturition (urination) reflex is controlled by a parasympathetic reflex pathway in which A) the preganglionic neurons are in the dorsal motor nucleus of the vagus in the brain. B) the preganglionic cell bodies are in the wall of the bladder. C) the preganglionic axons occur in pelvic splanchnic nerves. D) the postganglionic axons occur in the celiac plexus. Answer: C Diff: 3 Page Ref: 481-482 52) The trunk ganglia contain what kind of cell bodies? A) preganglionic parasympathetic B) postganglionic parasympathetic C) preganglionic sympathetic D) postganglionic sympathetic Answer: D Diff: 2 Page Ref: 468-469 53) Cell bodies of the preganglionic sympathetic neurons are located within the A) dorsal root ganglia. B) intramural ganglia. C) lateral gray horns of the spinal cord. D) sympathetic trunk. Answer: C Diff: 2 Page Ref: 468-469 11 Copyright © 2014 Pearson Education, Inc.

 

54) Which of the following autonomic plexi does not lie on the aorta or on the main branches of the aorta? A) celiac B) superior mesenteric C) inferior mesenteric D) inferior hypogastric Answer: D Diff: 2 Page Ref: 474 55) Which autonomic division increases heart rate? A) sympathetic B) parasympathetic C) somatic D) cranial Answer: A Diff: 1 Page Ref: 470 56) In which autonomic division do nerve cell bodies lie closest to the organs being innervated? A) sympathetic B) parasympathetic C) visceral sensory D) somatic motor Answer: B Diff: 1 Page Ref: 471 57) Postganglionic neurons in the pterygopalatine ganglion A) are very active when you cry. B) are adrenergic. C) run from the head to the abdomen. D) are primarily sympathetic fibers. Answer: A Diff: 3 Page Ref: 472-473 58) Parasympathetic postganglionic fibers of the head travel within the A) accessory nerve. B) facial nerve. C) trigeminal nerve. D) vestibulocochlear nerve. Answer: C Diff: 2 Page Ref: 473

12 Copyright © 2014 Pearson Education, Inc.

 

59) Parasympathetic ganglia that are located within the walls of the innervated organs are called A) collateral ganglia. B) dorsal root ganglia. C) intramural ganglia. D) paravertebral ganglia. Answer: C Diff: 2 Page Ref: 473 60) The part of the brain that exerts more control over autonomic functioning than any other part is the A) hypothalamus. B) thalamus. C) cerebral cortex. D) cerebellum. Answer: A Diff: 1 Page Ref: 483 61) Parasympathetic impulses to the stomach pass through the A) celiac plexus. B) esophageal plexus. C) inferior hypogastric plexus. D) superior mesenteric plexus. Answer: B Diff: 2 Page Ref: 473 62) The adrenal medulla is stimulated by preganglionic neurons localized to gray matter of the spinal cord A) in the cervical region. B) in the upper thoracic region. C) in the lower thoracic region. D) in the lower lumbar region. Answer: C Diff: 2 Page Ref: 480 63) Cell bodies of preganglionic sympathetic neurons are located within the A) dorsal root ganglia. B) intramural ganglia. C) lateral horns of the gray matter of the spinal cord. D) sympathetic trunk. Answer: C Diff: 2 Page Ref: 477

13 Copyright © 2014 Pearson Education, Inc.

 

64) The cell bodies of postganglionic neurons that stimulate secretion of the parotid gland are in the A) superior cervical ganglion. B) submandibular ganglion. C) otic ganglion. D) sphenopalatine ganglion. Answer: C Diff: 2 Page Ref: 473 65) Which one of these is solely innervated by the parasympathetic division? A) adipose tissue B) arrector pili muscle of the hair follicle C) ciliary muscle of the eye D) sweat glands Answer: C Diff: 2 Page Ref: 472 66) Identify the pathway that correctly traces the movement of preganglionic sympathetic fibers. A) dorsal root to gray ramus communicans to sympathetic trunk ganglion B) dorsal root to sympathetic trunk ganglion to gray ramus communicans C) ventral root to sympathetic trunk ganglion to white ramus communicans D) ventral root to white ramus communicans to sympathetic trunk ganglion Answer: D Diff: 3 Page Ref: 477 67) Another name for a chain ganglion is A) sympathetic trunk. B) paravertebral ganglion. C) prevertebral ganglion. D) intramural ganglion. Answer: B Diff: 2 Page Ref: 476 68) The ANS stimulates cardiac muscle as well as smooth muscle and glands. Which of the following nerves carries autonomic fibers that increase the rate of cardiac muscle contraction? A) phrenic nerves B) sympathetic nerves from the lower half of the thoracolumbar spinal cord C) vagus nerves D) sympathetic nerves from the middle and inferior cervical ganglion Answer: D Diff: 2 Page Ref: 476, 478-479

14 Copyright © 2014 Pearson Education, Inc.

 

69) The secretions of the adrenal medulla act to supplement the effects of A) parasympathetic innervation. B) sympathetic stimulation. C) vagus nerve activity. D) reflex control. Answer: B Diff: 1 Page Ref: 480 70) Sympathetic fibers leave the spinal cord in the A) craniosacral regions, and the postganglionic fibers secrete norepinephrine. B) thoracolumbar region, and the postganglionic fibers secrete acetylcholine. C) craniosacral regions, and the postganglionic fibers secrete acetylcholine. D) thoracolumbar region, and the postganglionic fibers secrete norepinephrine. Answer: D Diff: 3 Page Ref: 471 71) Where would you not find an autonomic ganglion? A) in the head B) in the cervical region C) in the armpit D) alongside the vertebral column Answer: C Diff: 2 Page Ref: 474 72) Cardiovascular effects of the sympathetic division include all of the following except A) constriction of most blood vessels. B) dilation of the vessels serving the skeletal muscles. C) increase in heart rate and force. D) dilation of the blood vessels serving the skin and digestive viscera. Answer: D Diff: 2 Page Ref: 478-480 73) Over 90% of all preganglionic parasympathetic fibers are in cranial nerve A) III. B) V. C) VII. D) X. Answer: D Diff: 2 Page Ref: 473

15 Copyright © 2014 Pearson Education, Inc.

 

74) Control of temperature, of autonomic nervous reflexes, of hunger, and of sleep are functions associated with the A) medulla. B) cerebellum. C) hypothalamus. D) thalamus. Answer: C Diff: 2 Page Ref: 483 75) This organ contains modified sympathetic postganglionic neurons that lack nerve processes. A) adrenal medulla B) collateral ganglia C) stellate ganglion D) sympathetic chain ganglia Answer: A Diff: 1 Page Ref: 480 76) Visceral pain results from all the following except A) cutting of an organ. B) chemical irritation of an organ. C) excessive stretch of an organ. D) inflammation of an organ. Answer: A Diff: 2 Page Ref: 481 77) What is the effect of parasympathetic stimulation of respiratory bronchioles in the lungs? A) constriction B) dilation C) decreased secretion of mucus D) no effect Answer: A Diff: 1 Page Ref: 475 78) Which of the following disorders of the sympathetic division does not involve a problem with blood vessels? A) mass reflex reaction B) congenital megacolon C) Raynaud's disease D) hypertension Answer: B Diff: 2 Page Ref: 484

16 Copyright © 2014 Pearson Education, Inc.

 

15.2 True/False Questions 1) Preganglionic fibers are myelinated, whereas postganglionic fibers are unmyelinated. Answer: TRUE Diff: 2 Page Ref: 470 2) Parasympathetic stimulation of blood vessels causes vasoconstriction and increased blood pressure. Answer: FALSE Diff: 2 Page Ref: 475 3) Fibers from the cranial and sacral regions of the spinal cord contribute to the sympathetic division. Answer: FALSE Diff: 1 Page Ref: 471 4) Postganglionic sympathetic fibers are relatively long compared to those of the parasympathetic division. Answer: TRUE Diff: 2 Page Ref: 471 5) The sympathetic and parasympathetic divisions can be distinguished by the amount of branching of the post-ganglionic fibers. Answer: TRUE Diff: 2 Page Ref: 471 6) Postganglionic sympathetic fibers release norepinephrine and are called cholinergic fibers. Answer: FALSE Diff: 2 Page Ref: 471 7) Adrenergic postganglionic neurons release the hormones norepinephrine and epinephrine. Answer: TRUE Diff: 1 Page Ref: 471 8) All parasympathetic postganglionic fibers of the head travel through the trigeminal nerve (V) to their final destinations. Answer: TRUE Diff: 3 Page Ref: 473 9) The cardiac center of the medulla oblongata regulates the diameter of blood vessels. Answer: FALSE Diff: 2 Page Ref: 475 10) Vagal stimulation of the heart decreases heart rate, resulting in a drop in blood pressure. Answer: TRUE Diff: 2 Page Ref: 475 17 Copyright © 2014 Pearson Education, Inc.

 

11) Sympathetic trunk ganglia contain the cell bodies of sensory neurons and are located in the dorsal root of the spinal cord. Answer: FALSE Diff: 3 Page Ref: 476 12) The main integration center of the ANS is the amygdala. Answer: FALSE Diff: 2 Page Ref: 483 13) Because the fibers of the sympathetic division arise from the thoracic and lumbar regions of the spinal cord, they cannot innervate structures of the head or pelvis. Answer: FALSE Diff: 1 Page Ref: 476 14) Parasympathetic stimulation of the pelvic organs cause defecation, voiding of urine, and erection. Answer: TRUE Diff: 2 Page Ref: 473 15) Parasympathetic fibers innervate the sweat glands, arrector pili, and smooth muscles of the arteries. Answer: FALSE Diff: 2 Page Ref: 475 15.3 Short Answer Questions 1) Vasoconstriction and bronchodilation result from stimulation by the ________ division. Answer: sympathetic Diff: 2 Page Ref: 475 2) Autonomic fibers emerging from the craniosacral region of the spinal cord belong to the ________ division. Answer: parasympathetic Diff: 2 Page Ref: 471 3) Postganglionic fibers of the parasympathetic system are called ________ because they release the neurotransmitter acetylcholine. Answer: cholinergic Diff: 2 Page Ref: 471 4) The ________ nervous system consists of complete three-neuron reflex arcs that exist entirely within the wall of the digestive tube. Answer: enteric Diff: 2 Page Ref: 482

18 Copyright © 2014 Pearson Education, Inc.

 

5) During development, all neurons with cell bodies in the PNS derive from the ________. Answer: neural crest Diff: 2 Page Ref: 484 6) The cell bodies of postganglionic parasympathetic fibers to the pupil and lens of the eye are located in the ________ ganglion. Answer: ciliary Diff: 2 Page Ref: 472 7) The ________ plexus is a network through which branches of the vagus pass to the intestines and liver. Answer: celiac Diff: 3 Page Ref: 473 8) Autonomic fibers to the pelvic organs pass through the pelvic plexus, also known as the ________ plexus. Answer: inferior hypogastric Diff: 3 Page Ref: 473 9) Visceral reflexes that do not involve the CNS but instead synapse in sympathetic ganglia are called ________ reflex arcs. Answer: peripheral Diff: 1 Page Ref: 482 10) Sympathetic preganglionic fibers leaving the thoracic spinal nerves may ascend within the ________ before synapsing so that they can exit near the head. Answer: sympathetic trunk Diff: 3 Page Ref: 478 11) Gray rami communicantes contain ________ sympathetic fibers that travel to peripheral structures. Answer: postganglionic Diff: 2 Page Ref: 476 12) The sensation of pain from a visceral organ that is perceived in the skin or outer body is the phenomenon called ________. Answer: referred pain Diff: 2 Page Ref: 481 13) The primary integrating center of the ANS is the ________ of the diencephalon. Answer: hypothalamus Diff: 2 Page Ref: 483

19 Copyright © 2014 Pearson Education, Inc.

 

14) The adrenal medulla is comprised of modified sympathetic ________ neurons which secretes epinephrine and norepinephrine. Answer: adrenergic or post-ganglionic Diff: 1 Page Ref: 480 15) Congenital megacolon, or Hirschsprung's disease, is a birth defect in which the ________ innervation of the distal region of the large intestine fails to develop normally. Answer: parasympathetic Diff: 2 Page Ref: 484 15.4 Essay Questions 1) Contrast the structure of the somatic and autonomic motor neurons. Answer: The somatic and autonomic nervous systems differ in the number of efferent motor fibers and in their diameter and presence of myelin. In both systems, the cell bodies of the somatic motor neuron and the autonomic preganglionic neuron are located within the CNS. The axons of these neurons are myelinated. Those of the somatic division are thicker in diameter. The preganglionic motor neurons of the ANS synapse with postganglionic neurons in a ganglion external to the CNS. The postganglionic fibers are thin and unmyelinated. Diff: 2 Page Ref: 468-470 2) Identify three anatomical differences that distinguish the sympathetic division from parasympathetic division. Answer: The two subdivisions of the ANS can be distinguished by (1) the location where the fibers leave the CNS, (2) the length of the postganglionic fibers and location of the ganglia, and (3) the degree of branching of the fibers. First, the sympathetic division arises from the thoracolumbar region of the spinal cord, whereas the parasympathetic arises from the cranial and sacral regions. Second, the postganglionic sympathetic fibers are longer than those of the parasympathetic division. The sympathetic fibers run from the sympathetic ganglia near the vertebral column to distant target tissues. The parasympathetic ganglia are often within the walls of the target tissue, giving rise to very short postganglionic fibers. Third, the fibers of the sympathetic division branch, allowing for innervation of multiple target tissues and simultaneous activation by the same preganglionic fiber. Diff: 2 Page Ref: 471 3) The sympathetic and parasympathetic divisions differ markedly in the branching of the postganglionic fibers. Explain the anatomical difference and its significance. Answer: The postganglionic fibers of the parasympathetic division are short and often enclosed within the tissues they innervate. In contrast, the postganglionic sympathetic fibers are highly branched and innervate multiple tissues; nerve impulses from one fiber will simultaneously activate multiple target tissues. The simultaneous activation of wide-ranging tissues and organs is necessary for a fight-or-flight response. Diff: 1 Page Ref: 470-471

20 Copyright © 2014 Pearson Education, Inc.

 

4) Explain the relationship between the vagus and pelvic splanchnic nerves, the cardiac and pulmonary plexus, celiac plexus, and hypogastric plexus. Answer: Some parasympathetic preganglionic fibers exit the CNS through the vagus nerve (CN X). Additional fibers exit the spinal cord as the pelvic splanchnic nerves at the level of the sacrum. These preganglionic fibers branch and interconnect to form plexi. The cardiac, pulmonary, and celiac plexi are derived from these pre-ganglionic vagus nerve fibers. The hypogastric plexus arises from thoracic, lumbar, pelvic and sacral splanchnic nerves. These plexi contain both sympathetic and parasympathetic fibers. Diff: 3 Page Ref: 473, 478-480 5) Describe the correct pathway that a nerve impulse to blood vessels and hair follicles (glands and arrector pili muscles) in the dermis must travel using the following structures: ventral ramus, dorsal root, ventral root, sympathetic trunk, gray ramus communicans, white ramus communicans, and spinal nerve. Answer: Efferent impulses leave the spinal cord through the ventral root to merge with a spinal nerve. The spinal nerve branches to form a dorsal and ventral ramus. The ventral ramus of the spinal nerve connects to the sympathetic trunk via a white ramus communicans. Sympathetic impulses move through preganglionic fibers within the sympathetic trunk. These fibers may either project inferiorly or superiorly through the chain ganglia. Within the ganglia, these preganglionic fibers synapse with postganglionic fibers that exits via gray rami communicantes. These postganglionic fibers will then proceed through the dorsal rami of spinal nerves to the skin on the posterior torso or through the ventral rami to the skin covering the rest of the body. Diff: 3 Page Ref: 478

21 Copyright © 2014 Pearson Education, Inc.

 

Human Anatomy, 7e (Marieb/Mitchell/Smith) Chapter 16 The Special Senses 16.1 Multiple Choice Questions

Figure 16.1 Use the diagram above to answer the following questions. 1) Identify the letter that indicates the structure that is important in equalizing air pressure on both sides of the eardrum. A) A B) B C) C D) D E) E Answer: E Diff: 2 Page Ref: 507-508 2) Identify the letter that indicates the structure that is the boundary between the external and middle ear. A) A B) B C) C D) D E) E Answer: A Diff: 2 Page Ref: 506-507

1 Copyright © 2014 Pearson Education, Inc.

 

3) Identify the letter that indicates the middle ear ossicle that is known as the stirrup. A) A B) B C) C D) D E) E Answer: B Diff: 2 Page Ref: 507-508 4) Identify the letter that indicates the structure that contains receptors for rotational acceleration. A) A B) B C) C D) D E) E Answer: C Diff: 2 Page Ref: 507, 513 5) Identify the letter that indicates the structure that is called the cochlea. A) A B) B C) C D) D E) E Answer: D Diff: 2 Page Ref: 507, 510

2 Copyright © 2014 Pearson Education, Inc.

 

Figure 16.2 Use the diagram above to answer the following questions. 6) Identify the letter that indicates the transparent portion of the fibrous layer. A) A B) B C) C D) D E) E Answer: D Diff: 2 Page Ref: 495-496 7) Identify the letter that indicates the portion of the fibrous layer known as the sclera. A) A B) B C) C D) D E) E Answer: A Diff: 2 Page Ref: 495-496

3 Copyright © 2014 Pearson Education, Inc.

 

8) Identify the letter that indicates the anterior segment, which is filled with aqueous humor. A) A B) B C) C D) D E) E Answer: E Diff: 2 Page Ref: 496 9) Identify the letter that indicates the ciliary body. A) A B) B C) C D) D E) E Answer: C Diff: 2 Page Ref: 496 10) Identify the letter that indicates the region of the retina that contains only cones and provides maximal visual acuity. A) A B) B C) C D) D E) E Answer: B Diff: 2 Page Ref: 496, 499 11) Microvilli from gustatory cells project through this structure. A) taste bud B) taste pore C) tongue papilla D) olfactory bulb E) gustatory epithelial cells Answer: B Diff: 2 Page Ref: 490 12) Sensory receptor for taste. A) taste bud B) taste pore C) tongue papilla D) olfactory bulb E) gustatory epithelial cells Answer: E Diff: 2 Page Ref: 490

4 Copyright © 2014 Pearson Education, Inc.

 

13) Filaments of the olfactory receptor cells synapse with these cells of the olfactory tract. A) mitral cells B) glomeruli C) support epithelial cells D) olfactory stem cells E) cells in the lamina propria Answer: A Diff: 2 Page Ref: 491 14) Region of the forebrain overlying the cribriform plate of the ethmoid. A) olfactory epithelium B) temporal lobe C) olfactory bulb D) thalamus E) insula Answer: C Diff: 2 Page Ref: 490-491 15) Transparent mucous membrane covering the inner surface of the eyelid. A) palpebrae B) tarsal glands C) lacrimal apparatus D) conjunctiva E) lacrimal caruncle Answer: D Diff: 2 Page Ref: 493 16) Transparent structure of the eye containing regularly aligned collagen fibers. A) sclera B) choroid C) ciliary body D) cornea E) lens Answer: D Diff: 2 Page Ref: 495 17) Melanin-containing layer of the eye's vascular tunic. A) sclera B) choroid C) ciliary body D) cornea E) lens Answer: B Diff: 2 Page Ref: 496

5 Copyright © 2014 Pearson Education, Inc.

 

18) Fluid filling the posterior segment of the eye. A) aqueous humor B) vitreous humor C) synovial fluid D) serous fluid E) endolymph Answer: B Diff: 2 Page Ref: 496, 500 19) Membrane attached to the stapes. A) tympanic membrane B) pharyngotympanic C) oval window D) round window E) tectorial membrane Answer: C Diff: 2 Page Ref: 508 20) Bony labyrinth structure containing the utricle and saccule. A) vestibule B) macula lutea C) macula densa D) ora serrate E) scala vestibule Answer: A Diff: 2 Page Ref: 512 21) Calcium carbonate crystals of the macula. A) stapedius B) modiolus C) otoliths D) helicotrema E) scala tympani Answer: C Diff: 2 Page Ref: 513 22) Endolymph-filled structure containing receptors for hearing. A) vestibule B) saccule C) utricle D) cochlear duct E) semicircular canals Answer: D Diff: 2 Page Ref: 510

6 Copyright © 2014 Pearson Education, Inc.

 

23) The basilar membrane supports the A) spiral lamina. B) modiolus. C) spiral ganglion. D) bony labyrinth. E) spiral organ. Answer: E Diff: 2 Page Ref: 510 24) Gel-like structure embedded with the tips of cochlear hair cells. A) basilar membrane B) tectorial membrane C) tympanic membrane D) scala tympani E) scala vestibule Answer: B Diff: 2 Page Ref: 511-512 25) Along with the saccule, this structure senses linear acceleration. A) vestibule B) spiral organ C) utricle D) cochlear duct E) semicircular canals Answer: C Diff: 2 Page Ref: 512-513 26) Which of the following structures in the eye are pigmented? A) the cornea B) sclera C) the retina D) lens Answer: C Diff: 1 Page Ref: 497 27) Which of the following areas has the highest concentration of cones? A) the optic disc B) the macula lutea C) the fovea centralis D) the ora serrata retinae Answer: C Diff: 2 Page Ref: 499

7 Copyright © 2014 Pearson Education, Inc.

 

28) Which of the following statements does not correctly describe the spiral organ of Corti? A) High-frequency sounds stimulate hair cells at the basal end of the basilar membrane. B) The "hairs" of the receptor cells are embedded in the tectorial membrane. C) The spiral organ is part of the cochlear duct, which equals the scala media. D) The tectorial membrane bends with vibrations, whereas the basilar membrane is rigid and fixed. Answer: D Diff: 2 Page Ref: 510-512 29) Endolymph is made A) from perilymph. B) from the dura mater. C) in the scala vestibuli. D) in the stria vascularis. Answer: D Diff: 1 Page Ref: 510 30) "Uncinate fits" are characterized by A) auditory hallucinations. B) photo sensitivity. C) imaginary odors. D) vertigo. Answer: C Diff: 1 Page Ref: 491 31) Of the following glands, which does not help keep the anterior part of the eye from drying out? A) lacrimal caruncle B) tarsal glands C) goblet cells in the conjunctiva D) lacrimal gland Answer: A Diff: 2 Page Ref: 492-493 32) The ora serrata is a A) type of papilla that houses taste buds. B) part of the modiolus. C) part of the choroid layer. D) part of the retina. Answer: D Diff: 1 Page Ref: 499

8 Copyright © 2014 Pearson Education, Inc.

 

33) Where do tears drain? A) into the Eustachian tube B) into the nasal cavity C) into the lacrimal gland D) into the pharynx Answer: B Diff: 1 Page Ref: 493 34) What axons decussate in the optic chiasma? A) the fibers in the optic radiation of white matter B) all fibers from both eyes C) those from the lateral half of each retina D) axons from the medial half of each eye Answer: D Diff: 2 Page Ref: 503 35) In Chapter 12 on the CNS, we learned that the inferior olivary nucleus and medial lemniscus are a relay nucleus and a fiber tract in general somatic sensory pathways in the brain. The superior olivary nucleus and the lateral lemniscus are entirely different structures belonging to what sensory pathway? A) taste B) olfactory C) visual D) auditory Answer: D Diff: 3 Page Ref: 515 36) Which is true about a retinal detachment? A) It can result from a blow to the eye. B) It is a detachment of the complete thickness of the retina from the choroid. C) It causes blindness immediately. D) The detached portion contains no capillaries. Answer: A Diff: 1 Page Ref: 500 37) Clouding of which of the following structures would lead to a clinical condition known as a cataract? A) lens B) cornea C) aqueous humor D) vitreous humor Answer: A Diff: 1 Page Ref: 501

9 Copyright © 2014 Pearson Education, Inc.

 

38) Of the following structures, which is the only one that contains perilymph (instead of endolymph)? A) scala media B) saccule C) scala tympani D) semicircular ducts Answer: C Diff: 1 Page Ref: 509-510 39) Hair cells are receptor cells for A) fine touch. B) both hearing and equilibrium. C) taste. D) smell. Answer: B Diff: 1 Page Ref: 512-513 40) Hyperopia and presbyopia may have some features in common, but a key difference between these two conditions is that A) one is farsightedness, and the other is nearsightedness. B) nearsighted people never develop presbyopia, but they can develop hyperopia. C) in hyperopia the lens can accommodate, but in presbyopia it cannot. D) people with astigmatism never develop presbyopia, but they can develop hyperopia. Answer: C Diff: 3 Page Ref: 502 41) Another name for the ciliary zonule is A) olfactory glomerulus. B) cochlear duct. C) the bitter taste zone on the tongue. D) the suspensory ligament of the lens. Answer: D Diff: 1 Page Ref: 496, 501 42) The middle ear cavity is normally filled with A) perilymph. B) endolymph. C) air. D) mucus. Answer: C Diff: 1 Page Ref: 508

10 Copyright © 2014 Pearson Education, Inc.

 

43) The cristae in the inner ear contain the receptors for A) rotational equilibrium. B) all aspects of hearing. C) all aspects of equilibrium. D) static equilibrium. Answer: A Diff: 2 Page Ref: 513-514 44) Which of the following would not be associated with strabismus? A) damage to the oculomotor nerve B) paralysis of the extrinsic muscles of the eye C) damage to the optic nerve D) lesions of the midbrain Answer: C Diff: 3 Page Ref: 495 45) The sensation of taste involves A) bending of cilia. B) chemicals binding to microvilli. C) movement of crystals embedded in gelatinous masses. D) photons altering pigment molecules. Answer: B Diff: 1 Page Ref: 490 46) Receptors for hearing are located in the A) cochlear duct. B) middle ear. C) semicircular canals. D) tympanic membrane. Answer: A Diff: 1 Page Ref: 510 47) The external ear consists of each of the following structures except the A) auricle. B) helix. C) lobule. D) vestibule. Answer: D Diff: 1 Page Ref: 506 48) Ringing in the ears, vertigo, and gradual loss of hearing typify the disorder called A) conjunctivitis. B) Ménière's syndrome. C) strabismus. D) glaucoma. Answer: B Diff: 2 Page Ref: 517 11 Copyright © 2014 Pearson Education, Inc.

 

49) The eyelids house all of the following except the A) tarsal glands. B) superior tarsal muscle. C) orbicularis oculi muscles. D) lacrimal sac. Answer: D Diff: 1 Page Ref: 492-493 50) Farsightedness is more properly called A) myopia. B) presbyopia. C) hyperopia. D) emmetropia. Answer: C Diff: 1 Page Ref: 502 51) The oval window of the ear is connected directly to which passageway? A) pharyngotympanic tube B) external auditory canal C) scala vestibuli D) scala tympani Answer: C Diff: 2 Page Ref: 510 52) The first "way station" (relay nucleus) in the visual pathway from the eye, after there has been partial crossover of the fibers in the optic chiasma, is the A) superior colliculi. B) lateral geniculate nucleus. C) primary visual cortex. D) temporal lobe. Answer: B Diff: 2 Page Ref: 503 53) An essential part of the maculae involved in static equilibrium is/are the A) spiral organ (of Corti). B) cupula. C) scala media. D) otoliths. Answer: D Diff: 2 Page Ref: 513

12 Copyright © 2014 Pearson Education, Inc.

 

54) Nearsightedness is more properly called A) myopia. B) hyperopia. C) presbyopia. D) emmetropia. Answer: A Diff: 1 Page Ref: 502 55) The oil component found in tears is produced by the A) lacrimal glands. B) tarsal glands. C) conjunctiva. D) endocrine glands. Answer: B Diff: 1 Page Ref: 493 56) Which of the following lies closest to the anterior pole of the eye? A) center of the lens B) center of the cornea C) tip of an eyelash D) sclera Answer: B Diff: 2 Page Ref: 495-496 57) The muscle that opens the eye is the A) superior rectus. B) orbicularis oculi. C) frontalis on forehead. D) levator palpebrae superioris. Answer: D Diff: 1 Page Ref: 494 58) What structure is handled by an "eye bank"? A) whole eyes for eye transplants B) just the lens C) just the cornea D) just the retina Answer: C Diff: 1 Page Ref: 495 59) Which cranial nerve does not innervate the extrinsic eye muscles? A) abducens B) facial C) oculomotor D) trochlear Answer: B Diff: 2 Page Ref: 494 13 Copyright © 2014 Pearson Education, Inc.

 

60) Scientists who are trying to find a way to make neurons divide to heal nerve injuries often study the body's few mitotic neurons. These neurons are the A) photoreceptors. B) olfactory stem cells. C) basal cells in the taste buds. D) auditory hair cells. Answer: B Diff: 1 Page Ref: 490 61) The bony labyrinth is located in which portion of the temporal bone? A) mastoid B) petrous C) squamous D) tympanic Answer: B Diff: 2 Page Ref: 509 62) The sclera of the eye develops from A) the lens placode. B) head mesenchyme. C) the optic cup. D) the optic stalk. Answer: B Diff: 2 Page Ref: 504-506 63) Nerve axons from the lateral portion of each retina A) cross over to the opposite side of the retina. B) pass posteriorly without crossing over at the chiasma. C) branch at the chiasma, some branches crossing and some not crossing. D) carry information from the lateral half of the visual field. Answer: B Diff: 2 Page Ref: 503-504 64) The transmission of sound vibrations through the external acoustic meatus occurs chiefly through A) nerve fibers. B) air. C) fluid. D) bone. Answer: B Diff: 1 Page Ref: 506

14 Copyright © 2014 Pearson Education, Inc.

 

65) Which of the following controls the amount of light entering the eye? A) ciliary muscle B) dilator/sphincter pupillae muscles C) levator palpebrae D) medial rectus Answer: B Diff: 2 Page Ref: 497 66) Which of the following membranes is not part of the cochlea? A) basilar B) tectorial C) tympanic D) vestibular Answer: C Diff: 2 Page Ref: 510-511 67) The nerve carrying taste information from the anterior two-thirds of the tongue is the A) hypoglossal. B) glossopharyngeal. C) olfactory. D) facial. Answer: D Diff: 2 Page Ref: 490 68) The superior oblique muscle turns the eye both laterally and A) medially. B) superiorly. C) inferiorly. D) laterally. Answer: C Diff: 2 Page Ref: 494 69) Each of the following structures participates in bending of light entering the eye except the A) cornea. B) iris. C) lens. D) vitreous humor. Answer: B Diff: 2 Page Ref: 501-502

15 Copyright © 2014 Pearson Education, Inc.

 

70) Abraham spoke so softly that Jason rolled a piece of paper into a funnel shape and put one end into his external auditory canal to signal Abraham to speak up. Abraham then shouted that a certain anatomical structure serves exactly the same function as Jason's paper funnel. That structure is the A) round window. B) pinna. C) pharyngotympanic tube. D) mastoid antrum. Answer: B Diff: 2 Page Ref: 506 71) The ossicle that is shaped like the stirrup of a saddle is the A) tympanic membrane. B) incus. C) malleus. D) stapes. Answer: D Diff: 1 Page Ref: 507-508 72) The cristae ampullares in the inner ear are located in the A) saccule. B) utricle. C) semicircular ducts. D) cochlear duct. Answer: C Diff: 2 Page Ref: 513-514 73) What structure regulates the amount of light passing to the visual receptors of the eye? A) aqueous humor B) vitreous humor C) lens D) pupil Answer: D Diff: 1 Page Ref: 497 74) Which of the following could not be seen as one looks into the eye with an ophthalmoscope? A) macula lutea B) optic chiasma C) fovea centralis D) optic disc Answer: B Diff: 2 Page Ref: 496, 499 -500

16 Copyright © 2014 Pearson Education, Inc.

 

75) The center for vision in the cerebral cortex is located in the A) temporal lobe. B) frontal lobe. C) occipital lobe. D) parietal lobe. Answer: C Diff: 1 Page Ref: 503 76) During embryonic development, the retina of the eye develops from A) the surface ectoderm. B) an outpocketing of the diencephalon. C) the mesoderm. D) the mesoderm and the endoderm. Answer: B Diff: 2 Page Ref: 504 77) Filaments that pass through the olfactory foramina of the cribriform plate belong to A) axons of the olfactory receptor cells. B) dendrites of the olfactory receptor cells. C) axons of the olfactory bulb mitral cells. D) dendrites of the olfactory bulb mitral cells. Answer: A Diff: 3 Page Ref: 491 78) Which of the following is not part of the flow of taste sensation along the gustatory pathway to the cerebral cortex? A) hypothalamic appetite centers B) solitary nucleus of the medulla oblongata C) thalamic nuclei D) vagus nerve Answer: A Diff: 3 Page Ref: 491 79) Ordinarily, it is not possible to transplant tissues from one person to another without rejection, yet corneas can be transplanted with impunity. This is because the cornea A) is not a human structure. B) has no nerve supply. C) has no blood supply, except around the periphery. D) is exposed and easily accessible. Answer: C Diff: 2 Page Ref: 495

17 Copyright © 2014 Pearson Education, Inc.

 

80) There are three layers of neurons in the retina. Which of the following neurons have axons that form the optic nerves? A) bipolar cells B) ganglion cells C) cone cells D) rod cells Answer: B Diff: 2 Page Ref: 497-498 81) Gustatory cells are located in all of the following areas except A) on the apical surface of vallate papillae on the tongue. B) on the fungiform papillae of the anterior tongue. C) on the inner surface of the cheeks. D) on the posterior wall of the pharynx. Answer: A Diff: 2 Page Ref: 489 82) The difference between the membranous ampulla and cupula of the ear is that A) the ampulla contains the crista ampullaris, which consists of hair cells and a cupula. B) the ampulla contains the utricle and saccule, whereas the cupula contains the crista ampullaris. C) the ampulla is the enlargement at the base of the semicircular canals, whereas the cupula is encrusted with otoliths and lies within the vestibule. D) the ampulla is the opening to the cochlea, whereas the cupula is found within the vestibule. Answer: A Diff: 3 Page Ref: 513-514 16.2 True/False Questions 1) Most of the special sensory receptors are free nerve endings. Answer: FALSE Diff: 1 Page Ref: 489 2) Taste sensation is carried by the facial, glossopharyngeal, and vagus nerves. Answer: TRUE Diff: 2 Page Ref: 490 3) Gustatory epithelial cells can be replaced if they are burned or scraped off during eating. Answer: TRUE Diff: 1 Page Ref: 490 4) Bipolar olfactory sensory neurons undergo continual renewal and replacement. Answer: TRUE Diff: 2 Page Ref: 490

18 Copyright © 2014 Pearson Education, Inc.

 

5) Lacrimal fluid is produced by the lacrimal sac in the medial canthus of the eye. Answer: FALSE Diff: 2 Page Ref: 493 6) The cone-filled region of the retina surrounding the fovea centralis is called the macula lutea. Answer: TRUE Diff: 2 Page Ref: 499 7) The transparency of the cornea is due to the presence of air-filled corneal epithelial cells. Answer: FALSE Diff: 2 Page Ref: 495 8) Light entering the eye passes first to the rod and cone cells, then to bipolar cells, and finally to ganglion cells adjacent to the pigmented retina. Answer: FALSE Diff: 2 Page Ref: 498-499 9) The superior oblique muscle of the eye enables the eye to look upward and laterally. Answer: FALSE Diff: 3 Page Ref: 494 10) The vestibule of the ear lies between the semicircular canals and the cochlea. Answer: TRUE Diff: 1 Page Ref: 512-513 11) The scala vestibuli is adjacent to the round window, whereas the scala tympani is adjacent to the oval window. Answer: FALSE Diff: 2 Page Ref: 510-511 12) The tips of cochlear hair cells are embedded in the basilar membrane. Answer: FALSE Diff: 2 Page Ref: 510-512 13) Infection and inflammation of the middle ear is called otitis media. Answer: TRUE Diff: 2 Page Ref: 508 14) The incus bridges the gap between the stapes connected to the oval window and the malleus connected to the tympanic membrane. Answer: TRUE Diff: 2 Page Ref: 508 15) Conduction deafness results from an inability of vibrations to pass to the inner ear. Answer: TRUE Diff: 1 Page Ref: 517 19 Copyright © 2014 Pearson Education, Inc.

 

16.3 Short Answer Questions 1) The special sense most like gustation (taste) in terms of type of receptor is ________. Answer: olfaction (smell) Diff: 2 Page Ref: 489 2) The synapse between an olfactory neuron and a mitral cell occurs in ________. Answer: glomeruli Diff: 2 Page Ref: 491 3) Smell is closely associated with the ________ region of the brain, which controls emotional responses. Answer: limbic Diff: 2 Page Ref: 491 4) In most Asians, a vertical fold of skin called the ________ often covers the medial corner of the eye. Answer: epicanthic fold Diff: 3 Page Ref: 492-493 5) The ________ links the middle ear to the pharynx and is important in equalizing air pressure on both sides of the eardrum. Answer: pharyngotympanic tube Diff: 2 Page Ref: 508 6) The lens of the eye is attached to stringlike ________, which are anchored by ciliary bodies attached to the sclera. Answer: ciliary zonules Diff: 2 Page Ref: 496-497 7) The sensory ________ of the eye consists of an outer pigmented layer and an inner neural layer. Answer: retina Diff: 1 Page Ref: 497 8) The posterior segment of the eye contains ________ humor. Answer: vitreous Diff: 2 Page Ref: 496, 500-501 9) Damage to the hair cells or any part of the auditory pathway to the brain results in ________. Answer: sensorineural deafness Diff: 2 Page Ref: 517 10) The tensor tympani and ________ are skeletal muscles that limit the vibrations of the ear ossicles to protect them from damage. Answer: stapedius Diff: 3 Page Ref: 509 20 Copyright © 2014 Pearson Education, Inc.

 

11) The ________ and saccule contain sensory organs of the vestibule that monitor static equilibrium and linear acceleration. Answer: utricle Diff: 2 Page Ref: 512-513 12) The crista ampullaris contains the jellylike ________, in which are embedded hair cells. Answer: cupula Diff: 2 Page Ref: 513-514 13) The three ________ are bony structures involved in the sense of rotational equilibrium. Answer: semicircular canals Diff: 1 Page Ref: 513 14) The ability of the eye to focus on nearby objects is a process called ________. Answer: accommodation Diff: 1 Page Ref: 502 15) The cochlear duct lies between two ________ -filled chambers known as the scala vestibuli and scala tympani. Answer: perilymph Diff: 2 Page Ref: 509-511 16.4 Essay Questions 1) Describe the production of aqueous and vitreous humors of the eye, and identify the structures involved in drainage. Answer: The vitreous humor is produced during embryo development and lasts a lifetime. The aqueous humor is a filtrate of the blood and is produced in the ciliary processes. The fluid exits the posterior chamber through the pupil into the anterior chamber of the eye. It drains via the scleral venous sinus. Diff: 2 Page Ref: 501-502 2) Describe the structure of photoreceptor cells and their arrangement within the retina. Answer: Photoreceptors consist of an outer segment containing disks of visual pigments. These rod- or cone-shaped segments are embedded in a pigmented epithelium of the retina. They are posterior to the inner segments. The inner segments are continuous with the cell body and synaptic endings of the cell that project anteriorly. Diff: 3 Page Ref: 497-498 3) Identify the routes by which middle ear infections can easily spread beyond the petrous region of the temporal bone. Answer: Infections can spread to the brain through the superior wall and the jugular vein through the floor, or inferior wall. A small canal, called the mastoid antrum, allows infections to spread to the mastoid air cells of the mastoid process. Infections can also spread from the pharynx to the middle ear through the pharyngotympanic tube. Diff: 2 Page Ref: 508 21 Copyright © 2014 Pearson Education, Inc.

 

4) Describe the anatomical similarities between the sensory organs of the vestibule (utricle and saccule) and semicircular canals. Answer: The sensory receptors in both the vestibule and semicircular canals are hair cells. In both cases, the tips of these cells are embedded within a gel-like structure. This structure is called the otolith membrane in the maculae of the sensory organs of the vestibule. This structure is called the cupula of the crista ampullaris in the semicircular canals. Movement of the gelatinous mass, either by fluid waves or gravity pulling on otoliths, causes the hair cells to depolarize and relay neural impulses to the brain. Diff: 2 Page Ref: 512-515 5) Identify the relationship between olfactory receptor cells, glomeruli, and mitral cells. Then explain how odor signals are amplified to the mitral cells. Answer: Odor molecules bind to olfactory receptor cells within the olfactory epithelium. These cells synapse with mitral cells located within the olfactory bulb. The synapse is called a glomerulus. Considering that many olfactory receptor cells converge and synapse with one mitral cell, the mitral cell receives a much stronger stimulus than it would if there were a one-to-one ratio of receptor and mitral cell. Diff: 2 Page Ref: 490-491

22 Copyright © 2014 Pearson Education, Inc.

 

Human Anatomy, 7e (Marieb/Mitchell/Smith) Chapter 17 The Endocrine System 17.1 Multiple Choice Questions

Figure 17.1 Use the diagram above to answer the following questions. 1) Identify the letter that indicates a gland that secretes hormone that controls metabolic rate. A) A B) B C) C D) D E) E Answer: B Diff: 2 Page Ref: 523, 532

1 Copyright © 2014 Pearson Education, Inc.

 

2) Identify the letter that indicates a gland that secretes corticosteroids. A) A B) B C) C D) D E) E Answer: D Diff: 2 Page Ref: 523, 532 3) Identify the letter that indicates a "master gland" that secretes at least nine hormones. A) A B) B C) C D) D E) E Answer: A Diff: 2 Page Ref: 523, 525 4) Secretes hormones that control blood sugar levels. A) A B) B C) C D) D E) E Answer: E Diff: 2 Page Ref: 523, 536-537 5) Immune organ that also secretes endocrine hormones. A) A B) B C) C D) D E) E Answer: C Diff: 2 Page Ref: 523, 537

2 Copyright © 2014 Pearson Education, Inc.

 

Figure 17.2 Use the diagram above to answer the following questions. 6) Identify the letter that indicates a gland that secretes melatonin. A) A B) B C) C D) D E) E Answer: C Diff: 2 Page Ref: 526, 536 7) Identify the letter that indicates a myelinated axon tract representative of a commissure connecting left and right cerebral hemispheres. A) A B) B C) C D) D E) E Answer: B Diff: 2 Page Ref: 526 8) Identify the letter that indicates the hypophysis, which consists of an anterior and posterior lobe. A) A B) B C) C D) D E) E Answer: A Diff: 2 Page Ref: 526 3 Copyright © 2014 Pearson Education, Inc.

 

9) Identify the letter that indicates a gland that regulates circadian rhythms. A) A B) B C) C D) D E) E Answer: C Diff: 2 Page Ref: 526, 536 10) Identify the letter that indicates a brain region (diencephalon) producing releasing and inhibiting hormones. A) A B) B C) C D) D E) E Answer: D Diff: 2 Page Ref: 526, 527 11) The anatomical stalk of the pituitary is also known as A) tuber cinerum. B) neurohypophysis. C) infundibulum. D) pars tuberalis. E) adenohypophysis. Answer: C Diff: 2 Page Ref: 525 12) Region of the hypophysis that contains secretory endocrine cells derived from the oral ectoderm. A) tuber cinerum B) neurohypophysis C) infundibulum D) pars tuberalis E) adenohypophysis Answer: E Diff: 2 Page Ref: 541 13) Region of the hypophysis that releases hormones stored in neurosecretory bodies in axon terminals. A) tuber cinerum B) neurohypophysis C) infundibulum D) pars tuberalis E) adenohypophysis Answer: B Diff: 2 Page Ref: 527-528 4 Copyright © 2014 Pearson Education, Inc.

 

14) Largest purely endocrine gland in the body. A) pancreas B) hypophysis C) adrenal gland D) thyroid gland E) thymus gland Answer: D Diff: 2 Page Ref: 531 15) Hormone that inhibits osteoclast activity in children. A) parathyroid hormone B) calcitonin C) thyroxine D) catecholamines E) oxytocin Answer: B Diff: 2 Page Ref: 532 16) Pyramid-shaped gland consisting of neural crest-derived cells and endocrine secretory cells. A) thyroid gland B) thymus gland C) gonadal cells of the ovaries/testes D) adrenal gland E) pancreas Answer: D Diff: 2 Page Ref: 534 17) Class of hormones secreted during chronic stress that depress immune function. A) mineralocorticoids B) glucocorticoids C) androgens D) gonadotrophins E) amino acid-based hormones Answer: B Diff: 2 Page Ref: 535 18) Pancreatic cell type that produces insulin. A) alpha cells B) beta cells C) delta cells D) gamma cells E) PP cells Answer: B Diff: 2 Page Ref: 536-537

5 Copyright © 2014 Pearson Education, Inc.

 

19) Gland of the posterior diencephalon. A) neurohypophysis B) adneohypophysis C) pineal gland D) thymus gland E) parathyroid gland Answer: C Diff: 2 Page Ref: 536 20) Pancreatic cell type that produces glucagon. A) alpha cells B) beta cells C) delta cells D) gamma cells E) PP cells Answer: A Diff: 2 Page Ref: 536 21) Endocrine gland of the thorax that is also important in immune function. A) neurohypophysis B) adneohypophysis C) pineal gland D) thymus gland E) parathyroid gland Answer: D Diff: 2 Page Ref: 537 22) Region of the adrenal cortex that secretes DHEA. A) zona glomerulosa B) zona fasiculata C) zona reticularis D) tuber cinerum E) chrommafin cells Answer: C Diff: 3 Page Ref: 536 23) Cells of the thyroid that produce calcitonin. A) follicular cells B) parafollicular cells C) parathyroid cells D) oxyphil cells E) pituicytes Answer: B Diff: 2 Page Ref: 532

6 Copyright © 2014 Pearson Education, Inc.

 

24) Class of hormones that regulates secretions of other endocrine glands. A) acidophilic B) basophilic C) tropic D) atrial naturetic peptide E) renin Answer: C Diff: 1 Page Ref: 525 25) Jellylike substance found in the thyroid follicles containing high concentrations of stored thyroglobulin. A) brain sand B) colloid C) acinar D) cortisol Answer: B Diff: 3 Page Ref: 531 26) Which of the following statements concerning the endocrine glands is incorrect? A) They are a group of ductless glands. B) They secrete messenger molecules called hormones. C) The hormones they secrete travel more quickly than nerve impulses. D) The hormones they secrete regulate processes such as growth and metabolism. Answer: C Diff: 1 Page Ref: 522 27) Which arrangement of endocrine cells does not allow for maximum contact with blood capillaries? A) branching networks B) clusters C) cords D) ducts Answer: D Diff: 1 Page Ref: 522 28) Which of the following hormones secreted by the pars distalis does not regulate the function of other endocrine glands? A) MSH B) TSH C) ACTH D) FSH Answer: A Diff: 1 Page Ref: 526

7 Copyright © 2014 Pearson Education, Inc.

 

29) Which of the following hormones is secreted by the posterior lobe of the pituitary? A) LH B) ADH C) glucagon D) adrenaline Answer: B Diff: 1 Page Ref: 528-529 30) Which endocrine gland stores enough of its hormone extracellularly to last several months? A) the pituitary B) the thyroid C) the pancreas D) the ovary Answer: B Diff: 2 Page Ref: 532 31) The action of the parathyroid hormone is counteracted by A) thyroxin. B) calcitonin. C) oxytocin. D) GH. Answer: B Diff: 3 Page Ref: 532-533 32) The adrenal medulla secretes A) aldosterone. B) cortisol. C) epinephrine and norepinephrine. D) glucocorticoids. Answer: C Diff: 1 Page Ref: 534 33) Which of the following is not part of the anterior lobe of the pituitary? A) pars distalis B) pars intermedia C) pars tuberalis D) pars nervosa Answer: D Diff: 2 Page Ref: 525

8 Copyright © 2014 Pearson Education, Inc.

 

34) Which endocrine gland stands out in X-ray images and helps radiologists get oriented in the brain? A) the pineal gland B) the pituitary gland C) the hypothalamus D) the thyroid gland Answer: A Diff: 1 Page Ref: 536 35) Which of the following minerals is essential for the formation of thyroid hormone? A) iodine B) calcium C) iron D) copper Answer: A Diff: 1 Page Ref: 532 36) Somatostatin A) increases the release of insulin. B) increases the release of glucagon. C) inhibits the secretion of insulin and glucagon. D) inhibits the exocrine activity of the pancreas. Answer: C Diff: 1 Page Ref: 537 37) Which of the following is a major source of steroid hormones in the body? A) the gonads B) the adrenal medulla C) the thyroid D) the pituitary Answer: A Diff: 1 Page Ref: 537 38) The hormone produced by the heart A) decreases urine output. B) increases calcium levels in the blood. C) is secreted in response to low blood volumes. D) increases the excretion of sodium in the urine. Answer: D Diff: 2 Page Ref: 537

9 Copyright © 2014 Pearson Education, Inc.

 

39) Which of the following endocrine glands develops primarily from the endoderm of the pharynx? A) anterior and posterior lobes of the pituitary B) thyroid and parathyroids C) pineal D) adrenal medulla Answer: B Diff: 2 Page Ref: 540-541 40) Which of the following hormones indirectly increases the secretion of aldosterone from the adrenal cortex? A) ACTH B) renin C) TSH D) corticotropin-releasing hormone Answer: B Diff: 3 Page Ref: 537-538 41) Erythropoietin is produced in the A) kidney. B) heart. C) bone marrow. D) thymus. Answer: A Diff: 1 Page Ref: 537 42) Which of the following is not part of the posterior lobe of the pituitary? A) infundibular stalk B) median eminence C) pars intermedia D) pars nervosa Answer: C Diff: 2 Page Ref: 525 43) The hypophyseal pouch is involved in the formation of the A) thyroid gland. B) thymus. C) pineal gland. D) pituitary gland. Answer: D Diff: 2 Page Ref: 540-541

10 Copyright © 2014 Pearson Education, Inc.

 

44) Vitamin D is synthesized and secreted by A) bone cells. B) the brain. C) the epithelium of the small intestine. D) the epidermis. Answer: D Diff: 3 Page Ref: 537 45) Melanocyte-stimulating hormone (MSH) is produced by A) corticotropic cells. B) gonadotropic cells. C) somatotropic cells. D) thyrotropic cells. Answer: A Diff: 2 Page Ref: 525 46) Prolactinoma is the most common tumor of the A) pineal gland. B) thyroid. C) adrenal medulla. D) pituitary gland. Answer: D Diff: 1 Page Ref: 541 47) The target of follicle-stimulating hormone (FSH) is A) the gonads. B) the parathyroid. C) the thymus. D) the thyroid. Answer: A Diff: 1 Page Ref: 525-526 48) Which of the following statements concerning the supraoptic nucleus is false? A) It is part of the hypothalamus. B) It gets its name because it is superior to the optic chiasma. C) It is a brain nucleus rather than a cell nucleus. D) It produces the hormone oxytocin. Answer: D Diff: 3 Page Ref: 528-529 49) Which of the following hormones is not produced by the placenta? A) estrogens B) testosterone C) progesterone D) hCG Answer: B Diff: 2 Page Ref: 537 11 Copyright © 2014 Pearson Education, Inc.

 

50) Of the following endocrine organs, which does not migrate away from its original site of formation in the embryo? A) thymus B) parathyroid glands C) thyroid gland D) posterior lobe of the pituitary gland Answer: D Diff: 1 Page Ref: 540-541 51) Compared to most other organs in the body, endocrine organs are A) well vascularized. B) poorly vascularized. C) have an average amount of vascularization. D) mainly epithelial cells. Answer: A Diff: 1 Page Ref: 523 52) The condition in which a neoplasm leads to hypersecretion of amine hormones by the adrenal medulla is called A) Graves' disease. B) pheochromocytoma. C) myxedema. D) acromegaly. Answer: B Diff: 1 Page Ref: 541 53) The secretion of hormones by the anterior lobe of the pituitary is controlled by the A) pars distalis. B) pancreatic islets. C) hypothalamus. D) atria of the heart. Answer: C Diff: 2 Page Ref: 527 54) Individuals with type 2 diabetes A) have an absolute requirement for insulin injections. B) have normal concentrations of glucose in their blood. C) may often control their disease by diet and exercise. D) have pancreases that produce and secrete very little insulin. Answer: C Diff: 2 Page Ref: 538-539

12 Copyright © 2014 Pearson Education, Inc.

 

55) The major stimulus for the release of thyroid hormone is A) hormonal. B) humoral. C) nervous. D) calcium. Answer: A Diff: 2 Page Ref: 524-525 56) The major stimulus for the release of the amine hormones of the adrenal gland is A) hormonal. B) humoral. C) nervous. D) adrenaline. Answer: C Diff: 2 Page Ref: 524 57) Which of the following hormones is secreted by neurons? A) oxytocin B) thyroid hormone C) growth hormone D) insulin Answer: A Diff: 2 Page Ref: 529 58) Which of the following hormones is secreted by cardiac muscle cells? A) ADH B) FSH C) atrial natriuretic hormone D) epinephrine Answer: C Diff: 2 Page Ref: 537 59) Which of the following hormones is secreted into the primary capillary plexus of the hypophyseal portal system? A) growth hormone-releasing hormone B) growth hormone C) ADH D) oxytocin Answer: A Diff: 2 Page Ref: 527

13 Copyright © 2014 Pearson Education, Inc.

 

60) The hypophyseal portal veins are primarily located in the A) pars nervosa. B) infundibulum. C) tuber cinereum. D) pars intermedia. Answer: B Diff: 2 Page Ref: 525, 527 61) Cells that manufacture and secrete aldosterone are located in the A) zona glomerulosa. B) renal cortex. C) pars distalis. D) pars nervosa. Answer: A Diff: 2 Page Ref: 535 62) Cells that manufacture and secrete ACTH are located in the A) adrenal cortex. B) adrenal medulla. C) pituitary gland. D) heart. Answer: C Diff: 1 Page Ref: 525 63) Cells that manufacture and secrete androgens are located in the A) bone tissue. B) pituitary gland. C) zona reticularis. D) nonpregnant uterus. Answer: C Diff: 2 Page Ref: 536 64) Cells that manufacture and secrete thyroid-stimulating hormone are located in the A) thyroid gland. B) hypothalamus. C) pars distalis. D) adrenal cortex. Answer: C Diff: 2 Page Ref: 525 65) Androgens are secreted and directly converted to estrogens and testosterone by the A) pituitary gland. B) peripheral reproductive cells. C) gonadotropic cells. D) interstitial cells. Answer: B Diff: 3 Page Ref: 536 14 Copyright © 2014 Pearson Education, Inc.

 

66) Pituitary disorders include all of the following except A) gigantism. B) acromegaly. C) pituitary dwarfism. D) Graves' disease. Answer: D Diff: 2 Page Ref: 538 67) The primary capillary plexus of the hypophyseal portal system is located in A) the hypothalamus. B) the infundibular stalk. C) the median eminence. D) the pars tuberalis. Answer: C Diff: 3 Page Ref: 525, 527 68) The artery that supplies most of the pituitary gland (but not the pars nervosa) is the A) primary plexus of the hypophyseal portal system. B) basilar artery on the brain stem. C) superior hypophyseal artery. D) hypophyseal branch of the external carotid artery. Answer: C Diff: 2 Page Ref: 525 69) Endocrine cells that secrete steroids have all of the following cytoplasmic features except A) abundant smooth endoplasmic reticulum. B) secretion via diffusion. C) lipid droplets. D) secretory granules. Answer: D Diff: 2 Page Ref: 523-524 70) Diabetes insipidus can be caused by trauma to the A) posterior pituitary gland. B) pancreas. C) gonads. D) kidney. Answer: A Diff: 2 Page Ref: 538 71) Which of the following hormones secreted by the pars distalis is a tropic hormone? A) MSH B) LH C) prolactin D) growth hormone Answer: B Diff: 2 Page Ref: 525 15 Copyright © 2014 Pearson Education, Inc.

 

72) The thyroid gland is located A) anterior to the aortic arch. B) immediately inferior to the larynx. C) on the superior aspect of the kidneys. D) posterior to the sternum. Answer: B Diff: 1 Page Ref: 531-532 73) Which of the following hormones is secreted by the pars intermedia of the pituitary gland? A) melatonin B) melanocyte-stimulating hormone C) prolactin-inhibiting hormone D) oxytocin Answer: B Diff: 2 Page Ref: 527 74) A cell type in the pars distalis that secretes two major kinds of hormones instead of one is the A) somatotropic cell. B) gonadotropic cell. C) thyrotropic cell. D) prolactin cell. Answer: B Diff: 2 Page Ref: 525 75) Which cells produce the thyroid hormone precursor thyroglobulin? A) parathyroid cells B) follicular cells C) oxyphil cells D) parafollicular cells Answer: B Diff: 2 Page Ref: 532 76) The beta type of endocrine cells in the pancreatic islets secretes a hormone that A) lowers blood sugar levels. B) raises blood sugar concentrations. C) targets liver cells. D) inhibits secretion by the exocrine pancreas. Answer: A Diff: 3 Page Ref: 536 77) Parathyroid hormone causes all of the following except A) activation of vitamin D. B) decreased secretion of calcium by the kidney. C) increase in plasma calcium. D) inhibition of osteoclasts. Answer: D Diff: 3 Page Ref: 533 16 Copyright © 2014 Pearson Education, Inc.

 

78) Cells of the adrenal medulla are most closely allied with A) the exocrine acinar cells of the pancreas. B) the pinealocytes. C) the sympathetic neurons. D) the T lymphocytes of the immune system. Answer: C Diff: 2 Page Ref: 534 79) Which region of the adrenal cortex consists of cells arranged in parallel cords? A) zona fasciculata B) zona glomerulosa C) zona pellucia D) zona reticularis Answer: A Diff: 2 Page Ref: 535 80) The cells that secrete parathyroid hormone are called A) parafollicular cells. B) oxyphil cells. C) parathyroid cells. D) follicular cells. Answer: C Diff: 2 Page Ref: 533 81) Target organs for the hormone oxytocin include the uterus and the A) lungs. B) mammary glands. C) biceps brachii muscle. D) thyroid gland. Answer: B Diff: 1 Page Ref: 529 82) Which of the following endocrine glands is capable of storing its hormones extracellularly and then slowly releasing them? A) thyroid gland B) pituitary gland C) adrenal gland D) pancreas Answer: A Diff: 2 Page Ref: 532

17 Copyright © 2014 Pearson Education, Inc.

 

17.2 True/False Questions 1) The pituitary gland atrophies with age and by age 80 is replaced by adipose tissue. Answer: FALSE Diff: 1 Page Ref: 541 2) All endocrine glands secrete their hormones into the bloodstream. Answer: TRUE Diff: 1 Page Ref: 522 3) An endocrine cell that secretes its hormone in response to the presence of another hormone is said to be controlled by humoral stimuli. Answer: FALSE Diff: 2 Page Ref: 524-525 4) Gonadotropic cells of the anterior lobe of the pituitary secrete prolactin. Answer: FALSE Diff: 2 Page Ref: 525-526 5) Gonadotropic cells of the pars distalis are the only class of cells there that produce more than one hormone. Answer: FALSE Diff: 2 Page Ref: 525 6) Thyroglobulin is produced by follicular cells of the thyroid gland, bound to thyroid hormone, and stored inside the follicle. Answer: TRUE Diff: 2 Page Ref: 532 7) Steroid hormones are synthesized from cholesterol. Answer: TRUE Diff: 1 Page Ref: 523-524 8) The adrenal medulla consists of three distinct zones and secretes steroid hormones. Answer: FALSE Diff: 2 Page Ref: 535-537 9) The endocrine cells of the pancreas are present in islets, whereas the exocrine cells are in acini. Answer: TRUE Diff: 2 Page Ref: 536 10) The heart contains endocrine cells that secrete atrial natriuretic peptide (ANP). Answer: TRUE Diff: 2 Page Ref: 537

18 Copyright © 2014 Pearson Education, Inc.

 

11) Releasing hormones from the hypothalamus diffuse from the secondary capillary plexus to activate target cells in the pars distalis. Answer: TRUE Diff: 2 Page Ref: 527-528 12) In diabetes mellitus, the posterior lobe of the pituitary does not produce sufficient antidiuretic hormone. Answer: FALSE Diff: 2 Page Ref: 538 13) Only GH, DHEA, and the sex hormones show marked decline in secretion with age. Answer: TRUE Diff: 2 Page Ref: 541 14) Hypersecretion of glucocorticoid hormones leads to Cushing's disease. Answer: TRUE Diff: 3 Page Ref: 539-540 15) Melatonin production by the pineal gland involves signals transmitted by thoracic and cervical nerves. Answer: TRUE Diff: 3 Page Ref: 536 17.3 Short Answer Questions 1) The parathyroid glands lie on the ________ surface of the thyroid gland. Answer: posterior Diff: 1 Page Ref: 532-533 2) The ability of a target tissue to respond to a particular hormone depends on the presence of a specific ________ that will bind the hormone on or inside the target cell. Answer: receptor Diff: 2 Page Ref: 524 3) Changing levels of ions or nutrients in the blood are examples of ________ stimuli that cause changes in endocrine secretion. Answer: humoral Diff: 2 Page Ref: 524 4) Cells of the kidney produce the hormone ________, which stimulates red blood cell production. Answer: erythropoietin Diff: 2 Page Ref: 537

19 Copyright © 2014 Pearson Education, Inc.

 

5) Releasing and inhibiting hormones travel from the hypothalamus through a vascular ________ portal system to the anterior lobe of the pituitary. Answer: hypophyseal Diff: 2 Page Ref: 527-528 6) The ________ hormone aldosterone is produced by the adrenal cortex to increase blood volume. Answer: mineralocorticoid, or corticosteroid Diff: 2 Page Ref: 535 7) The pineal gland is readily observed on X-ray images because of the presence of radiopaque ________, or calcium particles. Answer: pineal sand Diff: 2 Page Ref: 536 8) The ________ consists of four endocrine cell types: A, B, D, and PP. Answer: pancreas Diff: 2 Page Ref: 536-537 9) The thymic hormones are structurally ________ molecules. Answer: peptide Diff: 1 Page Ref: 537 10) Hyperthyroidism, known as ________ disease, is an autoimmune condition. Answer: Graves' Diff: 2 Page Ref: 539 11) Growth hormone is produced by ________ cells of the adenohypophysis. Answer: somatotropic Diff: 2 Page Ref: 526-527 12) The endocrine cells of the pancreas are located within spherical bodies called ________. Answer: pancreatic islets Diff: 1 Page Ref: 536 13) ________ produces contraction of the smooth musculature of the male and female reproductive tracts. Answer: Oxytocin Diff: 2 Page Ref: 529 14) The three layers of the adrenal cortex, from external to internal, are the zona glomerulosa, zona fasciculata, and zona ________. Answer: reticularis Diff: 3 Page Ref: 534

20 Copyright © 2014 Pearson Education, Inc.

 

15) The diffuse ________ system consists of endocrine cells scattered throughout the visceral organs. Answer: neuroendocrine Diff: 3 Page Ref: 537 17.4 Essay Questions 1) Explain what is meant by the term neuroendocrine, and give an example of a neuroendocrine gland. Answer: A neuroendocrine gland is an endocrine gland that contains a significant number of cells belonging to another organ system, specifically cells of the nervous system. The hypothalamus is a neuroendocrine gland because the neurohypophysis consists of neurons that store and release endocrine hormones. Diff: 1 Page Ref: 523, 525, 527-528 2) Contrast the major differences between the two anatomical divisions of the pituitary gland. Answer: The anterior lobe of the pituitary, or adenohypophysis, consists of several types of endocrine cells, all of which are epithelial in origin. These cells secrete their endocrine products in response to hormones released from the hypothalamus. Hypothalamic signaling molecules reach the adenohypophysis via a portal blood system.The posterior lobe of the pituitary, or neurohypophysis, consists of the axons and axon terminals of neurons whose cell bodies exist in the hypothalamus. Endocrine hormones are stored and released from their axon terminals. Diff: 2 Page Ref: 525-529 3) Two distinct endocrine glands are each a combination of two different embryonic tissues. Identify these glands, and describe their embryonic origins. Answer: Both the pituitary and adrenal glands are composed of two different embryonic tissues. The pituitary gland derives from projections from the roof of the mouth and the floor of the brain, the future anterior lobe, and the future posterior lobe, respectively. The adrenal gland consists of the cortex and medulla. The cortex derives from somatic mesoderm of the dorsal abdominal wall, whereas cells of the medulla are neural crest derivatives. The dual origin of these endocrine glands reflects the distinct function of their respective subdivisions. Diff: 3 Page Ref: 534, 541 4) Contrast the two anatomical regions of the adrenal glands. Answer: The adrenal gland consists of a superficial adrenal cortex and a deep adrenal medulla. The adrenal medulla consists of modified neural ganglia that release catecholamine neurotransmitters. In contrast, the adrenal cortex, consisting of three distinct layers or zones, secretes steroid hormones. These include the mineralocorticoids, glucocorticoids, and a weak androgen DHEA. Diff: 1 Page Ref: 534-536

21 Copyright © 2014 Pearson Education, Inc.

 

5) Distinguish the anatomical causes of diabetes mellitus and diabetes insipidus. Answer: Diabetes insipidus results when the posterior lobe of the pituitary produces insufficient antidiuretic hormone. One result is the production of large volumes of urine. In diabetes mellitus, the problem is that the pancreatic islet cells produce insufficient insulin, or that the tissues themselves don't respond to insulin. The production of large quantities of urine is also one sign of diabetes mellitus. Diff: 2 Page Ref: 538-539

22 Copyright © 2014 Pearson Education, Inc.

 

Human Anatomy, 7e (Marieb/Mitchell/Smith) Chapter 18 Blood 18.1 Multiple Choice Questions

Figure 18.1 Use the diagram above to answer the following questions. 1) Identify the letter that indicates the formed element of the blood that is packed with molecules of hemoglobin. A) A B) B C) C D) D E) E Answer: A Diff: 2 Page Ref: 551 2) Identify the letter that indicates formed elements of the blood that are important in blood clotting. A) A B) B C) C D) D E) E Answer: E Diff: 2 Page Ref: 551

1 Copyright © 2014 Pearson Education, Inc.

 

3) Identify the letter that indicates the most abundant class of leukocyte. A) A B) B C) C D) D E) E Answer: B Diff: 2 Page Ref: 551 4) Identify the letter that indicates the formed element of the blood that develops into phagocytic cells called macrophages. A) A B) B C) C D) D E) E Answer: D Diff: 2 Page Ref: 551 5) Identify the letter that indicates the formed element of the blood that produces antibodies. A) A B) B C) C D) D E) E Answer: C Diff: 2 Page Ref: 551

2 Copyright © 2014 Pearson Education, Inc.

 

Figure 18.2 Use the diagram above to answer the following questions. 6) Identify the letter that indicates the fraction of leukocytes that represents lymphocytes. A) A B) B C) C D) D E) E Answer: D Diff: 2 Page Ref: 550 7) Identify the letter that indicates the fraction that of leukocytes that represents monocytes. A) A B) B C) C D) D E) E Answer: E Diff: 2 Page Ref: 550 8) Identify the letter that indicates the fraction of leukocytes that represents neutrophils. A) A B) B C) C D) D E) E Answer: A Diff: 2 Page Ref: 550 3 Copyright © 2014 Pearson Education, Inc.

 

9) Identify the letter that indicates the fraction of leukocytes that represents eosinophils. A) A B) B C) C D) D E) E Answer: B Diff: 2 Page Ref: 550 10) Identify the letter that indicates the fraction of leukocytes that represents basophils. A) A B) B C) C D) D E) E Answer: C Diff: 2 Page Ref: 550 11) Along with leukocytes, these contribute less than 1% of whole blood. A) reticulocytes B) erythrocytes C) albumin D) platelets E) lymphocytes Answer: D Diff: 2 Page Ref: 547 12) Plasma minus clotting factors. A) hematocrit B) buffy coat C) serum D) albumin E) myeloid Answer: C Diff: 2 Page Ref: 547 13) Cell fragments, also called thrombocytes, involved in clotting. A) basophils B) globulins C) fibrinogen D) albumin E) platelets Answer: E Diff: 2 Page Ref: 547

4 Copyright © 2014 Pearson Education, Inc.

 

14) Fraction of blood comprised by erythrocytes. A) hematocrit B) buffy coat C) serum D) albumin E) myeloid Answer: A Diff: 2 Page Ref: 547 15) Blood protein that contributes osmotic pressure, keeping water from leaking out of the vessels. A) hematocrit B) buffy coat C) serum D) albumin E) myeloid Answer: D Diff: 2 Page Ref: 547 16) Most abundant formed element. A) erythrocyte B) myeloid C) eosinophil D) monocyte E) basophil Answer: A Diff: 2 Page Ref: 548 17) Leukocyte primarily responsible for destroying bacteria. A) erythrocyte B) lymphocyte C) eosinophil D) neutrophil E) basophil Answer: D Diff: 2 Page Ref: 551 18) Seen in abundance during parasitic infection. A) erythrocyte B) lymphocyte C) eosinophil D) neutrophil E) basophil Answer: C Diff: 2 Page Ref: 551

5 Copyright © 2014 Pearson Education, Inc.

 

19) Cell mediator of inflammation. A) erythrocyte B) lymphocyte C) eosinophil D) neutrophil E) basophil Answer: E Diff: 2 Page Ref: 551 20) Parent cell that transforms into a macrophage. A) monocyte B) lymphocyte C) eosinophil D) neutrophil E) basophil Answer: A Diff: 2 Page Ref: 551-552 21) Produces antibodies. A) erythrocyte B) lymphocyte C) eosinophil D) neutrophil E) basophil Answer: B Diff: 2 Page Ref: 551 22) Stem cell from which eosinophils and basophils develop. A) erythrocyte B) myeloid C) eosinophil D) monocyte E) basophil Answer: B Diff: 2 Page Ref: 554-555 23) Young erythrocyte. A) myeloid B) eosinophil C) reticulocyte D) lymphoid E) lymphocyte Answer: C Diff: 2 Page Ref: 555-556

6 Copyright © 2014 Pearson Education, Inc.

 

24) Cell containing a large, dark purple-staining spherical nucleus that almost completely fills the cell volume. A) myeloid B) eosinophil C) reticulocyte D) lymphoid E) lymphocyte Answer: E Diff: 3 Page Ref: 551-552 25) Disorders of red blood cells can be detected by obtaining counts of these immature structures. A) myeloid B) eosinophil C) reticulocyte D) lymphoid E) lymphocyte Answer: C Diff: 3 Page Ref: 556 26) Hematocrit measures the percentage of blood volume that consists of A) neutrophils. B) erythrocytes. C) platelets. D) plasma. Answer: B Diff: 1 Page Ref: 547 27) The most common formed elements in the blood are A) leukocytes. B) erythrocytes. C) platelets. D) macrophages. Answer: B Diff: 1 Page Ref: 548 28) In the cell line that forms erythrocytes in red bone marrow, all of the following occur except that A) the cytoplasm goes from basophilic (blue-staining) to eosinophilic (pink-staining). B) the nucleus is lost. C) hemoglobin accumulates in the cells. D) lysosome-like granules accumulate. Answer: D Diff: 2 Page Ref: 556

7 Copyright © 2014 Pearson Education, Inc.

 

29) Which of the following cells lack mitochondria? A) neutrophils B) platelets C) basophils D) erythrocytes Answer: D Diff: 2 Page Ref: 548-549 30) ________ is a condition in which the blood's capacity for carrying oxygen is diminished. A) Polycythemia B) Thrombocytopenia C) Anemia D) Leukemia Answer: C Diff: 1 Page Ref: 556 31) The unusual shape of the erythrocyte can be explained by which of the following? A) It is the best shape for a cell that must pass through narrow capillaries. B) It allows each cell to hold a maximum amount of hemoglobin. C) It increases surface area for respiratory exchange across the plasma membrane. D) It reflects the fact that erythrocytes are degenerating. Answer: C Diff: 2 Page Ref: 549 32) The most abundant white blood cell type is the A) neutrophil. B) monocyte. C) eosinophil. D) lymphocyte. Answer: A Diff: 1 Page Ref: 550-551 33) What is the anatomical difference between T lymphocytes and B lymphocytes? A) T cells are larger. B) B cells are larger. C) B cells have a darker-staining nucleus. D) They are structurally identical. Answer: D Diff: 1 Page Ref: 552 34) Which of the following is not phagocytic? A) lymphocyte B) neutrophil C) eosinophil D) basophil Answer: A Diff: 2 Page Ref: 550 8 Copyright © 2014 Pearson Education, Inc.

 

35) Which cells fight infection by producing antibodies? A) T lymphocytes B) B lymphocytes C) plasma cells D) eosinophils Answer: B Diff: 1 Page Ref: 551-552 36) What type of white blood cell increases dramatically during parasitic infections or allergic reactions? A) neutrophil B) monocyte C) basophil D) eosinophil Answer: D Diff: 1 Page Ref: 551-552 37) Which of the following organs does not form blood cells in the fetus? A) liver B) lung C) spleen D) yolk sac Answer: B Diff: 2 Page Ref: 557 38) Which white blood cells contain granules of histamine? A) eosinophils B) basophils C) neutrophils D) lymphocytes Answer: B Diff: 1 Page Ref: 551-552 39) The functions of platelets include all of the following except A) secretion of chemicals that call more platelets to the site of injury. B) formation of a temporary patch in the walls of damaged blood vessels. C) release of chemical signals that trigger the immune response. D) release of molecules that initiate clotting. Answer: C Diff: 2 Page Ref: 551, 553

9 Copyright © 2014 Pearson Education, Inc.

 

40) What is the difference between a thrombus and an embolus? A) One occurs in the bloodstream, whereas the other occurs outside the bloodstream. B) One occurs in arteries, the other in veins. C) One is a blood clot, whereas the other is a parasitic worm. D) A thrombus must travel to become an embolus. Answer: D Diff: 1 Page Ref: 553 41) Yellow bone marrow gets its color from A) the bone trabeculae in the center of this colorless marrow. B) the early stages of red blood cells being made there. C) low densities of red blood cells. D) fat cells. Answer: D Diff: 1 Page Ref: 553 42) Diapedesis is A) the process by which eosinophils attack worms. B) the ingestion of bacteria by macrophages. C) the exit of leukocytes from capillaries. D) amoeboid motion. Answer: C Diff: 2 Page Ref: 549 43) An average female has approximately ________ of blood. A) 0.5 liter B) 1 liter C) 4 liters D) 8 liters Answer: C Diff: 1 Page Ref: 547 44) When centrifuged, blood separates into the following sequence of layers, from lightest to heaviest: A) buffy coat, hematocrit, plasma. B) buffy coat, plasma, hematocrit. C) hematocrit, plasma, buffy coat. D) plasma, buffy coat, hematocrit. Answer: D Diff: 2 Page Ref: 547

10 Copyright © 2014 Pearson Education, Inc.

 

45) In adults, blood cells are manufactured in the marrow of all of the following bones except the A) pelvis. B) forearm bones (radius and ulna). C) clavicle. D) sternum. Answer: B Diff: 3 Page Ref: 553 46) Which of the following statements concerning the various leukocytes is false? A) All perform diapedesis. B) All are active in connective tissues but not in blood. C) All fight disease. D) All have distorted, lobed nuclei. Answer: D Diff: 2 Page Ref: 549, 551 47) The precursors of neutrophils, listed in their proper order from the least to the most differentiated cells, are: A) band cell, myeloblast, and myelocyte. B) metamyelocyte, myeloblast, and neutrophil. C) myeloblast, metamyelocyte, and band cell. D) metamyelocyte, neutrophil, and band cell. Answer: C Diff: 2 Page Ref: 555-556 48) The blood's globulins include A) albumin. B) antibodies. C) fibrinogen. D) hemoglobin. Answer: B Diff: 1 Page Ref: 547 49) T cells function to A) produce antibodies. B) destroy body cells infected with viruses. C) respond primarily to bacteria and bacterial toxins in body fluids. D) phagocytize antigen-antibody complexes. Answer: B Diff: 1 Page Ref: 552

11 Copyright © 2014 Pearson Education, Inc.

 

50) Leukocytes, ordered from most to least abundant, are the A) basophils, eosinophils, lymphocytes, monocytes, and neutrophils. B) basophils, eosinophils, monocytes, lymphocytes, and neutrophils. C) neutrophils, lymphocytes, monocytes, eosinophils, and basophils. D) neutrophils, monocytes, eosinophils, basophils, and lymphocytes. Answer: C Diff: 2 Page Ref: 550 51) The replacement of red bone marrow with yellow bone marrow in the limbs occurs A) in fetal life. B) by 2 years of age. C) before 8 years of age. D) between 8 and 18 years of age. Answer: D Diff: 2 Page Ref: 553 52) Which of the following cells develop into macrophages? A) monocytes B) neutrophils C) basophils D) lymphocytes Answer: A Diff: 1 Page Ref: 552 53) Which type of connective tissue occurs in the bone marrow cavity? A) cartilage B) loose areolar C) dense irregular D) reticular Answer: D Diff: 1 Page Ref: 554 54) The least abundant white blood cell type is the A) monocyte. B) eosinophil. C) basophil. D) lymphocyte. Answer: C Diff: 1 Page Ref: 551-552

12 Copyright © 2014 Pearson Education, Inc.

 

55) If examination of the blood reveals a large number of immature leukocytes, the individual is probably suffering from A) anemia. B) leukemia. C) sickle cell disease. D) thrombocytopenia. Answer: B Diff: 1 Page Ref: 557 56) Megakaryocytes A) circulate freely in the blood. B) give rise to platelets. C) are small compared to erythrocytes. D) are phagocytic. Answer: B Diff: 1 Page Ref: 556 57) Reticulocytes are precursors to mature A) erythrocytes. B) lymphocytes. C) neutrophils. D) basophils. Answer: A Diff: 2 Page Ref: 555-556 58) B cells fight infections by A) acting as killer cells. B) engulfing foreign pathogens. C) killing off body cells. D) producing antibodies. Answer: D Diff: 1 Page Ref: 552 59) The blood cell whose nucleus often resembles a telephone receiver is the A) erythrocyte. B) basophil. C) eosinophil. D) lymphocyte. Answer: C Diff: 1 Page Ref: 551-552

13 Copyright © 2014 Pearson Education, Inc.

 

60) Which of the following groups of cells are most structurally related? A) basophils, lymphocytes, and monocytes B) eosinophils, basophils, and neutrophils C) lymphocytes, erythrocytes, and basophils D) monocytes, platelets, and macrophages Answer: B Diff: 2 Page Ref: 550 61) ________ is a condition in which there is an abnormal excess of erythrocytes in the blood. A) Leukemia B) Anemia C) Polycythemia D) Thrombocytopenia Answer: C Diff: 2 Page Ref: 556 62) All of the following cell types are derived from the same precursor cell that generates neutrophils except the A) blood stem cell. B) plasma cell. C) myeloblast. D) myelocyte. Answer: B Diff: 2 Page Ref: 555 63) The formed element that is flattened, circular, lacks a nucleus, lacks mitochondria or ribosomes, and is red because of the presence of hemoglobin is A) an eosinophil. B) a basophil. C) an erythrocyte. D) a lymphocyte. Answer: C Diff: 1 Page Ref: 548-549 64) Hemopoiesis is A) different from hematopoiesis. B) a disease of erythrocytes. C) a stain for blood smears. D) blood cell production. Answer: D Diff: 1 Page Ref: 553

14 Copyright © 2014 Pearson Education, Inc.

 

65) The percentage of erythrocytes in a normal volume of blood is about A) 10%. B) 30%. C) 45%. D) 90%. Answer: C Diff: 2 Page Ref: 547 66) Which of the following does not remain in red bone marrow after hematopoiesis? A) lymphoid stem cells B) fat cells C) myeloid stem cells D) reticulocytes Answer: D Diff: 3 Page Ref: 554, 556 67) Which cells are the most responsible for the rejection of a transplanted organ, such as a heart? A) B cells B) macrophages C) eosinophils D) T cells Answer: D Diff: 3 Page Ref: 558 68) An eosinophil can be distinguished from a basophil because the eosinophil A) has a lobed (versus nonlobed) nucleus. B) contains cytoplasmic granules. C) has pink (versus dark purple) granules. D) is smaller. Answer: C Diff: 1 Page Ref: 551 69) Elevated numbers of reticulocytes may indicate A) a degenerative bone marrow disease. B) a parasitic infection. C) a person is adapting to life at high elevations. D) sickle cell disease. Answer: C Diff: 2 Page Ref: 556

15 Copyright © 2014 Pearson Education, Inc.

 

70) ________ is a condition in which there is an abnormally low concentration of platelets in the blood. A) Leukemia B) Anemia C) Polycythemia D) Thrombocytopenia Answer: D Diff: 2 Page Ref: 557 71) Giant cells located just outside of marrow sinusoids are called A) megakaryocytes. B) monoblasts. C) myeloblasts. D) reticulocytes. Answer: A Diff: 1 Page Ref: 556 72) Together, leukocytes and platelets constitute roughly which percentage of whole blood volume? A) 45% B) 55% C) 1% D) 10% Answer: C Diff: 1 Page Ref: 547 73) Which of the following is not a phase of erythropoiesis? A) production of ribosomes B) synthesis of hemoglobin C) ejection of the erythrocyte's nucleus D) production of vacuoles Answer: D Diff: 3 Page Ref: 556 74) Which of the following is the correct ranking of formed elements of blood by longevity (life span), from longest-lived to shortest-lived? A) neutrophils, basophils, erythrocytes B) basophils, erythrocytes, eosinophils C) erythrocytes, platelets, neutrophils D) monocytes, neutrophils, erythrocytes Answer: C Diff: 2 Page Ref: 551

16 Copyright © 2014 Pearson Education, Inc.

 

75) Which of the following precursors of erythrocytes are in their proper order, from least to most differentiated? A) proerythroblast, normocyte, reticulocyte B) erythroblast, reticulocyte, normocyte C) normocyte, proerythroblast, erythrocyte D) reticulocyte, normocyte, erythroblast Answer: A Diff: 2 Page Ref: 555-556 76) Which of the following is not a committed cell in a blood cell line? A) hemapoietic stem cell B) proerythroblast C) myeloblast D) megakaryoblast Answer: A Diff: 1 Page Ref: 555 77) Graft-versus-host disease occurs in many marrow transplant patients because of the activity of A) lymphocytes. B) macrophages. C) neutrophils. D) platelets. Answer: A Diff: 2 Page Ref: 558 78) In the body, erythrocytes are red because of the oxidized iron they contain. In stained blood smears, erythrocytes are A) also red from iron. B) light blue from methylene blue. C) purple from hematoxylin stain. D) pink from eosin. Answer: D Diff: 2 Page Ref: 548, 551 79) The defining characteristic of a reticulocyte in the blood is A) collagen (reticular) fibers. B) dark-staining nucleus. C) numerous endocytic vesicles. D) dark staining masses that represent degrading ribosomes. Answer: D Diff: 1 Page Ref: 556

17 Copyright © 2014 Pearson Education, Inc.

 

80) A plasma protein involved in blood clotting is A) a platelet. B) globulin. C) fibrin (and fibrinogen). D) albumin. Answer: C Diff: 1 Page Ref: 547 81) In the differentiating cell line that leads to neutrophils, which of the following events occurs? A) The nucleus assumes a band shape. B) Hemoglobin accumulates in the cytoplasm. C) Metamyelocytes become myeloblasts. D) The cells temporarily have the specific granules of eosinophils. Answer: A Diff: 2 Page Ref: 555-556 18.2 True/False Questions 1) Blood serum is plasma from which albumin has been removed. Answer: FALSE Diff: 2 Page Ref: 547 2) Hematocrit is the percentage of blood consisting of erythrocytes. Answer: TRUE Diff: 2 Page Ref: 547 3) Elevated numbers of band cells in differential WBC counts is considered an indicator of infection. Answer: TRUE Diff: 2 Page Ref: 555 — 556 4) All granulocytes have lobed nuclei and are phagocytic in function. Answer: TRUE Diff: 3 Page Ref: 550 5) Mast cells and basophils both release histamine at the site of inflammation. Answer: TRUE Diff: 2 Page Ref: 552 6) B lymphocytes attach to non-self cells and cause them to lyse. Answer: FALSE Diff: 2 Page Ref: 552 7) The least abundant type of leukocyte is the monocyte. Answer: FALSE Diff: 2 Page Ref: 552 18 Copyright © 2014 Pearson Education, Inc.

 

8) Leukocytes function primarily within the bloodstream. Answer: FALSE Diff: 1 Page Ref: 549 9) In adults, red bone marrow occurs in the skull, girdles, and diaphysis of the long bones. Answer: FALSE Diff: 2 Page Ref: 553 10) Erythrocytes and neutrophils both arise from myeloid stem cells. Answer: TRUE Diff: 2 Page Ref: 555 11) The reticulocytes secrete the reticular fiber network of bone marrow. Answer: FALSE Diff: 3 Page Ref: 555-556 12) Platelets are cell fragments arising from megakaryocytes located just outside the sinusoids. Answer: TRUE Diff: 2 Page Ref: 556 13) The liver and spleen are the major blood-producing organs between the second and the seventh month of fetal development. Answer: TRUE Diff: 2 Page Ref: 557 14) The most abundant type of leukocyte is the neutrophil. Answer: TRUE Diff: 2 Page Ref: 550-551 15) Erythrocytes eject their nuclei and other organelles prior to maturity. Answer: TRUE Diff: 2 Page Ref: 555-556 18.3 Short Answer Questions 1) The ________ is the fraction of blood made up of leukocytes. Answer: buffy coat Diff: 2 Page Ref: 547 2) ________ is an inherited disease caused by the lack of, or reduced amount of, a clotting factor. Answer: Hemophilia Diff: 2 Page Ref: 559 3) Blood clots are formed when cells become trapped among a tangle of the activated plasma protein ________ along with other clotting factors. Answer: fibrinogen Diff: 2 Page Ref: 547 19 Copyright © 2014 Pearson Education, Inc.

 

4) Leukocytes can exit the blood vessels through a process called ________. Answer: diapedesis Diff: 2 Page Ref: 549 5) ________ disease occurs in many bone marrow transplant patients because of the activity of the T lymphocytes. Answer: Graft-versus-host Diff: 2 Page Ref: 558 6) ________ turn off allergic responses by phagocytizing allergens and degrading histamine. Answer: Eosinophils Diff: 2 Page Ref: 551-552 7) The absence of a nucleus, mitochondria, and ribosomes is a telltale characteristic of ________. Answer: erythrocytes Diff: 1 Page Ref: 549 8) The presence of dark purple-stained granules along with a two-lobed nucleus is indicative of a ________. Answer: basophil Diff: 2 Page Ref: 552 9) Plasma minus clotting factors is called ________. Answer: serum Diff: 1 Page Ref: 547 10) Monocytes develop into ________, which are phagocytic cells that ingest foreign cells and debris. Answer: macrophages Diff: 2 Page Ref: 552 11) ________ are cell fragments that initiate a blood clot formed of blood cells trapped in a network of fibrin protein. Answer: Platelets Diff: 2 Page Ref: 553 12) The formation of blood is called ________. Answer: hematopoiesis, or hemopoiesis Diff: 2 Page Ref: 553 13) Newly developed blood cells enter the circulatory system by blood ________, or wide capillaries, within the marrow. Answer: sinusoids Diff: 2 Page Ref: 554 20 Copyright © 2014 Pearson Education, Inc.

 

14) For the majority of fetal development, blood formation occurs primarily in the ________ and later in the bone marrow. Answer: liver and spleen Diff: 2 Page Ref: 557 15) Insufficient vitamin B12 or folic acid can cause ________ and therefore lower a person's oxygen-carrying capacity. Answer: anemia Diff: 3 Page Ref: 556 18.4 Essay Questions 1) Describe how the structure of erythrocytes is advantageous for their function as oxygen transporters. Answer: Erythrocytes consist of hemoglobin molecules surrounded by a plasma membrane. They lack a nucleus and organelles. Therefore, the cells have little oxygen demand themselves, making them ideal for transporting oxygen to tissues. The biconcave shape enhances their ability to exchange gases across their surface. This shape is flexible, allowing the cells to deform slightly as they pass through capillaries that are not much wider than the cells. Diff: 2 Page Ref: 548-549 2) Describe the conclusions that can be drawn from a "CBC with diff" that comes back showing (a) elevated neutrophils, (b) with elevated eosinophils. Answer: (a) A complete blood count with differential white blood cell count that shows elevated neutrophils would suggest the presence of a bacterial infection. Neutrophils are primarily phagocytic against bacteria. (b) A test that reported elevated eosinophils would suggest either an allergic reaction or the presence of a parasitic infection. Diff: 2 Page Ref: 552 3) Explain how platelets are involved in stopping blood flow through a wound. Answer: Platelets attach themselves only to damaged blood vessels or those roughened by scarring, inflammation, or atherosclerosis. By adhering to a ruptured vessel, they form a temporary plug in the opening. Upon attaching to the collagen fibers, they release chemicals that attract additional platelets to the area and other chemicals that initiate a clotting cascade. This cascade involves the accumulation of fibrin protein in a meshlike plug that traps blood cells and platelets and seals the opening in the vessel wall. Diff: 2 Page Ref: 553 4) Describe the difference between myeloblastic and lymphoblastic leukemias, and identify the types of cells that might be abnormal in each disease. Answer: Myeloblastic leukemia is an abnormal proliferation of leukocytes derived from the myeloid stem cells. These would include the neutrophils, basophils, and eosinophils. In contrast, lymphoblastic leukemia is an abnormal proliferation of lymphocytes (B and T cells). In both cases, the bloodstream may have an abundance of these cells, the presence of abnormal leukocytes, and, in late stages, a decline in erythrocytes. Diff: 2 Page Ref: 557 21 Copyright © 2014 Pearson Education, Inc.

 

5) Define polycythemia, and describe a clinical blood test that would help in making this diagnosis. Answer: Polycythemia is an abnormal excess of red blood cells. It can be diagnosed by taking a blood sample and spinning it in a centrifuge. When the formed elements collect at the base of the centrifuge tube, the percentage of the sample consisting of red blood cells can be measured. This is the hematocrit. A significantly elevated hematocrit (above 47 for males or 42 for females) may suggest polycythemia. Diff: 3 Page Ref: 547, 556

22 Copyright © 2014 Pearson Education, Inc.

 

Human Anatomy, 7e (Marieb/Mitchell/Smith) Chapter 19 The Heart 19.1 Multiple Choice Questions

Figure 19.1 Use the diagram above to answer the following questions. 1) Identify the letter that indicates the left common carotid artery. A) A B) B C) C D) D E) E Answer: A Diff: 2 Page Ref: 567 2) Identify the letter that indicates the left auricle. A) A B) B C) C D) D E) E Answer: D Diff: 2 Page Ref: 567 1 Copyright © 2014 Pearson Education, Inc.

 

3) Identify the letter that indicates the ligamentum arteriosum. A) A B) B C) C D) D E) E Answer: C Diff: 2 Page Ref: 567 4) Identify the letter that indicates the left coronary artery. A) A B) B C) C D) D E) E Answer: E Diff: 2 Page Ref: 567 5) Identify the letter that indicates the aortic arch. A) A B) B C) C D) D E) E Answer: B Diff: 2 Page Ref: 567

2 Copyright © 2014 Pearson Education, Inc.

 

Figure 19.2 Use the diagram above to answer the following questions. 6) Identify the letter that indicates anchor points for chordae tendineae, comprised of cells from the myocardium. A) A B) B C) C D) D E) E Answer: B Diff: 2 Page Ref: 569 7) Identify the letter that indicates the tissue layer of the heart known as the epicardium. A) A B) B C) C D) D E) E Answer: E Diff: 2 Page Ref: 569, 565 8) Identify the letter that indicates the left atrioventricular valve. A) A B) B C) C D) D E) E Answer: A Diff: 2 Page Ref: 569, 570 3 Copyright © 2014 Pearson Education, Inc.

 

9) Identify the letter that indicates the location of the bundle branches. A) A B) B C) C D) D E) E Answer: C Diff: 3 Page Ref: 569, 578 10) Identify the letter that indicates the thicker myocardial layer of the left ventricle. A) A B) B C) C D) D E) E Answer: D Diff: 2 Page Ref: 569, 575 11) The region between the right and left pleural cavities is the A) pulmonary cavity. B) peritoneal cavity. C) pericardial cavity. D) vertebral cavity. E) mediastinum. Answer: E Diff: 2 Page Ref: 564 12) The accumulation of pericardial fluid due to inflammation or the accumulation of blood in the pericardial cavity can lead to A) pericarditis. B) pleuritis. C) cardiac tamponade. D) fasciae adherens. E) mitral valve prolapse. Answer: C Diff: 2 Page Ref: 565 13) The heart chamber that receives oxygenated blood from the pulmonary veins. A) right atrium B) right ventricle C) left atrium D) left ventricle E) right auricle Answer: C Diff: 2 Page Ref: 574

4 Copyright © 2014 Pearson Education, Inc.

 

14) The heart chamber that receives blood from the superior vena cava, inferior vena cava, and coronary sinus. A) right atrium B) right ventricle C) left atrium D) left ventricle E) left auricle Answer: A Diff: 2 Page Ref: 574 15) The internal C-shaped crest of the right atrium which indicates the openings for the Superior vena cava and Inferior vena cava is A) ligamentum arteriosum. B) crista terminalis. C) trabeculae carneae. D) pectinate muscles. E) fossa ovalis. Answer: B Diff: 2 Page Ref: 566 16) Threadlike structures of the endocardium that prevent prolapse of the atrioventricular valves. A) ligamentum arteriosum B) pectinate muscles C) trabeculae carneae D) chordae tendineae E) fossa ovalis Answer: D Diff: 2 Page Ref: 566 17) The valve responsible for preventing backflow of blood from the lungs into the heart. A) tricuspid valve B) bicuspid valve C) aortic semilunar valve D) pulmonary semilunar valve E) pectinate muscles Answer: D Diff: 2 Page Ref: 566, 571 18) Heart valve with two cusps. A) mitral valve B) pulmonary semilunar valve C) aortic semilunar valve D) fossa ovalis E) atrioventricular bundle Answer: A Diff: 2 Page Ref: 570-571 5 Copyright © 2014 Pearson Education, Inc.

 

19) Contraction of these structures tightens the chordae tendineae, preventing valve prolapse. A) trabeculae carneae B) pectinate muscles C) crista terminalis D) papillary muscles E) atrioventricular bundle Answer: D Diff: 2 Page Ref: 571 20) Cells of the conducting system located between the AV node and bundle branches. A) trabeculae carneae B) pectinate muscles C) crista terminalis D) papillary muscles E) atrioventricular bundle Answer: E Diff: 2 Page Ref: 577-5787 21) Large cardiac cells of the conducting system embedded in the ventricular walls between the endocardium and myocardium. A) atrioventricular bundle B) atrioventricular branches C) subendocardial conducting network (Purkinje fibers) D) sinoatrial node E) atrioventricular node Answer: C Diff: 2 Page Ref: 577-578 22) Parasympathetic impulses to the SA node are transmitted on this cranial nerve. A) Glossopharyngeal nerve B) Accessory nerve—spinal part C) Vagus nerve D) Trigeminal nerve E) Hypoglossal nerve Answer: C Diff: 2 Page Ref: 578 23) Coronary artery that supplies the left atrium. A) circumflex artery B) left anterior descending artery C) marginal artery D) posterior descending artery E) pulmonary artery Answer: A Diff: 2 Page Ref: 579

6 Copyright © 2014 Pearson Education, Inc.

 

24) Death of heart musculature due to lack of oxygen. A) cardiac tamponade B) valve insufficiency C) heart block D) myocardial infarction E) ventricular fibrillation Answer: D Diff: 2 Page Ref: 580 25) Enlargement of the heart with progressive decline in pumping efficiency. A) cardiac tamponade B) congestive heart failure C) heart block D) myocardial infarction E) hypertrophic cardiomyopathy Answer: B Diff: 2 Page Ref: 580 26) The pericardial cavity lies between A) the fibrous pericardium and the parietal pericardium. B) the parietal pericardium and the visceral pericardium. C) the serous pericardium and the epicardium. D) the fibrous pericardium and the diaphragm. Answer: B Diff: 1 Page Ref: 565 27) How did the sinoatrial (SA) node most likely get its name? A) It is on the side of the atrium. B) It lies at the opening of the coronary sinus. C) It is damaged by sinus infections (head colds). D) It develops from the sinus venosus and lies in an atrium. Answer: D Diff: 3 Page Ref: 582-583 28) A penetrating stab wound to the heart wall that causes blood to leak into the pericardial cavity would result in A) cardiac tamponade. B) myocardial infarction. C) hypertrophic cardiomyopathy. D) endocarditis. Answer: A Diff: 2 Page Ref: 565

7 Copyright © 2014 Pearson Education, Inc.

 

29) The inner endothelial layer that lines the heart is the A) epicardium. B) pericardium. C) myocardium. D) endocardium. Answer: D Diff: 2 Page Ref: 565-566 30) Which of the following vessels does not carry oxygen-poor blood to the heart? A) the superior vena cava B) the inferior vena cava C) the pulmonary vein D) the coronary sinus Answer: C Diff: 2 Page Ref: 573-574 31) The superior corner of the right atrium of the heart is located at the A) costal cartilage of the sixth rib, a finger's width lateral to the sternum. B) costal cartilage of the third rib where it attaches to the sternum. C) fifth intercostal space along a line extending inferiorly from the midpoint of the clavicle. D) midpoint of the jugular notch. Answer: B Diff: 3 Page Ref: 573 32) The auricles are A) earlike flaps on the surface of the ventricles. B) earlike flaps on the surface of the atria. C) projections of the endothelium into the ventricles. D) modifications of the pectinate muscles on the inner surface of the atria. Answer: B Diff: 1 Page Ref: 566 33) What structures anchor the chordae tendineae? A) trabeculae carneae B) papillary muscles C) pectinate muscles D) semilunar valves Answer: B Diff: 1 Page Ref: 566 34) Which structure develops from the embryological chamber called the bulbus cordis? A) the left atrium B) the right ventricle C) the left ventricle D) the sinoatrial node Answer: B Diff: 2 Page Ref: 582-583 8 Copyright © 2014 Pearson Education, Inc.

 

35) Which of the following structures is not found in the left ventricle? A) the pectinate muscles B) the mitral valve C) the trabeculae carneae D) the papillary muscles Answer: A Diff: 2 Page Ref: 571 36) The right ventricle pumps blood into which vessel? A) the aorta B) the pulmonary vein C) the pulmonary trunk D) the superior vena cava Answer: C Diff: 1 Page Ref: 566 37) A drop of blood returning to the heart from the head region would enter the heart through which vessel? A) a pulmonary vein B) the inferior vena cava C) the superior vena cava D) the coronary sinus Answer: C Diff: 2 Page Ref: 566 38) The cusps of the valves of the heart are covered by A) epicardium. B) myocardium. C) endocardium. D) modified pericardium. Answer: C Diff: 1 Page Ref: 566 39) Semilunar valves are located A) between the atria and the ventricles. B) between the ventricles and the great arteries. C) between the great veins and the atria. D) only between the left ventricle and the aorta. Answer: B Diff: 1 Page Ref: 570-571

9 Copyright © 2014 Pearson Education, Inc.

 

40) A condition in which the ventricles are unable to pump blood efficiently because of rapid, random contraction of cardiac muscle fibers is called A) atrial fibrillation. B) ventricular fibrillation. C) pulmonary arterial hypertension. D) congestive heart failure. Answer: B Diff: 2 Page Ref: 582 41) At which corner point of the heart does one listen for the sound of the closing aortic semilunar valve? A) superior right B) inferior right C) superior left D) inferior left Answer: A Diff: 3 Page Ref: 573 42) What is the effect of the parasympathetic fibers carried by the vagus nerve? A) They speed up the heartbeat. B) They increase the force of cardiac contractions. C) They slow the heartbeat. D) The heartbeat is not influenced by the vagus nerve. Answer: C Diff: 1 Page Ref: 578 43) The crista terminalis can be used to locate all of the following structures except the A) opening of the coronary sinus. B) opening of the inferior vena cava. C) opening of the pulmonary veins. D) opening of the superior vena cava. Answer: C Diff: 3 Page Ref: 566 44) Which vessel returns most of the venous blood from the heart to the right atrium? A) the great cardiac vein B) the coronary sinus C) the anterior cardiac vein D) the posterior interventricular vein Answer: B Diff: 2 Page Ref: 566, 580

10 Copyright © 2014 Pearson Education, Inc.

 

45) During ventricular systole, blood is A) flowing from the systemic and pulmonary circuits into both the atria and ventricles. B) forced from the atria into the ventricles. C) forced from the ventricles into the aorta and pulmonary trunk. D) not flowing into or out of the heart. Answer: C Diff: 2 Page Ref: 573 46) Blood within the pulmonary veins returns to the A) right atrium. B) left atrium. C) right ventricle. D) left ventricle. Answer: B Diff: 2 Page Ref: 573-574 47) The term for pain caused by deficient blood delivery to the heart wall is A) ischemia. B) angina pectoris. C) myocardial infarct. D) pericarditis. Answer: B Diff: 2 Page Ref: 580 48) Stenosis of the mitral valve may cause blood to back up into the A) venae cavae. B) pulmonary circulation. C) right ventricle. D) coronary circulation. Answer: B Diff: 3 Page Ref: 572 49) Blood is carried to capillaries in the myocardium by way of the A) coronary sinus. B) fossa ovalis. C) coronary arteries. D) coronary veins. Answer: C Diff: 1 Page Ref: 579-580 50) Which of the following veins does not deliver blood directly to the right atrium? A) superior vena cava B) inferior vena cava C) coronary sinus D) the great cardiac veins Answer: D Diff: 2 Page Ref: 580 11 Copyright © 2014 Pearson Education, Inc.

 

51) The desmosome-like structures that attach adjacent cardiac muscle cells are called A) fasciae adherens. B) gap junctions. C) intercalated disks. D) T tubules. Answer: A Diff: 2 Page Ref: 575 52) Oxygen-poor blood returns to the heart and enters the A) right atrium. B) left atrium. C) left ventricle. D) right ventricle. Answer: A Diff: 1 Page Ref: 566 53) Which structure develops from the embryological chamber called the sinus venosus? A) sinoatrial (SA) node B) the right ventricle C) the left atrium D) the pulmonary trunk Answer: A Diff: 2 Page Ref: 582-583 54) The small cardiac vein is present on the A) right ventricle. B) left ventricle. C) right atrium. D) left atrium. Answer: A Diff: 2 Page Ref: 568 55) The epicardium is the same as the A) visceral layer of serous pericardium. B) pericardium. C) endocardium. D) fibrous pericardium. Answer: A Diff: 2 Page Ref: 565-566

12 Copyright © 2014 Pearson Education, Inc.

 

56) A specific coronary vessel that lies in the anterior interventricular sulcus is the A) anterior interventricular artery/Left anterior descending artery (LAD). B) middle cardiac vein. C) coronary sinus. D) circumflex artery. Answer: A Diff: 2 Page Ref: 579 57) The artery that nourishes the walls of the left atrium is the A) anterior interventricular. B) circumflex. C) posterior interventricular. D) right coronary. Answer: B Diff: 2 Page Ref: 579-580 58) The heart chamber with the thickest wall is the A) right atrium. B) left atrium. C) right ventricle. D) left ventricle. Answer: D Diff: 3 Page Ref: 575 59) A specific coronary vessel that lies in the coronary sulcus is the A) posterior interventricular artery. B) right coronary artery. C) small cardiac vein. D) right marginal artery. Answer: B Diff: 2 Page Ref: 579 60) The heart chamber that pumps oxygenated blood around the systemic circuit is the A) right atrium. B) left atrium. C) right ventricle. D) left ventricle. Answer: D Diff: 2 Page Ref: 563, 573-574 61) If the beating heart makes a "lub-dup" sound, the "dup" sound is caused by A) the apex of the heart hitting the anterior chest wall. B) a stenotic atrioventricular valve. C) the large force of the contracting ventricles. D) vibrations that result from the semilunar valves slamming shut. Answer: D Diff: 3 Page Ref: 571, 573 13 Copyright © 2014 Pearson Education, Inc.

 

62) The base of the aorta derives from which of these "original" heart chambers in the embryo? A) sinus venosus B) atrium C) ventricle D) bulbus cordis Answer: D Diff: 3 Page Ref: 582-583 63) Destruction of which structure will result in electrical signals traveling to only one ventricle? A) atrioventricular bundle B) bundle branch C) internodal pathway D) sinoatrial node Answer: B Diff: 2 Page Ref: 577-578 64) Clinically, the posterior interventricular artery is referred to as the A) left artery ascending. B) left artery descending. C) posterior descending artery. D) posterior ascending artery. Answer: C Diff: 2 Page Ref: 579-580 65) Of the three layers of the heart wall, the layer that contains the cardiac muscle is the A) epicardium. B) visceral layer of serous pericardium. C) myocardium. D) endocardium. Answer: C Diff: 1 Page Ref: 565-566 66) Cells of the subendocardial conducting network A) are larger and have fewer myofilaments than other cardiac cells. B) are nonconducting cells that electrically insulate the bundle branches of the interventricular septum. C) are pacemaker cells located in the SA node that initiate each heartbeat. D) are sensory cells that monitor the stretch of the myocardium to prevent overexpansion by high blood pressure. Answer: A Diff: 3 Page Ref: 577

14 Copyright © 2014 Pearson Education, Inc.

 

67) There is a foramen ovale in the skull and another one in the heart. The foramen ovale in the heart gives rise to the A) openings between the atria and ventricles. B) openings between the ventricles. C) fossa ovalis. D) aortic semilunar valve. Answer: C Diff: 2 Page Ref: 583 68) Insertion of a stent to treat coronary artery disease (CAD) A) involves grafting a portion of the saphenous vein that contains the stent into the occluded artery. B) is accomplished by laparoscopic incision at the jugular notch. C) occurs through a catheter inserted in the femoral artery. D) requires open heart surgery. Answer: C Diff: 2 Page Ref: 581 69) Pericarditis can lead to all of the following except A) excess fluid in the pericardial cavity. B) pericardial friction rub. C) adhesions. D) a myocardial infarction. Answer: D Diff: 2 Page Ref: 565 70) The "heartstrings" are A) cusps of the atrioventricular valves. B) chordae tendineae. C) trabeculae carneae. D) papillary muscles. Answer: B Diff: 1 Page Ref: 566 71) The semilunar valves are closed when A) the ventricles are contracting. B) the ventricles are relaxing. C) the atrioventricular valves are closed. D) atria are contracting. Answer: B Diff: 3 Page Ref: 571, 573

15 Copyright © 2014 Pearson Education, Inc.

 

72) The atrioventricular node is located in the A) right atrium, just inferior to the opening of the superior vena cava. B) inferior part of the interatrial septum. C) interventricular septum, near the heart base. D) walls of the ventricles, with the other Purkinje fibers. Answer: B Diff: 1 Page Ref: 577 73) Of the following heart chambers, which is most affected by hypertrophic cardiomyopathy? A) right atrium B) right ventricle C) left ventricle D) left atrium Answer: C Diff: 2 Page Ref: 575-566 74) The left ventricular wall of the heart is thicker than the right ventricular wall so that it can A) accommodate a greater volume of blood. B) expand the thoracic cage during diastole. C) pump blood with greater pressure. D) pump blood through a smaller valve. Answer: C Diff: 3 Page Ref: 575 75) During left ventricular systole, blood exits the heart to enter the A) aorta. B) pulmonary trunk. C) pulmonary vein. D) venae cavae. Answer: A Diff: 2 Page Ref: 573-574 76) To listen for the aortic semilunar valve on the chest wall, one would place the stethoscope in the A) second intercostal space to the right of the sternum. B) second intercostal space to the left of the sternum. C) fifth intercostal space inferior to the left nipple. D) fifth right intercostal space. Answer: A Diff: 3 Page Ref: 573

16 Copyright © 2014 Pearson Education, Inc.

 

77) Which of the following statements about fetal heart development is false? A) The four heart chambers first develop during the third trimester. B) The heart begins as a pair of tubes in the midline of the thorax. C) The heart develops from mesodermal mesenchyme. D) The two atria are connected by a foramen ovale until birth. Answer: A Diff: 2 Page Ref: 582-583 78) The tricuspid valve is closed A) while the ventricle is in diastole. B) by movement of blood from atrium to ventricle. C) while the atrium is contracting. D) when the ventricle is in systole. Answer: D Diff: 2 Page Ref: 573 79) Which of the following is not an age-related change in the heart? A) thinning of the valve cusps B) decline in cardiac reserve C) fibrosis of cardiac muscle D) atherosclerosis Answer: A Diff: 3 Page Ref: 583-584 80) In the pericardial sac, the ________ lies directly deep to the fibrous pericardium. A) pericardial cavity B) visceral layer of serous pericardium C) parietal layer of the serous pericardium D) epicardium Answer: C Diff: 2 Page Ref: 565-566 81) This blood vessel is located in the anterior interventricular sulcus. A) anterior cardiac vein B) great cardiac vein C) middle cardiac vein D) small cardiac vein Answer: B Diff: 2 Page Ref: 580 82) The cardiac centers that control heart rate are located in the A) cerebral cortex of the temporal lobe. B) medulla oblongata of the brain stem. C) pituitary gland of the diencephalon. D) thoracic segments of the spinal cord. Answer: B Diff: 2 Page Ref: 578 17 Copyright © 2014 Pearson Education, Inc.

 

19.2 True/False Questions 1) Sounds of the aortic valve are heard in the second intercostal space at the right sternal margin. Answer: TRUE Diff: 3 Page Ref: 573 2) The correct sequence of heart wall layers from superficial to deep is epicardium, endocardium, and myocardium. Answer: FALSE Diff: 1 Page Ref: 565-566 3) Heart block interferes with the ability of the ventricles to receive the pacing impulses of the conducting system. Answer: TRUE Diff: 2 Page Ref: 578 4) Papillary muscles are horizontal ridges in the walls of the atrium. Answer: FALSE Diff: 1 Page Ref: 566, 569-570 5) Formation of the interatrial and interventricular septa occurs during the second month of embryonic development. Answer: TRUE Diff: 3 Page Ref: 582-583 6) Prolapse of the atrioventricular valves is prevented by the chordae tendineae. Answer: TRUE Diff: 2 Page Ref: 570-571 7) Oxygen-rich blood returning from the lungs enters the left atrium through two right and two left pulmonary veins. Answer: TRUE Diff: 2 Page Ref: 574 8) Contraction of the heart proceeds first on the right side of the heart and second on the left. Answer: FALSE Diff: 1 Page Ref: 573-574 9) The middle cardiac vein lies alongside the anterior interventricular artery. Answer: FALSE Diff: 3 Page Ref: 580 10) The electrical event that begins each heartbeat occurs at the sinoatrial (SA) node. Answer: TRUE Diff: 2 Page Ref: 577 18 Copyright © 2014 Pearson Education, Inc.

 

11) Contraction of the ventricles begins at the apex and proceeds superiorly. Answer: TRUE Diff: 2 Page Ref: 577 12) Parasympathetic fibers innervate the SA node, AV node, and cardiac musculature. Answer: FALSE Diff: 3 Page Ref: 578 13) The right and left coronary arteries arise from the descending aorta. Answer: FALSE Diff: 2 Page Ref: 579-580 14) The fibrous skeleton of the heart forces the transmission of electrical signals from the atria to the ventricles via the atrioventricular bundle. Answer: TRUE Diff: 2 Page Ref: 577 15) Atherosclerosis in the marginal artery may cause ischemia in the left side of the heart. Answer: FALSE Diff: 3 Page Ref: 579 19.3 Short Answer Questions 1) Inflammation of the heart's myocardium is called ________. Answer: myocarditis Diff: 2 Page Ref: 584 2) The sequence of pericardial layers from superficial to deep is the fibrous, ________, and visceral pericardium. Answer: parietal Diff: 2 Page Ref: 565-566 3) The right atrium receives blood from the superior vena cava, inferior vena cava, and ________. Answer: coronary sinus Diff: 2 Page Ref: 566 4) A heart valve that fails to close properly is considered to be ________ or to exhibit insufficiency. Answer: incompetent Diff: 2 Page Ref: 572 5) Irregular ridges in the walls of the ventricles are called ________. Answer: trabeculae carneae Diff: 2 Page Ref: 566

19 Copyright © 2014 Pearson Education, Inc.

 

6) The walls of the ________ are thicker so as to push blood through the systemic circuit. Answer: left ventricle Diff: 2 Page Ref: 575 7) The ________ valve prevents backflow of blood from the left ventricle to the left atrium. Answer: mitral, left atrioventricular, or bicuspid Diff: 2 Page Ref: 570 8) The atrioventricular valves cannot be inverted because attachment of ________. Answer: chordae tendineae Diff: 2 Page Ref: 570 9) The first heart sounds of "lub-dup" are produced by the closing of the right and left ________. Answer: atrioventricular valves Diff: 2 Page Ref: 570 10) The ________ of the heart prevents overdilation of the valve openings. Answer: cardiac skeleton Diff: 2 Page Ref: 571 11) The ________ are subdivisions of the atrioventricular bundle located within the upper half of the interventricular septum. Answer: bundle branches or crura Diff: 2 Page Ref: 577 12) The ________ artery supplies the left atrium and the posterior part of the left ventricle. Answer: circumflex artery Diff: 2 Page Ref: 579-580 13) Gap junctions and fasciae adherens are associated with ________ that join adjacent cardiac muscle cells. Answer: intercalated disks Diff: 2 Page Ref: 575 14) The technique called ________ uses a balloon catheter to expand stenotic coronary arteries. Answer: angioplasty Diff: 2 Page Ref: 581 15) Uncoordinated contractions of the ventricle, known as ________, result from irregularities in conducting system behavior. Answer: fibrillation Diff: 2 Page Ref: 582

20 Copyright © 2014 Pearson Education, Inc.

 

19.4 Essay Questions 1) Describe how the arrangement of cardiac muscle bundles "squeeze" the blood from the heart. Answer: The muscle bundles of the atria are circular. Contraction of these bundles forces the blood inferiorly through the AV valves into the ventricles. The bundles of the ventricles are arranged as a spiral. These bundles contract in a twisting fashion, beginning at the apex and wringing the blood superiorly through the semilunar valves of the pulmonary trunk and aorta. Diff: 2 Page Ref: 566 2) Compare the difference between incompetent and stenotic heart valves. Answer: Incompetent heart valves are valves that do not close properly and allow blood to leak past the cusps. In contrast, stenotic heart valves do not open properly because they have become stiff and rigid as a result of accumulation of calcium deposits. Although both conditions differ in their cause, their common effect is to weaken the heart and diminish its efficiency. Diff: 2 Page Ref: 572 3) Describe the function of the cardiac skeleton of the heart. Answer: The cardiac skeleton of the heart anchors the heart valves in position and prevents their openings from overdilating as blood flows through. The skeleton also anchors the cardiac muscle bundles and keeps the heart from tearing itself apart as different chambers contract. Finally, the cardiac skeleton serves as an electrical insulator that limits spread of the electrical depolarizations of the atria to the ventricles. Diff: 2 Page Ref: 571, 573 4) Identify the change in blood flow through the heart in the condition of a ventricular septal defect. Answer: A ventricular septal defect allows blood to flow directly from the right to left ventricles. When this happens, some of the blood from the right atrium has bypassed the pulmonary circuit and therefore remains deoxygenated. This means that blood pumped from the left ventricle through the systemic circuit consists of both oxygenated and deoxygenated blood. Diff: 3 Page Ref: 583 5) Describe the histological features of cardiac cells that allow for coordinated contractions. Answer: Individual cells of the myocardium are connected by intercalated disks. These disks have fingerlike projections that interlock with those of the adjacent cell. Within these interlocking disks, fasciae adherens are desmosome-like structures that transmit the contractile force of one cell to another. Additionally, gap junctions in the disks allow for the flow of activating ions (calcium and sodium) between cells so that synchronized contractions occur. Diff: 2 Page Ref: 575-576

21 Copyright © 2014 Pearson Education, Inc.

 

Human Anatomy, 7e (Marieb/Mitchell/Smith) Chapter 20 Blood Vessels 20.1 Multiple Choice Questions

Figure 20.1 Use the diagram above to answer the following questions. 1) Identify the letter that indicates the layer common to all blood vessels regardless of their size. A) A B) B C) C D) D E) E Answer: A Diff: 2 Page Ref: 589-590

1 Copyright © 2014 Pearson Education, Inc.

 

2) Identify the letter that is indicating endothelial cells. A) A B) B C) C D) D E) E Answer: E Diff: 2 Page Ref: 589-590 3) Identify the letter that indicates a connective tissue layer consisting of longitudinal collagen fibers. A) A B) B C) C D) D E) E Answer: D Diff: 2 Page Ref: 589-590 4) Identify the letter that indicates the blood vessel layer that is comprised of circular and longitudinal smooth muscle. A) A B) B C) C D) D E) E Answer: C Diff: 2 Page Ref: 589-590 5) Identify the letter that indicates the subendothelial layer associated with larger blood vessels. A) A B) B C) C D) D E) E Answer: B Diff: 2 Page Ref: 589-590

2 Copyright © 2014 Pearson Education, Inc.

 

Figure 20.2 Use the diagram above to answer the following questions. 6) Identify the letter that indicates the common carotid arteries. A) A B) B C) C D) D E) E Answer: C Diff: 2 Page Ref: 598 7) Identify the letter that indicates the common iliac artery. A) A B) B C) C D) D E) E Answer: A Diff: 2 Page Ref: 598 3 Copyright © 2014 Pearson Education, Inc.

 

8) Identify the letter that indicates the brachial artery. A) A B) B C) C D) D E) E Answer: D Diff: 2 Page Ref: 598 9) Identify the letter that indicates the renal artery as it branches from the Abdominal aorta. A) A B) B C) C D) D E) E Answer: B Diff: 1 Page Ref: 598 10) Identify the letter that indicates one of the primary arteries that contributes to the superficial palmar arch-the ulnar artery. A) A B) B C) C D) D E) E Answer: E Diff: 2 Page Ref: 598, 603

4 Copyright © 2014 Pearson Education, Inc.

 

Figure 20.3 Use the diagram above to answer the following questions. 11) Identify the letter that indicates the vessel that is easy to find in most people and is used to obtain blood. A) A B) B C) C D) D E) E Answer: D Diff: 2 Page Ref: 609, 613

5 Copyright © 2014 Pearson Education, Inc.

 

12) Identify the letter that indicates the longest vein in the body. A) A B) B C) C D) D E) E Answer: E Diff: 2 Page Ref: 609, 616 13) Identify the letter that indicates the vessel that arises from the union of the left and right brachiocephalic veins. A) A B) B C) C D) D E) E Answer: A Diff: 2 Page Ref: 609-610 14) Identify the letter that indicates the cephalic vein. A) A B) B C) C D) D E) E Answer: C Diff: 2 Page Ref: 609, 613 15) Identify the letter that indicates the internal jugular vein. A) A B) B C) C D) D E) E Answer: B Diff: 2 Page Ref: 609, 611 16) Layer of blood vessels innervated by sympathetic vasomotor fibers. A) tunica intima B) subendothelial layer C) tunica media D) tunica externa E) vasa vasorum Answer: C Diff: 2 Page Ref: 589

6 Copyright © 2014 Pearson Education, Inc.

 

17) Capillaries consist of only this layer. A) tunica intima B) subendothelial layer C) tunica media D) tunica externa E) vasa vasorum Answer: A Diff: 2 Page Ref: 590, 592 18) Structure that regulates blood flow into true capillaries. A) metarteriole B) thoroughfare channel C) precapillary sphincter D) sinusoids E) fenestrations Answer: C Diff: 2 Page Ref: 593 19) Wide leaky capillaries found in bone marrow and spleen. A) sinusoids B) fenestrated capillaries C) continuous capillaries D) metarterioles E) none of the above Answer: A Diff: 2 Page Ref: 593 20) A "coming together" of alternate pathways of blood vessels. A) thoroughfare channel B) vasa vasorum C) metarterioles D) vascular anastomosis E) converging veins Answer: D Diff: 2 Page Ref: 595 21) Precapillary sphincters allow blood to leave this structure and enter true capillaries. A) sinusoids B) fenestrated capillaries C) continuous capillaries D) metarterioles E) none of the above Answer: D Diff: 2 Page Ref: 593

7 Copyright © 2014 Pearson Education, Inc.

 

22) Collective name for the structures that drain the cranium. A) vascular anastomosis B) dural sinuses C) internal jugular vein D) cavernous sinuses E) inferior vena cava Answer: B Diff: 2 Page Ref: 610 23) Present in most capillaries, these structures are absent in those of the blood-brain barrier. A) tight junctions B) endothelial cells C) basement membrane D) intercellular clefts E) pericytes Answer: D Diff: 2 Page Ref: 593 24) Vessels of the small intestines, renal glomerulus, and synovial membranes that allow passage of fluid and solutes through "windows" in the endothelium. A) sinusoids B) fenestrated capillaries C) continuous capillaries D) metarterioles E) none of the above Answer: B Diff: 2 Page Ref: 593 25) Microvasculature that provides nourishment to the outer walls of the aorta. A) tunica intima B) subendothelial layer C) tunica media D) tunica externa E) vasa vasorum Answer: E Diff: 2 Page Ref: 589 26) Which layer of blood vessels contains smooth muscle tissue? A) tunica intima B) tunica media C) tunica externa D) tunica adventitia Answer: B Diff: 1 Page Ref: 589

8 Copyright © 2014 Pearson Education, Inc.

 

27) Most small molecules pass through a capillary wall through which route? A) via direct diffusion through the endothelium B) via pinocytotic vesicles C) through intercellular clefts D) through tight junctions Answer: C Diff: 1 Page Ref: 592 28) Functionally, there are no valves in arteries (as opposed to in veins) because A) valves direct blood only toward the heart and arterial blood passes away from the heart. B) valves would tear apart from the high arterial pressure. C) arteries get more atherosclerosis, so valves would cause lethal blood clotting. D) the blood pressure in arteries is high enough that there is no backflow of blood. Answer: D Diff: 2 Page Ref: 589-591, 595 29) Blood pressure is highest in the A) elastic arteries. B) arterioles. C) veins. D) capillaries. Answer: A Diff: 1 Page Ref: 589 30) The hepatic portal system has two distinct capillary beds separated by a portal vein. What are the functions of these two capillary beds? A) The first picks up digested nutrients, and the second delivers these nutrients to liver cells. B) The first nourishes the digestive tube, and the second picks up nutrients from the digestive tube. C) The first provides oxygen to the liver, and the second picks up nutrients from the liver. D) The first picks up toxins from the liver, and the second delivers them to the digestive tube for detoxification. Answer: A Diff: 2 Page Ref: 615-616 31) An aneurysm is A) a rupture in an artery. B) a buildup of fatty deposits on an arterial wall. C) a sac-like widening or outpocketing of an artery. D) a stroke. Answer: C Diff: 1 Page Ref: 618

9 Copyright © 2014 Pearson Education, Inc.

 

32) The pulse of the facial artery is palpated A) anterior to the auricle of the ear near the temple. B) anterior to the masseter muscle at the inferior margin of the mandible. C) anterior to the sternocleidomastoid. D) in the anterior triangle of the neck. Answer: B Diff: 2 Page Ref: 601 33) A blood vessel that ranges from 0.3 mm to about 1 cm in diameter and has a large tunica media relative to the size of the lumen is A) an elastic artery. B) a muscular artery. C) an arteriole. D) a capillary. Answer: B Diff: 1 Page Ref: 591 34) In a capillary bed, relaxation of the precapillary sphincters causes more blood to flow A) into the thoroughfare channels. B) into the arterioles. C) through the true capillaries. D) through the metarterioles. Answer: C Diff: 2 Page Ref: 593-594 35) The dorsalis pedis artery is located by A) finding the head of the fibula and palpating inferior to its neck. B) palpating between the first and second metatarsal. C) placing the fingers behind the knee. D) placing a finger behind the medial malleolus. Answer: B Diff: 2 Page Ref: 608 36) What artery enters the skull through the foramen spinosum and supplies the inner surface of the parietal bone, dura mater, and parts of the temporal bone? A) internal carotid artery B) middle cerebral artery C) middle meningeal artery D) basilar artery Answer: C Diff: 2 Page Ref: 601

10 Copyright © 2014 Pearson Education, Inc.

 

37) Two large (wide) arteries that have relatively superficial locations and are often wounded are the A) aorta and the popliteal artery. B) right and left coronary arteries. C) brachial artery and posterior intercostal arteries. D) common carotid artery and the femoral artery (in the superior thigh). Answer: D Diff: 3 Page Ref: 601, 608 38) If a physician cannot feel a pulse in the popliteal fossa, the ________ artery is most likely narrowed by atherosclerosis. A) dorsalis pedis B) femoral C) fibular D) greater saphenous Answer: B Diff: 2 Page Ref: 608 39) Of the following, the only unpaired dural sinus is the A) cavernous sinus. B) superior sagittal sinus. C) transverse sinus. D) carotid sinus. Answer: B Diff: 1 Page Ref: 610-611 40) Which vessel is missing in the following statement? "Tracing venous blood from the inferior left side of the posterior abdominal wall to the heart, we find that blood enters the posterior intercostal veins, the hemiazygos vein, the superior vena cava, and the right atrium." A) the azygos vein B) the hepatic portal vein C) the inferior vena cava D) the right brachiocephalic vein Answer: A Diff: 2 Page Ref: 611 41) The foramen ovale in the heart normally closes A) in the 2-month fetus. B) in the 7-month fetus. C) shortly after birth. D) never. Answer: C Diff: 1 Page Ref: 622-23

11 Copyright © 2014 Pearson Education, Inc.

 

42) Which vessel is most commonly used to bypass a damaged coronary artery in coronary bypass surgery? A) azygos vein B) great saphenous vein C) femoral artery D) internal carotid artery Answer: B Diff: 1 Page Ref: 617 43) What vessel in the fetus connects the pulmonary trunk to the aortic arch so that most of the blood bypasses the immature lungs? A) ductus venosus B) foramen ovale C) ductus arteriosus D) umbilical vein Answer: C Diff: 2 Page Ref: 622 44) Which of the following statements about arteries is false? A) Arterial walls are thicker than venous walls. B) Arteries have a smaller lumen than veins of similar size. C) Arteries carry oxygenated blood to the heart. D) Arteries have more elastin than veins. Answer: C Diff: 1 Page Ref: 589 45) Which branch (or branches) of the abdominal aorta supplies the stomach? A) celiac trunk B) superior mesenteric artery C) inferior phrenic arteries D) suprarenal arteries Answer: A Diff: 2 Page Ref: 603-604 46) The cerebral arterial circle forms a loop around which structures? A) the great vessels at the base of the heart B) the internal and external carotid arteries C) the cerebellum D) the pituitary gland and the optic chiasma Answer: D Diff: 2 Page Ref: 601

12 Copyright © 2014 Pearson Education, Inc.

 

47) A common theme to the development of atherosclerosis is A) a ballooning out of the vessel walls. B) a failure of the venous valves resulting in engorged and twisted vessels. C) an accumulation of glycoproteins in the basement membrane of capillaries. D) an inflammatory response to a damaged endothelium. Answer: D Diff: 2 Page Ref: 619 48) The abdominal aorta divides at its distal end into which arteries? A) the femoral arteries B) the internal iliac arteries C) the external iliac arteries D) the common iliac arteries Answer: D Diff: 2 Page Ref: 606-607 49) Which of the following is most likely to become a varicose vein? A) the femoral vein B) the saphenous vein C) the popliteal vein D) the fibular (peroneal) vein Answer: B Diff: 2 Page Ref: 616-617 50) Fenestrated capillaries A) are located in the central nervous system. B) have pores in their walls. C) permit the movement of very few molecules. D) occur in most of the organs of the body. Answer: B Diff: 1 Page Ref: 593 51) The correct proximal to distal sequence of the three vessels branching from the aortic arch is A) brachiocephalic, left common carotid, left subclavian. B) brachiocephalic, left subclavian, left common carotid. C) left common carotid, left subclavian, brachiocephalic. D) left subclavian, left common carotid, brachiocephalic. Answer: A Diff: 2 Page Ref: 599 52) What prevents the backflow of blood in veins? A) valves B) the narrowed lumen C) thick smooth muscle and elastic layers D) increased blood pressure Answer: A Diff: 1 Page Ref: 595 13 Copyright © 2014 Pearson Education, Inc.

 

53) The largest molecules that pass through the walls of typical capillaries are thought to use which route? A) direct diffusion through the endothelium B) pinocytotic vesicles C) intercellular clefts D) tight junctions Answer: B Diff: 3 Page Ref: 593 54) The internal carotid artery branches to form the A) anterior cerebral, middle cerebral, and ophthalmic arteries. B) facial, maxillary, and superficial temporal arteries. C) middle meningeal and middle cerebral arteries. D) posterior cerebral and posterior communicating arteries. Answer: A Diff: 2 Page Ref: 601 55) The major cause of atherosclerosis is due to the A) formation of atheromas. B) destruction of valves in veins. C) lack of formation of anastomosis between vessels. D) destruction of elastic fibers in artery walls. Answer: A Diff: 3 Page Ref: 619 56) Most systemic venous blood is both oxygen-poor and nutrient-poor. However, systemic venous blood that is not oxygen-poor and is nutrient-rich occurs in A) the renal vein. B) superficial veins of the limbs. C) the hepatic portal vein. D) the pulmonary veins. Answer: C Diff: 2 Page Ref: 613-615 57) The ________ delivers arterial blood to the rotator cuff muscles and thyroid gland. A) axillary artery B) costocervical trunk C) thyrocervical trunk D) vertebral artery Answer: C Diff: 2 Page Ref: 601

14 Copyright © 2014 Pearson Education, Inc.

 

58) The diameter of a typical capillary is similar to that of A) a venule. B) a sinusoid. C) an erythrocyte. D) a fat cell. Answer: C Diff: 2 Page Ref: 592 59) Which of the following statements about arterioles is false? A) They regulate the flow of blood to capillary beds through contraction and relaxation of the tunica media. B) They redirect blood flow in a sympathetic response to skeletal muscle. C) They can lead into metarterioles. D) They have the largest content of smooth muscle in their tunica media. Answer: D Diff: 2 Page Ref: 591-592 60) Systemic venous blood that is oxygen-poor but contains the lowest concentration of nitrogenous wastes occurs in the A) renal veins. B) hepatic portal vein. C) pulmonary veins. D) umbilical veins of the fetus. Answer: A Diff: 3 Page Ref: 613 61) A preferred site to insert intravenous catheters is into the A) brachial vein. B) dorsal venous network of the hand. C) great saphenous vein. D) superficial palmar venous arch of the hand. Answer: B Diff: 2 Page Ref: 613 62) Which artery arises from the inferior part of the abdominal aorta and supplies the distal half of the large intestine? A) gonadal artery B) median sacral artery C) superior phrenic artery D) inferior mesenteric artery Answer: D Diff: 1 Page Ref: 616

15 Copyright © 2014 Pearson Education, Inc.

 

63) The main arteries of the sole of the foot—the medial and lateral plantar arteries—arise behind the ankle from which artery? A) posterior tibial B) fibular C) saphenous D) dorsalis pedis Answer: A Diff: 1 Page Ref: 608 64) A dural sinus that contains a major artery and some cranial nerves within it is the A) superior sagittal. B) inferior sagittal. C) cavernous. D) transverse. Answer: C Diff: 2 Page Ref: 610-611 65) Which vessel is missing from the following statement? "Tracing blood that drains from the large intestine, we find that blood drains from the distal colon is collected in the inferior mesenteric vein, merges with the splenic vein then directed to the hepatic portal vein, the liver sinusoids, and the inferior vena cava." A) celiac vein B) umbilical vein C) hepatic vein D) azygos vein Answer: C Diff: 2 Page Ref: 613-616 66) In the adult, the hepatic portal system carries nutrients absorbed from the digestive tract to the liver. In the fetus, nutrients are absorbed at the placenta, and the vessel that carries these nutrients to the liver is the A) hepatic portal vein. B) placental vein. C) umbilical vein. D) internal iliac vein. Answer: C Diff: 2 Page Ref: 622-623 67) The right suprarenal and gonadal veins drain into the inferior vena cava, whereas the left suprarenal and gonadal veins drain into the A) superior vena cava. B) other side of the inferior vena cava. C) hepatic portal system. D) left renal vein. Answer: D Diff: 3 Page Ref: 613 16 Copyright © 2014 Pearson Education, Inc.

 

68) Phlebitis is A) inflammation of a vein. B) a condition characterized by excessively leaky capillaries. C) cancer of the tunica intima. D) ballooning of an artery. Answer: A Diff: 1 Page Ref: 623 69) By definition, veins are A) vessels that carry blood toward the heart. B) vessels that always carry nutrient-poor blood. C) the only vessels that lead from capillaries. D) vessels that carry oxygen-poor blood. Answer: A Diff: 1 Page Ref: 589, 594 70) Which body tissues lack capillaries? A) the myocardium and epicardium of the heart B) tendons and ligaments C) the lens and the cornea D) bones Answer: C Diff: 1 Page Ref: 593 71) Which arteries connect the basilar artery and the internal carotid artery forming the posterior aspect of the cerebral arterial circle? A) genicular arteries that encircle the knee B) posterior communicating arteries C) ulnar artery that encircles the elbow joint D) anterior communicating arteries Answer: B Diff: 2 Page Ref: 601 72) The extensor muscles of the forearm are supplied by which artery? A) radial B) posterior interosseous C) ulnar D) deep palmar arch Answer: B Diff: 2 Page Ref: 603

17 Copyright © 2014 Pearson Education, Inc.

 

73) The lumbar veins drain the inferior posterior abdominal wall and direct oxygen-poor blood into the A) superior vena cava. B) internal iliac vein. C) external iliac vein. D) inferior vena cava. Answer: D Diff: 2 Page Ref: 613 74) Which of the following is not a branch of the celiac trunk? A) left gastric artery B) common hepatic artery C) sigmoidal artery D) splenic artery Answer: C Diff: 2 Page Ref: 604-605 75) The right gonadal vein drains into the A) inferior vena cava. B) internal iliac vein. C) lumbar vein. D) renal vein. Answer: A Diff: 3 Page Ref: 613 76) The circulatory route that runs from the digestive tract to the liver is called A) coronary circulation. B) pulmonary circulation. C) hepatic portal circulation. D) cerebral circulation. Answer: C Diff: 2 Page Ref: 613-616 77) Blood passing through the fetal ductus arteriosus bypasses the A) lungs, left atrium, and ventricle. B) pulmonary trunk and lungs. C) right atrium and ventricle. D) right ventricle, pulmonary trunk, and lungs. Answer: A Diff: 3 Page Ref: 622-623

18 Copyright © 2014 Pearson Education, Inc.

 

20.2 True/False Questions 1) All types of blood vessels contain a tunica intima. Answer: TRUE Diff: 1 Page Ref: 589 2) All arteries carry oxygen-rich blood, whereas veins carry oxygen-poor blood. Answer: FALSE Diff: 2 Page Ref: 589 3) Systemic blood pressure is regulated by adjusting the diameter of arterioles. Answer: TRUE Diff: 2 Page Ref: 591-592 4) In metabolically active tissues, blood is present in metarterioles, and precapillary sphincters are constricted. Answer: FALSE Diff: 2 Page Ref: 593 5) Postcapillary venules function much like true capillaries. Answer: TRUE Diff: 2 Page Ref: 594 6) The pulse of the posterior tibial artery is palpated behind the knee. Answer: FALSE Diff: 2 Page Ref: 608 7) Veins have less elastin in their walls than do arteries. Answer: TRUE Diff: 2 Page Ref: 595 8) Arterioles and venules have a vasa vasorum to provide nutrients to their outer walls, whereas the inner walls receive nutrients from blood in the lumen. Answer: FALSE Diff: 2 Page Ref: 589 9) The middle and posterior cerebral arteries are connected by the posterior communicating arteries. Answer: TRUE Diff: 2 Page Ref: 601 10) The internal iliac arteries supply blood both to the pelvic organs and to the lower limbs. Answer: TRUE Diff: 2 Page Ref: 607

19 Copyright © 2014 Pearson Education, Inc.

 

11) The inferior vena cava ascends on the left side of the vertebral bodies and to the left of the abdominal aorta. Answer: FALSE Diff: 2 Page Ref: 610 12) Muscular arteries regulate blood flow to organs and groups of organs. Answer: TRUE Diff: 2 Page Ref: 591 13) The elastic arteries are the largest arteries near the heart. Answer: TRUE Diff: 1 Page Ref: 589-591 14) The saphenous vein is not paired with an artery with the same name. Answer: TRUE Diff: 2 Page Ref: 616-617 15) Paired fetal umbilical veins carry blood from the fetus to the placenta, whereas the unpaired umbilical artery returns blood from the placenta to the fetus. Answer: FALSE Diff: 2 Page Ref: 622 20.3 Short Answer Questions 1) The ________ pulse can be palpated near the temple anterior to the auricle of the ear. Answer: temporal Diff: 1 Page Ref: 601 2) The ________ artery descends along the arcuate line of the ilium and passes below the inguinal ligament. Answer: external iliac Diff: 2 Page Ref: 607-608 3) In a capillary bed, blood enters by a terminal arteriole, continues through a metarteriole, and exits to a venule by means of a(n) ________. Answer: thoroughfare channel Diff: 2 Page Ref: 593-594 4) The ________ vein descends through the transverse foramina of the first six cervical vertebrae. Answer: vertebral Diff: 2 Page Ref: 611 5) Capillaries that allow a high degree of flow between the blood and tissue fluid usually have pores called ________ in their membranes. Answer: fenestrations Diff: 2 Page Ref: 593 20 Copyright © 2014 Pearson Education, Inc.

 

6) The blood-brain barrier consists of capillaries with complete ________ and no intercellular clefts. Answer: tight junctions Diff: 2 Page Ref: 593 7) The presence of ________ in the veins and the action of skeletal muscle pressing on the walls ensure that blood return to the heart is in one direction. Answer: valves Diff: 2 Page Ref: 595 8) The largest blood vessels near the heart have their own blood supply network called the ________. Answer: vasa vasorum Diff: 2 Page Ref: 589 9) The ________ branches to form the right subclavian and right common carotid arteries. Answer: brachiocephalic trunk Diff: 2 Page Ref: 600-601 10) Cirrhosis of the liver causes backup of blood in the sinusoids, resulting in a condition known as ________ hypertension. Answer: portal Diff: 2 Page Ref: 616 11) Venous blood from the right posterior intercostal veins (except the first intercostal space) flows to the unpaired ________ vein and on to the superior vena cava. Answer: azygos Diff: 2 Page Ref: 611 12) Venous blood from the hands can be drained towards the heart through the radial, ulnar, median, cephalic, and ________ veins. Answer: basilic Diff: 2 Page Ref: 612-613 13) ________ channels, also known as arterial anastomoses, provide alternative pathways of blood to joints. Answer: Collateral Diff: 3 Page Ref: 595 14) Blood flow to the posterior brain follows the following sequence: subclavian artery, vertebral artery, ________, posterior cerebral arteries. Answer: basilar artery Diff: 3 Page Ref: 601

21 Copyright © 2014 Pearson Education, Inc.

 

15) Veins with compromised valves are called ________ veins. Answer: varicose Diff: 2 Page Ref: 595-596 20.4 Essay Questions 1) Identify the superficial veins of the forearm and arm, and describe their anatomical location. Answer: The superficial veins include the cephalic, basilic, median, and median cubital veins. The cephalic vein and basilic veins arise from the dorsal venous network. The cephalic ascends the lateral forearm, whereas the basilic ascends the medial aspect. The median cubital vein bridges these two veins; it runs obliquely across the cubital fossa from the cephalic to a more proximal union with the basilic vein. The median vein ascends the center of the forearm before passing the medial side of the cubital fossa. Diff: 2 Page Ref: 612-613 2) Describe how blood flow is regulated (a) to whole organs, (b) within tissues, and (c) to capillary beds. Answer: Blood flow to groups of organs or to individual organs is regulated by the muscular arteries. Sympathetic vasomotor fibers innervating their walls constrict or dilate the arteries to divert or enhance blood flow. A similar regulatory system occurs within tissues via vasoconstriction of arterioles. Sympathetic stimulation can cause vasoconstriction of arterioles to increase systemic blood pressure. Alternatively, local chemical conditions within metabolically active tissues can cause local vasodilation of the arterioles. These chemical factors also cause relaxation of the precapillary sphincters allowing greater perfusion of capillary beds within the tissues. Diff: 2 Page Ref: 589-592 3) Define and explain the significance of vascular anastomoses. Answer: A vascular anastomosis is a "coming together" of either arteries or veins. Vascular anastomoses are alternative pathways for blood flow. These pathways are called collateral channels. Arterial anastomoses often occur around joints providing alternative pathways for blood to flow when movement of a joint impinges upon flow through other vessels. The anastomosis ensures adequate perfusion of the tissue. Venous anastomoses provide multiple pathways for drainage. Blockage of a single vein rarely blocks blood flow or leads to tissue death. Diff: 2 Page Ref: 595-596 4) Define the term portal system, and describe the significance of portal-systemic anastomoses. Answer: A portal system consists of two separate capillary beds between the arterial and venous ends of the circuit. The capillary beds are joined to each other in sequence by a portal vein. In the hepatic portal system, destruction of the hepatic sinusoids can lead to blockage of blood flow through the liver, resulting in increased blood pressure throughout the liver and GI tract. Venous anastomoses along the GI tract provide a "safety valve" through which venous blood can drain, bypassing the blocked liver and relieving the hypertension. Diff: 3 Page Ref: 613-616

22 Copyright © 2014 Pearson Education, Inc.

 

5) Describe the function of the fetal shunts from the pulmonary circuit. Answer: The foramen ovale is a shunt between the right and left atria of the heart. This shunt allows blood to bypass the entire pulmonary circuit. Only about half of the blood entering the right atrium is diverted through the foramen ovale. Much of the remainder of the blood that proceeds through the right ventricle and into the pulmonary trunk then enters the second shunt. This ductus arteriosus connects the pulmonary trunk directly to the arch of the aorta. This blood then continues to the systemic circuit. Only a small quantity of the initial blood entering the heart goes to the developing lungs. Diff: 2 Page Ref: 622-623

23 Copyright © 2014 Pearson Education, Inc.

 

Human Anatomy, 7e (Marieb/Mitchell/Smith) Chapter 21 The Lymphatic and Immune Systems 21.1 Multiple Choice Questions

Figure 21.1 Use the diagram above to answer the following questions. 1) Identify the letter that indicates the unique arrangement of endothelial cells that comprise lymphatic capillaries. A) A B) B C) C D) D E) E Answer: E Diff: 2 Page Ref: 628-629 2) Identify the letter that indicates collagen fibers that anchor lymphatic capillaries to surrounding connective tissue. A) A B) B C) C D) D E) E Answer: D Diff: 2 Page Ref: 628-629

1 Copyright © 2014 Pearson Education, Inc.

 

3) Identify the letter that indicates the smallest lymphatic vessel that collects lymph in the peripheral tissue. A) A B) B C) C D) D E) E Answer: B Diff: 1 Page Ref: 628-629 4) Identify the letter that indicates a vessel that carries oxygenated blood to peripheral tissues. A) A B) B C) C D) D E) E Answer: C Diff: 2 Page Ref: 628-629 5) Identify the letter that indicates a vessel that carries deoxygenated blood away from peripheral tissues. A) A B) B C) C D) D E) E Answer: A Diff: 2 Page Ref: 628-629

2 Copyright © 2014 Pearson Education, Inc.

 

Figure 21.2 Use this illustration to answer the following questions.

6) Identify the letter that indicates lymph nodes that direct lymph fluid to the jugular trunks. A) A B) B C) C D) D E) E Answer: B Diff: 2 Page Ref: 630

3 Copyright © 2014 Pearson Education, Inc.

 

7) Identify the letter that indicates the large lymph vessel that drains three-quarters of the body. A) A B) B C) C D) D E) E Answer: C Diff: 2 Page Ref: 630-631 8) Identify the letter that indicates the union of the right jugular, subclavian, and bronchomediastinal trunks. A) A B) B C) C D) D E) E Answer: A Diff: 2 Page Ref: 630-631 9) Identify the letter that indicates inguinal lymph nodes that filter lymph from the lower limbs. A) A B) B C) C D) D E) E Answer: E Diff: 2 Page Ref: 630 10) Identify the letter that indicates the first structure to receive lymph from both the lumbar and intestinal trunks and is located on the surface of L1 and L2 vertebrae. A) A B) B C) C D) D E) E Answer: D Diff: 2 Page Ref: 630-631 11) Structures comprised of endothelial cells separated by flaplike minivalves that are opened as fluid accumulates in peripheral tissue describes A) blood capillaries. B) lymph nodes. C) lymph capillaries. D) cisterna chyli. E) thoracic duct. Answer: C Diff: 2 Page Ref: 628-629 4 Copyright © 2014 Pearson Education, Inc.

 

12) Set of lymphatic capillaries that collect fat-laden fluids from the intestinal tract. A) lumbar trunks B) subclavian trunks C) bronchomediastinal trunks D) lacteals E) intestinal trunk Answer: D Diff: 2 Page Ref: 629 13) Two-lobed lymphoid organ located posterior to the sternum. A) cisterna chyli B) thymus C) MALT D) lacteals E) crypts Answer: B Diff: 2 Page Ref: 637-638 14) Recycles red blood cells and initiates immune responses. A) tonsils B) lymph nodes C) MALT D) spleen E) thymus Answer: D Diff: 2 Page Ref: 638-639 15) Lymphoid organs located in the mucosal lining of the pharynx. A) tonsils B) salivary glands C) uvula D) pharygotympanic tube E) aggregated lymphoid nodules Answer: A Diff: 2 Page Ref: 640 16) Begins at the cisterna chyli. A) thymic corpuscles B) aggregated lymphoid nodules C) thoracic duct D) MALT E) inguinal lymph nodes Answer: C Diff: 3 Page Ref: 630-631

5 Copyright © 2014 Pearson Education, Inc.

 

17) Small, bean-shaped organs abundant in the cervical, axillary, and inguinal regions. A) lymphatic capillaries B) afferent lymphatic vessels C) lymphatic ducts D) lymph nodes E) lacteals Answer: D Diff: 2 Page Ref: 629-630 18) Large lymphatic vessel that if present, is comprised of the right subclavian trunk, right jugular trunk and right bronchomediastinal trunk. A) cisterna chyli B) MALT C) spleen D) thoracic duct E) right lymphatic duct Answer: E Diff: 2 Page Ref: 631-632 19) T cells gain immunocompetence in this lymphoid organ. A) thymus B) tonsils C) spleen—red pulp D) lymph nodes E) spleen—white pulp Answer: A Diff: 3 Page Ref: 637-638 20) The largest lymphoid organ. A) thymus B) tonsils C) aggregated lymphoid nodules D) lymph nodes E) spleen Answer: E Diff: 2 Page Ref: 638 21) Masses of lymphoid tissue within a lymph node. A) trabeculae B) hilum C) lymphoid follicles D) white pulp E) subcapsular sinus Answer: C Diff: 2 Page Ref: 631

6 Copyright © 2014 Pearson Education, Inc.

 

22) The lymphoid organ that atrophies with age. A) thymus B) tonsils C) aggregated lymphoid nodules D) lymph nodes E) spleen Answer: A Diff: 3 Page Ref: 637 23) Duct of the lymphatic system that drains the body's entire lower half. A) lumbar trunk B) intestinal trunk C) bronchomediastinal trunk D) thoracic duct E) right lymphatic duct Answer: D Diff: 2 Page Ref: 630-631 24) Lymphoid tissue associated with the digestive, respiratory, urinary, and reproductive tracts. A) aggregated lymphoid nodules B) tonsils C) MALT D) lacteals E) crypts Answer: C Diff: 2 Page Ref: 637, 640-641 25) Blind-end invaginations of tonsil epithelium that trap bacteria and particulate matter. A) aggregated lymphoid nodules B) tonsils C) MALT D) lacteals E) crypts Answer: E Diff: 2 Page Ref: 640 26) Which of these statements regarding lymph capillaries is false? A) They begin as closed-ended tubes. B) They have precapillary sphincters that regulate drainage of lymph from tissues. C) They lack barriers to stop cancer cells from entering the lymphatic system. D) They unite with each other to form collecting vessels. Answer: B Diff: 2 Page Ref: 628-629

7 Copyright © 2014 Pearson Education, Inc.

 

27) The sinuses within lymph nodes A) are separated from the bloodstream by a blood-node barrier. B) consist of star-shaped epithelial cells that secrete immune-boosting hormones. C) contain macrophages living along a reticular fiber network. D) surround clumps of lymphoid tissue called white pulp. Answer: C Diff: 2 Page Ref: 630 28) Given that a germinal center arises from a single activated lymphocyte, how many different kinds of antibodies are secreted by a single germinal center (and by the plasma cells it produces)? A) none; T cells do not produce antibodies B) one C) several dozen D) thousands Answer: B Diff: 3 Page Ref: 633-635 29) The largest kind of lymphatic vessel is a lymphatic A) duct. B) trunk. C) sinus. D) capillary. Answer: A Diff: 2 Page Ref: 630 30) Lymphatic capillaries are present in A) skin. B) teeth. C) the central nervous system. D) bone marrow. Answer: A Diff: 1 Page Ref: 629 31) Lymphocytes attack antigens mostly in lymphoid organs (except the thymus) and in A) loose connective tissue. B) the bloodstream. C) cartilage. D) the thyroid. Answer: A Diff: 2 Page Ref: 634

8 Copyright © 2014 Pearson Education, Inc.

 

32) In the spleen, lymphoid tissue is confined to the A) red pulp. B) white pulp. C) trabeculae. D) hilus. Answer: B Diff: 1 Page Ref: 638-639 33) Is it possible for lymphocytes to gain immunocompetence in the thymus of an 82-year-old person? A) Yes, but the thymus has shifted to the production of B lymphocytes. B) Yes, but fewer than in youth. C) No, because the thymus has completely degenerated. D) No, because all lymphocytes have turned into thymic corpuscles. Answer: B Diff: 2 Page Ref: 637 34) The immune system cell type that is preferentially destroyed by the Epstein-Barr virus in mononucleosis is the A) B lymphocyte. B) cytotoxic T lymphocyte. C) helper T lymphocyte. D) macrophage. Answer: A Diff: 1 Page Ref: 641 35) A bubo is A) an infected lymph node containing a large number of pathogens that are trapped in the node but not destroyed. B) a cancer-infiltrated lymph node. C) any enlarged lymph node. D) a structure that is unrelated to lymph nodes. Answer: A Diff: 1 Page Ref: 630 36) In humans, B lymphocytes originate in A) the thymus. B) the appendix. C) all lymphoid tissue. D) the bone marrow. Answer: D Diff: 2 Page Ref: 634

9 Copyright © 2014 Pearson Education, Inc.

 

37) Which of the following cells are the largest producers of antibodies? A) B lymphocytes B) T lymphocytes C) plasma cells D) macrophages Answer: C Diff: 2 Page Ref: 633 38) The lymph trunk that contains chyle and is unpaired is the A) intestinal. B) subclavian. C) jugular. D) right lymph trunk. Answer: A Diff: 1 Page Ref: 630 39) Which of the following statements about the right lymph duct is false? A) Not all people have one. B) This duct and its tributaries drain the superior right quarter of the body. C) It empties into the thoracic duct. D) It forms from the union of a jugular, subclavian, and bronchomediastinal trunk. Answer: C Diff: 2 Page Ref: 631-632 40) In a histological cross section through an artery, a vein, and a lymphatic vessel, one can recognize the lymphatic vessel because it A) has the fewest valves. B) has the thinnest walls. C) has the thickest tunica media. D) always lies closest to the artery, which has the thinnest walls. Answer: B Diff: 1 Page Ref: 6298 41) Which of the following is not a part of MALT? A) lymphoid tissue in the appendix B) lymphoid tissue in the tonsils C) aggregated lymphoid nodules D) lymphocytes in the spleen Answer: D Diff: 2 Page Ref: 637

10 Copyright © 2014 Pearson Education, Inc.

 

42) Which of the following lymphoid structures have a hilum? A) tonsils and appendix B) thymus and aggregated lymphoid nodules C) spleen and lymph nodes D) MALT Answer: C Diff: 2 Page Ref: 630, 638 43) Lymphatic ducts deliver lymph to the cardiovascular system proximal to the A) external jugular veins. B) inferior vena cava just below the right atrium. C) internal jugular and subclavian veins. D) superior vena cava and azygos vein. Answer: C Diff: 2 Page Ref: 630-632 44) In humans, T lymphocytes originate in A) the spleen. B) the thymus. C) the red bone marrow. D) the lymph nodes. Answer: C Diff: 2 Page Ref: 634 45) Blockage of the ________ duct would prevent drainage only from the ________ and lumbar trunks. A) right lymphatic; jugular B) right lymphatic; subclavian C) thoracic; axillary D) thoracic; intestinal Answer: D Diff: 3 Page Ref: 630-631 46) In lymph nodes, follicles are always located A) in the cortex. B) near the hilum. C) in the lymph sinuses. D) in the medullary cords. Answer: A Diff: 2 Page Ref: 631

11 Copyright © 2014 Pearson Education, Inc.

 

47) The lymph organ that lies in the superior part of the thoracic mediastinum is the A) cervical lymph node. B) spleen. C) tonsil. D) thymus. Answer: D Diff: 2 Page Ref: 637 48) Which of the following is not classified as a lymphoid organ? A) liver B) spleen C) tonsils D) aggregated nodules in the intestine Answer: A Diff: 2 Page Ref: 637 49) The thymus is most active during A) fetal development. B) adolescence. C) old age. D) childhood. Answer: D Diff: 2 Page Ref: 637 50) The tonsil located on the posterior surface of the tongue is the ________ tonsil. A) palatine B) pharyngeal C) tubal D) lingual Answer: D Diff: 1 Page Ref: 640 51) The ________ stores blood platelets. A) lymph node B) thymus C) tonsil D) spleen Answer: D Diff: 1 Page Ref: 638-639 52) The lymphatic capillaries are A) more permeable than blood capillaries. B) less permeable than blood capillaries. C) as permeable as blood capillaries. D) completely impermeable. Answer: A Diff: 2 Page Ref: 628 12 Copyright © 2014 Pearson Education, Inc.

 

53) B lymphocytes are to ________ as T lymphocytes are to ________. A) antibodies; antigens B) antibodies; programmed cell death C) antigens; antibodies D) programmed cell death; antibodies Answer: B Diff: 3 Page Ref: 633-635 54) Which cell has the responsibility of presenting foreign antigens to lymphocytes? A) cytotoxic CD8+ T lymphocyte B) dendritic cell C) helper CD4+ T lymphocyte D) natural killer Answer: B Diff: 2 Page Ref: 635 55) Lymph capillaries differ from blood capillaries in all of the following ways except that A) they are more permeable than most blood capillaries. B) they carry lymph instead of blood. C) they are blind-ended. D) their walls consist of endothelial cells. Answer: D Diff: 3 Page Ref: 628-629 56) Which of the following might explain why most people are sick more often as children than as adults in their 20s through 50s? A) The immune system does not appear until the preteen years. B) The thymus is so active in children that it fights bacteria so hard that it produces strong symptoms of disease. C) Because of their small size, children's bodies cannot contain as much lymphoid tissue as those of adults. D) We build up many varieties of memory lymphocytes during childhood, providing immunity to more and more antigens during adulthood. Answer: D Diff: 3 Page Ref: 635, 641-642 57) The bulges that give lymph vessels a string-of-beads appearance are really A) lymph nodes. B) pockets for valves. C) weakenings in the wall of the vessels. D) temporary constrictions caused by bulging skeletal muscles that squeeze the lymph vessel. Answer: B Diff: 1 Page Ref: 629

13 Copyright © 2014 Pearson Education, Inc.

 

58) Lymphoid tissue contains all of the following elements except A) T cells. B) B cells. C) follicles. D) thick collagen fibers. Answer: D Diff: 2 Page Ref: 635, 637 59) Clusters of lymph nodes occur in all of the following sites except A) along the aorta. B) in the mediastinum. C) in bones and bone marrow. D) in the inguinal region. Answer: C Diff: 1 Page Ref: 629-630 60) Because the appendix is a narrow tube, it traps microorganisms from the digestive tract, and these microorganisms enter its wall and activate memory lymphocytes for long-term immunity. Another immune organ that traps microorganisms is the A) thymus. B) bone marrow. C) palatine tonsil. D) the spleen. Answer: C Diff: 2 Page Ref: 640-641 61) The immune system cell type that is preferentially destroyed by the AIDS virus is the A) B lymphocyte. B) cytotoxic T lymphocyte. C) helper T lymphocyte. D) helper B lymphocyte. Answer: C Diff: 1 Page Ref: 636 62) Which of the following lymphoid organs does not directly fight antigens? A) tonsil B) spleen C) the thymus D) aggregated lymphoid nodules Answer: C Diff: 3 Page Ref: 637-638

14 Copyright © 2014 Pearson Education, Inc.

 

63) Which cell secretes cytokines that will stimulate the proliferation of other types of immune cells? A) cytotoxic CD8+ T lymphocyte B) dendritic cell C) helper CD4+ T lymphocyte D) natural killer cell Answer: C Diff: 2 Page Ref: 635 64) All of the following mechanisms help move lymph through the lymphatic vessels except A) contractions of smooth muscle in the wall of the lymph vessels. B) body movements during exercise. C) tiny lymph hearts. D) contraction of skeletal muscles. Answer: C Diff: 1 Page Ref: 629 65) The lymph trunk that drains the thoracic organs is called the A) posterior intercostal trunk. B) great cardiac trunk. C) bronchomediastinal trunk. D) azygos trunk. Answer: C Diff: 1 Page Ref: 630 66) The paired lymph trunks that carry lymph from the lower limbs are the A) cisterna chyli. B) iliac trunks. C) lumbar trunks. D) femoral trunks. Answer: C Diff: 1 Page Ref: 630 67) Lymphatic vessels perform all of the following functions except A) returning excess tissue fluid to the bloodstream. B) returning leaked blood proteins to the bloodstream. C) transporting digested fats. D) carrying blood. Answer: D Diff: 2 Page Ref: 628-629

15 Copyright © 2014 Pearson Education, Inc.

 

68) Macrophages that destroy aged and defective blood cells from circulation are present in the ________ of the spleen. A) hilum B) splenic cords C) venous sinuses D) white pulp Answer: B Diff: 2 Page Ref: 638-639 69) A lymphoid nodule is A) a small lymph node. B) a type of lymphoma. C) a bulge in a lymphatic collecting vessel. D) a cluster of densely packed lymphocytes. Answer: D Diff: 2 Page Ref: 640-641 70) Crypts of the tonsils A) are filled with lymphoid tissue. B) are specialized lymphatic capillaries. C) contain all the progeny of a single activated B lymphocyte. D) trap bacteria and particulate matter. Answer: D Diff: 2 Page Ref: 640 71) This organ has a superficial cortex filled with lymphocytes and an inner medulla with few lymphocytes and a sizable number of degenerating epithelial reticular cells. A) lymph node B) spleen C) thymus D) tonsil Answer: C Diff: 3 Page Ref: 637-638 72) Edema is A) a form of cancer. B) an infection of the lymphatic vessels. C) an infection of the lymph nodes. D) the accumulation of excess tissue fluid in loose connective tissue. Answer: D Diff: 1 Page Ref: 629

16 Copyright © 2014 Pearson Education, Inc.

 

73) One location where lymphoid tissue is found is "in the frequently infected mucous membranes of the digestive, respiratory, urinary, and reproductive tracts." Specifically, this refers to A) intestinal and bronchomediastinal lymph trunks. B) the intestinal and tracheobronchial lymph nodes. C) the cisterna chyli and lacteals. D) MALT. Answer: D Diff: 2 Page Ref: 635 74) A person with many enlarged lymph nodes that do not feel tender or sore is most likely to have A) lymphangitis. B) mononucleosis. C) Hodgkin's lymphoma. D) buboes. Answer: C Diff: 2 Page Ref: 641 75) Lymph vessels drain lymph from all the lymphoid organs (simply because lymph vessels drain almost all organs of the body). However, lymph vessels enter (empty into) only one type of lymphoid organ. The only organ with such afferent lymph vessels is A) a lymph node. B) the spleen. C) the appendix. D) the thymus. Answer: A Diff: 2 Page Ref: 630, 638 76) The distal region of the small intestine contains clumps of lymph follicles called A) pancreatic islets. B) aggregated lymphoid follicles. C) rugae. D) villi. Answer: B Diff: 2 Page Ref: 640 77) A patient with lymphangitis presents with visible red lines on the skin that are tender to the touch. The best explanation for this sign is that A) lack of movement has caused pooling of lymph in the vessels, which compromised the lymphatic valves, resulting in varicose lymph vessels. B) red blood cells have infiltrated the lymphatic system. C) the spleen has overdegraded red blood cells, releasing iron into the lymphatic system. D) the vasa vasorum on the surface of infected lymph vessels has become inflamed and congested with blood. Answer: D Diff: 3 Page Ref: 641 17 Copyright © 2014 Pearson Education, Inc.

 

78) Which of the following is not a normal component of lymph? A) water B) plasma proteins C) red blood cells D) ions Answer: C Diff: 1 Page Ref: 628 79) Which lymphatic structure drains lymph from the right upper limb? A) lumbar trunk B) thoracic duct C) right subclavian duct D) cisterna chyli Answer: C Diff: 1 Page Ref: 630 80) Lymph leaves a lymph node via A) afferent lymphatic vessels. B) efferent lymphatic vessels. C) the cortical lymph sinus. D) the subcapsular sinus. Answer: B Diff: 2 Page Ref: 630-631 81) By secreting hormones, the thymus causes which cells to become immunocompetent? A) T lymphocytes B) B lymphocytes C) macrophages D) monocytes Answer: A Diff: 2 Page Ref: 637 21.2 True/False Questions 1) Lymph is propelled through the lymph vessels by the force of the heartbeat. Answer: FALSE Diff: 1 Page Ref: 629 2) If lymphatic vessels are surgically removed, the resulting edema is permanent because these vessels cannot regenerate. Answer: FALSE Diff: 1 Page Ref: 629 3) Lymph capillaries are open-ended like drinking straws. Answer: FALSE Diff: 2 Page Ref: 628-629 18 Copyright © 2014 Pearson Education, Inc.

 

4) Epithelial cells within the thymus secrete hormones that are important to lymphocyte development. Answer: TRUE Diff: 2 Page Ref: 637-638 5) The right lymphatic duct drains the intestinal trunk. Answer: FALSE Diff: 2 Page Ref: 630-631 6) Lacteals are the micro-valves present in lymph collecting vessels. Answer: FALSE Diff: 2 Page Ref: 629 7) The thymus increases in mass through childhood and then atrophies during adulthood. Answer: TRUE Diff: 2 Page Ref: 637-638 8) Lymphangitis is inflammation of a lymphatic vessel. Answer: TRUE Diff: 1 Page Ref: 641 9) The spleen atrophies during adulthood. Answer: FALSE Diff: 2 Page Ref: 638-639 10) Lymph capillaries are absent from the central nervous system. Answer: TRUE Diff: 3 Page Ref: 629 11) Lymph transports fats absorbed from the small intestine to the vena cava. Answer: TRUE Diff: 3 Page Ref: 629 12) Deep lymphatic collecting vessels of the abdominal cavity travel alongside deep arteries. Answer: TRUE Diff: 3 Page Ref: 629 13) The thymus is the only lymphatic organ that does not directly fight antigens. Answer: TRUE Diff: 3 Page Ref: 637-638 14) To activate lymphocytes, bacteria must penetrate the epithelium of tonsils. Answer: TRUE Diff: 3 Page Ref: 640

19 Copyright © 2014 Pearson Education, Inc.

 

15) Vaccination mimics acquired immunity. Answer: TRUE Diff: 2 Page Ref: 635 21.3 Short Answer Questions 1) The tonsils located just behind the openings of the pharyngotympanic tubes in the pharynx are the ________ tonsils. Answer: tubal Diff: 2 Page Ref: 640 2) The ________ is the site of T cell maturation. Answer: thymus Diff: 1 Page Ref: 637-638 3) Tumorlike lesions of the skin and some internal organs caused by a type of herpesvirus is called ________. Answer: Kaposi's sarcoma Diff: 2 Page Ref: 642 4) ________ flows in one direction, toward the heart. Answer: Lymph Diff: 1 Page Ref: 629 5) The lumbar and intestinal trunks unite at the ________ of the thoracic duct. Answer: cisterna chyli Diff: 2 Page Ref: 630-631 6) ________ are lymphatic structures that collect fat-laden fluids from the intestinal tract. Answer: Lacteals Diff: 2 Page Ref: 629 7) The HIV that causes AIDS infects immune cells that have the surface protein called ________. Answer: CD4 Diff: 3 Page Ref: 636 8) The ________ prevents premature activation of immature thymic lymphocytes. Answer: blood-thymus barrier Diff: 2 Page Ref: 637-638 9) Congestion of the tonsils with bacteria results in ________. Answer: tonsillitis Diff: 2 Page Ref: 642

20 Copyright © 2014 Pearson Education, Inc.

 

10) The ________ tonsils are the ones most often infected and removed during a tonsillectomy. Answer: palatine Diff: 2 Page Ref: 640 11) The paired ________ drain lymph from the head and neck. Answer: jugular trunks Diff: 2 Page Ref: 630 12) The ________ duct drains three-quarters of the body. Answer: thoracic Diff: 2 Page Ref: 630-631 13) Lymphocyte ________ results in immune cells that have gained the ability to attack specific antigens. Answer: activation Diff: 2 Page Ref: 634-635 14) Lymphoid tissues that enclose the central arteries of the spleen are collectively called ________. Answer: white pulp Diff: 2 Page Ref: 638 15) Aggregated lymphoid nodules and the ________ contain dense concentrations of lymphoid tissue and are associated with the distal structures of the digestive tract. Answer: appendix Diff: 2 Page Ref: 640-641 21.4 Essay Questions 1) Explain why lymph from the right upper limb is drained into vessels on the right side of the neck, whereas lymph from the right lower limb is drained into vessels on the left side of the neck. Answer: Lymph from the right upper limb drains into the right subclavian trunk, which may then empty into the right lymphatic duct. These lymphatic vessels often return lymph fluid to the venous blood proximal to the union of the right internal jugular vein and right subclavian vein. In contrast, lymph from both lower limbs drains into the lumbar trunks, which empty into the thoracic duct at the cisterna chyli. In this manner, lymph from the right lower limb will drain, along with three-quarters of the body, into the vessels of the left side of the neck. Diff: 2 Page Ref: 630-631 2) Identify and describe the functions of the lymphatic system. Answer: The lymphatic system recovers tissue fluid and leaked plasma proteins and returns them to the bloodstream. By doing so, the system eliminates local variations in the chemical composition and volume of tissue fluid. The lymphatic system also distributes hormones, antibodies and cells of the immune system. Lastly, it picks up dietary fats from the intestines and transports them into the bloodstream. Diff: 2 Page Ref: 628 21 Copyright © 2014 Pearson Education, Inc.

 

3) How does the structure and the function of lymph-collecting vessels compare to those of veins? What tunics do they have in common? Answer: Lymph-collecting vessels transport fluids toward the heart in the same manner as veins. Both structures have three tunics, but the walls of the lymphatic vessels are thinner. Both structures transport low-pressure fluids and consequently have one-way pocket valves to prevent retrograde flow, although valves are more numerous in lymphatic vessels. They both rely on movement of the skeletal muscles to help propel their respective fluids. Diff: 2 Page Ref: 629 4) Patients often complain to their physician of "swollen glands" in their neck. What is the proper name of these "glands," and what is the significance of their swelling? Answer: These structures are properly known as lymph nodes, specifically cervical lymph nodes if they are located in the neck. The nodes contain large numbers of macrophages and lymphocytes. Large numbers of pathogens trapped in the lymph nodes can cause the node to enlarge and become tender. Diff: 2 Page Ref: 629-630 5) Describe the process of T cell activation and the significance of the blood-thymus barrier. Answer: Immature lymphocytes that originate in the bone marrow, travel to the thymus, where they mature into T lymphocytes. In the thymus they develop immunocompetence, the ability to recognize a specific antigen. These naive cells however, only become fully activated when they experience an antigen challenge. In this process the T cell is presented with its specific antigen by a macrophage or dendritic cell. Diff: 2 Page Ref: 634-635

22 Copyright © 2014 Pearson Education, Inc.

 

Human Anatomy, 7e (Marieb/Mitchell/Smith) Chapter 22 The Respiratory System 22.1 Multiple Choice Questions

Figure 22.1 Use the diagram above to answer the following questions. 1) Identify the letter that indicates structures that assist in filtering, heating, and humidifying inspired air. A) A B) B C) C D) D E) E Answer: A Diff: 2 Page Ref: 649

1 Copyright © 2014 Pearson Education, Inc.

 

2) Identify the letter that indicates a muscular tube that serves as a common passageway for food and air. A) A B) B C) C D) D E) E Answer: D Diff: 2 Page Ref: 649 3) Identify the letter that indicates a flexible tube that has C-shaped cartilaginous rings that keep it from collapsing. A) A B) B C) C D) D E) E Answer: E Diff: 2 Page Ref: 649 4) Identify the letter that indicates the opening of the pharyngotympanic tube. A) A B) B C) C D) D E) E Answer: B Diff: 2 Page Ref: 649 5) Identify the letter that indicates the uvula. A) A B) B C) C D) D E) E Answer: C Diff: 2 Page Ref: 649

2 Copyright © 2014 Pearson Education, Inc.

 

Figure 22.2 Use the diagram above to answer the following questions. 6) Identify the letter that indicates Ciliated cells in this layer transport mucus laden with dust, bacteria, pollen, and viruses towards the pharynx. A) A B) B C) C D) D E) E Answer: C Diff: 2 Page Ref: 655 7) Identify the letter that indicates the fibromusculocartilaginous layer with hyaline cartilaginous rings that allow the trachea to bend and elongate but not collapse. A) A B) B C) C D) D E) E Answer: D Diff: 2 Page Ref: 655 8) Identify the letter that indicates the trachealis muscle. A) A B) B C) C D) D E) E Answer: B Diff: 2 Page Ref: 655 3 Copyright © 2014 Pearson Education, Inc.

 

9) Identify the letter that indicates the soft wall of the trachea allows the esophagus to expand anteriorly. A) A B) B C) C D) D E) E Answer: B Diff: 2 Page Ref: 655 10) Identify the letter that indicates a muscular tube that propels swallowed food to the stomach. A) A B) B C) C D) D E) E Answer: A Diff: 2 Page Ref: 655

Figure 22.3 Use the diagram above to answer the following questions. 11) Identify the letter that indicates the cartilage that has a laryngeal prominence that is also known as the "Adam's apple." A) A B) B C) C D) D E) E Answer: D Diff: 2 Page Ref: 6652 4 Copyright © 2014 Pearson Education, Inc.

 

12) Identify the letter that indicates the vocal fold, or true vocal cord. A) A B) B C) C D) D E) E Answer: E Diff: 2 Page Ref: 652-653 13) Identify the letter that indicates the cartilage that folds over the opening of the larynx — the glottis. The function of this cartilage is to prevent aspiration of foods and liquids into the lower respiratory tubes. A) A B) B C) C D) D E) E Answer: B Diff: 2 Page Ref: 652-653 14) Identify the letter that indicates the vestibular fold, or false vocal cord. A) A B) B C) C D) D E) E Answer: C Diff: 2 Page Ref: 652-653 15) Identify the letter that indicates the cartilage that anchors the vocal cords posteriorly. A) A B) B C) C D) D E) E Answer: A Diff: 2 Page Ref: 652 16) Anatomical region of the nasal cavity containing nose hairs. A) nasal septum B) olfactory mucosa C) nasal conchae D) vestibule E) uvula Answer: D Diff: 2 Page Ref: 648 5 Copyright © 2014 Pearson Education, Inc.

 

17) The smallest airway of the bronchial tree that is primarily responsible for creating bronchoconstriction. A) primary bronchi B) bronchiopulmonary segment C) lobe of lung D) bronchiole E) alveoli sac Answer: D Diff: 2 Page Ref: 657 18) Alveolar cells that allow rapid diffusion of respiratory gases. A) Type I B) Type II C) alveolar macrophages D) endothelial cells E) pseudostratiied columnar epithelial cells Answer: A Diff: 2 Page Ref: 658 19) The serous membrane lining of the wall of the thoracic cavity. A) visceral pleura B) parietal pleura C) mucous membrane D) parietal pericardium E) visceral pericardium Answer: B Diff: 2 Page Ref: 662 20) The region of the lung served by secondary, or lobar, bronchi. A) bronchopulmonary segment B) lobe C) lobule D) lingular E) cardiac notch Answer: B Diff: 2 Page Ref: 656 21) Fluid that reduces surface tension of the alveolar walls. A) serous fluid B) pleural fluid C) surfactant D) lysozyme E) mucin Answer: C Diff: 3 Page Ref: 659

6 Copyright © 2014 Pearson Education, Inc.

 

22) These are peripheral chemoreceptors that monitor changes in respiratory gasses and blood acidity levels. A) ventral respiratory center B) vagus nerve C) reticular activating system D) carotid bodies E) medulla oblongata Answer: D Diff: 2 Page Ref: 666 23) The medial opening between the vocal folds through which air passes. A) glottis B) vestibule C) rima glottidis D) thyrohyoid membrane E) cricothyroid ligament Answer: C Diff: 2 Page Ref: 653 24) Specific location within the lungs where gas exchange occurs. A) terminal bronchioles B) carina C) bronchopulmonary segments D) alveoli E) respiratory membrane Answer: E Diff: 2 Page Ref: 658-659 25) Fissure separating the upper and middle lobes of the right lung. A) horizontal B) oblique C) carina D) superior E) inferior Answer: A Diff: 2 Page Ref: 659 26) The superior and middle nasal conchae are bony projections from the A) vomer. B) ethmoid bone. C) sphenoid bone. D) maxillary bone. Answer: B Diff: 2 Page Ref: 650

7 Copyright © 2014 Pearson Education, Inc.

 

27) Which cells produce surfactant? A) type I cells B) type II cells C) alveolar macrophages D) chondrocytes Answer: B Diff: 1 Page Ref: 658-659 28) Which part of the nose is composed of dense fibrous connective tissue? A) the apex B) the border where the nostril attaches to the maxilla C) the nasal septum D) the surface connecting to the nasal bones Answer: B Diff: 1 Page Ref: 648 29) The rubbing together of inflamed pleural membranes that produces a stabbing pain in the chest is called A) influenza. B) tuberculosis. C) rhinitis. D) pleurisy. Answer: D Diff: 2 Page Ref: 663 30) Which structure is not located within the nasopharynx? A) opening of the pharyngotympanic tube B) palatine tonsil C) pharyngeal tonsil D) tubal tonsil Answer: B Diff: 2 Page Ref: 651 31) Which of the following is not a function of the nasal conchae? A) They deflect particulates to the mucosa. B) They help warm the air. C) They decrease the turbulence in the flow of air through the nasal cavity. D) They moisten the air. Answer: C Diff: 2 Page Ref: 650

8 Copyright © 2014 Pearson Education, Inc.

 

32) When the diaphragm contracts, the size of the thoracic cavity ________, the pressure inside the thoracic cavity ________, and air flows ________ the lungs. A) decreases; rises; into B) increases; drops; into C) decreases; drops; out of D) increases; drops; out of Answer: B Diff: 3 Page Ref: 664-666 33) In lung cancer, the cancer cells usually arise from A) the smooth muscle fibers around the bronchioles. B) the epithelium lining a large bronchus. C) the alveoli. D) lymph nodes in the lung. Answer: B Diff: 2 Page Ref: 668 34) Vibrissae are A) specialized cells in the larynx that react to vibrations of air. B) large hairlike projections on epithelial cells that line the trachea. C) nose hairs. D) the sensory organs of the inner ear. Answer: C Diff: 2 Page Ref: 648 35) Which muscle contracts during forced expiration? A) internal oblique B) serratus anterior C) sternocleidomastoid D) pectoralis major Answer: A Diff: 2 Page Ref: 664-666 36) The part of the brain that generates the basic respiratory rhythm is the A) medulla oblongata. B) limbic system. C) hypothalamus. D) cerebrum. Answer: A Diff: 2 Page Ref: 666

9 Copyright © 2014 Pearson Education, Inc.

 

37) Sympathetic stimulation of terminal bronchioles causes A) bronchoconstriction. B) a decrease in blood supply to the bronchioles. C) bronchodilation. D) secretion of less surfactant. Answer: C Diff: 2 Page Ref: 657 38) The left lung A) has three lobes. B) is supplied entirely by a secondary bronchus. C) receives oxygenated blood from the heart via the left pulmonary artery. D) has a cardiac notch. Answer: D Diff: 1 Page Ref: 659 39) Which portion of the nasal cavity is lined with sebaceous and sweat glands and numerous hair follicles? A) hard and soft palate B) nasal conchae C) roof (ceiling) D) vestibule Answer: D Diff: 1 Page Ref: 648 40) The detergent-like molecule that keeps the alveoli from collapsing between breaths is called A) bile. B) hemoglobin. C) surfactant. D) oxygen. Answer: C Diff: 2 Page Ref: 659 41) The passageway between the nasopharynx and the middle ear is the A) oval window. B) internal acoustic meatus. C) mastoid sinus. D) pharyngotympanic tube. Answer: D Diff: 1 Page Ref: 651

10 Copyright © 2014 Pearson Education, Inc.

 

42) Of the three different layers of tissue in the wall of the trachea, the outermost layer is the A) adventitia. B) lamina propria. C) submucosa. D) muscularis. Answer: A Diff: 1 Page Ref: 656 43) What type of epithelium occurs in the respiratory mucosa? A) pseudostratified ciliated columnar epithelium B) simple squamous epithelium C) simple columnar epithelium D) stratified squamous epithelium Answer: A Diff: 1 Page Ref: 648 44) Some bronchi have names such as the "right upper-lobe bronchus," "left lower-lobe bronchus," and "right middle-lobe bronchus." These are ________ bronchi. A) primary B) secondary C) tertiary D) very small (about fifteenth-order) Answer: B Diff: 2 Page Ref: 656 45) The correct definition of bronchopulmonary segments is A) arbitrary subdivisions of a lung, about as large as a fist (0.3 liter). B) parts of the lung that are easy to remove during surgery. C) parts of the lungs that are separated by the oblique and horizontal fissures. D) parts of the lung that are supplied by a single tertiary bronchus (and its branches). Answer: D Diff: 1 Page Ref: 659 46) The ________ flaps over the glottis when we swallow food. A) uvula B) epiglottis C) palate D) vocal fold Answer: B Diff: 2 Page Ref: 652

11 Copyright © 2014 Pearson Education, Inc.

 

47) An infectious illness that results in the accumulation of fluid in the alveoli is A) tuberculosis. B) emphysema. C) bronchitis. D) pneumonia. Answer: D Diff: 1 Page Ref: 671 48) The respiratory mucosa is present throughout each of the following areas except the A) bronchi. B) nasal cavity. C) nasopharynx. D) superior laryngopharynx. Answer: D Diff: 2 Page Ref: 653-654 49) Most foreign substances in inspired air fail to reach the lungs because of the A) ciliated epithelium and mucus that line the respiratory passages. B) abundant blood supply to the nasal mucosa. C) porous structure of the nasal conchae. D) action of the epiglottis. Answer: A Diff: 2 Page Ref: 648 50) The ability to vary the pitch of the voice results from varying A) the force of air passing over the vocal folds. B) the tension in the vocal folds. C) the size of the laryngeal cartilages. D) the shape of the cricoid cartilage. Answer: B Diff: 2 Page Ref: 654 51) In the wall of the trachea and bronchi, the cartilage rings lie within which tissue layer? A) inner epithelium B) mucosa C) submucosa D) fibromusculocartilaginous layer Answer: D Diff: 2 Page Ref: 656 52) The aortic bodies A) sense the concentration of oxygen and carbon dioxide in the blood. B) give rise to the pulmonary arteries. C) are innervated by the glossopharyngeal nerve. D) are in the hilum of the lung. Answer: A Diff: 2 Page Ref: 666 12 Copyright © 2014 Pearson Education, Inc.

 

53) Of the following, which is the only organ that is not an upper respiratory structure? (Hint: Recall Figure 22.3.) A) larynx B) oropharynx C) vestibule of the nose D) nasal cavity Answer: A Diff: 1 Page Ref: 649 54) An aortic aneurysm that presses on the left recurrent laryngeal nerve could lead to A) paralysis of the diaphragm. B) hoarse speech. C) complete loss of speech. D) inability of the carotid sinus to monitor blood gases. Answer: B Diff: 3 Page Ref: 654 55) Lung cancer occurs slightly more frequently in the right lung than in the left lung (about 53% versus 47% of the time). The most logical explanation for this is that A) the tissue of the right lung is more susceptible to carcinogens. B) the right nostril is closed more often than the left one, so it blocks the exhalation of cigarette smoke and causes smoke to back up into the right lung. C) most people are right-handed, so everything affects the right side of the body more intensely. D) the right primary bronchus is wider than the left one (and the right lung is larger than the left lung), so the right lung receives slightly more carcinogenic cigarette smoke with each puff. Answer: D Diff: 3 Page Ref: 656, 668 56) The main function of the elaborate venous plexus in the nasal mucosa is A) diapedesis of leukocytes. B) to replace vessels broken in nose bleeds. C) to supply extra blood to the brain and supplement the dural sinuses. D) to warm blood and moisten inhaled air. Answer: D Diff: 2 Page Ref: 650 57) Which pulmonary disease is characterized by a permanent enlargement of the alveoli? A) emphysema B) tuberculosis C) pneumonia D) lung cancer Answer: A Diff: 2 Page Ref: 669

13 Copyright © 2014 Pearson Education, Inc.

 

58) The vocal ligaments attach between the thyroid cartilage and the A) arytenoid cartilage. B) corniculate cartilage. C) cricoid cartilage. D) cuneiform cartilage. Answer: A Diff: 2 Page Ref: 653 59) The trachealis muscle A) constricts the trachea. B) controls the length and tension of the vocal cords. C) initiates the cough reflex. D) raises the larynx when swallowing. Answer: A Diff: 2 Page Ref: 654-655 60) The pharyngeal tonsil is A) scattered throughout the pharynx. B) in the fauces. C) the largest tonsil. D) synonymous with the adenoids in the roof of the nasopharynx. Answer: D Diff: 2 Page Ref: 651 61) Embryologically, the nasal cavity develops from A) invagination of the ectoderm of the olfactory placode. B) the nasopharynx. C) a cranial extension of the mouth. D) the early precursor of the paranasal sinuses. Answer: A Diff: 2 Page Ref: 670 62) Supportive cartilage disappears from the bronchial tree at the level of the A) alveolar duct. B) bronchioles. C) lobar bronchi. D) respiratory bronchiole. Answer: B Diff: 2 Page Ref: 656

14 Copyright © 2014 Pearson Education, Inc.

 

63) The lobes of a lung are easy to recognize and distinguish from one another because A) they all have slightly different colors. B) they are separated by fissures. C) each is supplied by a primary bronchus, and the primary bronchi are easy to find. D) carbon from the air accumulates between and outlines the lobes in most people (even if they breathe almost-unpolluted air). Answer: B Diff: 1 Page Ref: 659 64) Alveolar pores A) allow gases to transfer from the alveoli to the blood. B) allow pleural fluid to enter the alveoli and keep their walls from sticking together. C) are the openings between the alveolar duct and the alveolus. D) equalize air pressure throughout the lung. Answer: D Diff: 2 Page Ref: 659 65) The space between the right and left vocal folds is called the A) laryngeal inlet. B) cricoid. C) vestibule. D) rima glottidis. Answer: D Diff: 2 Page Ref: 653 66) Which structure does not enter or exit from the hilum of a lung? A) bronchial vein B) phrenic nerve C) pulmonary artery D) pulmonary plexus Answer: B Diff: 3 Page Ref: 659-661 67) Which structure is both a lower respiratory structure and in the conducting zone? (Hint: Recall Figure 22.3.) A) the laryngopharynx B) a respiratory bronchiole C) any bronchus D) an alveolus Answer: C Diff: 2 Page Ref: 646-647, 649

15 Copyright © 2014 Pearson Education, Inc.

 

68) In a runner who is breathing heavily during a race, the rima glottidis is A) half closed. B) half open. C) open as far as possible. D) all the way closed. Answer: C Diff: 2 Page Ref: 653 69) It is easy to see on the outer surface of a smoker's lung hexagons approximately the size of the tip of a little finger and clearly outlined by black lines of carbon. These structures are A) lobes. B) bronchopulmonary segments. C) lobules. D) bronchioles. Answer: C Diff: 2 Page Ref: 659 70) A chronic respiratory disease that leads to an expanded "barrel chest" is A) emphysema. B) lung cancer. C) tuberculosis. D) pneumonia. Answer: A Diff: 1 Page Ref: 669-670 71) Which muscle is used for both quiet inspiration and forced expiration? A) external intercostals B) internal intercostals C) latissimus dorsi D) scalenes Answer: B Diff: 3 Page Ref: 664-666 72) Which of the following changes does not occur as the conducting tubes of the respiratory tree become smaller? A) Cartilage rings are replaced by irregular plates of cartilage. B) The lining epithelium thins. C) Elastin in the walls disappears. D) Cartilage disappears in the bronchioles. Answer: C Diff: 2 Page Ref: 655

16 Copyright © 2014 Pearson Education, Inc.

 

73) The ventral respiratory group (VRG) is a group of neurons located within the A) forebrain. B) medulla oblongata. C) midbrain. D) pons. Answer: B Diff: 2 Page Ref: 666 74) In a preserved cadaver, the first rib is likely to form a groove on which surface of the lung? A) basal B) mediastinal C) inferior costal D) apical (apex) Answer: D Diff: 2 Page Ref: 659 75) The lungs are located in the A) mediastinum. B) abdominopelvic cavity. C) thoracic cavity. D) pleura. Answer: C Diff: 1 Page Ref: 659 22.2 True/False Questions 1) The trachea, bronchi, and bronchioles are all located within the lungs. Answer: FALSE Diff: 2 Page Ref: 656 2) Changes in the size of the thoracic cavity bring about inspiration and expiration. Answer: TRUE Diff: 2 Page Ref: 664-666 3) Type II alveolar cells produce pleural fluid to keep the walls of the alveoli from collapsing or adhering together. Answer: FALSE Diff: 2 Page Ref: 659 4) Gas exchange occurs across the bronchi and bronchioles. Answer: FALSE Diff: 2 Page Ref: 657 5) The left lung has both a horizontal fissure and oblique fissure. Answer: FALSE Diff: 1 Page Ref: 659 17 Copyright © 2014 Pearson Education, Inc.

 

6) The pleural cavities extend two ribs below the inferior border of the lungs. Answer: TRUE Diff: 3 Page Ref: 663 7) The external intercostal muscles are involved in active inspiration. Answer: TRUE Diff: 2 Page Ref: 664 8) The groove through which air passes between nasal conchae is called a choanae. Answer: FALSE Diff: 2 Page Ref: 650 9) The respiratory zone begins at the large bronchioles. Answer: FALSE Diff: 1 Page Ref: 657 10) The only laryngeal cartilage to form a complete ring is the cuneiform. Answer: FALSE Diff: 2 Page Ref: 652 11) When the diaphragm contracts, it raises the floor of the thoracic cavity upward. Answer: FALSE Diff: 2 Page Ref: 664 12) The scalene muscles are involved in deep inspiration. Answer: TRUE Diff: 2 Page Ref: 665 13) The number of secondary, or lobar, bronchi is a distinguishing characteristic of the right and left lungs. Answer: TRUE Diff: 3 Page Ref: 656 14) The vocal ligaments are attached between the cricoid and arytenoid cartilages. Answer: FALSE Diff: 3 Page Ref: 653 15) The mucosal epithelium transitions from pseudostratified columnar to simple cuboidal along the path to the respiratory bronchioles. Answer: TRUE Diff: 3 Page Ref: 656

18 Copyright © 2014 Pearson Education, Inc.

 

22.3 Short Answer Questions 1) ________ fluid helps keep the lungs expanded against the thoracic walls. Answer: Pleural Diff: 1 Page Ref: 663 2) The ________ sinuses drain into the nasal cavity. Answer: paranasal Diff: 1 Page Ref: 650 3) The ________ intercostal muscles are involved in (forced) expiration. Answer: internal Diff: 1 Page Ref: 665-666 4) The presence of a(n) ________ in its medial aspect is unique to the left lobe. Answer: cardiac notch Diff: 2 Page Ref: 659 5) The function of type II alveolar cells is to produce ________. Answer: surfactant Diff: 2 Page Ref: 659 6) The ________ cartilages are the posterior anchor of the vocal cords. Answer: arytenoid Diff: 2 Page Ref: 653 7) The subdivisions of the lung from largest to smallest are the lobe, the segment, the ________, the alveolus. Answer: lobule Diff: 2 Page Ref: 659 8) The ________ is a groove inferior to the nasal conchae through which air passes. Answer: meatus Diff: 2 Page Ref: 650 9) The ________ tonsil, or adenoids, is located on the wall of the nasopharynx. Answer: pharyngeal Diff: 2 Page Ref: 651 10) The ________ is the portion of the soft palate that prevents food from entering the nasal cavity. Answer: uvula Diff: 2 Page Ref: 651

19 Copyright © 2014 Pearson Education, Inc.

 

11) Fusion of the alveolar and capillary membranes creates a(n) ________ membrane, or airblood barrier. Answer: respiratory Diff: 3 Page Ref: 658-659 12) ________ maneuver involves using the vocal folds as a sphincter to increase intra-abdominal pressure. Answer: Valsalva's Diff: 2 Page Ref: 654 13) The soft ________ muscle of the trachea allows the esophagus to expand anteriorly during swallowing. Answer: trachealis Diff: 2 Page Ref: 654-655 14) The unexpected death of an apparently healthy infant during sleep is called ________. Answer: sudden infant death syndrome (SIDS) Diff: 2 Page Ref: 671 15) The ________ marks the point where the trachea branches into the two main bronchi. Answer: carina Diff: 2 Page Ref: 655 22.4 Essay Questions 1) Describe the structure and function of the pleural cavity. Answer: The pleural cavity is a fluid-filled space between the parietal and visceral serous membranes of the thoracic cavity. These membranes, the pleurae, create pleural fluid that lubricates the contact between opposing surfaces of the lungs and thoracic cavity wall. This fluid prevents friction during inspiration and expiration. The surface tension of the fluid also keeps the lungs expanded against the thoracic walls, preventing collapse of the lung. Diff: 2 Page Ref: 663 2) Describe the effects of smoking on (a) alveolar structure, (b) cilia and alveolar macrophages, and (c) the bronchial epithelium. Answer: Components of tobacco smoke lead to destruction of alveolar walls. The resulting loss of surface area for gas exchange is a condition known as emphysema. Smoking also slows the activity of cilia and macrophages of the respiratory mucosa. This leads to accumulation of mucus, irritants, and carcinogens. The carcinogens lead to cancer growth in the bronchial epithelium and mucosal glands. Diff: 2 Page Ref: 668

20 Copyright © 2014 Pearson Education, Inc.

 

3) Describe the histological changes that occur in the walls of bronchial tree, beginning with the trachea proceeding to the alveoli. Answer: (1) The supportive cartilaginous rings of the trachea are replaced with irregular plates in the primary bronchi and eliminated altogether in the bronchioles. (2) The respiratory epithelium transitions from pseudostratified ciliated columnar cells to simple cuboidal cells in the bronchioles and simple squamous cells in the alveoli. (3) Goblet cells are present in the bronchi but absent elsewhere. (4) A layer of smooth muscle appears in the walls of the bronchi and is present throughout the tree to the respiratory bronchioles. Diff: 2 Page Ref: 656-657 4) Describe all the anatomical changes to the larynx that account for the deeper voice of men compared to women. Answer: The thyroid cartilage of males grows larger than in females, and its laryngeal prominence protrudes further anteriorly. The vocal ligaments, which stretch from the arytenoid cartilage to the thyroid cartilage, are therefore longer in males. Longer cords vibrate more slowly than do shorter cords, resulting in a deeper voice. Diff: 2 Page Ref: 654 5) Explain the difference in muscles used for active inspiration and expiration. Answer: Active inspiration involves the diaphragm and external intercostal muscles. Additionally, the scalenes, sternocleidomastoid, and pectoralis minor may also be involved. Active expiration involves relaxation of the diaphragm and contraction of the internal intercostals, oblique and transverse abdominis muscles, and, in certain circumstances, the latissimus dorsi. Diff: 2 Page Ref: 664-666

21 Copyright © 2014 Pearson Education, Inc.

 

Human Anatomy, 7e (Marieb/Mitchell/Smith) Chapter 23 The Digestive System 23.1 Multiple Choice Questions

Figure 23.1 Use the diagram above to answer the following questions. 1) Identify the letter that indicates the cardiac region of the stomach. A) A B) B C) C D) D E) E Answer: A Diff: 2 Page Ref: 694 2) Identify the letter that indicates the region of the stomach that regulates the passage of chyme into the small intestine. A) A B) B C) C D) D E) E Answer: E Diff: 2 Page Ref: 694-695

1 Copyright © 2014 Pearson Education, Inc.

 

3) Identify the letter that indicates folds that allow for expansion of the stomach. A) A B) B C) C D) D E) E Answer: C Diff: 2 Page Ref: 694-695 4) Identify the letter that indicates the curvature where the greater omentum attaches. A) A B) B C) C D) D E) E Answer: D Diff: 2 Page Ref: 694-695 5) Identify the letter that indicates the fundus of the stomach. A) A B) B C) C D) D E) E Answer: B Diff: 2 Page Ref: 694

2 Copyright © 2014 Pearson Education, Inc.

 

Figure 23.2 Use the diagram above to answer the following questions. 6) Identify the letter that indicates the cystic duct. A) A B) B C) C D) D E) E Answer: C Diff: 2 Page Ref: 698 7) Identify the letter that indicates the hepatic ducts as they exits the porta hepatis. A) A B) B C) C D) D E) E Answer: A Diff: 3 Page Ref: 698, 707

3 Copyright © 2014 Pearson Education, Inc.

 

8) Identify the letter that indicates the duct that carries digestive enzymes from acinar cells in the pancreas. A) A B) B C) C D) D E) E Answer: E Diff: 3 Page Ref: 698, 710 9) Identify the letter that indicates the duct that directs both digestive enzymes and bile to the duodenum. A) A B) B C) C D) D E) E Answer: D Diff: 2 Page Ref: 698 10) Identify the letter that indicates the duct formed by the union of the right and left hepatic ducts. A) A B) B C) C D) D E) E Answer: B Diff: 2 Page Ref: 698, 707

4 Copyright © 2014 Pearson Education, Inc.

 

Figure 23.3 Use the diagram above to answer the following questions. 11) Identify the letter that indicates the root canal. A) A B) B C) C D) D E) E Answer: E Diff: 2 Page Ref: 690 12) Identify the letter that indicates the crown. A) A B) B C) C D) D E) E Answer: A Diff: 2 Page Ref: 690

5 Copyright © 2014 Pearson Education, Inc.

 

13) Identify the letter that indicates the root. A) A B) B C) C D) D E) E Answer: C Diff: 2 Page Ref: 690 14) Identify the letter that indicates the surface of the tooth that is coated with the hardest substance in the body. A) A B) B C) C D) D E) E Answer: D Diff: 2 Page Ref: 690 15) Identify the letter on the diagram that represents the neck of the tooth. A) A B) B C) C D) D E) E Answer: B Diff: 2 Page Ref: 690 16) Retroperitoneal organs have a serosa facing the peritoneal cavity and a(n) ________ on the posterior side embedded in the abdominal wall. A) mesothelium B) adventitia C) muscularis externa D) vasa vasorum E) caveolae Answer: B Diff: 3 Page Ref: 685 17) Infoldings of the sarcolemma of smooth muscle fibers. A) mesothelium B) adventitia C) muscularis externa D) vasa vasorum E) caveolae Answer: E Diff: 2 Page Ref: 686 6 Copyright © 2014 Pearson Education, Inc.

 

18) Junction of the transverse and ascending colon. A) splenic flexure B) cecum C) hepatic flexure D) ileocecal valve E) haustra Answer: C Diff: 2 Page Ref: 701 19) The union of the cystic and common hepatic ducts. A) accessory pancreatic duct B) main pancreatic duct C) hepatopancreatic ampulla D) common bile duct E) bile canaliculi Answer: D Diff: 2 Page Ref: 710 20) Smooth muscle constriction between the ileum and cecum. A) splenic flexure B) cecum C) hepatic flexure D) ileocecal valve E) haustra Answer: D Diff: 2 Page Ref: 700-701 21) Attaches the liver to the lesser curvature of the stomach. A) haustra B) hepatopancreatic ampulla C) porta hepatis D) greater omentum E) lesser omentum Answer: E Diff: 2 Page Ref: 680 22) Attaches the liver to the anterior abdominal wall and diaphragm. A) greater omentum B) lesser omentum C) falciform ligament D) ligamentum teres E) porta hepatis Answer: C Diff: 2 Page Ref: 680

7 Copyright © 2014 Pearson Education, Inc.

 

23) Layer of the GI tract responsible for peristalsis and segmentation. A) muscularis mucosae B) muscularis externa C) lamina propria D) submucosa E) serosa Answer: B Diff: 2 Page Ref: 685 24) Bulblike union of the main pancreatic duct and bile duct. A) cystic duct B) hepatic duct C) porta hepatis D) hepatopancreatic ampulla E) accessory pancreatic duct Answer: D Diff: 2 Page Ref: 697 25) Three strips of longitudinal muscles of the muscularis of the colon causing it to pucker into sacs. A) ileocecal junction B) haustra C) teniae coli D) muscularis mucosae E) pyloric sphincter Answer: C Diff: 2 Page Ref: 700 26) Which layer of the digestive tract is responsible for the peristaltic waves that propel materials from one portion to another? A) muscularis externa B) serosa C) submucosa D) mucosa Answer: A Diff: 1 Page Ref: 685 27) Which of the following choices correctly pairs a type of cell in the stomach with its secretion? A) parietal cell; pepsinogen B) chief cell; pepsinogen C) parietal cell; mucus D) enteroendocrine; hydrochloric acid Answer: B Diff: 2 Page Ref: 695

8 Copyright © 2014 Pearson Education, Inc.

 

28) Which of the following is not a characteristic of the large intestine? A) It includes the ascending, transverse, and descending colon. B) It contains an abundant bacterial flora. C) It is the main site of nutrient absorption. D) It absorbs much of the water and salts remaining in the wastes. Answer: C Diff: 2 Page Ref: 700-701 29) The digestive organ primarily responsible for the absorption of water is the A) ileum. B) duodenum. C) anus. D) large intestine. Answer: D Diff: 1 Page Ref: 700 30) Another name for serosa is A) parietal peritoneum. B) serous gland. C) visceral peritoneum. D) mucosa. Answer: C Diff: 2 Page Ref: 685 31) Medial to both midclavicular lines and superior to the subcostal plane lies the A) appendix. B) cecum. C) jejunum. D) pyloric sphincter. Answer: D Diff: 2 Page Ref: 677 32) To say someone is "tongue-tied" means that the A) lips are exceptionally immobile. B) tongue muscles are weak. C) salivary glands produce little lubricant. D) lingual frenulum is short. Answer: D Diff: 1 Page Ref: 688 33) The lesser omentum extends between the A) greater curvature of the stomach and the posterior abdominal wall. B) lesser curvature of the stomach and the porta hepatis of the liver. C) transverse colon and the posterior abdominal wall. D) sigmoid colon and the posterior pelvic wall. Answer: B Diff: 2 Page Ref: 680 9 Copyright © 2014 Pearson Education, Inc.

 

34) The splenic, or left colic, flexure of the colon is located within the A) left hypochondriac region. B) left lumbar region. C) right hypochondriac region. D) right lumbar region. Answer: A Diff: 3 Page Ref: 677, 701 35) Which of the following is not contained in saliva? A) enzymes that begin the digestion of proteins B) enzymes that initiate the digestion of carbohydrates C) bicarbonate buffer D) bactericidal enzymes Answer: A Diff: 2 Page Ref: 691 36) How many deciduous teeth are there? A) 18 B) 20 C) 32 D) It varies from person to person. Answer: B Diff: 2 Page Ref: 689 37) Mucosa-associated lymphoid tissue (MALT) is primarily located within the A) lamina propria. B) muscularis mucosa. C) serosa. D) submucosa. Answer: A Diff: 2 Page Ref: 683 38) What is the function of the hepatopancreatic sphincter? A) It controls the entry of bile and pancreatic juices into the alimentary canal. B) As it contracts, it squeezes pancreatic secretions into the duodenum. C) It inhibits defecation in the upper alimentary canal while the anal sphincters do the same in the lower regions. D) It prevents the movement of bile into the gallbladder. Answer: A Diff: 2 Page Ref: 697-698

10 Copyright © 2014 Pearson Education, Inc.

 

39) Secretions of the parotid gland empty A) anterior to the frenulum of the tongue. B) between the lingual tonsil and epiglottis. C) lateral to the upper molars. D) through 10 ducts on the floor of the oral cavity. Answer: C Diff: 2 Page Ref: 691-692 40) The mesentery that suspends the small intestine is the A) falciform ligament. B) lesser omentum. C) greater omentum. D) mesentery proper. Answer: D Diff: 2 Page Ref: 680 41) Why are bacteria abundant in the large intestines but not in the stomach? A) Food enters the stomach first and does not spend much time there. B) The intestine is much warmer and moister, encouraging bacterial growth. C) The stomach wall contains so much lymphoid tissue that it destroys all bacteria there. D) Secretions of parietal cells kill bacteria in the stomach. Answer: D Diff: 3 Page Ref: 695 42) The stomach A) stores food for later use in the form of fat. B) absorbs most of the nutrients in food. C) churns food into a paste by mechanical means. D) dehydrates food materials before passing them to the small intestine. Answer: C Diff: 1 Page Ref: 694-695 43) Which of the following correctly describes the function of the greater omentum? A) It is a vestigial structure that has no known function. B) It stores fat. C) It absorbs heat from the digestive process and radiates it to the outside of the body. D) It wraps around most of the large intestine and anchors it to the anterior abdominal wall. Answer: B Diff: 3 Page Ref: 680 44) Which of the following is a secondarily retroperitoneal organ? A) descending colon B) ileum C) sigmoid colon D) transverse colon Answer: A Diff: 2 Page Ref: 680-681 11 Copyright © 2014 Pearson Education, Inc.

 

45) What is the function of the gallbladder? A) secretion of bile B) production of cholesterol C) secretion of gastrin D) storage of bile Answer: D Diff: 1 Page Ref: 710 46) The pancreas contains all of the following regions except a A) head. B) tail. C) hilum. D) body. Answer: C Diff: 2 Page Ref: 710 47) The largest salivary gland is the A) intrinsic. B) submandibular. C) sublingual. D) parotid. Answer: D Diff: 1 Page Ref: 691 48) Which of the following cells produce intrinsic factor? A) chief cells B) parietal cells C) mucous neck cells D) enteroendocrine cells Answer: B Diff: 2 Page Ref: 695 49) Which of the following is not a function of hepatocytes? A) producing digestive enzymes B) picking up and processing nutrients from the portal blood C) storing some vitamins D) detoxifying poisons Answer: A Diff: 2 Page Ref: 707, 710

12 Copyright © 2014 Pearson Education, Inc.

 

50) Which of the following applies to the small intestine? A) It is where foodstuffs first encounter protein-splitting enzymes. B) Its walls secrete most of the digestive enzymes that are active in its lumen. C) It is where carbohydrates and fats but not proteins are digested. D) Breakdown products of fats enter its lacteals. Answer: D Diff: 2 Page Ref: 697-698 51) All of the following structures have all four tissue layers in their walls except the A) esophagus. B) mouth. C) stomach. D) sigmoid colon. Answer: B Diff: 3 Page Ref: 688 52) The portion of the large intestine closest to the liver is the A) cecum. B) rectum. C) transverse colon. D) descending colon. Answer: C Diff: 2 Page Ref: 701 53) Digestion of which of the following would be affected the most if the bile-secreting liver were severely damaged? A) carbohydrates B) lipids C) proteins D) nucleic acids Answer: B Diff: 2 Page Ref: 706 54) The duodenum contains these structures whose products neutralize the acidic chyme. A) duodenal glands B) gastric glands C) intestinal glands D) Peyer's patches Answer: A Diff: 1 Page Ref: 699

13 Copyright © 2014 Pearson Education, Inc.

 

55) Which of the following statements about the duodenum is false? A) It receives chyme from the stomach. B) It is the site of action of liver and pancreas secretions. C) It is shorter than either the ileum or jejunum. D) It is more movable than the ileum or jejunum, which are retroperitoneal. Answer: D Diff: 2 Page Ref: 697-698 56) Which of the following layers is present in the mucosa of the stomach and intestines, but not in the mucosa of the mouth and pharynx? A) lining epithelium B) lamina propria C) muscularis mucosae D) lumen Answer: C Diff: 3 Page Ref: 688 57) The "mostly mucous" extrinsic salivary gland is the ________ gland. A) parotid B) submandibular C) sublingual D) intrinsic Answer: C Diff: 3 Page Ref: 692 58) Which of the following are the only mucosal folds that do not flatten out at all when the organ stretches? A) longitudinal folds in the esophagus B) rugae in the stomach C) circular folds in the small intestine D) mucosal folds in the gallbladder Answer: C Diff: 3 Page Ref: 697 59) The bare area of the liver A) contains the ligamentum teres. B) is covered with visceral peritoneum. C) is fused with the diaphragm. D) is on the liver's inferior and anterior surface. Answer: C Diff: 2 Page Ref: 707

14 Copyright © 2014 Pearson Education, Inc.

 

60) The epithelial lining of the mouth derives from A) ectoderm. B) mesoderm. C) endoderm. D) neural crest. Answer: A Diff: 3 Page Ref: 713 61) The liver and pancreas form as part of the embryonic A) foregut. B) midgut. C) hindgut. D) the hindgut and midgut. Answer: A Diff: 2 Page Ref: 713 62) Of the basic digestive processes, the one in which nutrients enter capillaries is called A) ingestion. B) propulsion. C) mechanical digestion. D) absorption. Answer: D Diff: 1 Page Ref: 682 63) Which of the following is not a characteristic of the rectum? A) lacks tenia coli B) has longitudinal folds called columns C) is secondarily retroperitoneal D) has transverse folds called rectal valves Answer: B Diff: 2 Page Ref: 701 64) If we say the pancreas is shaped like a tadpole, then the tadpole's head lies A) posterior to the fundus of the stomach. B) inside the mesentery proper. C) in the curvature formed by the duodenum. D) against the hilum of the spleen. Answer: C Diff: 2 Page Ref: 710 65) Which of the following is true of the pectinate line of the anal canal? A) It lies just below the level of the rectal valves. B) It is also called the anal columns. C) It divides regions of somatic and visceral innervation. D) All hemorrhoids occur there. Answer: C Diff: 3 Page Ref: 701 15 Copyright © 2014 Pearson Education, Inc.

 

66) Which of the following is not a characteristic of enteroendocrine cells? A) They are scattered throughout the lining epithelium of the stomach and intestines. B) They secrete hormones that help signal the events of digestion. C) They never secrete their product into the lumen of the digestive canal. D) They are scattered throughout the lining of the rectum. Answer: D Diff: 3 Page Ref: 695 67) The splenic flexure is the boundary between the A) spleen and stomach. B) transverse and descending colon. C) transverse and ascending colon. D) descending colon and sigmoid colon. Answer: B Diff: 2 Page Ref: 701 68) In the stomach, the undifferentiated epithelial stem cells lie near the junction between the gastric pits and gastric glands. In the intestine, the corresponding stem cells occur A) on the tips of the villi. B) where the intestinal crypts meet the villi. C) in the duodenal (Brunner's) glands. D) deep within the intestinal glands (crypts of Lieberkühn). Answer: D Diff: 3 Page Ref: 698 69) Some bacteria from the intestinal microbiota work their way into the intestinal wall and start to spread through the circulation. Many of these bacteria are stopped by MALT, while many more are destroyed by A) hepatocytes. B) hepatic macrophages. C) the walling-off action of the greater omentum. D) megakaryocytes. Answer: B Diff: 3 Page Ref: 707 70) Which of the following is a role of the levator ani muscle in defecation? A) It pushes down on the feces. B) It has no role in defecation, only in inhibiting defecation (it is the external sphincter muscle). C) It lifts the anal canal superiorly around the feces. D) Its stretch and proprioception properties initiate the defecation reflex. Answer: C Diff: 2 Page Ref: 702

16 Copyright © 2014 Pearson Education, Inc.

 

71) In mastication, the relative roles of an incisor versus a molar are A) piercing versus tearing. B) chewing versus holding food in the mouth. C) biting off pieces of food versus grinding. D) only incisors function in mastication. Answer: C Diff: 3 Page Ref: 690 72) Disease of which structure is the most common cause of tooth loss in adults? A) periodontal ligament B) crown C) enamel D) dentin Answer: A Diff: 3 Page Ref: 691 73) The lamina propria and submucosa of the stomach both derive from which embryonic layer? A) ectoderm B) intermediate mesoderm C) splanchnic mesoderm D) somatic mesoderm Answer: C Diff: 2 Page Ref: 713 74) Most of the gastrointestinal tract is innervated by the sympathetic and parasympathetic branches of the nervous system. Which parts are innervated by the somatic nervous system? A) esophagus and stomach B) pharynx and anal canal C) pyloric, ileocecal, and internal anal sphincters D) small and large intestines Answer: B Diff: 2 Page Ref: 702 75) Which of the following structures neither enters nor leaves the porta hepatis? A) hepatic veins B) branches of hepatic portal vein C) branches of hepatic artery D) hepatic ducts Answer: A Diff: 3 Page Ref: 707-708

17 Copyright © 2014 Pearson Education, Inc.

 

76) The parietal cells in the stomach produce A) mucin. B) pepsin. C) intrinsic factor and HCl. D) secretin. Answer: C Diff: 2 Page Ref: 695 77) Which of the following is not an accessory digestive organ? A) teeth B) salivary gland C) liver D) spleen Answer: D Diff: 1 Page Ref: 677 78) In most cases, the accessory pancreatic duct drains into the A) common bile duct. B) common hepatic duct. C) duodenum. D) jejunum. Answer: C Diff: 2 Page Ref: 710-711 79) The lining epithelium of the developing digestive tract (pharynx through anal canal) comes from A) ectoderm. B) mesoderm. C) endoderm. D) neural crest. Answer: C Diff: 2 Page Ref: 713 80) The terminal portion of the small intestine is the A) duodenum. B) ileum. C) jejunum. D) pyloric sphincter. Answer: B Diff: 1 Page Ref: 697

18 Copyright © 2014 Pearson Education, Inc.

 

81) The correct sequence of layers in the wall of the alimentary canal, from internal to external, is A) mucosa, muscularis, serosa, submucosa. B) mucosa, submucosa, muscularis, serosa. C) serosa, muscularis, mucosa, submucosa. D) submucosa, mucosa, serosa, muscularis. Answer: B Diff: 1 Page Ref: 683-685 82) The layer of the digestive tube that contains abundant elastin plus blood vessels, lymphoid nodules, and deep glands is the A) adventitia. B) submucosa. C) muscularis. D) serosa. Answer: B Diff: 2 Page Ref: 683 83) What is the distinction between Crohn's disease and ulcerative colitis? A) Although both diseases are caused by inflammation, Crohn's disease occurs only in the small intestine, whereas ulcerative colitis occurs only in the large intestine. B) Crohn's disease is caused by the bacterium H. pylori, whereas ulcerative colitis results from the failure of acid to be neutralized before it reaches the colon. C) Crohn's disease produces deeper erosions of the mucosa and occurs throughout the intestines, whereas ulcerative colitis occurs mostly in the rectum. D) Crohn's disease results in diverticula, caused by insufficient dietary fiber, whereas ulcerative colitis is caused by the bacterium H. pylori. Answer: C Diff: 3 Page Ref: 712 84) Which of the following statements about the large intestine is false? A) It has no villi. B) It exhibits external muscular bands called taeniae coli. C) It is longer than the small intestine. D) It has haustra. Answer: C Diff: 2 Page Ref: 700-701 23.2 True/False Questions 1) The vermiform appendix is suspended from the cecum. Answer: TRUE Diff: 1 Page Ref: 701 2) The terms taste bud and papillae are synonymous. Answer: FALSE Diff: 2 Page Ref: 689 19 Copyright © 2014 Pearson Education, Inc.

 

3) The lesser omentum directly attaches the stomach to the posterior abdominal wall. Answer: FALSE Diff: 2 Page Ref: 680 4) The pancreas and duodenum are secondarily retroperitoneal organs. Answer: TRUE Diff: 2 Page Ref: 680 5) Most of the ascending colon lies between the subcostal and transtubercular planes. Answer: TRUE Diff: 3 Page Ref: 677 6) From the lumen outward, the layers of the gastrointestinal tract are mucosa, submucosa, muscularis, serosa. Answer: TRUE Diff: 2 Page Ref: 683 7) The most superficial layer of the esophagus is the serosa. Answer: FALSE Diff: 3 Page Ref: 683-685, 693 8) Smooth muscle fibers differ from skeletal muscle in that they do not contain contractile myofilaments. Answer: FALSE Diff: 1 Page Ref: 685-686 9) Stretching of the anal sphincter initiates the defecation reflex. Answer: FALSE Diff: 3 Page Ref: 702 10) Villi are cytoplasmic projections on the surface of intestinal absorptive cells. Answer: FALSE Diff: 2 Page Ref: 698 11) Chief cells of the gastric glands secrete pepsinogen. Answer: TRUE Diff: 2 Page Ref: 695 12) Hepatic portal blood is mixed with blood from the hepatic artery in the liver. Answer: TRUE Diff: 3 Page Ref: 707 13) The small intestines contain bacteria that synthesize some essential vitamins. Answer: TRUE Diff: 3 Page Ref: 699 20 Copyright © 2014 Pearson Education, Inc.

 

14) A gallstone lodged in the cystic duct may also cause blockage of the pancreas. Answer: FALSE Diff: 3 Page Ref: 698, 710 15) The sinusoids of the liver lobule receive blood from the portal arteriole and deliver blood to the portal venule. Answer: FALSE Diff: 3 Page Ref: 707 23.3 Short Answer Questions 1) The ________ is connective tissue whose capillaries nourish the epithelium of the mucosa. Answer: lamina propria Diff: 2 Page Ref: 683 2) The right and left hepatic ducts join to form the ________ duct. Answer: common hepatic Diff: 2 Page Ref: 707 3) Blood flowing through the liver lobule enters at the portal venule and flows through ________ to the central vein and out of the liver through the hepatic vein. Answer: sinusoids Diff: 2 Page Ref: 707 4) The transverse colon bends inferiorly at the ________ to become the descending colon. Answer: splenic flexure Diff: 2 Page Ref: 700 5) The ________ region of the stomach is closest to the esophagus. Answer: cardiac Diff: 2 Page Ref: 694-695 6) The root of the tooth is held to the periodontal ligament by a calcified connective tissue called ________. Answer: cement Diff: 2 Page Ref: 690 7) The porta hepatis contains the hepatic arteries, hepatic ducts, and ________. Answer: hepatic portal veins Diff: 2 Page Ref: 707 8) The region between your teeth and lips is called the ________. Answer: oral vestibule Diff: 2 Page Ref: 688

21 Copyright © 2014 Pearson Education, Inc.

 

9) Thin myofilaments of smooth muscle fibers are anchored to intermediate filaments through ________. Answer: dense bodies Diff: 2 Page Ref: 686 10) The distal sigmoid colon is located within the ________ region of the abdominal pelvic cavity. Answer: hypogastric Diff: 2 Page Ref: 677 11) ________ is a dangerous inflammation of herniations in the colonic wall. Answer: Diverticulitis Diff: 2 Page Ref: 702 12) ________ is a condition in which inflammation of the intestines causes deep ulcers and fissures. Answer: Crohn's disease Diff: 2 Page Ref: 712 13) "Baby" teeth are more properly called ________ teeth. Answer: deciduous Diff: 2 Page Ref: 689 14) The external muscle layer of the pharynx consists of three ________ muscles composed of voluntary skeletal muscle. Answer: pharyngeal constrictor Diff: 2 Page Ref: 692 15) Longitudinal folds of the stomach mucosa are called ________. Answer: rugae Diff: 2 Page Ref: 694-695 23.4 Essay Questions 1) Explain the relationships and distinguish among the following structures of the small intestine: villi, microvilli, and circular folds. Answer: Plicae circulares or circular folds are large, 1-cm-high transverse folds of the small intestine. The surface of these folds has fingerlike projections called villi. They are approximately 1-mm-tall extensions of the mucosa that contain blood vessels and lacteals. The epithelial layer covering these villi consists of columnar absorptive cells each having mircovilli extensions on their apical surface. Diff: 2 Page Ref: 697-698

22 Copyright © 2014 Pearson Education, Inc.

 

2) Describe the structure and function of the intrinsic and extrinsic muscles of the tongue. Answer: The intrinsic muscles are confined within the tongue and are not attached to bone. They are responsible for changing the shape of the tongue but not its position in the mouth. This latter function belongs to the extrinsic muscles that are attached to bone. The genioglossus is responsible for protracting, retracting, and lateral movements of the tongue. The intrinsic muscles are arranged in transverse, longitudinal, and vertical layers, allowing the tongue to change shape while chewing and speaking. Diff: 2 Page Ref: 688 3) Describe the structure of a mesentery, and then identify and describe the location of two ventral and two dorsal mesenteries. Answer: Answer: A mesentery consists of two serous membranes that sandwich blood vessels and nerves in between. The mesentery extends from the abdominal wall to intraperitoneal organs, supporting the organs and fixing them to the wall. The two ventral mesenteries are the falciform ligament, which attaches the liver to the anterior wall and diaphragm, and the lesser omentum, which attaches the lesser curvature of the stomach to the liver. The dorsal mesenteries include the greater omentum, mesentery proper, transverse mesocolon, and sigmoid mesocolon. The greater omentum attaches the greater curvature of the stomach to the posterior abdominal wall. The mesentery proper connects the jejunum and ileum to the posterior abdominal wall. The two mesocolons attach to the region of the colon for which they are named. Diff: 2 Page Ref: 680-681 4) Identify the location and function of the enteric nerve plexus. Answer: The enteric nerve plexus consists of neurons located entirely within the walls of the GI tract. These neurons form reflex arcs that can function independently from the central nervous system. The enteric plexus includes neurons from both the submucosal and myenteric plexuses. The submucosal nerve plexus, located in the submucosa, innervates the secretory glands of the mucosa. It also causes some movements of the mucosa. The myenteric nerve plexus lies in the muscularis layer of the tract. It controls peristalsis and segmentation. Diff: 2 Page Ref: 686-687 5) Explain the relationship between a hiatal hernia and gastroesophageal reflux disease (GERD). Answer: In a hiatal hernia, the cardiac region of the stomach pushes superiorly through the esophageal hiatus of the diaphragm and into the thoracic cavity. Without the reinforcement of the surrounding diaphragmatic muscles, the effectiveness of the cardiac sphincter declines, and digestive juices are able to regurgitate from the stomach into the esophagus. This regurgitation and the associated inflammation and pain are called gastroesophageal reflux disease. Diff: 2 Page Ref: 693

23 Copyright © 2014 Pearson Education, Inc.

 

Human Anatomy, 7e (Marieb/Mitchell/Smith) Chapter 24 The Urinary System 24.1 Multiple Choice Questions

Figure 24.1 Use the diagram above to answer the following questions. 1) Identify the letter that indicates the major calyx. A) A B) B C) C D) D E) E Answer: E Diff: 2 Page Ref: 723 2) Identify the letter that indicates the blood vessel that branches from the abdominal aorta carrying oxygenated blood to the kidney. A) A B) B C) C D) D E) E Answer: C Diff: 2 Page Ref: 724 1 Copyright © 2014 Pearson Education, Inc.

 

3) Identify the letter that indicates the blood vessels that delineate the cortex from the medulla. A) A B) B C) C D) D E) E Answer: B Diff: 2 Page Ref: 724 4) Identify the letter that indicates an expanded portion of the ureter. A) A B) B C) C D) D E) E Answer: D Diff: 2 Page Ref: 723 5) Identify the letter that indicates a cortical radiate artery. A) A B) B C) C D) D E) E Answer: A Diff: 1 Page Ref: 724

2 Copyright © 2014 Pearson Education, Inc.

 

Figure 24.2 Use the diagram above to answer the following questions. 6) Identify the letter that indicates the afferent arteriole. A) A B) B C) C D) D E) E Answer: D Diff: 1 Page Ref: 729 7) Identify the letter that indicates the vasa recta. A) A B) B C) C D) D E) E Answer: E Diff: 1 Page Ref: 729 3 Copyright © 2014 Pearson Education, Inc.

 

8) Identify the letter that indicates the glomerulus. A) A B) B C) C D) D E) E Answer: B Diff: 2 Page Ref: 729 9) Identify the letter that indicates blood vessels that absorb solutes from the convoluted tubules. A) A B) B C) C D) D E) E Answer: A Diff: 2 Page Ref: 729-730 10) Identify the letter that indicates the region of the nephron with receptors sensitive to antidiuretic hormone (ADH). A) A B) B C) C D) D E) E Answer: C Diff: 2 Page Ref: 728-729

4 Copyright © 2014 Pearson Education, Inc.

 

Figure 24.3 Use the diagram above to answer the following questions. 11) Identify the letter that indicates the parietal layer of glomerular capsule. A) A B) B C) C D) D E) E Answer: D Diff: 2 Page Ref: 726 12) Identify the letter that indicates the proximal convoluted tubule. A) A B) B C) C D) D E) E Answer: E Diff: 2 Page Ref: 726 13) Identify the letter that indicates the efferent arteriole. A) A B) B C) C D) D E) E Answer: A Diff: 2 Page Ref: 726

5 Copyright © 2014 Pearson Education, Inc.

 

14) Identify the letter that indicates the visceral layer of the glomerular capsule. A) A B) B C) C D) D E) E Answer: C Diff: 2 Page Ref: 726 15) Identify the letter that indicates blood vessels covered by podocytes. A) A B) B C) C D) D E) E Answer: C Diff: 2 Page Ref: 726 16) Knot of capillaries that directs blood into the efferent arteriole. A) arcuate arteries B) cortical radiate arteries C) glomerulus D) afferent arterioles E) peritubular capillaries Answer: C Diff: 1 Page Ref: 730 17) The medial concave cleft in which vessels, ureters, and nerves enter/leave the kidney. A) cortex B) hilum C) renal corpuscle D) renal capsule E) sinus Answer: B Diff: 2 Page Ref: 721 18) A large fat-filled space within the kidney that also contains the renal pelvis, blood vessels, and nerves. A) sinus B) convoluted tubules C) renal corpuscle D) renal pyramids E) cortical columns Answer: A Diff: 2 Page Ref: 723

6 Copyright © 2014 Pearson Education, Inc.

 

19) Cup-shaped tubes that enclose the papillae of the pyramids. A) major calyx B) minor calyx C) renal sinus D) renal pelvis E) cortical columns Answer: B Diff: 2 Page Ref: 723 20) 85% of nephrons are described by this term that relates to their location in the kidney. A) sinusoidal B) trabecular C) medullary D) cortical E) extrinsic Answer: D Diff: 2 Page Ref: 728 21) Region of the distal tubule that monitors concentration of the filtrate. A) vasa recta B) collecting duct C) macula densa D) mesangial E) extraglomerular Answer: C Diff: 2 Page Ref: 730-731 22) The descriptive term for the location of the kidneys relative to the abdominal cavity. A) retroperitoneal B) intraperitoneal C) extraabdominal D) supraperitoneal E) subcapsular Answer: A Diff: 2 Page Ref: 721 23) Contraction of this muscle forces urine from the bladder. A) cremaster B) detrusor C) diaphragm D) vesicular E) dartos Answer: B Diff: 2 Page Ref: 733

7 Copyright © 2014 Pearson Education, Inc.

 

24) Ladderlike looping blood vessels within the medulla. A) vasa vasorum B) peritubular capillaries C) cortical radiate D) vasa recta E) interlobular Answer: D Diff: 2 Page Ref: 730 25) Blood vessel that contains granular cells involved in blood pressure regulation. A) afferent arteriole B) efferent arteriole C) segmental artery D) macula densa E) extraglomerular mesangial Answer: A Diff: 3 Page Ref: 730-731 26) Which of the following is not normally found in urine? A) urea B) glucose C) uric acid D) creatinine Answer: B Diff: 2 Page Ref: 720-721 27) If you looked closely at the tip of a renal papilla with a hand lens, the hundreds of little openings you would see on its surface are A) minor calyces. B) renal sinuses. C) openings of papillary ducts. D) glomeruli. Answer: C Diff: 2 Page Ref: 723,728 28) Vessels and nerves enter and leave the kidney through the A) fibrous capsule. B) lateral convex surface. C) inferior surface. D) medial hilum. Answer: D Diff: 1 Page Ref: 721

8 Copyright © 2014 Pearson Education, Inc.

 

29) The most superficial layer of the kidney is the A) cortex. B) medulla. C) renal pyramids. D) renal papilla. Answer: A Diff: 1 Page Ref: 722 30) How much of the fluid filtered by the kidney actually becomes urine? A) 1% B) 10% C) 50% D) 99% Answer: A Diff: 1 Page Ref: 730 31) Which of the following supportive tissues is most superficial? A) fibrous capsule B) pararenal fat C) perirenal fat D) renal fascia Answer: B Diff: 3 Page Ref: 721-722 32) Before it enters the ureter, urine collects in the A) renal sinus. B) renal pelvis. C) renal cortex. D) renal pyramids. Answer: B Diff: 2 Page Ref: 723 33) Infection of the renal pelvic and calyces is known as A) nephritis. B) cystitis. C) hydronephritis. D) pyelitis. Answer: D Diff: 1 Page Ref: 736 34) Which vessels lie within the renal columns? A) arcuate arteries B) cortical radiate arteries C) interlobar arteries D) segmental arteries Answer: C Diff: 2 Page Ref: 724 9 Copyright © 2014 Pearson Education, Inc.

 

35) Which of the following is not part of the filtration membrane? A) basement membrane B) capillary endothelium C) filtration slit diaphragm D) granular cells Answer: D Diff: 3 Page Ref: 725-726 36) Which of the following processes does not participate in the production of urine in the kidney? A) filtration B) secretion C) evaporation D) resorption Answer: C Diff: 1 Page Ref: 725 37) Which of the following is not found in the renal corpuscle? A) the glomerulus B) the nephron loop C) glomerular capsule D) podocytes Answer: B Diff: 1 Page Ref: 725 38) The only embryonic kidney that survives into adulthood is the A) pronephros. B) mesonephros. C) metanephros. D) paranephros. Answer: C Diff: 2 Page Ref: 737 39) The function of the collecting duct is to A) contract its muscular walls to expel urine from the cortex. B) determine the final volume and concentration of urine. C) drain blood from the kidney and deliver it to the renal vein. D) transport resorbed water back into the cardiovascular system. Answer: B Diff: 2 Page Ref: 728

10 Copyright © 2014 Pearson Education, Inc.

 

40) The kidneys lie behind the ________ cavity. A) abdominal B) pelvic C) thoracic D) cranial Answer: A Diff: 1 Page Ref: 721 41) The name of the inferior of the three openings in the trigone of the bladder is the A) ureteric orifice. B) urachus opening. C) internal urethral orifice. D) detrusor. Answer: C Diff: 2 Page Ref: 734 42) Renin is produced in A) the glomerulus. B) the renal medulla. C) the granular cells. D) glomerular capsules. Answer: C Diff: 2 Page Ref: 730-731 43) The mucosal folds in the bladder (rugae) A) act to increase the surface area for absorption. B) thicken the bladder wall so that it does not burst. C) are not present in life, only in cadavers. D) have the same basic function as transitional epithelium—accommodating stretch as the bladder fills. Answer: D Diff: 1 Page Ref: 733 44) Why are urinary tract infections more common in females than in males? A) In males the urethra is made up of three regions, whereas in females it is made of only one. B) In females the urethra is attached to the anterior vaginal wall by connective tissue. C) In females the urethra is shorter than in males. D) In males the urethra is shared by the both the reproductive and the urinary systems, whereas in females it is part of the urinary system only. Answer: C Diff: 2 Page Ref: 736

11 Copyright © 2014 Pearson Education, Inc.

 

45) Which of the following conditions is most often seen in elderly males with prostatic hyperplasia? A) urge incontinence B) overflow incontinence C) urinary retention D) stress incontinence Answer: C Diff: 2 Page Ref: 736 46) Which of the following would not inhibit micturition? A) stimulation of the somatic motor neurons to the external urethral sphincter B) activation of the sympathetic pathways C) relaxation of the internal urethal sphincter D) relaxation of the detrusor muscles Answer: C Diff: 3 Page Ref: 735-736 47) Which segment of the nephron has a simple squamous epithelium? A) ascending limb of the nephron loop B) descending limb of the nephron loop C) distal convoluted tubule D) proximal convoluted tubule Answer: B Diff: 2 Page Ref: 727-728 48) An important difference between a cuboidal epithelial cell of the proximal versus the distal convoluted tubules is that A) the former only secretes, the latter only filters. B) one lies in the renal cortex, the other in the medulla. C) only one has a folded basolateral membrane containing enzymes for ion transport. D) the proximal tubule cells have long microvilli that are fewer than or absent from the distal tubule cells. Answer: D Diff: 2 Page Ref: 727-728 49) An important difference between the peritubular capillaries and the vasa recta in the kidney is that A) only the vasa recta are supplied by efferent arterioles. B) the vasa recta drain into arterioles, whereas the peritubular capillaries drain into venules. C) the vasa recta are involved in filtration, whereas the peritubular capillaries are involved in resorption. D) the vasa recta are in the medulla, whereas the peritubular capillaries are in the cortex. Answer: D Diff: 2 Page Ref: 730

12 Copyright © 2014 Pearson Education, Inc.

 

50) From which embryonic tissue layer(s) does the kidney arise? A) ectoderm B) mesoderm C) endoderm D) all three layers Answer: B Diff: 1 Page Ref: 737-738 51) Which of the following is not usually associated with the formation of renal calculi? A) increased intake of calcium B) polyurea C) dehydration D) bacterial infection Answer: B Diff: 1 Page Ref: 736-737 52) The left renal vein ________ the right renal vein. A) is shorter than B) is the same length as C) is longer than D) carries less blood than Answer: C Diff: 2 Page Ref: 724 53) Which of the following statements about the urinary bladder is false? A) In females, the bladder lies posterior to the uterus but anterior to the rectum. B) The ureters attach to the bladder through oblique posterolateral orifices. C) Two ureteral openings and the internal urethral orifice bound the trigone of the bladder. D) When empty, the bladder lies inferior to the abdominal cavity. Answer: A Diff: 2 Page Ref: 732-734 54) Based on what you know about the location of the kidneys in the posterior abdominal wall, the hilum of the kidney must be at the level of which vertebra? A) T11 or T12 B) L1 or L2 C) L4 D) T8 Answer: B Diff: 2 Page Ref: 721

13 Copyright © 2014 Pearson Education, Inc.

 

55) Which of the following is found exclusively in the renal medulla? A) nephron loop of juxtamedullary nephrons B) cortical radiate arteries C) peritubular capillaries D) proximal convoluted tubules Answer: A Diff: 1 Page Ref: 728 56) The part of the nephron whose epithelial cells are most responsible for resorption and secretion is the A) glomerular capsule (podocytes). B) proximal tubule. C) thin segment. D) distal tubule. Answer: B Diff: 1 Page Ref: 727-728 57) The layer of podocytes is the same as the A) parietal layer. B) glomerulus. C) visceral layer of glomerular capsule. D) capsular space. Answer: C Diff: 3 Page Ref: 725-726 58) Of the following, the only epithelial type that does not line the urethra is A) simple squamous. B) pseudostratified columnar. C) stratified squamous. D) stratified columnar. Answer: A Diff: 2 Page Ref: 734 59) Which of the following statements about the internal urethral sphincter is false? A) It is a thickening of the detrusor muscle. B) It is located superior to the prostatic urethra in males. C) It is not consciously controlled. D) It is surrounded by the urogential diaphragm. Answer: D Diff: 2 Page Ref: 734

14 Copyright © 2014 Pearson Education, Inc.

 

60) Pyelography is A) kinking of the ureter. B) a type of X-ray procedure (radiology). C) a way to cure kidney stones. D) failure of the embryonic kidney to ascend. Answer: B Diff: 1 Page Ref: 731 61) Arteries that branch to form the afferent arterioles to the glomeruli are A) segmental arteries. B) arcuate arteries. C) cortical radiate arteries. D) interlobar arteries. Answer: C Diff: 2 Page Ref: 724 62) Which of the following structures is most numerous within a kidney? A) cortical radiate arteries B) minor calyces C) renal corpuscles D) renal papillae Answer: C Diff: 1 Page Ref: 724 63) The embryonic urogenital sinus A) derives from the cloaca. B) gives rise to the anus. C) gives rise to the rectum. D) gives rise to the ureters. Answer: A Diff: 2 Page Ref: 737-738 64) The ureters develop from A) the cloaca. B) the urogenital sinus. C) pronephric nephrons. D) a duct that branches from the mesonephric duct. Answer: D Diff: 3 Page Ref: 737-738 65) The external urethral sphincter is located A) at the external urethral orifice. B) at the junction of the bladder wall and urethra. C) at the ureteral orifice. D) at the urogenital diaphragm. Answer: D Diff: 1 Page Ref: 734 15 Copyright © 2014 Pearson Education, Inc.

 

66) The condition in which the metanephros has failed to ascend is A) horseshoe kidney. B) polycystic renal disease. C) hydronephrosis. D) pelvic kidney. Answer: D Diff: 2 Page Ref: 737-738 67) During a dissection, Arnie saw the ureters entering the lateral corners of the bladder but found the internal openings of the ureters in the trigone near the midline of the bladder. His observation necessarily means that A) the ureters run medially for some distance within the posterior bladder wall. B) the ureters and urethra are really the same tube. C) each ureter must have four distinct openings into the bladder. D) this bladder had a congenital defect. Answer: A Diff: 3 Page Ref: 731 68) The parts of the nephron whose epithelial cells contain the most mitochondria are the A) renal corpuscle and distal tubule. B) proximal and distal tubules. C) thin segment and glomerular capsule. D) vasa recta and collecting tubules. Answer: B Diff: 3 Page Ref: 728 69) In the micturition reflex, the detrusor muscle is stimulated to contract by A) sphincter neurons from the brain. B) sympathetic fibers. C) parasympathetic fibers. D) visceral sensory fibers from the vagus. Answer: C Diff: 2 Page Ref: 735-736 70) In the juxtaglomerular apparatus, the macula densa belongs to the A) terminal nephron loop. B) glomerulus. C) efferent arteriole. D) visceral layer of the glomerular capsule (podocytes). Answer: A Diff: 2 Page Ref: 730-731

16 Copyright © 2014 Pearson Education, Inc.

 

71) Another name for the neck of the bladder is the A) superior surface. B) anterior angle. C) inferior angle. D) trigone. Answer: C Diff: 2 Page Ref: 733 72) The longest of the three parts of the male urethra is the A) prostatic. B) membranous. C) spongy urethra. D) neck. Answer: C Diff: 2 Page Ref: 735 73) Which structure(s) is (are) most important for holding the kidney in place in the abdomen? A) renal vessels B) renal ligaments C) renal fascia D) the diaphragm, through its muscle tone Answer: C Diff: 2 Page Ref: 721-722 74) The micturition center is located in the A) cerebellum. B) detrusor muscle layer of the bladder wall. C) pons of the brain stem. D) sacral spinal cord. Answer: C Diff: 2 Page Ref: 735-736 75) Cortical nephrons are different from juxtamedullary nephrons in that A) they are much less abundant. B) they produce urine, whereas juxtamedullary nephrons do not. C) their nephron loop is shorter, with a shorter thin segment. D) they do not have a proximal convoluted tubule. Answer: C Diff: 2 Page Ref: 730

17 Copyright © 2014 Pearson Education, Inc.

 

76) When Melinda was asked to identify a "mystery" slide on a histology test, she immediately identified it as renal cortex because of the presence of scattered A) renal corpuscles. B) thin segments. C) renal papillae. D) vasa recta. Answer: A Diff: 2 Page Ref: 725 77) Which of the following regions of the nephron is most likely to be found in the renal medulla? A) glomerulus B) proximal convoluted tubule C) distal convoluted tubule D) collecting duct Answer: D Diff: 1 Page Ref: 728 78) Urine passes through the A) kidney hilum to the bladder to the ureter. B) renal pelvis to the ureter to the bladder to the urethra. C) glomerulus to the ureter to the nephron. D) hilus to the urethra to the bladder. Answer: B Diff: 1 Page Ref: 732-733 79) An increase in the permeability of collecting tubule cells to water is due to A) a decrease in the production of ADH. B) an increase in the production of ADH. C) a decrease in the concentration of solutes in the blood plasma. D) the presence of a salty urine in the bladder. Answer: B Diff: 1 Page Ref: 728 80) The epithelium lining the urinary bladder that permits distension is A) stratified squamous. B) transitional. C) simple squamous. D) pseudostratified columnar. Answer: B Diff: 1 Page Ref: 733

18 Copyright © 2014 Pearson Education, Inc.

 

81) Which gland sits atop each kidney? A) pancreas B) pituitary C) adrenal D) interlobar gland Answer: C Diff: 1 Page Ref: 721 24.2 True/False Questions 1) The glomerular capsule and the glomerular capillaries together make up the renal corpuscle. Answer: TRUE Diff: 2 Page Ref: 725 2) Urine flows from the kidney to the bladder by gravity and peristalsis. Answer: TRUE Diff: 2 Page Ref: 731-732 3) The epithelial cells of the distal convoluted tubule have an abundance of absorptive microvilli. Answer: FALSE Diff: 2 Page Ref: 728 4) The descending thin limb of the nephron loop consists of a simple squamous epithelium. Answer: TRUE Diff: 2 Page Ref: 728 5) In females, the urinary bladder lies anterior to the vagina and uterus. Answer: TRUE Diff: 2 Page Ref: 732 6) The location and length of the urethra are two factors that contribute to the high incidence of urinary tract infections in women. Answer: TRUE Diff: 2 Page Ref: 736 7) The urinary bladder, when empty, lies within both the abdominal and pelvic cavities. Answer: FALSE Diff: 2 Page Ref: 732 8) A network of peritubular capillaries surrounds the convoluted tubules and nephron loop for the purpose of filtration. Answer: FALSE Diff: 3 Page Ref: 730

19 Copyright © 2014 Pearson Education, Inc.

 

9) The internal urethral sphincter is composed of smooth muscle and is under involuntary control. Answer: TRUE Diff: 2 Page Ref: 734 10) The kidneys are located just below lumbar vertebrae L2 and L3. Answer: FALSE Diff: 3 Page Ref: 721 11) The correct sequence of arterial blood flow is the renal artery to the segmental arteries to the interlobar arteries. Answer: TRUE Diff: 3 Page Ref: 723-724 12) Venous structures of the kidney mirror those of the arterial circuit, except for the absence of segmental veins. Answer: TRUE Diff: 3 Page Ref: 723-724 13) Urine drains from the kidney in the following sequence: from the collecting duct to the minor calyx to the major calyx to the renal pelvis and then to the ureter. Answer: TRUE Diff: 3 Page Ref: 722-723 14) The ureters enter the bladder obliquely at the anterolateral corners. Answer: FALSE Diff: 3 Page Ref: 731 15) Cortical and juxtamedullary nephrons can be distinguished by the absence of a vasa recta in the cortical nephrons. Answer: TRUE Diff: 3 Page Ref: 728-730 24.3 Short Answer Questions 1) The ________ urethral sphincter is composed of skeletal muscle and under voluntary control. Answer: external Diff: 1 Page Ref: 734 2) The calyces and renal pelvis are all flared branches off the ________, which is an organ that transmits urine to the bladder. Answer: ureter Diff: 1 Page Ref: 722-723

20 Copyright © 2014 Pearson Education, Inc.

 

3) The ________, a clinical landmark in the urinary bladder, is bounded by the two ureteric openings and the internal urethral orifice. Answer: trigone Diff: 2 Page Ref: 733-734 4) Most water and solutes are resorbed from the convoluted renal tubules into the ________. Answer: peritubular capillaries Diff: 2 Page Ref: 730 5) The U-shaped ________ is located between the proximal and distal convoluted tubules. Answer: nephron loop Diff: 2 Page Ref: 727-728 6) ________ are large cells with complex "footlike" processes that wrap around the glomerular capillaries. Answer: Podocytes Diff: 2 Page Ref: 725-726 7) The ________ artery is located between the interlobar and cortical radiate arteries. Answer: arcuate Diff: 2 Page Ref: 724 8) Between the fibrous renal capsule and the renal fascia is a cushioning and supportive ________ fat capsule. Answer: perirenal Diff: 2 Page Ref: 721 9) Adjacent collecting ducts join to form larger ________ that drain into the minor calyx. Answer: papillary ducts Diff: 2 Page Ref: 728 10) Blood pressure is regulated by a specialized structure, known as the ________, that contacts both the afferent arteriole and the terminal end of the nephron loop. Answer: juxtaglomerular apparatus Diff: 2 Page Ref: 730-731 11) The ________ layer of the ureters propel urine to the bladder by peristalsis. Answer: muscularis Diff: 2 Page Ref: 732 12) Voiding of urine, known as ________, involves sympathetic, parasympathetic, and somatic fibers. Answer: micturition Diff: 2 Page Ref: 735-736

21 Copyright © 2014 Pearson Education, Inc.

 

13) Five ________ arteries branch from the renal artery and enter the hilus. Answer: segmental Diff: 2 Page Ref: 724 14) ________ cells are modified smooth muscle cells of the afferent and efferent arterioles that secrete the hormone renin. Answer: Granular (or Juxtaglomerular) Diff: 2 Page Ref: 731 15) Glomeruli arise from branches off the ________ arterioles of the cortex. Answer: afferent glomerular Diff: 2 Page Ref: 724 24.4 Essay Questions 1) Describe the structure and function of the filtration slit and membrane. Answer: Blood is filtered through filtration slits in the glomerular capillaries. The slits consist of three layers. The fenestrations in the endothelium of the capillaries create large pores through which the blood filtrate passes. A basement membrane superficial to the endothelium provides a second layer of the barrier. It prevents most large proteins from passing. Finally, the pedicles of the podocytes form the third layer of the membrane. These also prevent proteins from passing through the slits. Diff: 2 Page Ref: 725-726 2) Describe how the structure of the proximal convoluted tubule walls reflects its function in filtrate processing. Answer: The walls of the proximal tubule are simple cuboidal epithelium. The luminal surface of the cells is covered by microvilli. These structures increase the surface area of the proximal tubule thereby greatly enhancing the resorption of water, electrolytes, and other solutes from the filtrate. Diff: 2 Page Ref: 727-728 3) Compare and contrast the construction of the bladder and ureter walls. Answer: The walls of both the bladder and ureter consist of three layers: a mucosa of transitional epithelium, a muscularis layer, and an external adventitia. The muscularis of the ureter consists of two layers of smooth muscle, circular and inner longitudinal, for most of its length, then adds a third external longitudinal layer at its distal end. The bladder has all three layers of smooth muscle. The adventitia of the bladder is replaced with parietal peritoneum on its superior surface. Diff: 2 Page Ref: 731-733

22 Copyright © 2014 Pearson Education, Inc.

 

4) Explain the significance of the intimate anatomical relationship between the nephrons and the peritubular capillaries and vasa recta. Answer: The vast majority, 99%, of the filtrate that is removed from the blood must be resorbed from the nephrons. If this fluid were not resorbed, the body would quickly dehydrate. All ions, solutes, and water that are resorbed from the nephron pass into the surrounding peritubular capillaries and return to the cortical radiate vein. Sodium and water resorbed from the nephron loop are picked up by the vasa recta and returned to the cortical radiate vein. Diff: 2 Page Ref: 730 5) Describe the structure, function, and control of the internal and external urethral sphincters in micturition. Answer: The internal urethral sphincter is composed of smooth muscle and is under involuntary control. In contrast, the external urethral sphincter is skeletal muscle that can be voluntarily controlled. The internal sphincter prevents leakage of urine between voiding, and the external sphincter prevents urination until consciously desired. Diff: 2 Page Ref: 734

23 Copyright © 2014 Pearson Education, Inc.

 

Human Anatomy, 7e (Marieb/Mitchell/Smith) Chapter 25 The Reproductive System 25.1 Multiple Choice Questions

Figure 25.1 Use the diagram above to answer the following questions. 1) Identify the letter that indicates the ejaculatory duct. A) A B) B C) C D) D E) E Answer: A Diff: 2 Page Ref: 744 2) Identify the letter that indicates the site of sperm maturation and storage. A) A B) B C) C D) D E) E Answer: C Diff: 2 Page Ref: 744, 747-748

1 Copyright © 2014 Pearson Education, Inc.

 

3) Identify the letter that indicates the organ that directs sperm toward the seminal vesicles during ejaculation. A) A B) B C) C D) D E) E Answer: E Diff: 2 Page Ref: 744, 748 4) Identify the letter that indicates the gland that secretes neutralizing mucus prior to ejaculation. A) A B) B C) C D) D E) E Answer: B Diff: 2 Page Ref: 744, 751 5) Identify the letter that indicates the organ that serves as common passageway for semen and urine in men. A) A B) B C) C D) D E) E Answer: D Diff: 2 Page Ref: 744,752

2 Copyright © 2014 Pearson Education, Inc.

 

Figure 25.2 Use the diagram above to answer the following questions. 6) Identify the letter that indicates the cervix. A) A B) B C) C D) D E) E Answer: E Diff: 2 Page Ref: 756 7) Identify the letter that indicates the organ where fertilization normally occurs. A) A B) B C) C D) D E) E Answer: C Diff: 2 Page Ref: 756-757

3 Copyright © 2014 Pearson Education, Inc.

 

8) Identify the letter that indicates the region of the uterus known as the fundus. A) A B) B C) C D) D E) E Answer: B Diff: 2 Page Ref: 756 9) Identify the letter that indicates the tissue layer that is shed during menstruation. A) A B) B C) C D) D E) E Answer: D Diff: 2 Page Ref: 756, 759 10) Identify the letter that indicates the organ where ectopic pregnancies commonly occur. A) A B) B C) C D) D E) E Answer: A Diff: 2 Page Ref: 756, 758

4 Copyright © 2014 Pearson Education, Inc.

 

Figure 25.3 Use the diagram above to answer the following questions. 11) Identify the letter that indicates the "neck" of the uterus. A) A B) B C) C D) D E) E Answer: A Diff: 1 Page Ref: 756, 758 12) Identify the letter that indicates the infundibulum. A) A B) B C) C D) D E) E Answer: D Diff: 2 Page Ref: 756 13) Identify the letter that indicates the female erectile tissue that is homologous to the penis. A) A B) B C) C D) D E) E Answer: E Diff: 2 Page Ref: 756, 767 5 Copyright © 2014 Pearson Education, Inc.

 

14) Identify the letter that indicates the gland that secretes lubricating mucus into the vaginal orifice. A) A B) B C) C D) D E) E Answer: C Diff: 2 Page Ref: 756, 767 15) Identify the letter that indicates the organ which is commonly referred to as the birth canal. A) A B) B C) C D) D E) E Answer: B Diff: 2 Page Ref: 756, 766 16) Tubular organ that is transected during a vasectomy. A) epididymis B) rete testis C) ductus deferens D) spermatic cord E) pampiniform plexus Answer: C Diff: 2 Page Ref: 749 17) The fluid-filled cavity in a mature ovarian follicle is known as A) zona pellucidum. B) antrum. C) corona radiate. D) theca folliculi. E) granulaosa cells. Answer: B Diff: 2 Page Ref: 763 18) This layer of the uterus contracts during parturition to expel the baby. A) perimetrium B) stratum functionalis C) stratum basalis D) myometrium E) endometrium Answer: D Diff: 1 Page Ref: 759 6 Copyright © 2014 Pearson Education, Inc.

 

19) The opening of the oviduct is surrounded by these structures. A) internal os B) external os C) fimbriae D) isthmus E) cardinal ligaments Answer: C Diff: 2 Page Ref: 757 20) Fertilization of the ovum normally occurs in this region of the oviduct. A) isthmus B) fimbriae C) infundibulum D) ampulla E) intrauterine portion Answer: D Diff: 2 Page Ref: 757 21) Involuntary contraction of this smooth muscle layer wrinkles and thickens the walls of the scrotum to prevent heat loss. A) tunica albuginea B) tunica vaginalis C) dartos muscle D) cremaster muscle E) internal spermatic fascia Answer: C Diff: 2 Page Ref: 745 22) Tubules that connect testicular lobules with the efferent ductules of the epididymis. A) seminiferous tubules B) rete testis C) pampiniform plexus D) ductus deferens E) straight tubules Answer: B Diff: 2 Page Ref: 746 23) This multinucleate embryonic structure invades the uterus, digesting cells and maternal blood vessels to make way for development of the placenta. A) syncytiotrophoblast B) morula C) inner cell mass D) decidua basalis E) decidua capsularis Answer: A Diff: 3 Page Ref: 770 7 Copyright © 2014 Pearson Education, Inc.

 

24) Type of cell in the testes that produces testosterone. A) spermatogonium B) primary spermatocyte C) spermatid D) interstitial E) sustentocytes Answer: D Diff: 2 Page Ref: 747 25) Fingerlike placental structures where oxygen and nutrients from the maternal blood supply diffuse into the embryonic blood. A) syncytiotrophoblast B) allantois C) amnion D) chorionic villi E) extraembryonic membrane Answer: D Diff: 2 Page Ref: 772 26) The usual site of embryo implantation is the A) uterus. B) peritoneal cavity. C) vagina. D) uterine tube. Answer: A Diff: 1 Page Ref: 770 27) A factor that propels an oocyte through the uterine tube is A) pressure exerted by ovarian fluid. B) smooth muscle contraction (peristalsis). C) secretions of the nonciliated cells. D) amoeboid motion of the oocyte. Answer: B Diff: 2 Page Ref: 757 28) Sperm become motile in the A) ductus deferens. B) epididymis. C) seminiferous tubules. D) prostate. Answer: B Diff: 1 Page Ref: 748

8 Copyright © 2014 Pearson Education, Inc.

 

29) Which of the following is the primary sex organ in the male? A) the prostate B) the testis C) the epididymis D) the penis Answer: B Diff: 1 Page Ref: 744 30) The correct sequence of sperm-transporting tubules that transport spermatozoa to the epididymis is the A) seminiferous tubule to efferent ductule to straight tubule to rete testis. B) seminiferous tubule to rete testis to straight tubule to efferent ductule. C) seminiferous tubule to rete testis to efferent ductule to straight tubule. D) seminiferous tubule to straight tubule to rete testis to efferent ductule. Answer: D Diff: 2 Page Ref: 746 31) The female homologue of the male scrotum is the A) ovary. B) labia majora. C) penile urethra. D) bulb of the vestibule. Answer: B Diff: 2 Page Ref: 767 32) A woman's sex cells (germ cells) originate embryologically A) in the innermost part of the gonad. B) from the germinal epithelium of the ovary. C) from the follicular cells of the follicles. D) from yolk sac endoderm. Answer: D Diff: 2 Page Ref: 777 33) In cows, the nipples (teats) are in the groin region, not in the midthorax as in humans. What is the most logical reason for this difference between cows and people? A) Cows' mammary glands have entirely different embryonic origins from human ones. B) Male cows do not produce milk. C) Cows are more likely to develop inguinal hernias, and their mammary glands protect against that. D) Cows' mammary glands arise from a different part of the milk line. Answer: D Diff: 3 Page Ref: 767

9 Copyright © 2014 Pearson Education, Inc.

 

34) The function of the pampiniform plexus of veins is to A) help prevent hernias. B) help cool blood. C) form the outer wall of the spermatic cord. D) form hydroceles. Answer: B Diff: 1 Page Ref: 747 35) At what point during fetal development is the decidua basalis/chorionic villi collectively referred to as the placenta? A) the first month B) the ninth month C) the start of fourth month D) the end of second month Answer: C Diff: 2 Page Ref: 772 36) A woman has developed a cancerous lump in each breast, and the skin of the breasts is dimpled. What structure(s) is (are) causing the dimpling? A) pectoralis major muscle B) pectoralis minor muscle C) smooth muscle cells around the ducts of the large glands in the breast D) suspensory ligaments of the breast Answer: D Diff: 2 Page Ref: 767, 775 37) All of the following are adaptations for keeping the testes cool except the A) descent of the testes into the scrotum before birth. B) pampiniform plexus of veins. C) cremaster and dartos. D) testicular artery. Answer: D Diff: 3 Page Ref: 745, 747 38) The seminal vesicles A) store sperm. B) are glands that secrete most of the volume of the semen. C) secrete digestive enzymes when cancerous. D) empty into the epididymis. Answer: B Diff: 1 Page Ref: 750

10 Copyright © 2014 Pearson Education, Inc.

 

39) In breast cancer, the cancer cells usually originate from the A) lactiferous ductules. B) cuboidal acinar cells. C) fat cells in the breast. D) endothelium of the lymph vessels in the breasts. Answer: A Diff: 1 Page Ref: 775 40) Which testicular cells help deliver sperm to the epididymis by contracting rhythmically? A) interstitial B) myoid C) sustentocytes D) type A spermatocytes Answer: B Diff: 2 Page Ref: 747 41) In the early fetus, all of the following are parts of the chorionic villi except the A) syncytiotrophoblast. B) cytotrophoblast. C) extraembryonic mesoderm. D) decidua. Answer: D Diff: 3 Page Ref: 770, 772 42) Inguinal hernias A) always reach the scrotum. B) always enter the inguinal canal. C) occur with equal frequency in males and females. D) are signs of old age and almost never congenital. Answer: B Diff: 2 Page Ref: 749 43) Hypospadias A) never occurs in females. B) is characterized by the absence of a urethra. C) cannot be detected through an examination of the outside of a baby's body. D) has no effects until puberty, when males start producing sperm. Answer: A Diff: 1 Page Ref: 777 44) The function of the cremaster muscle is to A) squeeze semen from the urethra during ejaculation. B) maintain erection in both sexes. C) increase intra-abdominal pressure during childbirth. D) alter the position of the testes in the scrotal sac with increases in ambient temperature. Answer: D Diff: 1 Page Ref: 745 11 Copyright © 2014 Pearson Education, Inc.

 

45) By undergoing meiosis, each primary spermatocyte ultimately gives rise to how many sperm cells? A) two B) four C) eight D) billions Answer: B Diff: 2 Page Ref: 754 46) Of the following tubes, which is the only one that lies partly outside the testis? A) tubulus rectus B) seminiferous tubule C) rete testis D) efferent ductule Answer: D Diff: 1 Page Ref: 746 47) Which of the following anchors the uterus into an anteverted position in the pelvis? A) broad ligament B) ovarian ligament C) round ligament D) suspensory ligament Answer: C Diff: 2 Page Ref: 759 48) The ovarian cortex consists of A) follicles and connective tissue. B) the largest blood vessels of the ovary. C) the hilus of the ovary. D) the mesovarium. Answer: A Diff: 1 Page Ref: 761-763 49) The female uterine cycle begins A) at ovulation. B) on the first day of menstruation. C) at the end of menstruation. D) every six weeks. Answer: B Diff: 2 Page Ref: 763

12 Copyright © 2014 Pearson Education, Inc.

 

50) In addition to remaining granulosa cells, which of the following cell types make up the corpus luteum? A) oocyte B) theca cells C) cells of the corona radiata D) interstitial cells Answer: B Diff: 2 Page Ref: 763 51) The largest macroscopic region of the uterus is its A) fundus. B) body. C) cervix. D) fornix. Answer: B Diff: 1 Page Ref: 758 52) The zona pellucida A) is the portion of the placenta in which maternal blood contacts fetal tissues. B) is a protective shell around the oocyte that sperm must penetrate for fertilization to occur. C) is the region of the uterine tube where fertilization occurs. D) is the remnant of the vesicular follicle that becomes the corpus luteum. Answer: B Diff: 2 Page Ref: 763 53) The prepuce of the clitoris is formed by A) the fourchette. B) the hymen of the vagina. C) the labia minora. D) the mons pubis. Answer: C Diff: 1 Page Ref: 767 54) The milk-producing cells in the breast are A) adipose cells (because milk contains 4% fat). B) columnar epithelial cells in lobules. C) simple cuboidal epithelial cells in alveoli. D) endocrine cells. Answer: C Diff: 2 Page Ref: 768

13 Copyright © 2014 Pearson Education, Inc.

 

55) In females, the paramesonephric ducts give rise to A) the ovaries. B) the vestibule in the vulva. C) the corpora cavernosa (erectile bodies). D) the uterine tubes and uterus. Answer: D Diff: 2 Page Ref: 777 56) The epithelium lining the vagina is A) simple columnar. B) stratified squamous. C) pseudostratified columnar. D) simple squamous. Answer: B Diff: 1 Page Ref: 766 57) The most common site of ectopic pregnancy is the A) uterus. B) peritoneal cavity. C) vagina. D) uterine tube. Answer: D Diff: 1 Page Ref: 758 58) Which of the following events occurs when the testes do not descend before birth? A) Male sex hormones will not circulate in the body. B) Sperm will have no route of exit from the body. C) Inadequate blood supply will retard the development of the testes. D) Viable sperm will not be produced. Answer: D Diff: 2 Page Ref: 778 59) Which of the following organs does not contribute secretion(s) during the male sexual response? A) prostate B) seminal vesicles C) bulbourethral glands D) corpus cavernosum Answer: D Diff: 2 Page Ref: 750-751

14 Copyright © 2014 Pearson Education, Inc.

 

60) During the proliferative phase of the uterine cycle, these vessels grow and proliferate. A) arcuate arteries B) radial arteries C) spiral arteries D) straight arteries Answer: C Diff: 2 Page Ref: 759 61) Which female structure is homologous to the corpus spongiosum of the penis? A) clitoris B) crura C) bulb of the vestibule D) labia minora Answer: C Diff: 3 Page Ref: 767 62) Erection is caused by A) a jointed bone in the penis that unfolds. B) the penis filling up with semen that is about to be ejaculated. C) erectile bodies filling with blood. D) impulses from the sympathetic nervous system. Answer: C Diff: 2 Page Ref: 752 63) Sperm are moved into the epididymis from the testis by A) swimming under their own power. B) gravity. C) suction. D) testicular fluid moved by cilia and smooth muscle cells. Answer: D Diff: 2 Page Ref: 747 64) The deep inguinal ring is formed by A) extension of the external oblique muscle. B) extension of the internal oblique muscle. C) fascia layer deep to the transversus abdominis muscle. D) cremaster muscle. Answer: C Diff: 2 Page Ref: 749 65) A hydrocele A) occurs in females only. B) is a series of varicosities of the pampiniform plexus. C) is a sac filled with intestines that have pushed through the abdominal wall. D) is excess serous fluid in the scrotum. Answer: D Diff: 1 Page Ref: 780 15 Copyright © 2014 Pearson Education, Inc.

 

66) A difference between a primary follicle and a primordial follicle is that A) one has an antrum and the other does not. B) the oocyte is larger in the primordial follicle. C) the primary follicle is an earlier stage than the primordial follicle. D) the primordial follicle has only a single layer of flat follicular cells, whereas the primary follicle is comprised of cuboidal follicle cells. Answer: D Diff: 2 Page Ref: 763 67) The acrosomal reaction occurs A) just before fertilization. B) during meiosis of the sperm. C) during spermiogenesis. D) during placentation. Answer: A Diff: 2 Page Ref: 769 68) Which structure develops into the umbilical cord? A) body stalk B) chorionic villi C) decidua capsularis D) placenta Answer: A Diff: 1 Page Ref: 772 69) The placenta consists of which two structures? A) chorionic villi and amnion B) chorionic villi and decidua basalis C) cytotrophoblast and syncytiotrophoblast D) decidua basalis and decidua capsularis Answer: B Diff: 3 Page Ref: 772 70) Which of the following pairs of structures is not derived from the same embryonic tissues? A) ductus deferens, uterine tube B) penis, clitoris C) spongy urethra, labia minora D) testes, ovaries Answer: A Diff: 3 Page Ref: 777

16 Copyright © 2014 Pearson Education, Inc.

 

71) Which structure is not derived from the embryonic paramesonephric ducts? A) cervix B) clitoris C) upper vagina D) uterine tube Answer: B Diff: 2 Page Ref: 777 72) The structural basis of the blood-testis barrier is A) the basal lamina around the seminiferous tubules. B) the endothelial cells of the continuous (nonfenestrated) capillaries that supply the seminiferous tubules. C) the tight junctions between sustentocytes. D) the thick basal lamina around the capillaries that supply the seminiferous tubules. Answer: C Diff: 1 Page Ref: 755 73) In the late stages of pregnancy, the placental barrier of the chorionic villi consists only of which two layers? A) decidua capsularis and decidua basalis B) syncytiotrophoblast and cytotrophoblast C) syncytiotrophoblast and capillary endothelium D) cytotrophoblast and extraembryonic mesoderm Answer: C Diff: 3 Page Ref: 772 74) An episiotomy during childbirth is performed to A) save the baby's life. B) minimize tearing of the central tendon and muscles of the pelvic floor. C) prevent the anal canal from being crushed. D) eliminate the pain to the mother. Answer: B Diff: 1 Page Ref: 767 75) Which of the following statements about pelvic inflammatory disease is incorrect? A) It can lead to sterility. B) It is usually caused by chlamydia or gonorrhea. C) It includes salpingitis. D) It inhibits ovulation. Answer: D Diff: 2 Page Ref: 758

17 Copyright © 2014 Pearson Education, Inc.

 

76) All of the following are functions of the sustentocytes in the seminiferous tubules except A) secretion of androgen-binding protein. B) secretion of testicular fluid. C) secretion of testosterone. D) phagocytosis of excess cytoplasm from spermatids during spermiogenesis. Answer: C Diff: 2 Page Ref: 747 77) The typical (diploid) cells of the body have 46 chromosomes; how many chromosomes does each gamete have? A) 23 B) 46 C) 46 pairs D) 184 Answer: A Diff: 2 Page Ref: 754, 760 78) All of the following are located within the spermatic cord except the A) superficial inguinal ring. B) ductus deferens. C) pampiniform plexus. D) testicular artery. Answer: A Diff: 2 Page Ref: 747, 749 79) The embryonic urethral folds develop into the female A) bulb of the vestibule. B) labia majora. C) labia minora. D) urethra. Answer: C Diff: 2 Page Ref: 777 80) The cells that secrete estrogens in females are A) oocytes. B) cells in the pituitary gland. C) granulosa cells. D) germinal epithelium of the ovary. Answer: C Diff: 2 Page Ref: 763

18 Copyright © 2014 Pearson Education, Inc.

 

81) Technically, the theca folliculi is not part of the ovarian follicle, although it is associated with it. Instead, the theca folliculi arises from the A) zona pellucida. B) basal lamina around the follicle. C) connective tissue that fills the area between follicles. D) ovarian medulla. Answer: C Diff: 2 Page Ref: 763 82) In which part of the uterine tube does fertilization usually occur? A) ampulla B) isthmus C) infundibulum D) It doesn't occur in the uterine tubes. Answer: A Diff: 1 Page Ref: 757 83) Most support of the uterus is provided by the A) mesometrium. B) round ligament. C) cardinal ligament. D) muscles of the pelvic floor. Answer: D Diff: 1 Page Ref: 759 84) The vaginal fornix A) is in the superior part of the vestibule. B) surrounds the tip of the cervix. C) is in the middle layer of the vaginal wall. D) surrounds the external vaginal orifice. Answer: B Diff: 1 Page Ref: 766 85) In an adult woman who has never been pregnant, the breasts have poorly developed or no A) lobules of acini/alveoli. B) lactiferous sinuses. C) suspensory ligaments. D) areolar glands. Answer: A Diff: 2 Page Ref: 768

19 Copyright © 2014 Pearson Education, Inc.

 

86) The capillaries within a chorionic villus of the placenta contain blood A) from the fetus. B) from the mother. C) from both the fetus and the mother. D) that doesn't belong to the fetus or the mother. Answer: A Diff: 3 Page Ref: 772 87) In males, the paramesonephric (Müllerian) ducts A) give rise to the penile urethra. B) give rise to the prostate gland. C) give rise to the vas deferens. D) degenerate and don't form any structures in males. Answer: D Diff: 2 Page Ref: 777 25.2 True/False Questions 1) The spermatic cord is another name for the ductus deferens. Answer: FALSE Diff: 1 Page Ref: 748-749 2) Breast milk collects in the lactiferous sinus when a woman breastfeeds her infant. Answer: TRUE Diff: 2 Page Ref: 768 3) Primordial follicles develop into primary oocytes within the germinal epithelium of the ovary. Answer: FALSE Diff: 2 Page Ref: 761-763 4) Although the ovaries and uterine tubes are retroperitoneal, oocytes are ovulated into the peritoneal cavity. Answer: TRUE Diff: 3 Page Ref: 763 5) The ductus deferens and uterine tube are homologous structures, each deriving from the same embryonic duct. Answer: FALSE Diff: 2 Page Ref: 777 6) Sustentocytes support developing sperm and form a blood-testis barrier. Answer: TRUE Diff: 2 Page Ref: 747 7) Spermatogonia are located near the lumen of seminiferous tubules. Answer: FALSE Diff: 2 Page Ref: 752-754 20 Copyright © 2014 Pearson Education, Inc.

 

8) The ejaculatory ducts are formed by the union of the ductus deferens and urethra. Answer: FALSE Diff: 2 Page Ref: 748, 750 9) Granulosa cells of a maturing ovarian follicle convert androgens to estrogens that will stimulate proliferation of the endometrium. Answer: TRUE Diff: 2 Page Ref: 763 10) Breast milk production and secretion follows the following sequence: lactiferous sinus, alveoli, lactiferous ducts, nipple. Answer: FALSE Diff: 2 Page Ref: 767-768 11) Seminiferous tubules are the site of sperm storage. Answer: FALSE Diff: 2 Page Ref: 745-746, 752 12) Cryptorchidism is a congenital condition in which one or both testes fail to descend into the scrotum. Answer: TRUE Diff: 2 Page Ref: 778 13) Oogenesis takes years to complete. Answer: TRUE Diff: 2 Page Ref: 760 14) The epididymis, ductus deferens, and seminal vesicles are all derived from mesonephric (Wolffian) ducts. Answer: TRUE Diff: 3 Page Ref: 776-777 15) Lobules and alveoli of the breast develop during puberty. Answer: FALSE Diff: 3 Page Ref: 768 25.3 Short Answer Questions 1) The ________ is formed by the union of the ductus deferens and seminal vesicles. Answer: ejaculatory duct Diff: 2 Page Ref: 748, 750 2) The ________ of the female genitalia is a central space surrounded by the labia minora. Answer: vestibule Diff: 2 Page Ref: 767 21 Copyright © 2014 Pearson Education, Inc.

 

3) The bulb of the vestibule and the ________ are female erectile bodies. Answer: clitoris Diff: 2 Page Ref: 767 4) The uterus consists of the fundus, body, cervix, and ________. Answer: isthmus Diff: 2 Page Ref: 758 5) Ovarian follicles contain ________ and thecal cells that, after ovulation, differentiate into the corpus luteum Answer: granulosa cells Diff: 2 Page Ref: 763 6) An ovulated oocyte swept into the oviduct via fimbriae moves from the ________ of the oviduct to the ampula, then the isthmus, and finally into the uterus. Answer: infundibulum Diff: 2 Page Ref: 757 7) Contraction of the ________ in the wall of the spermatic cord raises the testes closer to the body cavity. Answer: cremaster muscle Diff: 2 Page Ref: 745 8) The uterus is composed of three basic layers. The inner layer is the ________. Answer: endometrium Diff: 2 Page Ref: 759 9) The first menstruation is called ________. Answer: menarche Diff: 2 Page Ref: 779 10) Testes descend through the ________ into the scrotum. Answer: inguinal canal Diff: 2 Page Ref: 777-778 11) Contraction of the fibromuscular ________ of the prostate squeezes prostatic secretions into the urethra during ejaculation. Answer: stroma Diff: 2 Page Ref: 750-751 12) During the menstrual phase of the uterine cycle, the inner ________ of the endometrium is shed. Answer: functional layer Diff: 2 Page Ref: 759

22 Copyright © 2014 Pearson Education, Inc.

 

13) The crura are extensions of the ________, the paired erectile bodies of the penis, and attach the penis to the pubic arch. Answer: corpora cavernosa Diff: 3 Page Ref: 752 14) The secretory phase of the uterine cycle corresponds to the ________ phase of the ovarian cycle. Answer: luteal Diff: 2 Page Ref: 763-764 15) The most effective way to detect cervical cancer in its earliest stage is by a(n) ________. Answer: Papanicolaou (Pap) smear Diff: 2 Page Ref: 775 25.4 Essay Questions 1) Describe the structures involved in the descent of the testes. Answer: The testes descend from the lumbar region of the abdominal cavity to the pelvic region. The vaginal process, an extension of the parietal peritoneum protrudes through an opening in the muscles of the anterior abdominal wall to create the inguinal canal. The testes descend through this canal into the scrotum during the seventh month of fetal development. The mechanism for the descent of the testes is not known, however their movement from the abdominopelvic cavity is guided by a fibrous cord called the gubernaculum. This cord connects from the scrotal floor to the testis. As it shortens, the testes move through the inguinal canal. Diff: 2 Page Ref: 777-778 2) Describe the mechanisms of propulsion by which sperm are transported from the seminiferous tubules to the epididymis and, during ejaculation, from the epididymis out of the body. Answer: Sperm are carried to the epididymis on a current of testicular fluid that is continuously produced by sustentocytes. The positive pressure of the fluid production forces the nonmotile sperm forward. Additionally, myoid cells that surround the seminiferous tubules rhythmically contract, helping to squeeze the sperm out of the testes. During ejaculation, smooth muscle cells in the epididymis and ductus deferens propel sperm by strong peristaltic contractions. Finally, the bulbospongiosus muscle covering the bulb of the penis contracts, propelling semen through the spongy urethra. Diff: 2 Page Ref: 747-748, 752 3) Identify the space into which an oocyte is ovulated, and describe the structures and mechanisms involved in transporting it to the uterus. Answer: The oocyte is ovulated into the peritoneal cavity. Currents, created by the motion of the fimbria of the oviduct, draw the oocyte into the infundibulum. Ciliated columnar cells lining the oviduct create currents that move the oocyte through the ampulla, to the isthmus, and into the uterus. The journey of the oocyte is assisted by peristaltic contractions of the smooth muscle in the oviduct walls. Diff: 2 Page Ref: 757

23 Copyright © 2014 Pearson Education, Inc.

 

4) Menstruation occurs as a result of the absence (or low level) of a particular hormone. Identify (a) this hormone, (b) the specific arteries of the uterine lining that collapse/fracture, and (c) the specific layer of the uterus that is shed. Answer: Menstruation, or shedding of the endometrium, occurs due to the rapid decline of progesterone. Without progesterone, the spiral arteries of the endometrium (located in the functional layer) spasm, collapse, and degenerate. Diff: 3 Page Ref: 759 5) Describe the structures involved in the transfer of nutrients and wastes between the fetal and maternal bloodstreams. Answer: Nutrients and wastes cross the placenta, which consists of both embryonic and maternal tissues. Extensions of the embryonic tissue are called chorionic villi. These extensions protrude into lacunae, or cavities in the endometrium, which contain maternal blood. Adjacent to these lacunae are maternal blood vessels located in a region called the decidua basalis. Together, the decidua basalis and chorionic villi, when fully established at the fourth month of fetal development, are called the placenta. Diff: 3 Page Ref: 772

24 Copyright © 2014 Pearson Education, Inc.

View more...

Comments

Copyright ©2017 KUPDF Inc.
SUPPORT KUPDF